Sir Stanley Davidson(1894–1981)
This famous textbook was the brainchild of one of the great Professors of
Medicine of the 20th century. Stanley Davidson was born in Sri Lanka and
began his medical undergraduate training at Trinity College, Cambridge;
this was interrupted by World War I and later resumed in Edinburgh. He
was seriously wounded in battle, and the carnage and shocking waste of
young life that he encountered at that time had a profound effect on his
subsequent attitudes and values.
In 1930 Stanley Davidson was appointed Professor of Medicine at
the University of Aberdeen, one of the rst full-time Chairs of Medicine
anywhere and the rst in Scotland. In 1938 he took up the Chair of
Medicine at Edinburgh and was to remain in this post until retirement
in 1959. He was a renowned educator and a particularly gifted teacher
at the bedside, where he taught that everything had to be questioned
and explained. He personally gave most of the systematic lectures in
Medicine, which were made available as typewritten notes that empha-
sised the essentials and far surpassed any textbook available at the time.
Principles and Practice of Medicine was conceived in the late 1940s
with its origins in those lecture notes. The rst edition, published in 1952,
was a masterpiece of clarity and uniformity of style. It was of modest
size and price, but sufciently comprehensive and up to date to provide
students with the main elements of sound medical practice. Although
the format and presentation have seen many changes in 23 subsequent
editions, Sir Stanley’s original vision and objectives remain. More than
half a century after its rst publication, his book continues to inform and
educate students, doctors and health professionals all over the world.
4.
24th Edition
Edited by
IanD Penman
BSc(Hons), MBChB, MD, FRCPE
Consultant Gastroenterologist, Royal Inrmary of Edinburgh;
Honorary Senior Lecturer, University of Edinburgh, UK
Stuart H Ralston
MBChB, MD, FRCP, FMedSci, FRSE, FFPM(Hon)
Professor of Rheumatology, Centre for Genomic and Experimental Medicine,
Institute of Genetics and Cancer University of Edinburgh;
Honorary Consultant Rheumatologist, Western General Hospital,
Edinburgh, UK
Mark WJ Strachan
BSc(Hons), MBChB(Hons), MD, FRCPE
Consultant Endocrinologist, Metabolic Unit, Western General Hospital,
Edinburgh; Honorary Professor, University of Edinburgh, UK
Richard P Hobson
MBBS, LLM, PhD, MRCP(UK), FRCPath
Consultant Microbiologist, Harrogate and District NHS Foundation Trust;
Honorary Senior Lecturer, University of Leeds, UK
Illustrations by Robert Britton
London New York Oxford Philadelphia St Louis Sydney 2023
Medicine
Davidson’s
Principles and Practice of
Contents
Preface ix
Contributors xi
InternationalAdvisory Board xv
Acknowledgements xvii
Introduction xix
PART 1 FUNDAMENTALS OF MEDICINE 1
1. Clinical decision-making 1
N Cooper, AL Cracknell
2. Clinical therapeutics and good prescribing 13
SRJ Maxwell
3. Clinical genetics 37
K Tatton-Brown
4. Clinical immunology 59
SL Johnston
5. Population health and epidemiology 87
H Campbell, DA McAllister
6. Principles of infectious disease 97
JAT Sandoe, DH Dockrell
7. Oncology 127
S Clive, M Stares
8. Pain and palliative care 153
LA Colvin, M Fallon
PART 2 EMERGENCY AND CRITICAL CARE MEDICINE 175
9. Acute medicine and critical illness 175
VR Tallentire, MJ MacMahon
10. Poisoning 219
SHL Thomas
7.
vi CONTENTS
11.Envenomation 239
J White
12. Medicine in austere environments 251
AC Baker
PART 3 CLINICAL MEDICINE 261
13. Infectious disease 261
DH Dockrell, S Sundar, BJ Angus
14. HIV infection and AIDS 349
G Maartens
15. Sexually transmitted infections 369
DJ Clutterbuck
16. Cardiology 385
DE Newby, NR Grubb
17. Respiratory medicine 479
IJ Clifton, DAB Ellames
18. Nephrology and urology 557
B Conway, PJ Phelan, GD Stewart
19. Clinical biochemistry and metabolic medicine 613
A Mather, DR Sullivan, E Miller-Hodges
20. Endocrinology 647
JDC Newell-Price, FW Gibb
21. Diabetes mellitus 703
JR Petrie, JG Boyle
22. Nutritional factors in disease 755
AG Shand, MEJ Lean
23. Gastroenterology 781
A Rej, TS Chew, DS Sanders
24. Hepatology 859
MJ Williams, TT Gordon-Walker
25. Haematology and transfusion medicine 921
HG Watson, DJ Culligan, LM Manson
26. Rheumatology and bone disease 989
GPR Clunie, SH Ralston
27. Dermatology 1063
SH Ibbotson
28. Neurology 1119
DPJ Hunt, MD Connor
8.
CONTENTS vii
29.Stroke medicine 1201
W Whiteley, R Woodeld
30. Medical ophthalmology 1217
JA Olson
31. Medical psychiatry 1235
RM Steel, SM Lawrie
32. Maternal medicine 1263
L Mackillop, FEM Neuberger
33. Adolescent and transition medicine 1281
RJ Mann
34. Ageing and disease 1295
TJ Quinn
35. Laboratory reference ranges 1309
SJ Jenks
Index 1317
Get the most out of your Davidson’s!
Redeem your unique e-content PIN code today (see inside front cover) to access:
The complete, downloadable eBook – for quick reference, anytime access
BONUS NEW self-assessment material – 166 interactive questions and answers
supplement each chapter, to help test your understanding of key points and aid exam
preparation
Preface
Well over 2.5million copies of Davidson’s Principles and Practice of
Medicine have been sold since it was rst published in 1952. Now in
its 24th Edition, Davidson’s is regarded as a ‘must-have’ textbook for
thousands of medical students, doctors and health professionals across
the world, describing the pathophysiology and clinical features of the
most important conditions encountered in the major specialties of adult
medicine and explaining how to investigate, diagnose and manage
them. The book is the winner of numerous prizes and awards and has
been translated into many languages. Taking its origins from Sir Stanley
Davidson’s much-admired lecture notes, the book has endured because
it continues to keep pace with how modern medicine is taught and to
provide a wealth of information in an easy-to-read, concise and beauti-
fully illustrated format.
Davidson’s strives to ensure that readers can not only recognise the
clinical features of a disease, but also understand the underlying causes.
To achieve this, each chapter begins with a summary of the relevant
pre-clinical science, linking pathophysiology with clinical presentation
and treatment so that students can use the book from the start of their
medical studies right through to their nal examinations and beyond.
The regular introduction of new authors and editors is important for
maintaining freshness. On this occasion, 21 new authors have joined
our existing contributors to make up an outstanding team of authorities
in their respective elds. As well as recruiting authors from around the
globe, particularly for topics such as infectious diseases, HIV and enven-
omation, we welcome members from 10 countries on to our International
Advisory Board. These leading experts provide detailed comments that
are crucial to our revision of each new edition. A particularly important
aspect in planning the revision is for the editors to meet students and
faculty in medical schools in those countries where the book is most
widely read, so that we can respond to the feedback of our global read-
ership and their tutors. We use this feedback, along with the information
we gather via detailed student reviews and surveys, to craft each edition.
The authors, editors and publishing team aim to ensure that readers all
over the world are best served by a book that integrates medical science
with clinical medicine to convey key knowledge and practical advice in
an accessible and readable format. The amount of detail is tailored to the
needs of medical students working towards their nal examinations, as
well as candidates preparing for Membership of the Royal Colleges of
Physicians (MRCP) or its equivalent.
With this new edition we have introduced several changes in both
structure and content. The opening eight chapters provide an account of
the principles of genetics, immunology, infectious diseases, population
health, oncology and pain management, along with a discussion of the
core principles behind clinical decision-making and good prescribing.
Subsequent chapters discuss medical emergencies in poisoning, enven-
omation and medicine in austere environments, while common presenta-
tions in acute medicine, including recognition and management of the
critically ill patient, are also addressed. The disease-specic chapters
that follow cover the major medical specialties, each one thoroughly
revised and updated to ensure that readers have access to the ‘cutting
edge’ of medical knowledge and practice. As we publish the 24th edi-
tion, the world is in the grip of the COVID-19 pandemic and while our
knowledge of virology, epidemiology, clinical impact and management
of SARS-CoV-2 is still evolving, we have dedicated a new section on
core aspects of this hugely important topic in Chapter 13, but also in
Chapter6 and, as appropriate, elsewhere throughout the book.
The innovations introduced in recent editions have been maintained
and, in many cases, developed. The highly popular ‘Clinical Examination’
overviews have been extended and updated. The ‘Presenting Problems’
sections continue to provide an invaluable overview of the most common
presentations in each disease area. The ‘Emergency’ and ‘Practice Point’
boxes have been retained along with the ‘In Old Age’, ‘In Pregnancy’
and ‘In Adolescence’ boxes, which emphasise key practical points in
the presentation and management of the older adult, women with medi-
cal disorders who are pregnant or planning pregnancy, and adolescents
transitioning between paediatric and adult services.
Education is achieved by assimilating information from many sources
and readers of this book can enhance their learning experience by using
several complementary resources. We developed a self-testing compan-
ion book entitled Davidson’s Assessment in Medicine, containing over
1250 multiple choice questions specically tailored to the contents of
Davidson’s for the 23rd edition and have added more new online MCQs
to accompany this edition. The long-standing association of Davidson’s
with its sister books, Macleod’s Clinical Examination and Principles and
Practice of Surgery, still holds good. Our ‘family’ has also expanded with
the publication of Davidson’s Essentials of Medicine, a pocket-sized
version of the main text, now in its 3rd edition; and Macleod’s Clinical
Diagnosis, which describes a systematic approach to the differential
diagnosis of symptoms and signs. We congratulate the editors and
authors of these books for continuing the tradition of easily digested and
expertly illustrated texts.
We all take immense pride in continuing the great tradition rst estab-
lished by Sir Stanley Davidson and in producing an outstanding book for
the next generation of doctors.
IDP, SHR, MWJS, RPH
Edinburgh 2022
Contributors
Brian J AngusMD, FRCP, DTM&H
Associate Professor, Nufeld Department of Medicine,
Oxford University, Oxford, UK
Adam C Baker BM, BMedSci(Hons), MSc ExMed
Pre-Hospital Emergency Medicine Fellow, Honorary Senior Clinical
Research Fellow, Department of Emergency Medicine,
University Hospitals Plymouth NHS Trust, Devon Air Ambulance,
University of Exeter, Devon, UK
James G Boyle MBChB, MD, MSc, FRCP
Consultant Diabetologist, Department of Diabetes, Endocrinology and
Clinical Pharmacology, Glasgow Royal Inrmary; Honorary Associate
Clinical Professor, School of Medicine, Dentistry and Nursing,
University of Glasgow, Glasgow, UK
Harry Campbell MBChB, MD, FRCPE, FFPH, FRSE, FMedSci
Professor of Genetic Epidemiology and Public Health,
Centre for Global Health, Usher Institute, University of Edinburgh,
Edinburgh, UK
Thean Soon Chew MBChB, MRCP, PhD
Consultant Gastroenterologist and Honorary Senior Lecturer in
Gastroenterology, Academic Unit of Gastroenterology,
University of Shefeld, Shefeld, UK
Ian J Clifton MD, FRCP
Consultant Respiratory Physician and Honorary Senior Lecturer,
Department of Respiratory Medicine, St James’s University Hospital,
Leeds, UK
Sally Clive BMedSci(Hons), MBChB, MD, FRCPE
Consultant Medical Oncologist, Edinburgh Cancer Centre, Western
General Hospital, Edinburgh, UK
Gavin PR Clunie MD, FRCP
Consultant Rheumatologist and Metabolic Bone Physician,
Department of Rheumatology, Cambridge University Hospital NHS
Foundation Trust, Addenbrooke’s Hospital, Cambridge, UK
Daniel J Clutterbuck BSc(Hons), FRCP
Consultant in Genitourinary and HIV Medicine, NHS Lothian;
Chalmers Sexual Health Centre, Edinburgh, UK
Lesley A Colvin MBChB, PhD, FRCA, FFPMRCA, FRCPE
Professor of Pain Medicine, Division of Population Health
and Genomics, University of Dundee School of Medicine,
Dundee, UK
Myles D Connor MBBCh, FCP(SA), FCNeurol(SA), PhD, FRCPE
Consultant Neurologist, Department of Neurology, NHS Borders,
Melrose, UK; Honorary Senior Lecturer, Centre for Clinical Brain
Sciences, University of Edinburgh, Edinburgh, UK; Honorary Senior
Researcher, School of Public Health,
University of the Witwatersrand, Johannesburg, South Africa
Bryan Conway MB, MRCP, PhD
Senior Lecturer and Honorary Consultant Nephrologist, Department of
Renal Medicine, University of Edinburgh and Edinburgh Royal Inrmary,
Edinburgh, UK
Nicola Cooper MBChB, MMedSci, FRCPE, FRACP, FAcadMEd,
SFHEA
Consultant Physician and Clinical Associate Professor in Medical
Education, Department for Acute Internal Medicine, University Hospitals
of Derby & Burton NHS Foundation Trust, Derby, UK
Alison L Cracknell MBChB, FRCP
Consultant in Medicine for Older People, Department of Medicine for
Older People, Leeds Teaching Hospitals NHS Trust; Honorary Clinical
Associate Professor, University of Leeds, Leeds, UK
Dominic J Culligan BSc, MD, FRCP, FRCPath
Consultant Haematologist and Honorary Senior Lecturer,
Department of Haematology, Aberdeen Royal Inrmary,
Aberdeen, UK
David H Dockrell MD, FRCPI, FRCPG, FACP
Chair of Infection Medicine, Centre for Inammation Research,
University of Edinburgh; Professor of Infection Medicine,
UoE Centre for Inammation Research, University of Edinburgh,
Edinburgh, UK
Deborah AB Ellames BSc(Hons), MBChB
Consultant Physician in Respiratory Medicine, Department of
Respiratory Medicine, Leeds Teaching Hospitals NHS Trust, Leeds, UK
Marie Fallon MBChB, MD, FRCPG, FRCPE, DCH, DRCOG,
MRCGP
St Columba’s Hospice Chair of Palliative Medicine, Edinburgh Cancer
Research Centre, Institute for Genetics and Cancer, University of
Edinburgh, Edinburgh, UK
Fraser W Gibb MBChB, BSc(Hons), FRCP, PhD
Consultant Physician/Honorary Clinical Reader, Edinburgh Centre for
Endocrinology and Diabetes, Royal Inrmary of Edinburgh/
University of Edinburgh, Edinburgh, UK
13.
xii CONTRIBUTORS
TimothyT Gordon-Walker MBChB, PhD, MRCP
Consultant Hepatologist, Department of Gastroenterology and
Scottish Liver Transplant Unit, The Royal Inrmary of Edinburgh,
Edinburgh, UK
Neil R Grubb MD, FRCP
Consultant in Cardiology and Cardiac Electrophysiology,
Department of Cardiology, Royal Inrmary of Edinburgh, Edinburgh, UK
David PJ Hunt MB BChir, PhD, FRCP
Wellcome Senior Clinical Fellow, Department of Clinical Brain Sciences,
University of Edinburgh, Edinburgh, UK
Sally H Ibbotson BSc(Hons), MD, FRCPE
Professor of Photodermatology, University of Dundee; Honorary
Consultant Dermatologist and Head of Photobiology Unit,
Ninewells Hospital and Medical School, Dundee, UK
Sara J Jenks MBChB, MRCP, FRCPath
Consultant in Metabolic Medicine, Department of Clinical Biochemistry,
Royal Inrmary of Edinburgh, Edinburgh, UK
Sarah L Johnston FRCP, FRCPath
Consultant Immunologist, Department of Immunology and
Immunogenetics, North Bristol NHS Trust, Bristol, UK
Stephen M Lawrie MD(Hons), FRCPsych, Hon FRCPE, FRSE
Professor of Psychiatry and Neuroimaging, Division of Psychiatry,
Centre for Clinical Brain Sciences, University of Edinburgh,
Edinburgh, UK
Michael EJ Lean MA, MB, BChir, MD, FRCP, FRSE
Professor of Human Nutrition, Human Nutrition, University of Glasgow,
Glasgow, UK
Gary Maartens MBChB, MMed, FCP(SA)
Chair of Clinical Pharmacology, Department of Medicine,
University of Cape Town, Cape Town, South Africa
Lucy H Mackillop BM BCh, MA(Oxon), FRCP
Consultant Obstetric Physician, Oxford University Hospitals,
NHS Foundation Trust; Honorary Senior Clinical Lecturer,
Nufeld Department of Women’s and Reproductive Health,
University of Oxford, Oxford, UK
Michael J MacMahon FRCA, EDIC
Consultant Intensivist, Department of Anaesthesia and Intensive Care,
Victoria Hospital, Kirkcaldy, UK
Rebecca J Mann BMedSci, BMBS, MRCP, FRCPCh
Consultant Paediatrician, Department of Paediatrics, Taunton and
Somerset NHS Foundation Trust, Taunton, UK
Lynn M Manson MBChB, MD
Consultant Haematologist, Department of Transfusion Medicine,
Royal Inrmary of Edinburgh, Edinburgh, UK
Amanda Mather MBBS, FRACP, PhD
Renal Staff Specialist, Department of Renal Medicine, Royal North
Shore Hospital; Conjoint Senior Lecturer, Faculty of Medicine,
University of Sydney, Sydney, NSW, Australia
Simon RJ Maxwell MD, PhD, FRCP, FRCPE, FBPhS
Professor of Student Learning (Clinical Pharmacology and Prescribing),
Clinical Pharmacology Unit, University of Edinburgh, Edinburgh, UK
David A McAllister MBChB, MPH, MD, MRCP, MFPH
Wellcome Trust Intermediate Clinical Fellow and Beit Fellow and
Honorary Consultant in Public Health Medicine, Institute of Health and
Wellbeing, University of Glasgow, Glasgow, UK
Eve Miller-Hodges MBChB, PhD
Senior Clinical Lecturer and Honorary Consultant in Inherited Metabolic
Disorders and Renal Medicine, Centre for Cardiovascular Science &
Scottish IMD Service, University of Edinburgh, Edinburgh, UK
Francesca EM Neuberger MBChB, FRCP
Consultant Physician in Acute and Obstetric Medicine, Medical Division,
North Bristol NHS Trust, Bristol, UK
David E Newby BA BSc(Hons), PhD, BM DM DSc, FRSE, FESC,
FACC, FMedSci
British Heart Foundation Duke of Edinburgh Chair of Cardiology, Centre
for Cardiovascular Science, University of Edinburgh, Edinburgh, UK
John DC Newell-Price MA, PhD, FRCP
Professor of Endocrinology, Department of Oncology and Metabolism,
University of Shefeld, Shefeld, UK
John A Olson MD, FRCPE, FRCOphth
Consultant Ophthalmic Physician, Aberdeen Royal Inrmary; Honorary
Reader, University of Aberdeen, Aberdeen, UK
John R Petrie BSc, MBChB, PhD, FRCPE, FRCPG
Professor of Diabetic Medicine, Institute of Cardiovascular and Medical
Sciences, University of Glasgow, Glasgow, UK
Paul J Phelan MD, FRCPE
Consultant Nephrologist and Renal Transplant Physician,
Department of Nephrology, Royal Inrmary of Edinburgh, Edinburgh, UK
Terence J Quinn FRCP, MD, MBChB, BSc
Senior Clinical Lecturer and Honorary Consultant in Geriatric Medicine,
Institute of Cardiovascular and Medical Sciences, University of
Glasgow, Glasgow, UK
Stuart H Ralston MBChB, MD, FRCP, FFPM(Hon), FMedSci, FRSE
Professor of Rheumatology, Centre for Genomic and Experimental
Medicine, Institute of Genetics and Cancer, University of Edinburgh,
Western General Hospital, Edinburgh, UK
Anupam Rej MBChB, BMedSci(Hons), MRCP(UK)
Clinical Research Fellow, Academic Unit of Gastroenterology,
Royal Hallamshire Hospital, Shefeld, UK
David S Sanders MBChB, MD, FACG
Professor of Gastroenterology, Academic Unit of Gastroenterology,
Royal Hallamshire Hospital and University of Shefeld, Shefeld, UK
Jonathan AT Sandoe FRCPath, PhD
Associate Clinical Professor, Consultant Microbiologist,
Department of Microbiology, University of Leeds and
Leeds Teaching Hospitals NHS Trust, Leeds, UK
Alan G Shand MD, FRCPE
Consultant Gastroenterologist, Gastrointestinal Unit,
Western General Hospital, Edinburgh, UK
Mark Stares MBBS, MD(Res), MRes, BSc, MRCP
Medical Oncologist, Edinburgh Cancer Centre, Western General
Hospital, NHS Lothian, Edinburgh, UK
14.
CONTRIBUTORS xiii
RobbyM Steel MA, MD, FRCPsych
Consultant Liaison Psychiatrist and Honorary (Clinical) Senior Lecturer
in Psychiatry, Department of Psychological Medicine, Royal Inrmary of
Edinburgh, Edinburgh, UK
Grant D Stewart BSc, MBChB, PhD Edin, MA Cantab,
FRCSE (Urol)
Professor of Surgical Oncology, Department of Surgery,
University of Cambridge, Cambridge, and Honorary Consultant
Urological Surgeon, Addenbrooke’s Hospital, Cambridge, UK
David R Sullivan MBBS, FRACP, FRCPA, FCSANZ
Head of Department, Department of Chemical Pathology,
NSW Health Pathology, Sydney, NSW, Australia
Shyam Sundar MD, FRCP, FAMS, FASc, FNA
Distinguished Professor, Department of Medicine, Institute of Medical
Sciences, Banaras Hindu University, Varanasi, India
Victoria R Tallentire MBChB, MD, FRCP
Consultant Acute Physician and Associate Postgraduate Dean,
Medical Directorate, NHS Education for Scotland, Edinburgh, UK
Katrina Tatton-Brown BM BCh, BA, MD
Professor of Clinical Genetics and Genomic Education and Consultant
in Clinical Genetics, South West Thames Regional Genetics Service,
St George’s University Hospitals NHS Foundation Trust, London, UK
Simon HL Thomas BSc, MD, FRCP, FRCPE, FEAPCCT, FACCT
Professor of Clinical Pharmacology and Therapeutics,
Translational and Clinical Research Institute, Newcastle University,
Newcastle-upon-Tyne, UK
Henry G Watson MD, FRCP, FRCPath
Consultant Haematologist and Honorary Professor of Medicine,
Department of Haematology, Aberdeen Royal Inrmary, Aberdeen, UK
Julian White MD, FACTM
Clinical Toxinologist and Head of Toxinology, Toxinology Department
and University of Adelaide Department of Paediatrics,
Women’s & Children’s Hospital, North Adelaide, SA, Australia
William Whiteley BA, BM BCh, MSc, PhD, FRCP
Reader in Neurology, Centre for Clinical Brain Sciences,
University of Edinburgh, Edinburgh, UK;
Senior Research Fellow, Nufeld Department of Population Health,
University of Oxford, Oxford, UK
Michael J Williams BM BCh, PhD
Consultant Hepatologist, Scottish Liver Transplant Unit, Royal Inrmary
of Edinburgh, Edinburgh, UK
Rebecca Woodeld MA, MB BChir, MRCP, PhD
Consultant Geriatrician and Stroke Physician, Department of Medicine
for the Elderly, Western General Hospital, Edinburgh, UK
International Advisory Board
ABMAbdullah MRCP(UK), FRCPE
UGC Professor, Department of Medicine, Bangabandhu Sheikh Mujib
Medical University, Dhaka, Bangladesh
Amitesh Aggarwal MD, FRCPE, FACP(USA), FRCPG, FICP,
FIACM, FIMSA, FISE, FIAMS, FUPDA, FGSI, FISH, FRSSDI
Professor, Department of Medicine, University College of Medical
Sciences (University of Delhi) and GTB Hospital, Delhi, India
Matthew A Brown MBBS, MD, FRACP, FAHMS, FAA
Director, Guy’s and St Thomas’ NHS Foundation Trust and King’s
College London NIHR Biomedical Research Centre, King’s College
London, London, UK
Arnold N Cohen MD, FACP, FACG, AGAF
Professor of Medicine, Department of Medical Education and Science,
Elson S. Floyd College of Medicine of Washington State University;
Associate Clinical Professor of Medicine Emeritus, University of
Washington School of Medicine, Spokane, Washington, USA
Mradul Kumar Daga MD, FRCP, FCCP
Professor of Medicine and In-Charge Medical ICU, Department of
Internal Medicine, Maulana Azad Medical College, New Delhi, India
D Dalus MD, PhD, FRCP, FRCPE, FRCPG
Professor of Medicine and Senior Consultant, Department of Internal
Medicine, Cosmopolitan Hospitals, Trivandrum, Kerala, India
Sydney C Dsouza MD
Professor and Head, Department of Internal Medicine, Yenepoya
Medical College, Mangaluru, Karnataka, India
Tarun K Dutta MBBS, MD
Emeritus Professor, Department of General Medicine, Mahatma Gandhi
Medical College and Research Institute, Puducherry, India
M Abul Faiz MBBS, FCPS, PhD
Professor of Medicine (Retired), Department of Medicine, Sir Salimullah
Medical College and Dev Care Foundation, Dhaka, Bangladesh
Albert G Frauman MD, FRACP, FACP, FACCP, FBPhS
Professor of Clinical Pharmacology and Therapeutics, The University
of Melbourne; Director, Department of Clinical Pharmacology and
Therapeutics, Austin Health, Victoria, Australia
Sujoy Ghosh MD, DM, FRCP, FRCPE, FRCPG, FACE
Professor, Department of Endocrinology, Institute of Post Graduate
Medical Education and Research, Kolkata, West Bengal, India
Hadi Goubran MBBCh, MSc, MD, FACP, FRCPE
Professor, Division of Hematology and Oncology, Saskatoon Cancer
Centre and College of Medicine, University of Saskatchewan,
Saskatoon, Saskatchewan, Canada
Rajiva Gupta MD, MRCP, FRCP
Vice Chairman and Head, Department of Rheumatology and Clinical
Immunology, Medanta The Medicity, Gurgaon, India
Quazi T Islam FCPS, FRCP, FRCPE, MACP
Professor of Medicine, Department of Medicine, Popular Medical
College, Dhaka, Bangladesh
Saroj Jayasinghe MBBS, MD, MRCP(UK), PhD, FRCP, FCCP,
FNASSL
Consultant Physician and former Chair Professor of Medicine,
Department of Clinical Medicine, University of Colombo, Colombo,
Sri Lanka
AL Kakrani MD, FICP
Professor of Clinical Eminence and Director of Academic
Collaborations, Department of Medicine, Dr. D.Y. Patil Medical College,
Hospital and Research Centre, Pune, Maharashtra, India
Vasantha Kamath MD, FICP
Senior Professor in Internal Medicine, Department of Internal Medicine,
MVJ Medical College and Research Hospital, Hoskote, Bangalore,
Karnataka, India
Ammar F Mubaidin MD, FRCPE, FRCPG
Professor of Clinical Neurology, Department of Neurology, Al Khalidi
Medical Centre, Amman, Jordan
Milind Y Nadkar MD, FICP, FACP
Additional Dean (Academic) and Professor and Head, Department of
Medicine and Rheumatology Services, Seth G S Medical College and
KEM Hospital, Mumbai, Maharashtra, India
Viswanathan Neelakantan MBBS, DNB, FRCPG, FRCP,
FACP, AB
Senior Professor of Medicine and Consultant in Tropical Medicine,
Department of Internal Medicine, Sri Manakula Vinayagar Medical
College Hospital, Puducherry, India
Matthew Ng FRCP, FRCPE
Honorary Clinical Professor, Department of Medicine, University of Hong
Kong, Pok Fu Lam, Hong Kong
17.
xvi INTERNATIONALADVISORY BOARD
Ami Prakashvir Parikh MD
Professor and Head, Department of Medicine, Smt. NHL Municipal
Medical College and SVP Hospital, Ahmedabad, Gujarat, India
Medha Y Rao MBBS, MD, PGDHHM
Senior Professor of Internal Medicine and Principal and Dean,
Department of General Medicine, M.S. Ramaiah Medical College,
Bangalore, Karnataka, India
NR Rau MD, FICP
Consultant Physician, Anugraha Medical Center, Udupi, Karnataka, India;
Consultant Physician and Head, Department of Internal Medicine,
Adarsh Hospital, Udupi, Karnataka, India; Former Professor and
Head, Department of Medicine, Kasturba Medical College,
Manipal, India
Jacek Rozanski MD, PhD
Professor of Medicine, Department of Nephrology, Transplantology, and
Internal Medicine, Pomeranian Medical University, Szczecin, Poland
Sarkar Nirmalendu MD
Honorary Professor, Department of General Medicine, Ramakrishna
Mission Sevapratishthan and Vivekananda Institute of Medical
Sciences, Kolkata, West Bengal, India
Surendra K Sharma MD (Internal Medicine), PhD
Adjunct Professor, Department of Molecular Medicine, Jamia Hamdard
Institute of Molecular Medicine, Hamdard University, Hamdard Nagar,
Delhi, India
Arvind K Vaish MD, FICP
Professor and Head, Department of Medicine, Hind Institute of Medical
Sciences; Former Professor and Head of Medicine, King George’s
Medical University, Lucknow, India
Josanne Vassallo MD, PhD, FRCP, FACP, FACE
Professor and Consultant Endocrinologist, Department of Medicine,
Division of Endocrinology, University of Malta Medical School, Msida,
Malta
Special Content Advisor for the Indian
National Medical Commission undergraduate
curriculum
Professor Dilip R Karnad MD, FRCPG, FACP
Senior Consultant in Critical Care, Jupiter Hospital, Thane, India
18.
Acknowledgements
The editors wouldlike to acknowledge and offer grateful thanks for the
input of all previous editions’ contributors, without whom this new edition
would not have been possible.
In particular we are indebted to those former authors who step down
with the arrival of this new edition. They include Quentin M Anstee,
Leslie Burnett, Mark Byers, Graham G Dark, Richard J Davenport, Emad
El-Omar, David R FitzPatrick, J Alastair Innes, David EJ Jones, Peter
Langhorne, John Paul Leach, Sara E Marshall, Rory J McCrimmon,
Mairi H McLean, Ewan R Pearson, Peter T Reid, Gordon R Scott, Peter
Stewart, John PH Wilding and Miles D Witham.
We are grateful to members of the International Advisory Board, all
of whom provided detailed suggestions that have improved the book.
Several members have now retired from the Board and we are grate-
ful for their support during the preparation of previous editions. They
include Ragavendra Bhat, Khalid I Bzeizi, Piotr Kuna, Pravin Manga,
Moffat Nyirenda, Tommy Olsson, KR Sethuraman, Ibrahim Sherif, Ian J
Simpson, SG Siva Chidambaram and Josanne Vassallo. We are equally
grateful to new members of our International Advisory Board, who have
given us valuable advice as we prepared this new edition, including ABM
Abdullah, Quazi T Islam, Viswanathan Neelakantan and Jacek Rozanski.
We would like to extend special thanks to Professor Dilip Karnad,
Jupiter Hospital, Thane, India, who thoroughly reviewed all chapters of
this 24th edition in draft form, to help ensure the coverage of this edition
is more relevant than ever to our large readership in India, Pakistan and
Bangladesh. He provided invaluable advice to the editorial team dur-
ing preparation of the 24th edition and exhaustive feedback on how the
content aligns with competencies in the current Indian National Medical
Commission undergraduate curriculum. Readers of the International
Edition of this book can now access a comprehensive Competency
Mapping Guide with full page references for the rst time as a result of
this thorough review.
Detailed chapter reviews were commissioned to help plan this new
edition and we are grateful to all those who assisted, including Professor
Rustam Al-Shahi, Dr Daniel Beckett, Dr Helen Cohen, Dr Ian Edmond,
Dr David Enoch, Professor Tonks Fawcett, Dr Colin Forfar, Professor
Richard Gilson, Dr Helena Gleeson, Dr Peter Hall, Dr Greg Heath,
Dr Richard Herriot, Dr Robert Lindsay, Dr Catherine Nelson-Piercy and
Dr Alex Rowe.
The Editors and Publisher would like to thank all those who have pro-
vided valuable feedback on this textbook and whose comments have
helped shape this new edition. We would particularly like to extend
our thanks to the many readers who contact us with suggestions for
improvements. This input has been invaluable and is much appreciated;
we regret the names are too numerous to mention individually.
The authors of Chapter 20 would like to thank Dr Drew Henderson,
who reviewed the ‘Diabetic nephropathy’ section, and we are indebted
to Dr Ruth Darbyshire for assistance with the Ophthalmology multiple
choice questions to accompany Chapter 30.
Two short sections in Chapter 3 on array comparative genomic
hybridisation and single-molecule sequencing are adapted from Dr K
Tatton-Brown’s Massive Open Online Course for FutureLearn. We would
like to thank the Open University and St George’s, University of London,
for permission to use this material.
We are especially grateful to Laurence Hunter and Wendy Lee from
Elsevier for their endless support and expertise in the shaping, collation
and publication of Davidson’s over many years and who have now retired.
We have thoroughly enjoyed working with a new team including Jeremy
Bowes, Siân Jarman and Anne Collett who have seamlessly taken over
the reins. We are delighted that Robert Britton continues to work on the
book and illustrate it beautifully. We are proud of this new edition and are
condent it will remain an essential and invaluable resource for readers
the world over.
Introduction
The opening chaptersof the book, making up Part 1 on ‘Fundamentals
of Medicine’, provide an account of the principles of genetics, immunol-
ogy, infectious diseases and population health, oncology and pain man-
agement, along with a discussion of the core principles behind clinical
decision-making and good prescribing. Subsequent chapters in Part 2,
‘Emergency and Critical Care Medicine’, discuss medical emergencies
in poisoning, envenomation and medicine in austere environments, while
Chapter 9 explores common presentations in acute medicine, as well
as the recognition and management of the critically ill. The third part,
‘Clinical Medicine’, is devoted to the major medical specialties. Each
chapter has been written by experts in the eld to provide the level of
detail expected of trainees in their discipline. To maintain the book’s virtue
of being concise, care has been taken to avoid unnecessary duplication
between chapters.
The system-based chapters in Part 3 follow a standard format, begin-
ning with an overview of the relevant aspects of clinical examination,
followed by an account of functional anatomy, physiology and investi-
gations, then the common presentations of disease, and details of the
individual diseases and treatments relevant to that system. In chapters
that describe the immunological, cellular and molecular basis of disease,
this problem-based approach brings the close links between modern
medical science and clinical practice into sharp focus.
The methods used to present information are described below.
Clinical examination overviews
The value of good clinical skills is highlighted by a two-page overview
of the important elements of the clinical examination at the beginning
of most chapters. The left-hand page includes a mannikin to illustrate
key steps in examination of the relevant system, beginning with simple
observations and progressing in a logical sequence around the body.
The right-hand page expands on selected themes and includes tips on
examination technique and interpretation of physical signs. These over-
views are intended to act as an aide-mémoire and not as a replacement
for a detailed text on clinical examination, as provided in our sister title,
Macleod’s Clinical Examination
Presenting problems
Medical students and junior doctors must not only assimilate a great many
facts about various disorders, but also develop an analytical approach to
formulating a differential diagnosis and a plan of investigation for patients
who present with particular symptoms or signs. In Davidson’s this is
addressed by incorporating a ‘Presenting Problems’ section into all rel-
evant chapters. Nearly 250 presentations are included, which represent
the most common reasons for referral to each medical specialty.
Boxes
Boxes are a popular way of presenting information and are particularly
useful for revision. They are classied by the type of information they
contain, using specic symbols.
General Information
These include causes, clinical features, investigations, treatments and
other useful information.
Practice Point
There are many practical skills that students and doctors must master.
These vary from inserting a nasogastric tube to reading an ECG or X-ray,
or interpreting investigations such as arterial blood gases or thyroid func-
tion tests. ‘Practice Point’ boxes provide straightforward guidance on
how these and many other skills can be acquired and applied.
Emergency
These boxes describe the management of many of the most common
emergencies in medicine.
In Old Age
Life expectancy is increasing in many countries and older people are
the chief users of health care. While they contract the same diseases
as those who are younger, there are often important differences in the
way they present and how they are best managed. Chapter 34, ‘Ageing
and disease’, concentrates on the principles of managing the frailest
group, who suffer from multiple comorbidity and disability, and who tend
to present with non-specic problems such as falls or delirium. Many
older people, though, also suffer from specic single-organ pathology. ‘In
Old Age’ boxes are thus included in each chapter and describe common
presentations, implications of physiological changes of ageing, effects of
age on investigations, problems of treatment in old age, and the benets
and risks of intervention in older people.
In Pregnancy
Many conditions are different in the context of pregnancy, while some
arise only during or shortly after pregnancy. Particular care must be taken
with investigations (for example, to avoid radiation exposure to the fetus)
and treatment (to avoid the use of drugs that harm the fetus). These
issues are highlighted by ‘In Pregnancy’ boxes distributed throughout the
book, which complement Chapter 32, ‘Maternal medicine’.
21.
xx INTRODUCTION
InAdolescence
Although paediatric medicine is not covered in Davidson’s, many chronic
disorders begin in childhood, and physicians who look after adults
often contribute to multidisciplinary teams that manage young patients
‘in transition’ between paediatric and adult health-care services. This group
of patients often presents a particular challenge, due to the physiological
and psychological changes that occur in adolescence, and which can
have a major impact on the disease and its management. Adolescents
can be encouraged to take over responsibility from their parents/carers in
managing their disease, but are naturally rebellious and often struggle to
adhere to the impositions of chronic treatment. Chapter 33, ‘Adolescent
and transition medicine’, highlights these issues, alongside the ‘In
Adolescence’ boxes that appear in relevant chapters.
Terminology
The recommended International Non-proprietary Names (INNs) are used
for all drugs, with the exception of adrenaline and noradrenaline. British
spellings have been retained for drug classes and groups (e.g. amphet-
amines not amfetamines).
Units of measurement
The International System of Units (SI units) is the recommended means
of presentation for laboratory data and has been used throughout
Davidson’s. We recognise, however, that many laboratories around
the world continue to provide data in non-SI units, so these have been
included in the text for the commonly measured analytes. Both SI and
non-SI units are also given in Chapter 35, which describes the reference
ranges used in laboratories in Edinburgh. It is important to appreciate that
these reference ranges may vary from those used in other laboratories.
Finding what you are looking for
A contents list is given on the opening page of each chapter. In addi-
tion, the book contains cross-references to help readers nd their way
around, along with an extensive index. A list of up-to-date reviews and
useful websites with links to management guidelines appears at the end
of each chapter.
Giving us your feedback
The Editors and Publisher hope that you will nd this edition of
Davidson’s informative and easy to use. We would be delighted to
hear from you if you have any comments or suggestions to make
for future editions of the book. Please contact us by e-mail at:
[email protected]. All comments received will be
much appreciated and will be considered by the editorial team.
22.
Clinical decision-making
1
N Cooper
ALCracknell
Introduction 2
The problem of diagnostic error 2
Clinical reasoning: denitions 2
History and physical examination 2
Use and interpretation of diagnostic tests 3
Normal values 3
Factors other than disease that inuence test results 4
Operating characteristics 4
Sensitivity and specicity 4
Prevalence of disease 5
Dealing with uncertainty 5
Problem representation 5
Cognitive biases 6
Type 1 and type 2 thinking 6
Common cognitive biases in medicine 7
Thinking about thinking 7
Human factors 7
Shared decision-making 9
Patient-centred evidence-based medicine 9
Effective team communication 9
Using clinical prediction rules and other decision aids 9
Reducing errors in clinical decision-making 9
Deliberate practice 9
Cognitive debiasing strategies 9
Clinical decision-making: putting it all together 10
Answers to problems 11
23.
2 CLINICALDECISION-MAKING
Introduction
A great deal of knowledge and skill is required to practise as a doctor.
Physicians in the 21st century need to have a comprehensive knowl-
edge of basic and clinical sciences, have good communication skills, be
able to perform procedures, work effectively in a team and demonstrate
professional and ethical behaviour. But how doctors think, reason and
make decisions is arguably their most critical skill. Knowledge is neces-
sary, but not sufcient on its own for good performance and safe care.
This chapter describes the principles of clinical decision-making, also
known as ‘clinical reasoning’.
The problem of diagnostic error
It is estimated that diagnosis is wrong 10%–15% of the time in spe-
cialties such as emergency medicine, internal medicine and general
practice. Diagnostic error is associated with greater morbidity than other
types of medical error, and the majority is considered to be preventable.
For every diagnostic error there are usually several root causes. Studies
identify three main categories, shown in Box 1.1. However, ‘human cog-
nitive error’ appears to play a signicant role in the majority of diagnostic
errors.
Human cognitive error occurs when the clinician has all the information
necessary to make the diagnosis, but then makes the wrong diagnosis.
Why does this happen? Three main reasons have been identied:
knowledge gaps
misinterpretation of diagnostic tests
cognitive biases.
Examples of errors in these three categories are shown in Box 1.2.
Clearly, clinical knowledge is required for sound clinical reasoning, and
an incomplete knowledge base or inadequate experience can lead to
diagnostic error. However, this chapter will focus on some other aspects
of knowledge that are important for effective clinical reasoning, including
use and interpretation of diagnostic tests, cognitive biases and human
factors.
Clinical reasoning: denitions
‘Clinical reasoning’ describes the thinking and decision-making processes
associated with clinical practice. Our understanding of clinical reason-
ing derives from the elds of education, cognitive psychology, studies of
expertise and the diagnostic error and health systems literature.
Clinical reasoning can be conceptualised as a process with different
components, each requiring specic knowledge, skills and behaviours.
The UK Clinical Reasoning in Medical Education group (see ‘Further
information’) broadly lists these components as:
history and physical examination
use and interpretation of diagnostic tests
problem identication and management
shared decision-making.
Not all of these components are necessary for effective clinical rea-
soning and they do not necessarily happen in this order. They also occur
in contexts that impact on decision-making, which will be explored later.
Underpinning all of this is formal and experiential knowledge of basic
sciences and clinical medicine. The knowledge required for effective clin-
ical reasoning includes factual knowledge, but also conceptual knowl-
edge (how things t together) as well as procedural knowledge (how
to do something, what techniques to use) plus an awareness of and
an ability to think about one’s own thinking (also known as metacogni-
tive knowledge). This is where an understanding of cognitive biases and
human factors is important.
Fig. 1.1 shows the key components involved in clinical reasoning that
will be explored further in this chapter.
History and physical examination
Even with major advances in medical technology, the history remains
the most important part of the clinical decision-making process. Studies
show that physicians make a diagnosis in 70%–90% of cases from the
history alone. It is important to remember that the history is explored
not only with the patient, but also (and with consent if required) from all
available sources if necessary: for example, paramedic and emergency
department notes, eye-witnesses, relatives and/or carers.
However, clinicians need to be aware of the diagnostic usefulness of
clinical features in the history and physical examination. For example,
students are often taught that meningitis classically presents with the
following features:
headache
fever
meningism (photophobia, neck stiffness and other signs of menin-
geal irritation, such as Kernig’s and Brudzinski’s signs).
However, knowing the frequency with which patients present with
certain features and the diagnostic weight of each feature are impor-
tant in clinical decision-making. Many patients with meningitis do not
1.1 Root causes of diagnostic error in studies
Error category Examples
No fault Unusual presentation of a disease
Missing information
System error Inadequate diagnostic support
Results not available
Error-prone processes
Poor supervision of inexperienced staff
Poor team communication
Human cognitive error Inadequate data-gathering
Errors in reasoning
Adapted from Graber M, Gordon R, Franklin N. Reducing diagnostic errors in medicine: what is
the goal? Acad Med 2002; 77:981–992.
1.2 Reasons for errors in clinical reasoning
Source of error Examples
Knowledge gaps Telling a patient she cannot have
biliary colic because she has had her
gallbladder removed – gallstones can
form in the bile ducts in patients who
have had a cholecystectomy
Misinterpretation of diagnostic
tests
Deciding a patient has not had a
stroke because his brain scan is
normal – computed tomography and
even magnetic resonance imaging,
especially when performed early, may
not identify an infarct
Cognitive biases Accepting a diagnosis handed over
to you without question (the ‘framing
effect’) instead of asking yourself
‘What is the evidence that supports
this diagnosis?’
24.
Use and interpretationof diagnostic tests 3
1
have classical signs of meningism and the clinical presentation varies
among different patient populations and in different parts of the world.
In one prospective study conducted in the United States, nearly all adult
patients with meningitis had headache and a fever, but less than half
had neck stiffness on examination and only 5% of patients had Kernig’s
and Brudzinski’s signs. All three signs had a likelihood ratio of around 1,
meaning their presence or absence was of little use in deciding whether
a patient had meningitis or not (Fig. 1.2).
Likelihood ratios (LR) are clinical diagnostic weights. An LR of greater
than 1 increases the probability of disease (the higher the value, the
greater the probability). Similarly, an LR of less than 1 decreases the
probability of disease. LRs are developed against a diagnostic standard
(in the case of meningitis, lumbar puncture results), so do not exist for all
clinical ndings. LRs illustrate how an individual clinical nding changes
the probability of a disease. For example, in a person presenting with
headache and fever, the clinical nding of neck stiffness may carry little
weight in deciding whether to perform a lumbar puncture because LRs
do not determine the prior probability of disease; they reect only how a
single clinical nding changes it. Clinicians have to take all the available
information from the history and physical examination into account. If
the overall clinical probability is high to begin with, a clinical nding with
an LR of around 1 does not change this.
‘Evidence-based history and examination’ is a term used to describe
how clinicians incorporate knowledge about the prevalence and diag-
nostic weight of clinical ndings into their decision-making. In studies,
students who are taught the probabilities of features being present in
specic diseases rather than lists of features have better diagnostic
accuracy. This is improved further by understanding the basic science
explanation for symptoms and signs: bedside signs of meningism iden-
tify patients with severe meningeal inammation but do not pick up those
with early or mild inammation.
Evidence-based history and examination is important because esti-
mating the clinical (pre-test) probability is vital not only for diagnostic
accuracy, but also in the use and interpretation of diagnostic tests.
Use and interpretation of diagnostic tests
There is no such thing as a perfect diagnostic test. Test results give us
test probabilities, not real probabilities. Test results have to be interpreted
because they are affected by the following:
how ‘normal’ is dened
factors other than disease
operating characteristics
sensitivity and specicity
prevalence of disease in the population.
Normal values
Most tests provide quantitative results (i.e. a value on a continuous
numerical scale). In order to classify quantitative results as normal or
abnormal, it is necessary to dene a cut-off point. Many quantitative
measurements in populations have a Gaussian or ‘normal’ distribution.
By convention, the normal range is dened as those values that encom-
pass 95% of the population, or 2 standard deviations above and below
the mean. This means that 2.5% of the normal population will have
values above, and 2.5% will have values below the normal range. For this
reason, it is more appropriate to talk about the ‘reference range’ rather
than the ‘normal range’ (Fig. 1.3).
Test results in abnormal populations also have a Gaussian distribution,
with a different mean and standard deviation. In some diseases there is
no overlap between results from the abnormal and normal population.
However, in many diseases there is overlap; in these circumstances, the
greater the difference between the test result and the limits of the refer-
ence range, the higher the chance that the person has the disease.
However, there are also situations in medicine when ‘normal’ is abnor-
mal and ‘abnormal’ is normal. For example, in the context of a severe
asthma attack a ‘normal’ PaCO2
is abnormal and means the patient has
life-threatening asthma. Conversely, a low ferritin in a young menstruating
woman is not considered to be abnormal at all.
Laboratory results (e.g. cholesterol, thyroid-stimulating hormone) also
vary from day to day in the same person in the absence of a real change
because of biological variation and laboratory variation. The extent to
which a blood test is allowed to vary before it has truly changed is called
Clinical
reasoning
(knowledge, skills
and behaviours)
History and
physical
examination
Cognitive biases
and human
factors
Thinking about
thinking
Shared
decision-making
Use and
interpretation of
diagnostic
tests
Problem
representation
Fig. 1.1 Elements of clinical reasoning.
Change in
probability
of disease
10
Infinity
Zero
5
2
1
0.5
0.2
0.1
+ 45%
+ 30%
+ 15%
Kernig’s sign
Brudzinski’s sign
Nuchal rigidity
Increase
probability
LR
Decrease
probability
– 15%
– 30%
– 45%
Fig. 1.2 Likelihood ratio (LR) of Kernig’s sign, Brudzinski’s sign and nuchal
rigidity in the clinical diagnosis of meningitis.
LRs are also used for diagnostic tests; here a physical examination nding can be
considered a diagnostic test. Data from Thomas KE, Hasbun R, Jekel J, et al. The
diagnostic accuracy of Kernig’s sign, Brudzinski’s sign, and nuchal rigidity in adults
with suspected meningitis. Clin Infect Dis 2002; 35:46–52.
LR =
probability of finding in patients with disease
probab
bility of finding in patients without disease
25.
4 CLINICALDECISION-MAKING
the ‘critical difference’. The critical difference is different for each test,
and can be high – 17% in the case of cholesterol and higher for some
other tests.
Factors other than disease that inuence test
results
A number of factors other than disease inuence test results:
age
ethnicity
pregnancy
sex
spurious (in vitro) results.
Box 1.3 gives some examples.
Operating characteristics
Tests are also subject to operating characteristics. This refers to the
way the test is performed. Patients need to be able to comply fully with
some tests, such as spirometry (p. 501), and if they cannot, the test
result will be affected. Some tests are very dependent on the skill of
the operator and are also affected by the patient's body habitus and
clinical state; ultrasound of the heart and abdomen are examples. A
common mistake is when doctors refer to a test result as ‘no abnor-
mality detected’ when, in fact, the report describes a technically dif-
cult and incomplete scan that should more accurately be described as
‘non-diagnostic’.
Some conditions are paroxysmal. For example, around half of patients
with epilepsy have a normal standard electroencephalogram (EEG). A
normal EEG therefore does not exclude epilepsy. On the other hand,
around 10% of patients who do not have epilepsy have epileptiform
discharges on their EEG. This is referred to as an ‘incidental nding’.
Incidental ndings are common in medicine, and are increasing in inci-
dence with the greater availability of more sensitive tests. Test results
should always be interpreted in the light of the patient’s history and phys-
ical examination.
Sensitivity and specicity
Diagnostic tests have characteristics termed ‘sensitivity’ and ‘specicity’.
Sensitivity is the ability to detect true positives; specicity is the ability to
detect true negatives. Even a very good test, with 95% sensitivity, will
miss 1 in 20 people with the disease. Every test therefore generates
‘false positives’ and ‘false negatives’ (Box 1.4).
A very sensitive test will detect most disease but may generate abnor-
mal ndings in healthy people. A negative result will therefore reliably
exclude disease but a positive result does not mean the disease is pres-
ent – it means further evaluation is required. On the other hand, a very
specic test may miss signicant pathology but is likely to establish the
diagnosis beyond doubt when the result is positive. All tests differ in their
sensitivity and specicity, and clinicians require a working knowledge of
the tests they use in order to accurately interpret them.
In choosing how a test is used to guide decision-making there is a
trade-off between sensitivity versus specicity. For example, dening
an exercise electrocardiogram (p. 393) as abnormal if there is at least
0.5mm of ST depression would ensure that very few cases of coronary
artery disease are missed but would generate many false-positive results
(high sensitivity, low specicity). On the other hand, a cut-off point of
2.0mm of ST depression would detect most cases of important coronary
artery disease with far fewer false positives. This trade-off is calculated
using the receiver operating characteristic curve of the test (Fig. 1.4).
An extremely important concept in clinical decision-making is this: the
probability that a person has a disease depends on the pre-test proba-
bility, and the sensitivity and specicity of the test. For example, imagine
an older woman has fallen and hurt her left hip. On examination, the hip is
extremely painful to move and she cannot stand. However, her hip X-rays
are normal. Does she have a fracture?
The sensitivity of plain X-rays of the hip performed in the emergency
department for suspected hip fracture is around 95%. A small percent-
age of fractures are therefore missed. If our patient has (or is at risk of)
osteoporosis, has severe pain on hip movement and cannot bear weight
on the affected side, then the clinical probability of hip fracture is high. If,
on the other hand, she is unlikely to have osteoporosis, has no pain on
Normal
population
Number
of
people
having
each
value
Abnormal
populations
Mean
– 2SD
Mean
+ 2SD
Mean
‘Reference range’
Value
Fig. 1.3 Normal distribution and reference range. For many tests, the frequency
distribution of results in the normal healthy population (red line) is a symmetrical
bell-shaped curve. The mean ±2 standard deviations (SD) encompasses 95% of the
normal population and usually denes the ‘reference range’; 2.5% of the normal
population have values above, and 2.5% below, this range (shaded areas). For some
diseases (blue line), test results overlap with the normal population or even with the
reference range. For other diseases (green line), tests may be more reliable because
there is no overlap between the normal and abnormal population.
1.3 Examples of factors other than disease that inuence test
results
Factor Examples
Age Creatinine is lower in old age (due to
relatively lower muscle mass) – an older
person can have a signicantly reduced
eGFR with a ‘normal’ creatinine
Ethnicity Healthy people of African ancestry have
lower white cell counts
Pregnancy Several tests are affected by late
pregnancy, due to the effects of a growing
fetus, including:
Reduced urea and creatinine
(haemodilution)
Iron deciency anaemia (increased
demand)
Increased alkaline phosphatase (produced
by the placenta)
Raised D-dimer (physiological changes in
the coagulation system)
Mild respiratory alkalosis (physiological
maternal hyperventilation)
ECG changes (tachycardia, left axis
deviation)
Sex Males and females have different reference
ranges for many tests, e.g. haemoglobin
Spurious (in vitro) results A spurious high potassium is seen
in haemolysis and in thrombocytosis
(‘pseudohyperkalaemia’)
(ECG = electrocardiogram; eGFR = estimated glomerular ltration rate, a better estimate of renal
function than creatinine)
26.
Problem representation 5
1
hip movement and is able to bear weight, then the clinical probability of
hip fracture is low.
Doctors are continually making judgements about whether something is
true, given that something else is true. This is known as ‘conditional prob-
ability’. Bayes’ Theorem (named after English clergyman Thomas Bayes,
1702–1761) is a mathematical way to describe the post-test probability
of a disease by combining pre-test probability, sensitivity and specicity.
In clinical practice, doctors are not able to make complex mathematical
calculations for every decision they make. In practical terms, the answer
to the question of whether there is a fracture is that in a high-probability
patient a normal test result does not exclude it, but in a low-probability
patient it (virtually) does. This principle is illustrated in Fig. 1.5
Sox and colleagues (see ‘Further information’) state a fundamental
assertion, which they describe as a profound and subtle principle of clin-
ical medicine: the interpretation of new information depends on what you
believed beforehand. In other words, the interpretation of a test result
depends on the probability of disease before the test.
Prevalence of disease
Consider this problem that was posed to a group of Harvard doctors.
The problem originates from a 1978 article in the New England Journal
of Medicine (Casscells et al, see ‘Further information’): if a test to detect
a disease whose prevalence is 1:1000 has a false-positive rate of 5%,
what is the chance that a person found to have a positive result actually
has the disease, assuming you know nothing about the person’s symp-
toms and signs? Assume the test generates no false negatives and take
a moment to work this out. In this problem, we have removed clinical
probability and are only considering prevalence. The answer is at the end
of the chapter (p. 11).
Predictive values combine sensitivity, specicity and prevalence.
Sensitivity and specicity are characteristics of the test; the population
does not change this. However, as doctors, we are interested in the
question, ‘What is the probability that a person with a positive test actu-
ally has the disease?’ This is illustrated in Box 1.5
Post-test probability and predictive values are different. Post-test
probability is the probability of a disease after taking into account new
information from a test result. Bayes’ Theorem can be used to calculate
post-test probability for a patient in any population. The pre-test prob-
ability of disease is decided by the doctor; it is a judgement based on
information gathered prior to ordering the test. Predictive value is the
proportion of patients with a test result who have the disease (or no
disease) and is calculated from a table of results in a specic population
(see Box 1.5). It is not possible to transfer this value to a different popula-
tion. This is important to realise because published information about the
performance of diagnostic tests may not apply to different populations.
In deciding the pre-test probability of disease, clinicians often neglect
to take prevalence into account and this distorts their estimate of proba-
bility. To estimate the probability of disease in a patient more accurately,
clinicians should anchor on the prevalence of disease in the subgroup to
which the patient belongs and then adjust to take the individual factors
into account.
Dealing with uncertainty
Clinical ndings are imperfect and diagnostic tests are imperfect. It is
important to recognise that clinicians frequently deal with uncertainty. By
expressing uncertainty as probability, new information from diagnostic
tests can be incorporated more accurately. However, subjective estimates
of probability can sometimes be unreliable. As the section on cognitive
biases will demonstrate (see below), intuition can be a source of error.
Knowing the patient’s true state is often unnecessary in clinical deci-
sion-making. Sox and colleagues (see ‘Further information’) argue that
there is a difference between knowing that a disease is present and act-
ing as if it were present. The requirement for diagnostic certainty depends
on the penalty for being wrong. Different situations require different levels
of certainty before starting treatment. How we communicate uncertainty
to patients will be discussed later in this chapter (see Fig. 1.9).
The ‘treatment threshold’ combines factors such as the risks of the
test, and the risks versus benets of treatment. The point at which the
factors are all evenly weighed is the threshold. If a test or treatment for
a disease is effective and low risk (e.g. giving antibiotics for a suspected
urinary tract infection), then there is a lower threshold for going ahead.
On the other hand, if a test or treatment is less effective or high risk
(e.g. starting chemotherapy for a malignant brain tumour), then greater
condence is required in the clinical diagnosis and potential benets of
treatment rst. In principle, if a diagnostic test will not change the man-
agement of the patient, then careful consideration should be given to
whether it is necessary to do the test at all.
In summary, test results shift our thinking, but rarely give a ‘yes’ or a
‘no’ answer in terms of a diagnosis. Sometimes tests shift the probability
of disease by less than we realise. Pre-test probability is key, and this
is derived from the history and physical examination, combined with a
sound knowledge of medicine and an understanding of the prevalence
of disease in the particular care setting or the population to which the
patient belongs.
Problem representation
Many students are taught to formulate a differential diagnosis after the
history, physical examination and initial test results, but the ability to
accurately articulate a ‘problem representation’ (or problem list) rst and
then construct a prioritised differential diagnosis based on this, including
relevant ‘must-not-miss’ diagnoses, is a key step in the clinical reasoning
process. A problem representation refers to how information about a
problem is mentally organised. Studies show that expert clinicians spend
far more time on dening a problem before trying to solve it compared
with novices, and novices are more likely to be unsuccessful in solving
1.4 Sensitivity and specicity
Disease No disease
Positive test A (True positive) B (False positive)
Negative test C (False negative) D (True negative)
Sensitivity = A/(A+C) × 100
Specicity = D/(D+B) × 100
1.0
0.8
0.6
0.4
0.2
0.0
1.0 0.8 0.6
Specificity
0.4 0.2 0
Sensitivity
Fig. 1.4 Receiver operating characteristic graph illustrating the trade-off
between sensitivity and specicity for a given test. The curve is generated by
‘adjusting’ the cut-off values dening normal and abnormal results, calculating the
effect on sensitivity and specicity and then plotting these against each other. The
closer the curve lies to the top left-hand corner, the more useful the test. The red line
illustrates a test with useful discriminant value and the green line illustrates a less
useful, poorly discriminant test.
27.
6 CLINICALDECISION-MAKING
a problem because they have not accurately represented the problem
in the rst place. The ability to synthesise all the available information
and encapsulate it into a problem representation using precise medical
language is an important skill that helps to organise and retrieve knowl-
edge from long-term memory relevant to the case and is associated with
signicantly higher diagnostic accuracy, particularly in complex cases
(Bordage, 1994; see ‘Further information’).
Formulating a problem representation (e.g. a 30-year-old pregnant
woman with acute left-sided pleuritic chest pain and breathlessness) or a
problem list (e.g. persistent vomiting, hypokalaemia, acute kidney injury),
not only helps to organise and retrieve knowledge relevant to the case
which helps in diagnosis, it also helps to formulate an action plan while
further information is being gathered when a diagnosis is not yet possible.
Cognitive biases
Advances in cognitive psychology in recent decades have demonstrated
that human thinking and decision-making are prone to error. Cognitive
biases are subconscious errors that lead to inaccurate judgement and
illogical interpretation of information. They are prevalent in everyday life;
as the famous saying goes, ‘to err is human’.
Take a few moments to look at this simple puzzle. Do not try to solve
it mathematically but listen to your intuition:
A bat and ball cost £1.10.
The bat costs £1 more than the ball.
How much does the ball cost?
The answer is at the end of the chapter. Most people get the answer
to this puzzle wrong – even though they have all the knowledge and
experience they need to solve this problem. Why? British psychologist
and patient safety pioneer James Reason said that, ‘Our propensity
for certain types of error is the price we pay for the brain’s remarkable
ability to think and act intuitively – to sift quickly through the sensory
information that constantly bombards us without wasting time trying to
work through every situation anew.’ This property of human thinking is
highly relevant to clinical decision-making.
Type 1 and type 2 thinking
Decades of studies in cognitive psychology and, more recently, func-
tional magnetic resonance imaging demonstrate two distinct types of
processes when it comes to decision-making: humans have a fast, intu-
itive, pattern-recognising way of thinking which uses little cognitive effort
(known as type 1 thinking) and a more deliberate, analytical one which
engages our working memory (known as type 2 thinking). This is known
as ‘dual process theory’ and Box 1.6 explains this in more detail.
Psychologists estimate that we spend 95% of our daily lives engaged in
type 1 thinking – the intuitive, fast, subconscious mode of decision-mak-
ing. Imagine driving a car, for example: it would be impossible to function
efciently if every decision and movement were as deliberate, conscious,
slow and effortful as in our rst driving lesson. With experience, complex
procedures become automatic, fast and effortless. The same applies to
medical practice. There is evidence that expert decision-making is well
served by intuitive thinking. The problem is that although intuitive process-
ing is highly efcient in many circumstances, in others it is prone to error.
Clinicians use both type 1 and type 2 thinking, and both types are
important in clinical decision-making. When encountering a problem
that is familiar, clinicians employ pattern recognition and reach a work-
ing diagnosis or differential diagnosis quickly (type 1 thinking). When
encountering a problem that is more complicated, they use a slower,
systematic approach (type 2 thinking). Both types of thinking interplay –
they are not mutually exclusive in the diagnostic process. Fig. 1.6 illus-
trates the interplay between type 1 and type 2 thinking in clinical practice.
Errors can occur in both type 1 and type 2 thinking. For example,
people can apply the wrong rules or make errors in their application while
using type 2 thinking. However, it has been argued that the common
cognitive biases encountered in medicine tend to occur when clinicians
are engaged in type 1 thinking.
For example, imagine being asked to see a young woman who is
drowsy. She is handed over to you as a ‘probable overdose’ because she
has a history of depression and a packet of painkillers was found beside
her at home. Her observations show she has a Glasgow Coma Scale
score of 10/15, heart rate 100 beats/min, blood pressure 100/60mmHg,
respiratory rate 14 breaths/min, oxygen saturations 98% on air and tem-
perature 37.5°C. Already your mind has reached a working diagnosis.
It ts a pattern (type 1 thinking). You think she has taken an overdose. At
Patient A
90% chance
of having the
disease before
the test is done
34.6% chance
of having the
disease if the
test is negative
0 10 20 30 40 50
Percentage probability of having the disease
60 70 80 90 100 0 10 20 30 40 50 60 70 80 90 100
98.3% chance
of having the
disease if the
test is positive
Patient B
50% chance
of having the
disease before
the test is done
86.4% chance
of having the
disease if the
test is positive
5.6% chance
of having the
disease if the
test is negative
Fig. 1.5 The interpretation of a test result depends on the probability of the disease before the test is carried out. In the example shown, the test being carried out
has a sensitivity of 95% and a specicity of 85%. Patient A has very characteristic clinical ndings, which make the pre-test probability of the condition for which the test
is being used very high – estimated as 90%. Patient B has more equivocal ndings, such that the pre-test probability is estimated as only 50%. If the result in Patient A is
negative, there is still a signicant chance that he has the condition for which he is being tested; in Patient B, however, a negative result makes the diagnosis very unlikely.
1.5 Predictive values:‘What is the probability that a person with
a positive test actually has the disease?’
Disease No disease
Positive test A (True positive) B (False positive)
Negative test C (False negative) D (True negative)
Positive predictive value = A/(A+B) × 100
Negative predictive value = D/(D+C) × 100
28.
Cognitive biases 7
1
this point you can stop to think about your thinking (rational override in
Fig. 1.6): ‘What is the evidence for this diagnosis? What else could it be?’
On the other hand, imagine being asked to assess a patient who has
been admitted with syncope. There are several different causes of syn-
cope and a systematic approach is required to reach a diagnosis (type 2
thinking). However, you recently heard about a case of syncope due to a
painless leaking abdominal aortic aneurysm. At the end of your assess-
ment, following evidence-based guidelines, it is clear the patient can be
discharged. Despite this, you decide to observe the patient overnight
just in case (irrational override in Fig. 1.6). In this example, your intuition
is actually ‘availability bias’ (when things are at the forefront of your mind)
which has signicantly distorted your estimate of probability.
Common cognitive biases in medicine
Figure 1.7 illustrates the common cognitive biases prevalent in medical
practice. Biases often work together; for example, in overcondence bias
(the tendency to believe we know more than we actually do), too much
faith is placed in opinion instead of gathered evidence. This bias can be
augmented by the availability bias and nally by commission bias (the
tendency towards action rather than inaction) – sometimes with disas-
trous results.
The mark of a well-calibrated thinker is the ability to recognise what
mode of thinking is being employed and to anticipate and recognise situ-
ations in which cognitive biases and errors are more likely to occur.
Thinking about thinking
Research has highlighted the importance of being aware of and being
able to think about one’s own thinking (also known as metacognition).
Several studies have demonstrated that engaging in reection during
diagnostic decision-making improves performance (Prakash et al, see
‘Further information’). This can be as simple as asking, ‘What is the
evidence for this diagnosis? What else could it be?’ Reection is most
effective when the case is more complex (relative to the clinician).
Some people have a natural tendency to look for evidence, weigh
things up, be aware of context, not take things at face value, and think
about their own thinking. Others can learn to do so. Psychologists con-
sider this ‘reective mind’ a subset of analytical type 2 thinking. The ten-
dency to engage in reection during decision-making is independent of
knowledge and cognitive ability, and accounts for the greatest variation
in individual performance on many reasoning tasks.
With increasing knowledge and experience, intuitive type 1 think-
ing is used more; it is fast and highly accurate and commonly used
by experts. Novices cannot do this because they have not yet built
a database of patterns in their long-term memory, known as illness
scripts. Reection interrupts type 1 processing, even briey, to sim-
ulate alternatives. So as you can see, good decision-making relies
on knowledge and experience, but also the ability to ‘stop and think’
when needed. However, this requires cognitive effort (type 2 thinking),
which may be impaired by things like fatigue, interruptions and cog-
nitive overload.
Human factors
‘Human factors’ is the science of the limitations of human performance,
and how technology, the work environment and team communication can
adapt for this to reduce diagnostic and other types of error. Analysis of
serious adverse events in clinical practice shows that human factors and
poor team communication play a signicant role when things go wrong.
Research shows that many errors are beyond an individual’s con-
scious control and are precipitated by many factors. The discipline of
human factors seeks to understand interactions between:
people and tasks or technology
people and their work environment
people in a team.
1.6 Type 1 and type 2 thinking
Type 1 Type 2
Intuitive, heuristic (pattern
recognition)
Analytical, systematic
Automatic, subconscious Deliberate, conscious
Fast, effortless Slow, effortful
Low/variable reliability High/consistent reliability
Vulnerable to error Less prone to error
Highly affected by context Less affected by context
High emotional involvement Low emotional involvement
Low scientic rigour High scientic rigour
Experience
Context
Ambient conditions
Education
Training
Logical competence
Clinical
presentation
Recognised
Not
recognised
Type 2
processes
Type 1
processes
Cognitive biases more likely
Irrational
override
Rational
override
Working
diagnosis
Fig. 1.6 The interplay between type 1 and type 2 thinking in the diagnostic process. Adapted from Croskerry P. A universal model of diagnostic reasoning. Acad Med
2009; 84:1022–1028.
29.
8 CLINICALDECISION-MAKING
An understanding of these interactions makes it easier for health-care
professionals, who are committed to ‘rst, do no harm’, to work in the safest
way possible. For example, performance is adversely affected by factors
such as poorly designed processes and equipment, fatigue and poor com-
munication. So simple, clear processes, good design of equipment and
shift patterns, and clear team communication all help to minimise errors.
The areas of the brain required for type 2 processing are most affected
by things like fatigue and cognitive overload, and the brain reverts to
type 1 processing to conserve cognitive energy. Figure 1.8 illustrates
some of the internal and external factors that affect human judgement
and decision-making. An awareness of the factors that impact on
decision-making can allow clinicians to take steps to mitigate these.
Anchoring
The common human tendency
to rely too heavily on the first piece
of information offered (the
‘anchor’) when making decisions
Diagnostic momentum
Once a diagnostic label has been
attached to a patient (by the patient
or other health-care professionals),
it can gather momentum with each
review, leading others to exclude
other possibilities in their thinking
Premature closure
The tendency to close the decision-
making process prematurely and
accept a diagnosis before it, and
other possibilities, have been fully
explored
Ascertainment bias
We sometimes see what we
expect to see (‘self-fulfilling
prophecy’). For example, a
frequent self-harmer attends the
emergency department with
drowsiness; everyone assumes he
has taken another overdose and
misses a brain injury
Psych-out error
Psychiatric patients who present
with medical problems are under-
assessed, under-examined and
under-investigated because
problems are presumed to be due
to, or exacerbated by, their
psychiatric condition
Framing effect
How a case is presented – for
example, in handover – can
generate bias in the listener. This
can be mitigated by always having
‘healthy scepticism’ about other
people’s diagnoses
Availability bias
Things may be at the forefront of
your mind because you have seen
several cases recently or have
been studying that condition in
particular. For example, when one
of the authors worked in an
epilepsy clinic, all blackouts were
possible seizures
Hindsight bias
Knowing the outcome may
profoundly influence the perception
of past events and decision-making,
preventing a realistic appraisal of
what actually occurred – a major
problem in learning from
diagnostic error
Search satisficing
We may stop searching because we
have found something that fits or is
convenient, instead of
systematically looking for the best
alternative, which involves more
effort
Base rate neglect
The tendency to ignore the
prevalence of a disease, which
then distorts Bayesian reasoning.
In some cases, clinicians do this
deliberately in order to rule out an
unlikely but worst-case scenario
Omission bias
The tendency towards inaction,
rooted in the principle of ‘first, do
no harm.’ Events that occur through
natural progression of disease are
more acceptable than those that
may be attributed directly to the
action of the health-care team
Triage-cueing
Triage ensures patients are sent to
the right department. However, this
leads to ‘geography is destiny’. For
example, a diabetic ketoacidosis
patient with abdominal pain and
vomiting is sent to surgery. The
wrong location (surgical ward)
stops people thinking about
medical causes of abdominal pain
and vomiting
Commission bias
The tendency towards action
rather than inaction, on the
assumption that good can come
only from doing something
(rather than ‘watching and
waiting’)
Overconfidence bias
The tendency to believe we know
more than we actually do, placing
too much faith in opinion instead of
gathered evidence
Unpacking principle
Failure to ‘unpack’ all the available
information may mean things are
missed. For example, if a thorough
history is not obtained from either
the patient or carers (a common
problem in geriatric medicine),
diagnostic possibilities may be
discounted
Confirmation bias
The tendency to look for
confirming evidence to support a
theory rather than looking for
disconfirming evidence to refute
it, even if the latter is clearly
present. Confirmation bias is
common when a patient has been
seen first by another doctor
Posterior probability
Our estimate of the likelihood of
disease may be unduly influenced
by what has gone on before for a
particular patient. For example, a
patient who has been extensively
investigated for headaches
presents with a severe headache,
and serious causes are discounted
Visceral bias
The influence of either negative or
positive feelings towards patients,
which can affect our decision-
making
Fig. 1.7 Common cognitive biases in medicine. Adapted from Croskerry P. Achieving quality in clinical decision-making: cognitive strategies and detection of bias. Acad
Emerg Med 2002; 9:1184–1204.
30.
1
Reducing errors inclinical decision-making 9
Several studies demonstrate that we focus our attention to lter out
distractions. This is advantageous in many situations (for example, when
performing a procedure), but in focusing on what we are trying to see we
may not notice the unexpected. In a team context, what is obvious to
one person may be completely missed by someone else. Safe and effec-
tive team communication therefore requires us never to assume, and to
verbalise things, even though they may seem ‘obvious’.
Shared decision-making
Shared decision-making refers to the fact that clinical reasoning does
not necessarily take place solely within a clinician’s head. Good deci-
sion-making is often shared with patients, relatives and carers, within
teams, and by using guidelines, clinical prediction rules and other
decision aids. Integral to all of this is the ability to engage in optimal
decision-making behaviours: involving the patient and/or carers in the
diagnostic and management process, listening to others, following evi-
dence-based guidelines, asking for help when needed, and using clear
communication, especially when handing over care.
Patient-centred evidence-based medicine
‘Patient-centred evidence-based medicine’ refers to the application of
best available research evidence while taking individual patient factors
into account; these include both clinical and non-clinical factors (e.g.
the patient’s social circumstances, values and wishes). For example, a
95-year-old man with dementia and a recent gastrointestinal bleed is
admitted with an inferior myocardial infarction. He is clinically well. Should
he be treated with dual antiplatelet therapy and low-molecular-weight
heparin as recommended in clinical guidelines?
As this chapter has described, clinicians frequently deal with uncer-
tainty/probability. Clinicians need to be able to explain risks and benets
of treatment in an accurate and understandable way. Providing the rel-
evant statistics is seldom sufcient to guide decision-making because a
patient’s perception of risk may be inuenced by irrational factors as well
as individual values.
Research evidence provides statistics but these can be confusing.
Terms such as ‘common’ and ‘rare’ are nebulous. Whenever possible,
clinicians should quote numerical information using consistent denomi-
nators (e.g. ‘90 out of 100 patients who have this operation feel much
better, 1 will die during the operation and 2 will suffer a stroke’). Visual
aids can be used to present complex statistical information (Fig. 1.9).
How uncertainty is conveyed to patients is important. Many studies
demonstrate a correlation between effective clinician–patient communi-
cation and improved health outcomes. If patients feel they have been
listened to and understand the problem and proposed treatment plan,
they are more likely to follow the plan and less likely to re-attend.
Effective team communication
In increasingly complex health-care systems, patients are looked after by
a wide variety of professionals, each of whom has access to important
information required to make clinical decisions. Strict hierarchies are haz-
ardous to patient safety if certain members of the team do not feel able
to speak up.
The SBAR system of communication has been recommended by the
UK’s Patient Safety First campaign. It is a structured way to communicate
about a patient with another health-care professional (e.g. during handover
or when making a referral) and increases the amount of relevant informa-
tion being communicated in a shorter time. It is illustrated in Box 1.7
Using clinical prediction rules and other
decision aids
A clinical prediction rule is a statistical model of the diagnostic process.
When clinical prediction rules are matched against the opinion of experts,
the model usually outperforms the experts, because it is applied con-
sistently in each case. However, it is important that clinical prediction
rules are used correctly – that is, applied to the patient population that
was used to create the rule. Clinical prediction rules force a scientic
assessment of the patient’s symptoms, signs and other data to develop
a numerical probability of a disease or an outcome. They help clinicians
to estimate probability more accurately.
A good example of a clinical prediction rule to estimate pre-test prob-
ability is the Wells score in suspected deep vein thrombosis (see Box
9.18). Other commonly used clinical prediction rules predict outcomes
and therefore guide the management plan. These include the GRACE
score in acute coronary syndromes (see Fig. 16.61) and the CURB-65
score in community-acquired pneumonia (see Fig. 17.32).
Reducing errors in clinical decision-making
Deliberate practice
Studies of expertise reveal that extensive experience is necessary to
achieve high levels of performance, but extensive experience does not
by itself lead to expert levels of achievement. Something else is required,
encapsulated in the term ‘deliberate practice’. This involves effortful
practice, coaching and feedback, and the ability of an individual to plan,
assess, reect and adjust so their performance improves.
Cognitive debiasing strategies
Knowledge and experience alone, however, do not eliminate errors.
There are some simple and established techniques that can be used to
avoid cognitive biases and errors in clinical decision-making.
History and physical examination
Taking a history and performing a physical examination may seem
obvious, but these are sometimes carried out inadequately. This is
the ‘unpacking principle’: failure to unpack all the available information
means things can be missed and lead to error.
Error
Type 1 thinking/
conservation of
cognitive effort
Cognitive and
affective biases
Internal factors
Knowledge
Training
Beliefs and values
Emotions
Sleep/fatigue
Stress
Physical illness
Personality type
External factors
Interruptions
Cognitive overload
Time pressure
Ambient conditions
Insufficient data
Team factors
Patient factors
Poor feedback
Fig. 1.8 Factors that affect our judgement and decision-making. (Type 1 thinking = fast, intuitive, subconscious, low effort)
31.
10 CLINICALDECISION-MAKING
Problem representation/list and differential
diagnosis
Once all the available data from history, physical examination and (some-
times) initial test results are available, these need to be synthesised into
a problem representation or problem list. The process of generating a
problem list ensures nothing is missed. The process of generating a
differential diagnosis works against anchoring on a particular diagnosis
too early, thereby avoiding search satiscing and premature closure (see
Fig. 1.6).
Mnemonics and checklists
These are used frequently in medicine in order to reduce reliance on
fallible human memory. ABCDE (airway, breathing, circulation, disability,
exposure/examination) is probably the most successful checklist in med-
icine, used during the assessment and treatment of critically ill patients
(ABCDE is sometimes prexed with ‘C’ for ‘control of any obvious prob-
lem’; see p. 191). Checklists ensure that important issues have been
considered and completed, especially under conditions of complexity,
stress or fatigue.
Red ags and ROWS (‘rule out worst-case scenario’)
These are strategies that force doctors to consider serious diseases that
can present with common symptoms. Red ags in back pain are listed
in Box 26.19. Considering and investigating for possible pulmonary
embolism in patients who present with pleuritic chest pain and breath-
lessness is a common example of ruling out a worst-case scenario, as
pulmonary embolism can be fatal if missed. Red ags and ROWS help
to avoid cognitive biases such as the ‘framing effect’ and ‘premature
closure’.
Newer strategies to avoid cognitive biases and errors in decision-mak-
ing are emerging. These involve explicit training in clinical reasoning and
human factors. In theory, if doctors are aware of the science of human
thinking and decision-making, then they are more able to think about
their thinking, understand situations in which their decision-making may
be affected, and take steps to mitigate this.
Clinical decision-making: putting it all
together
The following is a practical example that brings together many of the
concepts outlined in this chapter:
A 25-year-old woman presents with right-sided pleuritic chest pain
and breathlessness. She reports that she had an upper respiratory tract
infection a week ago and was almost back to normal when the symp-
toms started. The patient has no past medical history and no family his-
tory, and her only medication is the combined oral contraceptive pill.
On examination, her vital signs are normal (respiratory rate 19 breaths/
min, oxygen saturations 98% on air, blood pressure 115/60mmHg, heart
rate 90 beats/min, temperature 37.5°C) and the physical examination is
also normal. You have been asked to assess her for the possibility of a
pulmonary embolism.
(More information on pulmonary embolism can be found in Chapter 17.)
Evidence-based history and examination
Information from the history and physical examination is vital in deciding
whether this could be a pulmonary embolism. Pleurisy and breathless-
ness are common presenting features of this disease but are also com-
mon presenting features in other diseases. There is nothing in the history
to suggest an alternative diagnosis (e.g. high fever, productive cough,
recent chest trauma). The patient’s vital signs are normal, as is the phys-
ical examination. However, very few individual ndings help to distinguish
patients with pulmonary embolism from those without it. The presence of
wheeze and a high fever modestly decrease the probability of pulmonary
embolism. The presence of hypoxaemia is unhelpful (likelihood ratio not
signicant).
Deciding pre-test probability
The prevalence of pulmonary embolism (PE) in 25-year-old women is
low. We anchor on this prevalence and then adjust for individual patient
factors. This patient has no major risk factors for pulmonary embolism.
To assist our estimate of pre-test probability, we could use a clinical
1.7 The SBAR system of communicating
SBAR Example (a telephone call to the
Intensive Care team)
Situation I am [name] calling from [place] about
a patient with a NEWS* of 10.
Background [Patient’s name], 30-year-old woman,
no past medical history, was admitted
last night with community-acquired
pneumonia. Since then her oxygen
requirements have been steadily
increasing.
Assessment Her vital signs are: blood pressure
115/60mmHg, heart rate 120 beats/
min, temperature 38°C, respiratory
rate 32 breaths/min, oxygen
saturations 89% on 15L via reservoir
bag mask.
An arterial blood gas shows H+
50nmol/L (pH 7.3), PaCO2
4.0kPa
(30mmHg), PaO2
7kPa (52.5mmHg),
standard bicarbonate 14mmol/L.
Chest X-ray shows extensive right
lower zone consolidation.
Recommendation Please can you come and see her as
soon as possible? I think she needs
admission to Intensive Care.
*NEWS = National Early Warning Score; a patient with normal vital signs scores 0. See Royal
College of Physicians. National Early Warning Score (NEWS) 2: Standardising the assessment of
acute-illness severity in the NHS. Updated report of a working party. London: RCP; December 2017.
Feel better
No difference
Stroke
Dead
Fig. 1.9 Visual portrayal of benets and risks. The image refers to an operation
that is expected to relieve symptoms in 90% of patients, but cause stroke in 2% and
death in 1%. From Edwards A, Elwyn G, Mulley A. Explaining risks: turning numerical
data into meaningful pictures. BMJ 2002; 324:827–830, reproduced with permission
from the BMJ Publishing Group.
32.
Further information 11
1
prediction rule: in this case, the modied Wells score for pulmonary
embolism, which would give a score of 3 (low probability – answering
yes only to the criterion ‘PE is the number one diagnosis, an alternative
is less likely’).
Interpreting test results
Imagine the patient went on to have a normal chest X-ray and blood
results, apart from a raised D-dimer of 900 (normal <500ng/mL). A nor-
mal chest X-ray is a common nding in pulmonary embolism. Several
studies have shown that the D-dimer assay has at least 95% sensitivity
in acute pulmonary embolism but it has a low specicity. A very sensitive
test will detect most disease but generate abnormal ndings in healthy
people. On the other hand, a negative result virtually, but not completely,
excludes the disease. It is important at this point to realise that a raised
D-dimer result does not mean this patient has a pulmonary embolism;
it just means that we have not been able to exclude it. Since pulmonary
embolism is a potentially fatal condition we need to rule out the worst-
case scenario (ROWS), and the next step is therefore to arrange further
imaging. What kind of imaging depends on individual patient character-
istics and what is available.
Treatment threshold
The treatment threshold combines factors such as the risks of the test,
and the risks versus benets of treatment. A CT pulmonary angiogram
(CTPA) could be requested for this patient, although in some circum-
stances ventilation–perfusion single-photon emission computed tomog-
raphy (V/QSPECT, p. 547) may be a more suitable alternative. However,
what if the scan cannot be performed until the next day? Because pul-
monary embolism is potentially fatal and the risks of treatment in this
case are low, the patient should be started on treatment while awaiting
the scan.
Post-test probability
The patient’s scan result is subsequently reported as ‘no pulmonary
embolism’. Combined with the low pre-test probability, this scan result
reliably excludes pulmonary embolism.
Cognitive biases
Imagine during this case that the patient had been handed over to you
as ‘nothing wrong – probably a pulled muscle’. Cognitive biases (sub-
conscious tendencies to respond in a certain way) would come into play,
such as the ‘framing effect’, ‘conrmation bias’ and ‘search satiscing’.
The normal clinical examination might conrm the diagnosis of mus-
culoskeletal pain in your mind, despite the examination being entirely
consistent with pulmonary embolism and despite the lack of history and
examination ndings (e.g. chest wall tenderness) to support the diagno-
sis of musculoskeletal chest pain.
Human factors
Imagine that, after you have seen the patient, a colleague hands you
some blood forms and asks you what tests you would like to request
on ‘this lady’. You request blood tests including a D-dimer on the wrong
patient. Luckily, this error is intercepted.
Shared decision-making
The diagnosis of pulmonary embolism can be difcult. Clinical prediction
rules (e.g. modied Wells score), guidelines (e.g. from the UK’s National
Institute for Health and Care Excellence, or NICE) and decision aids (e.g.
simplied pulmonary embolism severity index, or PESI) are frequently
used in combination with the doctor’s opinion, derived from information
gathered in the history and physical examination.
The patient is treated according to evidence-based guidelines that
apply to her particular situation. Tests alone do not make a diagnosis
and at the end of this process the patient is told that the combination
of history, examination and test results mean she is extremely unlikely
to have a pulmonary embolism. Viral pleurisy is offered as an alternative
diagnosis and she is reassured that her symptoms are expected to settle
over the coming days with analgesia. She is advised to re-present to
hospital if her symptoms get worse.
Answers to problems
Harvard problem (p. 5)
Almost half of doctors surveyed said 95%, but they neglected to take
prevalence into account. If 1000 people are tested, there will be 51 pos-
itive results: 50 false positives and 1 true positive (assuming 100% sen-
sitivity). The chance that a person found to have a positive result actually
has the disease is therefore only 1/51 or ~2%.
Bat and ball problem (p. 6)
This puzzle is from the book Thinking, Fast and Slow, by Nobel laureate
Daniel Kahneman (see ‘Further information’). He writes, ‘A number came
to your mind. The number, of course, is 10p. The distinctive mark of this
easy puzzle is that it evokes an answer that is intuitive, appealing – and
wrong. Do the math, and you will see.’ The correct answer is 5p.
Further information
Books and journal articles
Bordage G. Elaborated knowledge: a key to successful diagnostic thinking. Acad
Med 1994;69:883–885.
Casscells W, Schoenberger A, Graboys TB. Interpretation by physicians of clinical
laboratory results. N Engl J Med 1978;299:999–1001.
Cooper N, Frain J, eds. ABC of clinical reasoning. Oxford: Wiley–Blackwell; 2016.
Croskerry P. A universal model of diagnostic reasoning. Acad Med
2009;84:1022–1028.
Kahneman D. Thinking, fast and slow. London: Penguin; 2012.
McGee S. Evidence-based physical diagnosis, 4th edn. Philadelphia: Elsevier;
2018.
Prakash S, Sladek RM, Schuwirth L. Interventions to improve diagnostic decision
making: a systematic review and meta-analysis on reective strategies. Med
Teacher 2019;41:517–524.
Sox H, Higgins MC, Owens DK. Medical decision making, 2nd edn. Chichester:
Wiley–Blackwell; 2013.
Websites
creme.org.uk UK Clinical Reasoning in Medical Education Group.
improvediagnosis.org Society to Improve Diagnosis in Medicine (USA).
chfg.org UK Clinical Human Factors Group.
clinical-reasoning.org Clinical reasoning resources.
vassarstats.net/index.html Suite of calculators for statistical computation
(Calculator 2 is a calculator for predictive values and likelihood ratios) .
Multiple Choice Questions
1.1.Which of the following best describes what is meant by the term
‘shared decision-making’?
A. Allowing the patient to make decisions about his or her care
B. Asking for help when needed
C. Making decisions with the wider clinical team
D. Patient-centred evidence-based medicine
E. Sharing decision-making with the patient, relatives, clinical
team, guidelines and decision aids
Answer: E.
Shared decision-making refers to the fact that clinical reasoning does
not necessarily take place solely within a clinician’s head. Good deci-
sion-making is often shared with patients, relatives and carers, within
teams, and by using guidelines, clinical prediction rules and other
decision aids. Integral to all of this is the ability to engage in optimal
decision-making behaviours: involving the patient and/or carers in the
diagnostic and management process, listening to others, following evi-
dence-based guidelines, asking for help when needed, and using clear
communication, especially when handing over care.
1.2. Which of the following best describes what is meant by the term
‘metacognitive knowledge’?
A. An awareness of and an ability to think about one’s own
thinking
B. An understanding of how things t together
C. An understanding of how to do something, what techniques to
use
D. The tendency to engage in deliberate practice
E. The tendency to look for evidence, weigh things up and be
aware of context
Answer: A.
Metacognition literally means ‘thinking about thinking’, ‘knowing
about knowing’, and being ‘aware of one’s awareness’. Metacognitive
knowledge is about being aware of one’s own cognitive processes
and is a key ingredient for successful learning and the development
of expertise.
1.3. When using a clinical prediction rule for a specic condition, which
of the following is true?
A. The tool can be applied to any patient with the condition
B. The tool can determine the clinical probability that a patient has
a disease
C. The tool is not necessarily based on evidence
D. The tool uses the receiver operating characteristic curve to
statistically model the diagnostic process
E. Use of the tool removes the need for clinical decision-making
on the part of the clinician
Answer: B.
A clinical prediction rule is a statistical model of the diagnostic pro-
cess. When clinical prediction rules are matched against the opinion of
experts, the model usually outperforms the experts, because it is applied
consistently in each case. However, it is important that clinical prediction
rules are used correctly – that is, applied to the patient population that
was used to create the rule. Clinical prediction rules force a scientic
assessment of the patient’s symptoms, signs and other data to develop
a numerical probability of a disease or an outcome. They help clinicians
to estimate probability more accurately. A good example of a clinical
prediction rule to estimate pre-test probability is the Wells score in sus-
pected deep vein thrombosis.
1.4. Regarding normal values, which of the following statements is
true?
A. A normal result means that disease is not present
B. A normal result specic to a particular disease is not found in a
person with the disease
C. Normal values do not vary signicantly within the same person
D. Normal values follow a Poisson distribution
E. One in 20 healthy people will have values that lie outside the
reference range
Answer: E.
Most tests provide quantitative results on a continuous numerical
scale. In order to classify quantitative results as normal or abnormal, it is
necessary to dene a cut-off point. Many quantitative measurements in
populations have a Gaussian or ‘normal’ distribution. By convention, the
normal (reference) range is dened as those values that encompass 95%
of the population, or 2 standard deviations above and below the mean.
This means that 2.5% of the normal population will have values above,
and 2.5% will have values below the normal range. In some diseases
there is no overlap between results from the abnormal and normal pop-
ulation. However, in many diseases there is overlap. So a healthy person
can have an ‘abnormal’ result, and a person with disease can have a
‘normal’ result. Normal values also vary from day to day in the same per-
son in the absence of a real change because of biological variation and
laboratory variation. The extent to which a blood test is allowed to vary
before it has truly changed is called the ‘critical difference’.
1.5. An 80-year-old man presented with a fall on a background of a
2-week history of his legs ‘giving way’ and a history of increasing
low back pain. In this scenario, which of the following strategies to
reduce errors in clinical reasoning would be most appropriate?
A. An assessment of his mobility
B. An X-ray of the lumbar spine
C. Generating a problem representation
D. Use of a clinical prediction rule
E. Using red ags and ‘ROWS’ (rule out worst-case scenario)
Answer: E.
Various strategies can be used to reduce errors in clinical reasoning
including: taking a proper history and performing a thorough physical
examination, generating a problem representation, and using mnemon-
ics and checklists. Using red ags and ROWS (rule out worst-case
scenario) forces clinicians to think about serious diseases that can
present with common symptoms. In this example, it forces clinicians
to consider the possibility of cord compression. Red ags and ROWS
helps to avoid cognitive biases such as the ‘framing effect’ and ‘pre-
mature closure’.
35.
Clinical therapeutics and
goodprescribing
Principles of clinical pharmacology 13
Pharmacodynamics 13
Pharmacokinetics 16
Inter-individual variation in drug responses 19
Adverse outcomes of drug therapy 19
Adverse drug reactions 19
Drug interactions 23
Medication errors 24
Drug regulation and management 25
Drug development and marketing 25
Managing the use of medicines 27
Prescribing in practice 28
Decision-making in prescribing 28
Prescribing in special circumstances 31
Writing prescriptions 32
Monitoring drug therapy 35
2
SRJ Maxwell
36.
14 CLINICALTHERAPEUTICS AND GOOD PRESCRIBING
Prescribing medicines is the major tool used by doctors to restore or
preserve the health of patients. Medicines contain drugs (the specic
chemical substances with pharmacological effects), either alone or in
combination with additional drugs, in a formulation mixed with other
ingredients. The benecial effects of medicines must be weighed against
their cost and potential adverse drug reactions and interactions. The lat-
ter two factors are sometimes caused by injudicious prescribing deci-
sions and by prescribing errors. The modern prescriber must meet the
challenges posed by the increasing number of drugs and formulations
available, of indications for prescribing them and the greater complexity
of treatment regimens followed by individual patients (‘polypharmacy’, a
particular challenge in the ageing population). The purpose of this chap-
ter is to elaborate on the principles and practice that underpin good pre-
scribing (Box 2.1).
at a particular subtype of receptor to exhibit some effect at other
subtypes. For example, β-adrenoceptors can be subtyped on the
basis of their responsiveness to the endogenous agonist noradren-
aline (norepinephrine): the concentration of noradrenaline required
to cause bronchodilatation (via β2
-adrenoceptors) is 10 times higher
than that required to cause tachycardia (via β1
-adrenoceptors).
‘Cardioselective’ β-adrenoceptor antagonists (β-blockers) have
anti-anginal effects on the heart (β1
), but may still cause broncho-
spasm in the lung (β2
) and are contraindicated for asthmatic patients.
Agonists bind to a receptor to produce a conformational change
that is coupled to a biological response. As agonist concentration
increases, so does the proportion of receptors occupied, and hence
the biological effect. Partial agonists activate the receptor, but can-
not produce a maximal signalling effect equivalent to that of a full
agonist even when all available receptors are occupied.
Antagonists bind to a receptor, but do not produce the confor-
mational change that initiates an intracellular signal. A competitive
antagonist competes with endogenous agonist ligands to occupy
receptor-binding sites, with the resulting antagonism depending
on the relative afnities and concentrations of the antagonist drug
and endogenous agonist. Non-competitive antagonists inhibit the
effect of an agonist by mechanisms other than direct competition
for receptor binding with the agonist (e.g. by affecting post-receptor
signalling).
Dose–response relationships
Plotting the logarithm of drug dose against drug response typically pro-
duces a sigmoidal dose–response curve (Fig. 2.2). Progressive increases
in drug dose (which, for most drugs, is proportional to the plasma drug
concentration) produce increasing response, but only within a relatively
narrow range of dose; further increases in dose beyond this range pro-
duce little extra effect. The following characteristics of the drug response
are useful in comparing different drugs:
Efcacy describes the extent to which a drug can produce a tar-
get-specic response when all available receptors or binding sites
are occupied (i.e. Emax
on the dose–response curve). A full agonist
can produce the maximum response of which the receptor is capa-
ble, while a partial agonist at the same receptor will have lower
efcacy. Therapeutic efcacy describes the effect of the drug on a
desired biological endpoint and can be used to compare drugs that
act via different pharmacological mechanisms (e.g. loop diuretics
2.1 Steps in good prescribing
Make a diagnosis
Consider factors that might inuence the patient’s response to therapy (age,
concomitant drug therapy, renal and liver function etc.)
Establish the therapeutic goal*
Choose the therapeutic approach*
Choose the drug and its formulation (the ‘medicine’)
Choose the dose, route and frequency
Choose the duration of therapy
Write an unambiguous prescription (or ‘medication order’)
Inform the patient about the treatment and its likely effects
Monitor treatment effects, both benecial and harmful
Review/alter the prescription
*These steps in particular take the patient’s views into consideration to establish a therapeutic
partnership that aims to achieve ‘concordance’ based on shared decision-making
Dosage
regimen
Plasma
concentration
Concentration at
the site of action
Pharmacological
effects
Pharmacokinetics ‘what the body does
to a drug’
Monitoring
Measure plasma drug
concentration
‘what a drug does
to the body’
Monitoring
Measure clinical
effects
Time
Concentration
Pharmacodynamics
Concentration
Effect
Fig. 2.1 Pharmacokinetics and pharmacodynamics.
Principles of clinical pharmacology
Prescribers need to understand what the drug does to the body (phar-
macodynamics) and what the body does to the drug (pharmacokinetics)
(Fig. 2.1). Although this chapter is focused on the most common drugs,
which are synthetic small molecules, the same principles apply to the
increasingly numerous ‘biologic’ therapies (sometimes abbreviated to
‘biologics’) now in use, which include peptides, proteins, enzymes and
monoclonal antibodies.
Pharmacodynamics
Drug targets and mechanisms of action
Modern drugs are usually discovered by screening compounds for activ-
ity either to stimulate or to block the function of a specic molecular tar-
get, which is predicted to have a benecial effect in a particular disease
(Box 2.2). Other drugs have useful but less selective chemical properties,
such as chelators (e.g. for treatment of iron or copper overload), osmotic
agents (used as diuretics in cerebral oedema) or general anaesthetics
(that alter the biophysical properties of lipid membranes). The following
characteristics of the interaction of drugs with receptors illustrate some
of the important determinants of the effects of drugs:
Afnity describes the propensity for a drug to bind to a receptor
and is related to the ‘molecular t’ and the strength of the chemi-
cal bond. Some drug–receptor interactions are irreversible, either
because the afnity is so strong or because the drug modies the
structure of its molecular target.
Selectivity describes the propensity for a drug to bind to one target
rather than another. Selectivity is a relative term, not to be con-
fused with absolute specicity. It is common for drugs targeted
37.
Principles of clinicalpharmacology 15
2
2.2 Examples of target molecules for drugs
Drug target Description Examples
Receptors
Channel-linked receptors Ligand binding controls a linked ion channel, known as ‘ligand-
gated’ (in contrast to ‘voltage-gated’ channels that respond to
changes in membrane potential)
Nicotinic acetylcholine receptor
GABA receptor
Sulphonylurea receptor
G-protein-coupled receptors (GPCRs) Ligand binding affects one of a family of ‘G-proteins’ that mediate
signal transduction either by activating intracellular enzymes
(such as adenylate or guanylate cyclase, producing cyclic AMP or
GMP, respectively) or by controlling ion channels
Muscarinic acetylcholine receptor
β-adrenoceptors
Dopamine receptors
5-Hydroxytryptamine (5-HT, serotonin) receptors
Opioid receptors
Kinase-linked receptors Ligand binding activates an intracellular protein kinase that
triggers a cascade of phosphorylation reactions
Insulin receptor
Cytokine receptors
Transcription factor receptors Intracellular and also known as ‘nuclear receptors’; ligand binding
promotes or inhibits gene transcription and hence synthesis of
new proteins
Steroid receptors
Thyroid hormone receptors
Vitamin D receptors
Retinoid receptors
PPARγ and α receptors
Other targets
Voltage-gated ion channels Mediate electrical signalling in excitable tissues (muscle and
nervous system)
Na+
channels
Ca2+
channels
Enzymes Catalyse biochemical reactions. Drugs interfere with binding of
substrate to the active site or of co-factors
Cyclo-oxygenase
ACE
Xanthine oxidase
Transporter proteins Carry ions or molecules across cell membranes 5-HT re-uptake transporter
Na+
/K+
ATPase
Cytokines and other signalling molecules Small proteins that are important in cell signalling (autocrine,
paracrine and endocrine), especially affecting the immune
response
Tumour necrosis factors
Interleukins
Cell surface antigens Block the recognition of cell surface molecules that modulate
cellular responses
Cluster of differentiation molecules (e.g. CD20,
CD80)
(ACE = angiotensin-converting enzyme; AMP = adenosine monophosphate; ATPase = adenosine triphosphatase; GABA = γ-aminobutyric acid; GMP = guanosine monophosphate; PPAR =
peroxisome proliferator-activated receptor)
Hypersusceptibility Side-effects
100
80
60
40
20
0
0.0001 0.001 0.01 0.1 1 10 100 1000
Therapeutic index
100/0.1 = 1000
Drug dose (mg)
Response
(%
of
maximum)
Toxic effects
Adverse
effect
ED50 =100 mg
Beneficial
effect
ED50 =0.1 mg
Emax
ED50 ED50
Fig. 2.2 Dose–response curve. The green curve represents the benecial effect of the drug. The maximum response on the curve is the Emax
and the dose (or concentration)
producing half this value (Emax
/2) is the ED50
(or EC50
). The red curve illustrates the dose–response relationship for the most important adverse effect of this drug. This occurs at
much higher doses; the ratio between the ED50
for the adverse effect and that for the benecial effect is the ‘therapeutic index’, which indicates how much margin there is for
prescribers when choosing a dose that will provide benecial effects without also causing this adverse effect. Adverse effects that occur at doses above the therapeutic range
are normally called ‘toxic effects’, while those occurring within the therapeutic range are ‘side-effects’ and those below it are ‘hyper-susceptibility effects’.
38.
16 CLINICALTHERAPEUTICS AND GOOD PRESCRIBING
induce a greater diuresis than thiazide diuretics and so have greater
therapeutic efcacy).
Potency describes the amount of drug required for a given
response. More potent drugs produce biological effects at lower
doses, so they have a lower ED50
. A less potent drug can still have
an equivalent or greater efcacy if it is given in higher doses.
The dose–response relationship for a specic drug varies between
patients because of variations in the many determinants of pharma-
cokinetics and pharmacodynamics. In clinical practice, the prescriber is
unable to construct a dose–response curve for each individual patient.
Therefore, most drugs are licensed for use within a recommended range
of doses that is expected to reach close to the top of the dose–response
curve for most patients. However, it is sometimes possible to achieve the
desired therapeutic efcacy at doses towards the lower end of, or even
below, the recommended range.
Therapeutic index
The adverse effects of drugs are often dose-related in a similar way to the
benecial effects, although the dose–response curve for these adverse
effects is normally shifted to the right (see Fig. 2.2). The ratio of the
ED50
for therapeutic efcacy and for a major adverse effect is known as
the ‘therapeutic index’. In reality, drugs have multiple potential adverse
effects, but the concept of therapeutic index is usually based on adverse
effects that might require dose reduction or discontinuation. For most
drugs, the therapeutic index is greater than 100, but there are some
notable exceptions with therapeutic indices of less than 10 (e.g. digoxin,
warfarin, insulin, phenytoin, opioids). The doses of such drugs have to
be titrated carefully for individual patients to maximise benets, but avoid
adverse effects.
Desensitisation and withdrawal effects
Desensitisation refers to the common situation in which the biological
response to a drug diminishes when it is given continuously or repeat-
edly. It may be possible to restore the response by increasing the dose
of the drug but, in some cases, the tissues may become completely
refractory to its effect.
Tachyphylaxis describes desensitisation that occurs very rap-
idly, sometimes with the initial dose. This rapid loss of response
implies depletion of chemicals that may be necessary for the
pharmacological actions of the drug (e.g. a stored neurotransmitter
released from a nerve terminal) or receptor phosphorylation.
Tolerance describes a more gradual loss of response to a drug that
occurs over days or weeks. This slower change implies changes in
receptor numbers or the development of counter-regulatory physio-
logical changes that offset the actions of the drug (e.g. accumulation
of salt and water in response to vasodilator therapy).
Drug resistance is a term normally reserved for describing the loss
of effectiveness of an antimicrobial (p. 113) or cancer chemotherapy
drug.
In addition to these pharmacodynamic causes of desensitisation,
reduced response may be the consequence of lower plasma and
tissue drug concentrations as a result of altered pharmacokinetics
(see below).
When drugs induce chemical, hormonal and physiological changes
that offset their actions, discontinuation may allow these changes to
cause ‘rebound’ withdrawal effects (Box 2.3).
Pharmacokinetics
Understanding ‘what the body does to the drug’ (Fig. 2.3) is extremely
important for prescribers because this forms the basis on which the opti-
mal route of administration and dose regimen are chosen and explains
the majority of inter-individual variation in the response to drug therapy.
Drug absorption and routes of administration
Absorption is the process by which drug molecules gain access to the
blood stream. The rate and extent of drug absorption depend on the
route of administration (see Fig. 2.3).
Enteral administration
These routes involve administration via the gastrointestinal tract:
Oral. This is the most common route of administration because it
is simple, convenient and readily used by patients to self-admin-
ister their medicines. Absorption after an oral dose is a complex
process that depends on the drug being swallowed, surviving
exposure to gastric acid, avoiding unacceptable food binding, being
absorbed across the small bowel mucosa into the portal venous
system, and surviving metabolism by gut wall or liver enzymes
2.3 Examples of drugs associated with withdrawal effects
Drug Symptoms Signs Treatment
Alcohol Anxiety, panic, paranoid delusions,
visual and auditory hallucinations
Agitation, restlessness, delirium,
tremor, tachycardia, ataxia,
disorientation, seizures
Treat immediate withdrawal syndrome with
benzodiazepines
Barbiturates, benzodiazepines Similar to alcohol Similar to alcohol Transfer to long-acting benzodiazepine then
gradually reduce dosage
Glucocorticoids Weakness, fatigue, decreased
appetite, weight loss, nausea,
vomiting, diarrhoea, abdominal pain
Hypotension, hypoglycaemia Prolonged therapy suppresses the
hypothalamic–pituitary–adrenal axis and
causes adrenal insufciency requiring
glucocorticoid replacement. Withdrawal
should be gradual after prolonged therapy
(p. 684)
Opioids Rhinorrhoea, sneezing, yawning,
lacrimation, abdominal and leg
cramping, nausea, vomiting,
diarrhoea
Dilated pupils Transfer addicts to long-acting agonist
methadone
Selective serotonin re-uptake
inhibitors (SSRIs)
Dizziness, sweating, nausea,
insomnia, tremor, delirium,
nightmares
Tremor Reduce SSRIs slowly to avoid withdrawal
effects
39.
Principles of clinicalpharmacology 17
2
on some degree of manual dexterity and timing (see Fig. 17.23).
Patients who nd these difcult may use a ‘spacer’ device to
improve drug delivery. A special mode of inhaled delivery is via a
nebulised solution created by using pressurised oxygen or air to
break up solutions and suspensions into small aerosol droplets that
can be directly inhaled from the mouthpiece of the device.
Drug distribution
Distribution is the process by which drug molecules transfer between
the circulating blood, interstitial space and intracellular uid. This is
inuenced by the drug’s molecular size and lipid solubility, the extent to
which it binds to proteins in plasma, its susceptibility to drug transporters
expressed on cell surfaces and its binding to its molecular target and
to other cellular proteins (which can be irreversible). Most drugs diffuse
passively across capillary walls down a concentration gradient into the
interstitial uid until the concentration of free drug molecules in the inter-
stitial uid is equal to that in the plasma. As drug molecules in the blood
are removed by metabolism or excretion, the plasma concentration falls
and drug molecules diffuse back from the tissue compartment into the
blood until eventually all are eliminated. Note that this reverse movement
of drug away from the tissues will be prevented if further drug doses are
administered and absorbed into the plasma.
Volume of distribution
The apparent volume of distribution (Vd
) is the volume into which a drug
appears to have distributed following intravenous injection. It is calcu-
lated from the equation
Vd
= D.C0
where D is the amount of drug given and C0
is the initial plasma concen-
tration (Fig. 2.4A). Drugs that are highly bound to plasma proteins may
have a Vd
below 10L (e.g. warfarin, aspirin), while those that diffuse into
the interstitial uid but do not enter cells because they have low lipid sol-
ubility may have a Vd
between 10 and 30L (e.g. gentamicin, amoxicillin).
It is an ‘apparent’ volume because those drugs that are lipid-soluble and
highly tissue-bound may have a Vd
of greater than 100L (e.g. digoxin,
amitriptyline). Drugs with a larger Vd
have longer half-lives (see below),
take longer to reach steady state on repeated administration and are
eliminated more slowly from the body following discontinuation.
Drug elimination
Drug metabolism
Metabolism is the process by which drugs are chemically altered from
a lipid-soluble form suitable for absorption and distribution to a more
water-soluble form that is necessary for excretion. Some drugs, known
as ‘prodrugs’, are inactive in the form in which they are administered, but
are converted to an active metabolite in vivo.
(‘rst-pass metabolism’). As a consequence, absorption is frequently
incomplete following oral administration. The term ‘bioavailability’
describes the proportion of the dose that reaches the systemic
circulation intact.
Buccal and sublingual (SL). These routes have the advantage of
enabling rapid absorption into the systemic circulation without
the uncertainties associated with oral administration (e.g. organic
nitrates for angina pectoris, triptans for migraine, opioid analgesics).
Rectal (PR). The rectal mucosa is occasionally used as a site of
drug administration when the oral route is compromised because of
nausea and vomiting or unconsciousness (e.g. diazepam in status
epilepticus).
Parenteral administration
These routes avoid absorption via the gastrointestinal tract and rst-pass
metabolism in the liver:
Intravenous (IV). The IV route enables all of a dose to enter the sys-
temic circulation reliably, without any concerns about absorption
or rst-pass metabolism (i.e. the dose is 100% bioavailable), and
rapidly achieve a high plasma concentration. It is ideal for very ill
patients when a rapid, certain effect is critical to outcome (e.g. ben-
zylpenicillin for meningococcal meningitis).
Intramuscular (IM). IM administration is easier to achieve than the
IV route (e.g. adrenaline (epinephrine) for acute anaphylaxis), but
absorption is less predictable and depends on muscle blood ow.
Subcutaneous (SC). The SC route is ideal for drugs that have to
be administered parenterally because of low oral bioavailability, are
absorbed well from subcutaneous fat and can ideally be injected by
patients themselves (e.g. insulin, heparin).
Transdermal. A transdermal patch can enable a drug to be
absorbed through the skin and into the circulation (e.g. oestrogens,
nicotine, nitrates).
Nasal. The nasal mucosa provides another potential route for
absorption of some drugs with systemic action (e.g. sumatriptan,
calcitonin, naloxone, testosterone, desmopressin).
Other routes of administration
Topical application of a drug involves direct administration to the site
of action (e.g. skin, eye, ear). This has the advantage of achieving
sufcient concentration at this site while minimising systemic expo-
sure and the risk of adverse effects elsewhere.
Inhaled (INH) administration allows drugs to be delivered directly
to a target in the respiratory tree, usually the small airways (e.g.
salbutamol, beclometasone). However, a signicant proportion of
the inhaled dose may be absorbed from the lung or is swallowed
and can reach the systemic circulation. The most common mode
of delivery is the metered-dose inhaler, but its success depends
I
terst tial fluid
Intracellular
fluid
Kidney
Liver
Parenteral
Mouth
Stomach
Small
intestine
Large
intestine
Rectum
Buccal
Excretion
in urine
Excretion
in faeces
Portal
venous system
Intestinal wall enzymes
Liver enzymes
Metabolism
Circulating
plasma
Rectal
Oral
Fig. 2.3 Pharmacokinetics summary. Most drugs are taken orally,
are absorbed from the intestinal lumen and enter the portal venous
system to be conveyed to the liver, where they may be subject to
rst-pass metabolism and/or excretion in bile. Active drugs then enter
the systemic circulation, from which they may diffuse (or sometimes
be actively transported) in and out of the interstitial and intracellular
uid compartments. Drug that remains in circulating plasma is subject
to liver metabolism and renal excretion. Drugs excreted in bile may
be reabsorbed, creating an enterohepatic circulation. First-pass
metabolism in the liver is avoided if drugs are administered via the
buccal or rectal mucosa, or parenterally (e.g. by intravenous injection).
40.
18 CLINICALTHERAPEUTICS AND GOOD PRESCRIBING
Phase I metabolism involves oxidation, reduction or hydrolysis to make
drug molecules suitable for phase II reactions or for excretion. Oxidation
is by far the most common form of phase I reaction and chiey involves
members of the cytochrome P450 family of membrane-bound enzymes
in the endoplasmic reticulum of hepatocytes.
Phase II metabolism involves combining phase I metabolites with an
endogenous substrate to form an inactive conjugate that is much more
water-soluble. Reactions include glucuronidation, sulphation, acetyla-
tion, methylation and conjugation with glutathione. This is necessary to
enable renal excretion, because lipid-soluble metabolites will simply dif-
fuse back into the body after glomerular ltration (p. 617).
Drug excretion
Excretion is the process by which drugs and their metabolites are
removed from the body.
Renal excretion is the usual route of elimination for drugs or their
metabolites that are of low molecular weight and sufciently water-soluble
to avoid reabsorption from the renal tubule. Drugs bound to plasma pro-
teins are not ltered by the glomeruli. The pH of the urine is more acidic
than that of plasma, so that weakly acidic drugs (e.g. salicylates) become
un-ionised and tend to be reabsorbed. Alkalination of the urine can has-
ten excretion (e.g. after a salicylate overdose; p. 225). For some drugs,
active secretion into the proximal tubule lumen, rather than glomerular
ltration, is the predominant mechanism of excretion (e.g. methotrexate,
penicillins).
Faecal excretion is the predominant route of elimination for drugs
with high molecular weight, including those that are excreted in the bile
after conjugation with glucuronide in the liver and any drugs that are not
absorbed after enteral administration. Molecules of drug or metabolite
that are excreted in the bile enter the small intestine where they may, if
they are sufciently lipid-soluble, be reabsorbed through the gut wall and
return to the liver via the portal vein (see Fig. 2.3). This recycling between
the liver, bile, gut and portal vein is known as ‘enterohepatic circulation’
and can signicantly prolong the residence of drugs in the body (e.g.
digoxin, morphine, levothyroxine).
Elimination kinetics
The net removal of drug from the circulation results from a combination of
drug metabolism and excretion and is usually described as ‘clearance’,
i.e. the volume of plasma that is completely cleared of drug per unit time.
For most drugs, elimination is a high-capacity process that does not
become saturated, even at high dosage. The rate of elimination is, there-
fore, directly proportional to the drug concentration because of the ‘law
of mass action’, whereby higher drug concentrations will drive faster met-
abolic reactions and support higher renal ltration rates. This results in
‘rst-order’ kinetics, when a constant fraction of the drug remaining in the
circulation is eliminated in a given time and the decline in concentration
over time is exponential (see Fig. 2.4A). This elimination can be described
by the drug’s half-life (t1/2
), i.e. the time taken for the plasma drug concen-
tration to halve, which remains constant throughout the period of drug
elimination. The signicance of this phenomenon for prescribers is that
the effect of increasing doses on plasma concentration is predictable – a
doubled dose leads to a doubled concentration at all time points.
For a few drugs in common use (e.g. phenytoin, alcohol), elimination
capacity is exceeded (saturated) within the usual dose range. This is
called ‘zero-order’ kinetics. Its signicance for prescribers is that, if the
rate of administration exceeds the maximum rate of elimination, the drug
will accumulate progressively, leading to serious toxicity.
Repeated dose regimens
The goal of therapy is usually to maintain drug concentrations within the
therapeutic range (see Fig. 2.2) over several days (e.g. antibiotics) or
even for months or years (e.g. antihypertensives, lipid-lowering drugs,
thyroid hormone replacement therapy). This goal is rarely achieved with
single doses, so prescribers have to plan a regimen of repeated doses.
This involves choosing the size of each individual dose and the frequency
of dose administration.
As illustrated in Figure 2.4B, the time taken to reach drug concentra-
tions within the therapeutic range depends on the half-life of the drug.
Typically, with doses administered regularly, it takes approximately 5
half-lives to reach a ‘steady state’ in which the rate of drug elimination
is equal to the rate of drug administration. This applies when starting
new drugs and when adjusting doses of current drugs. With appropri-
ate dose selection, steady-state drug concentrations will be maintained
within the therapeutic range. This is important for prescribers because it
means that the effects of a new prescription, or dose titration, for a drug
with a long half-life (e.g. digoxin – 36 hours) may not be known for a few
days. In contrast, drugs with a very short half-life (e.g. dobutamine – 2
minutes) have to be given continuously by infusion, but reach a new
steady state within minutes.
Time (hours)
A constant fraction of drug
is cleared in unit time
t1/2
= 8 hours
C0
Plasma
drug
concentration
6 12 18 24
A
Loading
dose
Dose
Dose
Dose Dose Dose Dose Dose
Subtherapeutic
Dose interval = 24 hours
Time (days)
Plasma
drug
concentration
1 2 3 4 5 6
Therapeutic range
Adverse effects
t1/2= 30 hours
B
Fig. 2.4 Drug concentrations in plasma following single and multiple drug
dosing.
the time period required for the plasma drug concentration to halve (half-life, t1/2
) remains
concentration rises if each dose is administered before the previous dose has been
entirely cleared. In this example, the drug’s half-life is 30 hours, so that with daily dosing
the peak, average and trough concentrations steadily increase as drug accumulates
in the body (black line). Steady state is reached after approximately 5 half-lives, when
the rate of elimination (the product of concentration and clearance) is equal to the rate
of drug absorption (the product of rate of administration and bioavailability). The long
half-life in this example means that it takes 6 days for steady state to be achieved and,
for most of the rst 3 days of treatment, plasma drug concentrations are below the
therapeutic range. This problem can be overcome if a larger loading dose (red line) is
used to achieve steady-state drug concentrations more rapidly.
41.
Adverse outcomes ofdrug therapy 19
2
For drugs with a long half-life, if it is unacceptable to wait for 5 half-
lives until concentrations within the therapeutic range are achieved, then
an initial ‘loading dose’ can be given that is much larger than the main-
tenance dose and equivalent to the amount of drug required in the body
at steady state. This achieves a peak plasma concentration close to the
plateau concentration, which can then be maintained by successive
maintenance doses.
‘Steady state’ actually involves uctuations in drug concentrations,
with peaks just after administration followed by troughs just prior to the
next administration. The manufacturers of medicines recommend dosing
regimens that predict that, for most patients, these oscillations result in
troughs within the therapeutic range and peaks that are not high enough
to cause adverse effects. The optimal dose interval is a compromise
between convenience for the patient and a constant level of drug expo-
sure. More frequent administration (e.g. 25mg 4 times daily) achieves a
smoother plasma concentration prole than 100mg once daily, but is
much more difcult for patients to sustain. A solution to this need for
compromise in dosing frequency for drugs with half-lives of less than
24 hours is the use of ‘modied-release’ (m/r) formulations. These allow
drugs to be absorbed more slowly from the gastrointestinal tract and
reduce the oscillation in plasma drug concentration prole, which is espe-
cially important for drugs with a low therapeutic index (e.g. levodopa).
Inter-individual variation in drug responses
Prescribers have numerous sources of guidance about how to use drugs
appropriately (e.g. dose, route, frequency, duration) for many conditions.
However, this advice is based on average dose–response data derived
from observations in many individuals. When applying this information
to an individual patient, prescribers must take account of inter-individ-
ual variability in response. Some of this variability is predictable and
good prescribers are able to anticipate it and adjust their prescriptions
accordingly to maximise the chances of benet and minimise harm. Inter-
individual variation in responses also mandates that effects of treatment
should be monitored (p. 35).
Some inter-individual variation in drug response is accounted for by
differences in pharmacodynamics. For example, the benecial natriuresis
produced by the loop diuretic furosemide is often signicantly reduced
at a given dose in patients with renal impairment, while delirium caused
by opioid analgesics is more likely in the elderly. However, it is differences
in pharmacokinetics that more commonly account for different drug
responses. Examples of factors inuencing the absorption, metabolism
and excretion of drugs are shown in Box 2.4.
It is hoped that a signicant proportion of the inter-individual varia-
tion in drug responses can be explained by studying genetic differences
in single genes (‘pharmacogenetics’; Box 2.5) or the effects of multiple
gene variants (‘pharmacogenomics’). The aim is to identify those patients
most likely to benet from specic treatments and those most suscepti-
ble to adverse effects. In this way, it may be possible to select drugs and
dose regimens for individual patients to maximise the benet-to-hazard
ratio (‘personalised medicine’).
Adverse outcomes of drug therapy
The decision to prescribe a drug always involves a judgement of the
balance between therapeutic benets and risk of an adverse outcome.
Both prescribers and patients tend to be more focused on the former,
but a truly informed decision requires consideration of both.
Adverse drug reactions
Some important denitions for the adverse effects of drugs are:
Adverse event. A harmful event that occurs while a patient is taking
a drug, irrespective of whether the drug is suspected of being the
cause.
Adverse drug reaction (ADR). An unwanted or harmful reaction that
is experienced following the administration of a drug or combination
of drugs under normal conditions of use and is suspected to be
related to the drug. An ADR will usually require the drug to be dis-
continued or the dose reduced.
Side-effect. Any effect caused by a drug other than the intended
therapeutic effect, whether benecial, neutral or harmful. The term
‘side-effect’ is often used interchangeably with ‘ADR’, although the
former usually implies an ADR that occurs during exposure to nor-
mal therapeutic drug concentrations (e.g. vasodilator-induced ankle
oedema).
Hypersensitivity reaction. An ADR that occurs as a result of an
immunological reaction and often after exposure to subtherapeutic
drug concentrations. These include: (1) acute anaphylaxis (p. 183) –
the result of an interaction between drug antigens and immuno-
globulin E (IgE) on mast cells and basophils, resulting in a release
of vasoactive biomolecules (e.g. penicillin, suxamethonium); this is
also known as a type I hypersensitivity reaction. (2) ‘Anaphylactoid’
2.4 Patient-specic factors that inuence pharmacokinetics
Age
Drug metabolism is low in the fetus and newborn, may be enhanced in young
children, and becomes less effective with age
Drug excretion falls with the age-related decline in renal function
Sex
Women have a greater proportion of body fat than men, increasing the volume of
distribution and half-life of lipid-soluble drugs
Body weight
Obesity increases volume of distribution and half-life of lipid-soluble drugs
Patients with higher lean body mass have larger body compartments into which
drugs are distributed and may require higher doses
Liver function
Metabolism of most drugs depends on several cytochrome P450 enzymes that are
impaired in patients with advanced liver disease
Hypoalbuminaemia inuences the distribution of drugs that are highly protein-
bound
Kidney function
Renal disease and the decline in renal function with ageing may lead to drug
accumulation
Gastrointestinal function
Small intestinal absorption of oral drugs may be delayed by reduced gastric
motility
Absorptive capacity of the intestinal mucosa may be reduced in disease (e.g.
Crohn’s or coeliac disease) or after surgical resection
Food
Food in the stomach delays gastric emptying and reduces the rate (but not usually
the extent) of drug absorption
Some food constituents bind to certain drugs and prevent their absorption
Smoking
Tar in tobacco smoke stimulates the oxidation of certain drugs
Alcohol
Regular alcohol consumption stimulates liver enzyme synthesis, while binge
drinking may temporarily inhibit drug metabolism
Drugs
Drug–drug interactions cause marked variation in pharmacokinetics
(see Box 2.12)
42.
20 CLINICALTHERAPEUTICS AND GOOD PRESCRIBING
reactions – these present in a similar manner to acute anaphylaxis,
but are a consequence of non-IgE-mediated degranulation of mast
cells and basophils or direct complement activation (e.g. aspirin,
non-steroidal anti-inammatory drugs, opiates). (3) Types II–IV
hypersensitivity reactions – these occur via other mechanisms, such
as antibody-dependent (IgM or IgG), immune complex-mediated
or cell-mediated pathways (p. 78); examples of such reactions are
listed in Box 2.6.
Drug toxicity. Adverse effects of a drug that occur because the dose
or plasma concentration has risen above the therapeutic range,
either unintentionally or intentionally (drug overdose; see Fig. 2.2 and
p. 224).
Drug misuse. The misuse of recreational or therapeutic drugs that
may lead to addiction or dependence, serious physiological injury
(such as liver damage), psychological harm (abnormal behaviour
patterns, hallucinations, memory loss) or death (p. 1240).
Prevalence of ADRs
ADRs are a common cause of illness, accounting in the UK for approx-
imately 3% of consultations in primary care, 7% of emergency admis-
sions to hospital and affecting around 15% of hospital inpatients. Many
‘disease’ presentations are eventually attributed to ADRs, emphasising
the importance of always taking a careful drug history (Box 2.7). Factors
2.5 Examples of pharmacogenetic variations that inuence drug response
Genetic variant Drug affected Clinical outcome
Pharmacokinetic
Aldehyde dehydrogenase-2 deciency Ethanol Elevated blood acetaldehyde causes facial ushing and increased
heart rate in ~50% of Japanese, Chinese and other Asian
populations
Acetylation Isoniazid, hydralazine, procainamide Increased responses in slow acetylators, up to 50% of some
populations
Oxidation (CYP2D6) Nortriptyline
Codeine
Increased risk of toxicity in poor metabolisers
Reduced responses with slower conversion of codeine to more
active morphine in poor metabolisers, 10% of European populations
Increased risk of toxicity in ultra-fast metabolisers, 3% of
Europeans but 25% of North Africans
Oxidation (CYP2C9) Warfarin Polymorphisms known to inuence dosages
Oxidation (CYP2C19) Clopidogrel
Proguanil
Reduced enzymatic activation results in reduced antiplatelet effect
Reduced efcacy with slower conversion to active cycloguanil in
poor metabolisers
Sulphoxidation Penicillamine Increased risk of toxicity in poor metabolisers
Pseudocholinesterase deciency Suxamethonium (succinylcholine) Decreased drug inactivation leads to prolonged paralysis and
sometimes persistent apnoea requiring mechanical ventilation until
the drug can be eliminated by alternate pathways; occurs in 1 in
1500 people
Pharmacodynamic
Glucose-6-phosphate dehydrogenase (G6PD)
deciency
Oxidant drugs, including
antimalarials (e.g. chloroquine,
primaquine)
Risk of haemolysis in G6PD deciency
Acute intermittent porphyria Enzyme-inducing drugs Increased risk of an acute attack
SLC01B1 polymorphism Statins Increased risk of rhabdomyolysis
HLA-B*5701 polymorphism Abacavir Increased risk of skin hypersensitivity reaction
HLA-B*5801 polymorphism Allopurinol Increased risk of rashes in Han Chinese
HLA-B*1502 polymorphism Carbamazepine Increased risk of serious dermatological reactions
(e.g. Stevens–Johnson syndrome)
Hepatic nuclear factor 1 alpha (HNF1A)
polymorphism
Sulphonylureas Increased sensitivity to the blood glucose-lowering effects
Human epidermal growth factor receptor 2 (HER2)-
positive breast cancer cells
Trastuzumab Increased sensitivity to the inhibitory effects on growth and division
of the target cancer cells
accounting for the rising prevalence of ADRs are the increasing age of
patients, polypharmacy (higher risk of drug interactions), increasing avail-
ability of over-the-counter medicines, increasing use of herbal or tradi-
tional medicines and the increase in medicines available via the Internet
that can be purchased without a prescription from a health-care profes-
sional. Risk factors for ADRs are shown in Box 2.8.
ADRs are important because they reduce quality of life for patients,
reduce adherence to and therefore efcacy of benecial treatments,
cause diagnostic confusion, undermine the condence of patients in
their health-care professional(s) and consume health-care resources.
Retrospective analysis of ADRs has shown that more than half could have
been avoided if the prescriber had taken more care in anticipating the poten-
tial hazards of drug therapy. For example, non-steroidal anti-inammatory
drug (NSAID) use accounts for many thousands of emergency admissions,
gastrointestinal bleeding episodes and a signicant number of deaths. In
many cases, the patients are at increased risk due to their age, interacting
drugs (e.g. aspirin, warfarin) or a past history of peptic ulcer disease. Drugs
that commonly cause ADRs are listed in Box 2.9.
Prescribers and their patients ideally want to know the frequency with
which ADRs occur for a specic drug. Although this may be well char-
acterised for more common ADRs observed in clinical trials, it is less
clear for rarely reported ADRs when the total numbers of reactions and
patients exposed are not known. The words used to describe frequency
43.
Adverse outcomes ofdrug therapy 21
2
who experience type B reactions are generally ‘hyper-susceptible’
because of unpredictable immunological or genetic factors (e.g.
anaphylaxis caused by penicillin, peripheral neuropathy caused by
isoniazid in poor acetylators).
This simple classication has shortcomings, and a more detailed clas-
sication based on dose (see Fig. 2.2), timing and susceptibility (DoTS) is
now used by those analysing ADRs in greater depth (Box 2.10). The AB
classication can be extended as a reminder of some other types of ADR:
Type C (‘chronic/continuous’) ADRs. These occur only after pro-
longed continuous exposure to a drug. Examples include osteopo-
rosis caused by glucocorticoids, retinopathy caused by (hydroxy)
chloroquine and tardive dyskinesia caused by phenothiazines.
Type D (‘delayed’) ADRs. These are delayed until long after drug
exposure, making diagnosis difcult. Examples include malignancies
that may emerge after immunosuppressive treatment post-trans-
plantation (e.g. azathioprine, tacrolimus) and vaginal cancer occur-
ring many years after exposure to diethylstilboestrol.
Type E (‘end-of-treatment’) ADRs. These occur after abrupt drug
withdrawal (see Box 2.3).
2.6 Examples of drug-mediated types I-IV hypersensitivity
reactions1
Type I (mediated by IgE antibodies and mast cell degranulation)
Acute anaphylaxis
Penicillins
Chemotherapy
Monoclonal antibodies
Neuromuscular blocking drugs
Type II (mediated by IgM and IgG antibodies)
Haemolytic anaemia (latency less than 7 days)
Methyldopa
Penicillins
Cephalosporins
NSAIDs
Quinidine/quinine
Neutropenia/agranulocytosis (latency days to weeks)
Antiarrhythmics
Antibiotics
NSAIDs
Antimalarials
Antithyroid drugs
Clozapine
Ticlopidine
Thrombocytopenia (latency 1–2 weeks)
Carbamazepine
Heparin
Penicillins
Cephalosporins
NSAIDs
Quinidine/quinine
Thiazide diuretics
Type III (mediated by immune complex deposition)
Glomerulonephritis, serum sickness, vasculitis
Penicillins
Cephalosporins
Sulphonamides
Type IV (mediated by T cells)
Skin eruptions, including maculopapular rash, lichenoid or pemphigoid-like
reaction, drug rash with eosinophilia and systemic symptoms (DRESS), Stevens–
Johnson syndrome (SJS), toxic epidermal necrolysis (TEN)2
Carbamazepine
Abacavir
Trimethoprim
Dapsone
Drug-induced lupus
Allopurinol
Thiazide diuretics
Hydralazine
2.7 How to take a drug history
Information from the patient (or carer)
Use language that patients will understand (e.g.‘medicines’ rather than ‘drugs’,
which may be mistaken for drugs of abuse) while gathering the following information:
Current prescribed drugs, including formulations (e.g. modied-release tablets),
doses, routes of administration, frequency and timing, duration of treatment
Other medications that are often forgotten (e.g. contraceptives, over-the-counter
drugs, herbal remedies, vitamins)
Drugs that have been taken in the recent past and reasons for stopping them
Previous drug hypersensitivity reactions, their nature and time course (e.g. rash,
anaphylaxis)
Previous ADRs, their nature and time course (e.g. ankle oedema with amlodipine)
Adherence to therapy (e.g. ‘Are you taking your medication regularly?’)
Information from GP medical records and/or pharmacist
Up-to-date list of medications
Previous ADRs
Last order dates for each medication
Inspection of medicines
Drugs and their containers (e.g. blister packs, bottles, vials) should be inspected
for name, dosage and the number of dosage forms taken since dispensed
(ADR = adverse drug reaction)
can be misinterpreted by patients, but widely accepted meanings
include: very common (10% or more), common (1%–10%), uncommon
(0.1%–1%), rare (0.01%–0.1%) and very rare (0.01% or less).
Classication of ADRs
ADRs have traditionally been classied into two major groups:
Type A (‘augmented’) ADRs. These are predictable from the known
pharmacodynamic effects of the drug and are dose-dependent,
common (detected early in drug development) and usually mild.
Examples include constipation caused by opioids, hypotension
caused by antihypertensives and dehydration caused by diuretics.
Type B (‘bizarre’) ADRs. These are not predictable, are not obvi-
ously dose-dependent in the therapeutic range, are rare (remaining
undiscovered until the drug is marketed) and often severe. Patients
2.8 Risk factors for adverse drug reactions
Patient factors
Advanced age (e.g. low physiological reserve)
Gender (e.g. ACE inhibitor-induced cough in women)
Polypharmacy (e.g. drug interactions)
Genetic predisposition (see Box 2.5)
Hypersensitivity/allergy (e.g. β-lactam antibiotics)
Diseases altering pharmacokinetics (e.g. hepatic or renal impairment) or
pharmacodynamic responses (e.g. bladder instability)
Adherence problems (e.g. cognitive impairment)
Drug factors
Steep dose–response curve (e.g. insulin)
Low therapeutic index (e.g. digoxin, cytotoxic drugs)
Prescriber factors
Inadequate understanding of the principles of clinical pharmacology
Inadequate knowledge of the patient
Inadequate knowledge of the prescribed drug
Inadequate instructions and warnings provided to patients
Inadequate monitoring arrangements planned
(ACE = angiotensin-converting enzyme)
2
See also Boxes 27.34 and 27.35.
1
Type I is often known as ‘immediate’ hypersensitivity while types II–IV are known as ‘delayed’
hypersensitivity.
44.
22 CLINICALTHERAPEUTICS AND GOOD PRESCRIBING
A teratogen is a drug with the potential to affect the development of
the fetus in the rst 10 weeks of intrauterine life (e.g. phenytoin, warfa-
rin). The thalidomide disaster in the early 1960s highlighted the risk of
teratogenicity and led to mandatory testing of all new drugs. Congenital
defects in a live infant or aborted fetus should provoke suspicion of an
ADR and a careful exploration of maternal drug exposures (including
self-medication and herbal remedies).
Detecting ADRs – pharmacovigilance
Type A ADRs become apparent early in the development of a new
drug. By the time a new drug is licensed and launched on to a possi-
ble worldwide market, however, a relatively small number of patients
(just several hundred) may have been exposed to it, meaning that
rarer but potentially serious type B ADRs may remain undiscovered.
Pharmacovigilance is the process of detecting (‘signal generation’)
and evaluating ADRs in order to help prescribers and patients to be
better informed about the risks of drug therapy. Drug regulatory agen-
cies may respond to this information by placing restrictions on the
licensed indications, reducing the recommended dose range, adding
special warnings and precautions for prescribers in the product litera-
ture, writing to all health-care professionals or withdrawing the product
from the market.
Voluntary reporting systems allow health-care professionals and
patients to report suspected ADRs to the regulatory authorities. A
good example is the ‘Yellow Card’ scheme that was set up in the UK
in response to the thalidomide tragedy. Reports are analysed to assess
the likelihood that they represent a true ADR (Box 2.11). Although vol-
untary reporting is a continuously operating and effective early-warning
system for previously unrecognised rare ADRs, its weaknesses include
low reporting rates (only 3% of all ADRs and 10% of serious ADRs are
ever reported), an inability to quantify risk (because the ratio of ADRs
to prescriptions is unknown) and the inuence of prescriber awareness
on likelihood of reporting (reporting rates rise rapidly following publicity
about potential ADRs).
More systematic approaches to collecting information on ADRs
include ‘prescription event monitoring’, in which a sample of prescribers
of a particular drug are issued with questionnaires concerning the clini-
cal outcome for their patients and the collection of population statistics.
Many health-care systems routinely collect patient-identiable data on
prescriptions (a surrogate marker of exposure to a drug), health-care
events (e.g. hospitalisation, operations, new clinical diagnoses) and other
2.9 Drugs that are common causes of adverse drug reactions
Drug or drug class Common adverse drug reactions
ACE inhibitors
(e.g. lisinopril)
Renal impairment
Hyperkalaemia
Antibiotics
(e.g. amoxicillin)
Nausea
Diarrhoea
Anticoagulants
(e.g. warfarin, heparin)
Bleeding
Antipsychotics
(e.g. haloperidol)
Falls
Sedation
Delirium
Aspirin Gastrotoxicity (dyspepsia,
gastrointestinal bleeding)
Benzodiazepines
(e.g. diazepam)
Drowsiness
Falls
β-blockers
(e.g. atenolol)
Cold peripheries
Bradycardia
Calcium channel blockers
(e.g. amlodipine)
Ankle oedema
Digoxin Nausea and anorexia
Bradycardia
Diuretics
(e.g. furosemide,
bendroumethiazide)
Dehydration
Electrolyte disturbance (hypokalaemia,
hyponatraemia)
Hypotension
Renal impairment
Insulin Hypoglycaemia
NSAIDs
(e.g. ibuprofen)
Gastrotoxicity (dyspepsia,
gastrointestinal bleeding)
Renal impairment
Opioid analgesics
(e.g. morphine)
Nausea and vomiting
Delirium
Constipation
(ACE = angiotensin-converting enzyme; NSAID = non-steroidal anti-inammatory drug)
2.10 DoTS classication of adverse drug reactions
Category Example
Dose
Below therapeutic dose
In the therapeutic dose range
At high doses
Anaphylaxis with penicillin
Nausea with morphine
Hepatotoxicity with paracetamol
Timing
With the rst dose
Early stages of treatment
On stopping treatment
Signicantly delayed
Anaphylaxis with penicillin
Hyponatraemia with diuretics
Benzodiazepine withdrawal syndrome
Clear-cell cancer with
diethylstilboestrol
Susceptibility See patient factors in Box 2.8
2.11 TREND analysis of suspected adverse drug reactions
Factor Key question Comment
Temporal relationship What is the time
interval between the
start of drug therapy
and the reaction?
Most ADRs occur soon
after starting treatment
and within hours in the
case of anaphylactic
reactions
Re-challenge What happens when
the patient is re-
challenged with the
drug?
Re-challenge is rarely
possible because of the
need to avoid exposing
patients to unnecessary
risk
Exclusion Have concomitant
drugs and other non-
drug causes been
excluded?
ADR is a diagnosis of
exclusion following
clinical assessment and
relevant investigations for
non-drug causes
Novelty Has the reaction been
reported before?
The suspected ADR may
already be recognised
and mentioned in the
SPC approved by the
regulatory authorities
De-challenge Does the reaction
improve when the
drug is withdrawn or
the dose is reduced?
Most, but not all,
ADRs improve on drug
withdrawal, although
recovery may be slow
(ADR = adverse drug reaction; SPC = summary of product characteristics)
45.
Adverse outcomes ofdrug therapy 23
2
clinical data (e.g. haematology, biochemistry). If these records can be
linked, with appropriate safeguards for condentiality and data protec-
tion, they may provide a much more powerful mechanism for assessing
both the harms and benets of drugs.
All prescribers will inevitably see patients experiencing ADRs caused
by prescriptions written by themselves or their colleagues. It is important
that these are recognised early. In addition to the features in Box 2.11,
features that should raise suspicion of an ADR and the need to respond
(by drug withdrawal, dosage reduction or reporting to the regulatory
authorities) include:
concern expressed by a patient that a drug has harmed them
abnormal clinical measurements (e.g. blood pressure, temperature,
pulse, blood glucose and weight) or laboratory results (e.g. abnor-
mal liver or renal function, low haemoglobin or white cell count) while
on drug therapy
new therapy started that could be in response to an ADR (e.g. ome-
prazole, allopurinol, naloxone)
the presence of risk factors for ADRs (see Box 2.8).
Drug interactions
A drug interaction has occurred when the administration of one
drug increases or decreases the benecial or adverse responses to
another drug. Although the number of potential interacting drug com-
binations is very large, only a small number are common in clinical
practice. Important drug interactions are most likely to occur when
the affected drug has a low therapeutic index, steep dose–response
curve, high rst-pass or saturable metabolism, or a single mechanism
of elimination.
Mechanisms of drug interactions
Pharmacodynamic interactions occur when two drugs produce additive,
synergistic or antagonistic effects at the same drug target (e.g. receptor,
enzyme) or physiological system (e.g. electrolyte excretion, heart rate).
These are the most common interactions in clinical practice and some
important examples are given in Box 2.12.
2.12 Common drug interactions
Mechanism Object drug Precipitant drug Result
Pharmaceutical*
Chemical reaction Sodium bicarbonate Calcium gluconate Precipitation of insoluble calcium carbonate
Pharmacokinetic
Reduced absorption Tetracyclines Calcium, aluminium and
magnesium salts
Reduced tetracycline absorption
Reduced protein binding Phenytoin Aspirin Increased unbound and reduced total phenytoin plasma
concentration
Reduced metabolism:
CYP3A4 Amiodarone Grapefruit juice Cardiac arrhythmias because of prolonged QT
interval (p. 418)
Warfarin Clarithromycin Enhanced anticoagulation
CYP2C19 Phenytoin Miconazole Phenytoin toxicity
CYP2D6 Haloperidol Fluoxetine Haloperidol toxicity
Xanthine oxidase Azathioprine Allopurinol Azathioprine toxicity
Monoamine oxidase Catecholamines Monoamine oxidase inhibitors Hypertensive crisis due to monoamine toxicity
Increased metabolism (enzyme
induction)
Ciclosporin St John’s wort Loss of immunosuppression
Reduced renal elimination Lithium Diuretics Lithium toxicity
Methotrexate NSAIDs Methotrexate toxicity
Pharmacodynamic
Direct antagonism at same receptor Opioids Naloxone Reversal of opioid effects used therapeutically
Salbutamol Atenolol Inhibits bronchodilator effect
Direct potentiation in same organ
system
Benzodiazepines Alcohol Increased sedation
ACE inhibitors NSAIDs Increased risk of renal impairment
Indirect potentiation by actions in
different organ systems
Digoxin Diuretics Digoxin toxicity enhanced because of hypokalaemia
Warfarin Aspirin, NSAIDs Increased risk of bleeding because of gastrotoxicity and
antiplatelet effects
Diuretics ACE inhibitors Blood pressure reduction (may be therapeutically
advantageous) because of the increased activity of the
renin–angiotensin system in response to diuresis
*Pharmaceutical interactions are related to the formulation of the drugs and occur before drug absorption.
(ACE = angiotensin-converting enzyme; NSAID = non-steroidal anti-inammatory drug)
46.
24 CLINICALTHERAPEUTICS AND GOOD PRESCRIBING
Pharmacokinetic interactions occur when the administration of a sec-
ond drug alters the concentration of the rst at its site of action. There are
numerous potential mechanisms:
Absorption interactions. Drugs that either delay (e.g. anticholinergic
drugs) or enhance (e.g. prokinetic drugs) gastric emptying inuence
the rate of rise in plasma concentration of other drugs, but not the
total amount of drug absorbed. Drugs that bind to form insoluble
complexes or chelates (e.g. aluminium-containing antacids binding
with ciprooxacin) can reduce drug absorption.
Distribution interactions. Co-administration of drugs that compete
for protein binding in plasma (e.g. phenytoin and diazepam) can
increase the unbound drug concentration, but the effect is usually
short-lived due to increased elimination and hence restoration of the
pre-interaction equilibrium.
Metabolism interactions. Many drugs rely on metabolism by different
isoenzymes of cytochrome P450 (CYP) in the liver. CYP enzyme
inducers (e.g. phenytoin, rifampicin) generally reduce plasma con-
centrations of other drugs, although they may enhance activation
of prodrugs. CYP enzyme inhibitors (e.g. clarithromycin, cimetidine,
grapefruit juice) have the opposite effect. Enzyme induction effects
usually take a few days to manifest because of the need to synthesise
new CYP enzyme, in contrast to the rapid effects of enzyme inhibition.
Excretion interactions. These primarily affect renal excretion. For
example, drug-induced reduction in glomerular ltration rate (e.g.
diuretic-induced dehydration, angiotensin-converting enzyme (ACE)
inhibitors, NSAIDs) can reduce the clearance and increase the
plasma concentration of many drugs, including some with a low
therapeutic index (e.g. digoxin, lithium, aminoglycoside antibiotics).
Less commonly, interactions may be due to competition for a com-
mon tubular organic anion transporter (e.g. methotrexate excretion
may be inhibited by competition with NSAIDs).
Avoiding drug interactions
Drug interactions are increasing as patients are prescribed more medi-
cines (polypharmacy). Prescribers can avoid the adverse consequences
of drug–drug interactions by taking a careful drug history (see Box 2.7)
before prescribing additional drugs, only prescribing for clear indications
and taking special care when prescribing drugs with a narrow therapeu-
tic index (e.g. warfarin). When prescribing an interacting drug is una-
voidable, good prescribers will seek further information and anticipate
the potential risk. This will allow them to provide special warnings for
the patient and arrange for monitoring, either of the clinical effects (e.g.
coagulation tests for warfarin) or of plasma concentration (e.g. digoxin).
Medication errors
A medication error is any preventable event that may lead to inappropri-
ate medication use or patient harm while the medication is in the control
of the health-care professional or patient. Errors may occur in prescrib-
ing, dispensing, preparing solutions, administration or monitoring. Many
ADRs are considered in retrospect to have been ‘avoidable’ with more
care or forethought; in other words, an adverse event considered by one
prescriber to be an unfortunate ADR might be considered by another to
be a prescribing error.
Medication errors are very common. Several thousand medication
orders are dispensed and administered each day in a medium-sized
hospital. Recent UK studies suggest that 7%–9% of hospital prescrip-
tions contain an error, and most are written by junior doctors. Common
prescribing errors in hospitals include omission of medicines (especially
failure to prescribe regular medicines at the point of admission or dis-
charge, i.e. ‘medicines reconciliation’), dosing errors, unintentional pre-
scribing and poor use of documentation (Box 2.13).
Most prescription errors result from a combination of failures by the
individual prescriber and the health-service systems in which they work
(Box 2.14). Health-care organisations increasingly encourage reporting
of errors within a ‘no-blame culture’ so that they can be subject to ‘root
cause analysis’ using human error theory (Fig. 2.5). Prevention is tar-
geted at the factors in Box 2.14 and can be supported by prescribers
communicating and cross-checking with colleagues (e.g. when calculat-
ing doses adjusted for body weight, or planning appropriate monitoring
after drug administration). Prescription errors may also be reduced by
clinical pharmacist support (e.g. to check the patient’s previous medica-
tions and current prescriptions) and electronic prescribing (which avoids
errors due to illegibility or serious dosing mistakes and may be combined
with a clinical decision support system to take account of patient char-
acteristics and drug history, and provide warnings of potential contrain-
dications and drug interactions).
Responding when an error is discovered
All prescribers will make errors. When they do, their rst duty is to pro-
tect the patient’s safety. This will involve a clinical review and the taking
of any steps that will reduce harm (e.g. remedial treatment, monitoring,
recording the event in the notes, informing colleagues). Patients should
be informed if they have been exposed to potential harm. For errors that
do not reach the patient, it is the prescriber’s duty to report them, so that
others can learn from the experience and take the opportunity to reect
on how a similar incident might be avoided in the future.
2.13 Hospital prescribing errors
Error type
Approximate
% of total
Omission on admission 30
Underdose 11
Overdose 8
Strength/dose missing 7
Omission on discharge 6
Administration times incorrect/missing 6
Duplication 6
Product or formulation not specied 4
Incorrect formulation 4
No maximum dose 4
Unintentional prescribing 3
No signature 2
Clinical contraindication 1
Incorrect route 1
No indication 1
Intravenous instructions incorrect/missing 1
Drug not prescribed but indicated 1
Drug continued for longer than needed 1
Route of administration missing 1
Start date incorrect/missing 1
Risk of drug interaction < 0.5
Controlled drug requirements incorrect/missing < 0.5
Daily dose divided incorrectly < 0.5
Signicant allergy < 0.5
Drug continued in spite of adverse effects < 0.5
Premature discontinuation < 0.5
Failure to respond to out-of-range drug level < 0.5
47.
Drug regulation andmanagement 25
2
2.14 Causes of prescribing errors
Systems factors
Working hours of prescribers (and others)
Patient throughput
Professional support and supervision by colleagues
Availability of information (medical records)
Design of prescription forms
Distractions
Availability of decision support
Checking routines (e.g. clinical pharmacy)
Reporting and reviewing of incidents
Prescriber factors
Knowledge
Clinical pharmacology principles
Drugs in common use
Therapeutic problems commonly encountered
Knowledge of workplace systems
Skills
Taking a good drug history
Obtaining information to support prescribing
Communicating with patients
Numeracy and calculations
Prescription writing
Attitudes
Coping with risk and uncertainty
Monitoring of prescribing
Checking routines
Planned
action
Prescribing
Intended
action
Correct
action
Intended
outcome
Unintended
action
Lapse
Slip
Wrong plan selected
(Causes include
poor training and
lack of experience)
Correct plan known
but not executed
(Causes include
workload, time
pressures, distractions)
Prescription not
as intended
Prescriber unaware
Prescription incomplete
or forgotten
Prescriber may remember
Violation
Mistake
Prescription as intended
but written based on
the wrong principles or
lack of knowledge
Prescriber unaware
Deliberate deviations
from standard practice
Prescriber aware
Fig. 2.5 Human error theory. Unintended errors may occur because the prescriber
fails to complete the prescription correctly (a slip; e.g. writes the dose in ‘mg’ not
‘micrograms’) or forgets part of the action that is important for success (a lapse; e.g.
forgets to co-prescribe folic acid with methotrexate); prevention requires the system
to provide appropriate checking routines. Intended errors occur when the prescriber
acts incorrectly due to lack of knowledge (a mistake; e.g. prescribes atenolol for a
patient with known severe asthma because of ignorance about the contraindication);
prevention must focus on training the prescriber.
Drug regulation and management
Given the powerful benecial and potentially adverse effects of drugs, the
production and use of medicines are strictly regulated (e.g. by the Food
and Drug Administration in the United States, Medicines and Healthcare
Products Regulatory Agency in the UK, and Central Drugs Standard
Control Organisation in India). Regulators are responsible for licensing
medicines, monitoring their safety (pharmacovigilance), approving clinical
trials, and inspecting and maintaining standards of drug development
and manufacture.
In addition, because of the high costs of drugs and their adverse effects,
health-care services must prioritise their use in light of the evidence of their
benet and harm, a process referred to as ‘medicines management’.
Drug development and marketing
Naturally occurring products have been used to treat illnesses for
thousands of years and some remain in common use today. Examples
include morphine from the opium poppy (Papaver somniferum), dig-
italis from the foxglove (Digitalis purpurea), curare from the bark of
a variety of species of South American trees, and quinine from the
bark of the Cinchona species. Although plants and animals remain a
source of discovery, the majority of new drugs come from drug dis-
covery programmes that aim to identify small-molecule compounds
with specic interactions with a molecular target that will induce a
predicted biological effect.
The usual pathway for development of these small molecules includes:
identifying a plausible molecular target by investigating pathways in dis-
ease; screening a large library of compounds for those that interact with
the molecular target in vitro; conducting extensive medicinal chemistry to
optimise the properties of lead compounds; testing efcacy and toxicity
of these compounds in vitro and in animals; and undertaking a clinical
development programme (Box 2.15). This process typically takes longer
than 10 years and may cost up to US$2 billion. Manufacturers have a
dened period of exclusive marketing of the drug while it remains pro-
tected by an original patent, typically 10–15 years, during which time
they must recoup the costs of developing the drug. Meanwhile, com-
petitor companies will often produce similar ‘me too’ drugs of the same
class. Once the drug’s patent has expired, ‘generic’ manufacturers may
step in to produce cheaper formulations of the drug. Paradoxically, if a
generic drug is produced by only one manufacturer, the price may actu-
ally rise, sometimes substantially.
2.15 Clinical development of new drugs
Phase I
Healthy volunteers (20–80)
These involve initial single-dose, ‘rst-into-man’ studies, followed by
repeated-dose studies. They aim to establish the basic pharmacokinetic and
pharmacodynamic properties, and short-term safety
Duration: 6–12 months
Phase II
Patients (100–200)
These investigate clinical effectiveness (‘proof of concept’), safety and dose–
response relationship, often with a surrogate clinical endpoint, in the target
patient group to determine the optimal dosing regimen for larger conrmatory
studies
Duration: 1–2 years
Phase III
Patients (100s–1000s)
These are large, expensive clinical trials that conrm safety and efcacy in
the target patient population, using relevant clinical endpoints. They may be
placebo-controlled studies or comparisons with other active compounds
Duration: 1–2 years
Phase IV
Patients (100s–1000s)
These are undertaken after the medicine has been marketed for its rst
indication to evaluate new indications, new doses or formulations, long-term
safety or cost-effectiveness
48.
26 CLINICALTHERAPEUTICS AND GOOD PRESCRIBING
New therapeutic agents
The traditional approach of targeting membrane-bound receptors and
enzymes with small molecules (see Box 2.2) is now giving way to drug
development that focuses on new targets, such as complex second-
messenger systems, cytokines, nucleic acids and cellular networks
(Box 2.16). These require the development of novel therapeutic agents,
which are typically large molecules (e.g. human recombinant antibod-
ies) manufactured by biological processes (‘biologics’) (Fig. 2.6). These
present new challenges for ‘translational medicine’, the discipline of con-
verting scientic discoveries into a useful medicine with a well-dened
benet–risk prole, and also for health-care providers and prescribers.
Manufacturers
There are multiple challenges at every stage of the development and
quality assurance of high-quality biological products. These include
the initial molecular cloning process, development of clone expres-
sion by stable cell lines, purication and characterisation procedures,
performing post-translation modications, ensuring chemical stability,
deploying novel sensitive bio-analytical methods and meeting strin-
gent regulatory expectations. There are also new hurdles to overcome
in the clinical development phase, including denition of the clinical
indication with linked inclusion and exclusion criteria, study design
and appropriate comparator product, selecting the appropriate ther-
apeutic dose based on interpretation of pharmacokinetic/pharma-
codynamic (PK/PD) data, and addressing specic safety concerns
(notably immunogenicity) that arise after exposure to large biological
molecules.
Health services
Biological drugs are often much more expensive than conventional
synthetic molecules because of the complex manufacturing process
outlined above, but also because they often have much narrower indi-
cations for use based on specic molecular proling of the recipients.
This means that the development costs have to be recouped from a rel-
atively smaller patient group. After the patent for the originator product
expires, other manufacturers may develop similar products (‘biosimi-
lars’) that have the same pharmacological actions, but are not com-
pletely identical because of inevitable differences that arise during a
complex manufacturing process (e.g. glycosylation). For that reason,
‘biosimilars’ are not considered to be ‘generic’ medications, although
they are usually considerably cheaper than the originator product.
Nevertheless, the use of expensive biological agents is subject to par-
ticular scrutiny with regard to cost-effectiveness and health services
may put additional requirements in place prior to access (e.g. failure of
conventional treatments).
Prescribers
For prescribers there are some important considerations when assessing
the balance of benecial and adverse effects of biologics, for example, the
use of recombinant antibodies for autoimmune inammatory conditions.
Drugs such as iniximab or adalimumab carry a signicantly increased
risk of infection when compared to classical disease-modifying anti-rheu-
matic drugs (cDMARDs) and this risk is increased by age, co-morbidities
and concomitant use of other immunosuppressant drugs (e.g. gluco-
corticoids). Patients should be protected with appropriate vaccinations
(e.g. inuenza, pneumococcus) and treatment is contra-indicated in the
presence of active infection (e.g. hepatitis B, tuberculosis). The increased
immunogenicity of large biological molecules has two important conse-
quences. First, there is an increased chance of immediate and delayed
hypersensitivity reactions (see Box 2.6). Second, the development of anti-
drug antibodies can precipitate loss of effect. A recent Cochrane review
of exposure to nine commonly used biologics for up to 5 years concluded
that there was a 5% absolute increase in all adverse effects combined, a
1% absolute increase in risk of serious infection, but little or no increased
risk of cancer or other serious outcomes.
Licensing new medicines
New drugs are given a ‘market authorisation’, based on the evidence of
quality, safety and efcacy presented by the manufacturer. The regulator
not only will approve the drug, but also will take great care to ensure
that the accompanying information reects the evidence that has been
presented. The summary of product characteristics (SPC), or ‘label’,
provides detailed information about indications, dosage, adverse effects,
warnings and monitoring requirements. If approved, drugs can be made
available with different levels of restriction:
Controlled drug (CD). These drugs are subject to strict legal controls
on supply and possession, usually due to their abuse potential (e.g.
opioid analgesics).
Prescription-only medicine (PoM). These are available only from a
pharmacist and can be supplied only if prescribed by an appropriate
practitioner.
Pharmacy (P). These are available only from a pharmacist, but can
be supplied without a prescription.
General sales list (GSL). These medicines may be bought ‘over the
counter’ (OTC) from any shop and without a prescription.
Although the regulators take great care to agree the exact indica-
tions for prescribing a medicine, based on the evidence provided by the
manufacturer, there are some circumstances in which prescribers may
2.16 Novel therapeutic alternatives to conventional small-molecule drugs
Approaches Therapeutic indications Challenges
Monoclonal antibodies
Targeting of receptors or other molecules with
relatively specic antibodies
Cancer
Chronic inammatory diseases (e.g. rheumatoid
arthritis, inammatory bowel disease)
Selectivity of action
Complex manufacturing process
Small interfering RNA (siRNA)
Inhibition of gene expression Macular degeneration Delivery to target
Gene therapy
Delivery of modied genes that supplement or alter
host DNA
Cystic brosis
Cancer
Cardiovascular disease
Delivery to target
Adverse effects of delivery vector (e.g. virus)
Stem cell therapy
Stem cells differentiate and replace damaged host
cells
Parkinson’s disease
Spinal cord injury
Ischaemic heart disease
Delivery to target
Immunological compatibility
Long-term effects unknown
49.
Drug regulation andmanagement 27
2
Monoclonal antibodies
e.g. infliximab, adalimumab
150000 Da
Small-molecule drugs
e.g. allopurinol
136 Da
Other proteins
e.g. interferons, enzymes,
cytokines
10000 – 500000 Da
A B
Polypeptides
e.g. insulin, glucagon
5000 – 50000 Da
α
β
7 20
7 19
s
s
s
s
s
s
Blood products
e.g. red blood cells, white cells,
platelets
1000000000000 Da
C D
Gene therapies
e.g. alipogene tiparvovec
1000000 Da
Antisense oligonucleotides
e.g. nusinersin, inotersin
10000 Da
mRNA
E F
Cell therapy
e.g. CAR-T cells, embryonic
stem cells
1000000000000 Da
Vaccines
e.g. Haemophilus, polio
50000000 Da
G H
Fig. 2.6 The mechanism of action and molecular weight of various types of biologic therapies.
Cell therapies include injection of cells that replace damaged cells (e.g. embryonic stem cells) or manipulated immune cells that are targeted
Vaccines are used to stimulate the immune system to respond to and remember specic antigens associated with viral or bacterial
infections. (CAR-T cells = chimeric antigen recipient T cells; Da = Daltons; mRNA = messenger ribonucleic acid; TNF = tumour necrosis factor)
direct its use outside the terms stated in the SPC (‘off-label’ prescribing).
Common situations where this might occur include prescribing outside
the approved age group (e.g. prescribing for children) or using an alter-
native formulation (e.g. administering a medicine provided in a solid form
as an oral solution). Other important examples might include prescribing
for an indication for which there are no approved medicines or where all
of the approved medicines have caused unacceptable adverse effects.
Occasionally, medicines may be prescribed when there is no marketing
authorisation in the country of use. Examples include when a medicine
licensed in another country is imported for use for an individual patient
(‘unlicensed import’) or when a patient requires a specic preparation of
a medicine to be manufactured (‘unlicensed special’). When prescribing
is ‘off-label’ or ‘unlicensed’, there is an increased requirement for pre-
scribers to be able to justify their actions and to inform and agree the
decision with the patient.
Drug marketing
The marketing activities of the pharmaceutical industry are well resourced
and are important in the process of recouping the massive costs of drug
development. In some countries, such as the United States, it is possible
to promote a new drug by direct-to-consumer advertising, although this
is illegal in the UK and countries of the European Union. A major focus
is on promotion to prescribers via educational events, sponsorship of
meetings, advertisements in journals, involvement with opinion leaders
and direct contact by company representatives. Such largesse has the
potential to cause signicant conicts of interest and might tempt pre-
scribers to favour one drug over another, even in the face of evidence on
effectiveness or cost-effectiveness.
Managing the use of medicines
Many medicines meet the three key regulatory requirements of quality,
safety and efcacy. Although prescribers are legally entitled to prescribe
any of them, it is desirable to limit the choice so that treatments for spe-
cic diseases can be focused on the most effective and cost-effective
options, prescribers (and patients) gain familiarity with a smaller number
of medicines, and pharmacies can concentrate stocks on them.
The process of ensuring optimal use of available medicines is known
as ‘medicines management’ or ‘quality use of medicines’. It involves
careful evaluation of the evidence of benet and harm from using the
medicine, an assessment of cost-effectiveness and support for pro-
cesses to implement the resulting recommendations. These activities
usually involve both national (e.g. National Institute for Health and Care
Excellence (NICE) in the UK) and local organisations (e.g. drug and ther-
apeutics committees).
50.
28 CLINICALTHERAPEUTICS AND GOOD PRESCRIBING
Evaluating evidence
Drugs are often evaluated in high-quality randomised controlled trials,
the results of which can be considered in systematic reviews (Fig. 2.7).
Ideally, data are available not only for comparison with placebo, but also
for ‘head-to-head’ comparison with alternative therapies. Trials are con-
ducted in selected patient populations and so may not be representative
of every clinical scenario; therefore, extrapolation to individual patients
is not always straightforward. Other subtle bias may be introduced
because of the sources of funding (e.g. pharmaceutical industry) and
the interests of the investigators in being involved in research that has a
‘positive’ impact. These biases may be manifest in the way the trials are
conducted or in how they are interpreted or reported. A common exam-
ple of the latter is the difference between relative and absolute risk of clin-
ical events reported in prevention trials. If a clinical event is encountered
in the placebo arm at a rate of 1 in 50 patients (2%), but only 1 in 100
patients (1%) in the active treatment arm, then the impact of treatment
can be described as either a 50% relative risk reduction or 1% absolute
risk reduction. Although the former sounds more impressive, it is the
latter that is of more importance to the individual patient. It means that
the number of patients that needed to be treated (NNT) for 1 to benet
(compared to placebo) was 100. This illustrates how large clinical trials of
new medicines can produce highly statistically signicant and impressive
relative risk reductions and still predict a very modest clinical impact.
Evaluating cost-effectiveness
New drugs often represent an incremental improvement over the current
standard of care, but are usually more expensive. Health-care budgets are
limited in every country and so it is impossible to fund all new medicines.
This means that very difcult nancial decisions have to be taken with
due regard to the principles of ethical justice. The main approach taken is
cost-effectiveness analysis (CEA), where a comparison is made between
the relative costs and outcomes of different courses of action. CEA is usu-
ally expressed as a ratio where the denominator is a gain in health and the
numerator is the cost associated with the health gain. A major challenge is
to compare the value of interventions for different clinical outcomes. One
method is to calculate the quality-adjusted life years (QALYs) gained if the
new drug is used rather than standard treatment. This analysis involves
estimating the ‘utility’ of various health states between 1 (perfect health)
and 0 (dead). If the additional costs and any savings are known, then it is
possible to derive the incremental cost-effectiveness ratio (ICER) in terms
of cost/QALY. These principles are exemplied in Box 2.17. There are,
however, inherent weaknesses in this kind of analysis: it usually depends
on modelling future outcomes well beyond the duration of the clinical trial
data; it assumes that QALYs gained at all ages are of equivalent value;
and the appropriate standard care against which the new drug should be
compared is often uncertain.
These pharmacoeconomic assessments are challenging and
resource-intensive, and are undertaken at national level in most coun-
tries, e.g. in the UK by NICE.
Implementing recommendations
Many recommendations about drug therapy are included in clinical guide-
lines written by an expert group after systematic review of the evidence.
Guidelines provide recommendations rather than obligations for prescribers
and are helpful in promoting more consistent and higher-quality prescrib-
ing. They are often written without concern for cost-effectiveness, however,
and may be limited by the quality of available evidence. Guidelines cannot
anticipate the extent of the variation between individual patients who may,
for example, have unexpected contraindications to recommended drugs
or choose different priorities for treatment. When deviating from respected
national guidance, prescribers should be able to justify their practice.
Additional recommendations for prescribing are often implemented
locally or imposed by bodies responsible for paying for health care. Most
health-care units have a drug and therapeutics committee (or equivalent)
comprised of medical staff, pharmacists and nurses, as well as manag-
ers (because of the implications of the committee’s work for governance
and resources). This group typically develops local prescribing policy and
guidelines, maintains a local drug formulary and evaluates requests to
use new drugs. The local formulary contains a more limited list than any
national formulary (e.g. British National Formulary) because the latter lists
all licensed medicines that can be prescribed legally, while the former
contains only those that the health-care organisation has approved for
local use. The local committee may also be involved, with local special-
ists, in providing explicit protocols for management of clinical scenarios.
Prescribing in practice
Decision-making in prescribing
Prescribing should be based on a rational approach to a series of chal-
lenges (see Box 2.1).
Odds ratio
Favours treatment
0.1 0.2 0.5 1 2 5 10
Favours placebo
Fig. 2.7 Systematic review of the evidence from randomised controlled
clinical trials. This forest plot shows the effect of warfarin compared with placebo
on the likelihood of stroke in patients with atrial brillation in ve randomised
controlled trials that passed the quality criteria required for inclusion in a meta-
analysis. For each trial, the purple box is proportionate to the number of participants.
The tick marks show the mean odds ratio and the black lines indicate its 95%
condence intervals. Note that not all the trials showed statistically signicant effects
(i.e. the condence intervals cross 1.0). However, the meta-analysis, represented
by the black diamond, conrms a highly signicant statistical benet. The overall
odds ratio is approximately 0.4, indicating a mean 60% risk reduction with warfarin
treatment in patients with the characteristics of the participants in these trials.
2.17 Cost-effectiveness analysis
A clinical trial lasting 2 years compares two interventions for the treatment of colon
cancer:
Treatment A: standard treatment, cost £1000/year, oral therapy
Treatment B: new treatment, cost £6000/year, monthly intravenous infusions,
often followed by a week of nausea.
The new treatment (B) signicantly increases the average time to progression
(18 months versus 12 months) and reduces overall mortality (40% versus
60%). The health economist models the survival curves from the trial in order to
undertake a cost–utility analysis and concludes that:
Intervention A: allows an average patient to live for 2 extra years at a utility
0.7 = 1.4 QALYs (cost £2000)
Intervention B: allows an average patient to live for 3 extra years at a utility
0.6 = 1.8 QALYs (cost £18000).
The health economists conclude that treatment B provides an extra 0.4 QALYs
at an extra cost of £16000, meaning that the ICER = £40000/QALY. They
recommend that the new treatment should not be funded on the basis that their
threshold for cost acceptability is £30000/QALY.
(ICER = incremental cost-effectiveness ratio; QALY = quality-adjusted life year)
51.
Prescribing in practice 29
2
Making a diagnosis
Ideally, prescribing should be based on a conrmed diagnosis but, in
reality, many prescriptions are based on the balance of probability, taking
into account the differential diagnosis (e.g. proton pump inhibitors for
post-prandial retrosternal discomfort).
Establishing the therapeutic goal
The goals of treatment are usually clear, particularly when relieving symp-
toms (e.g. pain, nausea, constipation). Sometimes the goal is less obvi-
ous to the patient, especially when aiming to prevent future events (e.g.
ACE inhibitors to prevent hospitalisation and extend life in chronic heart
failure). Prescribers should be clear about the therapeutic goal against
which they will judge success or failure of treatment. It is also important
to establish that the value placed on this goal by the prescriber is shared
by the patient (concordance).
Choosing the therapeutic approach
For many clinical problems, drug therapy is not absolutely mandated.
Having taken the potential benets and harms into account, prescribers
must consider whether drug therapy is in the patient’s interest and is
preferred to no treatment or one of a range of alternatives (e.g. physio-
therapy, psychotherapy, surgery). Assessing the balance of benet and
harm is often complicated and depends on various features associated
with the patient, disease and drug (Box 2.18).
Choosing a drug
For most common clinical indications (e.g. type 2 diabetes, depression),
more than one drug is available, often from more than one drug class.
Although prescribers often have guidance about which represents the
rational choice for the average patient, they still need to consider whether
this is the optimal choice for the individual patient. Certain factors may
inuence the choice of drug:
Absorption
Patients may nd some formulations easier to swallow than others or
may be vomiting and require a drug available for parenteral administration.
Distribution
Distribution of a drug to a particular tissue sometimes dictates choice
(e.g. tetracyclines and rifampicin are concentrated in the bile, and linco-
mycin and clindamycin in bones).
Metabolism
Drugs that are extensively metabolised should be avoided in severe liver
disease (e.g. opioid analgesics).
Excretion
Drugs that depend on renal excretion for elimination (e.g. digoxin, amino-
glycoside antibiotics) should be avoided in patients with impaired renal
function if suitable alternatives exist.
Efcacy
Prescribers normally choose drugs with the greatest efcacy in achieving
the goals of therapy (e.g. proton pump inhibitors rather than H2
-receptor
antagonists). It may be appropriate, however, to compromise on efcacy
if other drugs are more convenient, safer to use or less expensive.
Avoiding adverse effects
Prescribers should be wary of choosing drugs that are more likely to
cause adverse effects (e.g. cephalosporins rather than alternatives
for patients allergic to penicillin) or worsen coexisting conditions (e.g.
β-blockers as treatment for angina in patients with asthma).
Features of the disease
This is most obvious when choosing antibiotic therapy, which should be
based on the known or suspected sensitivity of the infective organism.
Severity of disease
The choice of drug should be appropriate to disease severity (e.g. par-
acetamol for mild pain, morphine for severe pain).
Coexisting disease
This may be either an indication or a contraindication to therapy.
Hypertensive patients might be prescribed a β-blocker if they also have
left ventricular impairment, but not if they have asthma.
Avoiding adverse drug interactions
Prescribers should avoid giving combinations of drugs that might inter-
act, either directly or indirectly (see Box 2.12).
Patient adherence to therapy
Prescribers should choose drugs with a simple dosing schedule or eas-
ier administration (e.g. the ACE inhibitor lisinopril once daily rather than
captopril 3 times daily for hypertension).
Cost
Prescribers should choose the cheaper drug (e.g. a generic or biosimilar) if
two drugs are of equal efcacy and safety. Even if cost is not a concern for
the individual patient, it is important to remember that unnecessary expendi-
ture will ultimately limit choices for other prescribers and patients. Sometimes
a more costly drug may be appropriate (e.g. if it yields improved adherence).
Genetic factors
There are already a small number of examples where genotype inu-
ences the choice of drug therapy (see Box 2.5).
Choosing a dosage regimen
Prescribers have to choose a dose, route and frequency of administra-
tion (dosage regimen) to achieve a steady-state drug concentration that
provides sufcient exposure of the target tissue without producing toxic
effects. Manufacturers draw up dosage recommendations based on
average observations in many patients, but the optimal regimen that will
maximise the benet to harm ratio for an individual patient is never cer-
tain. Rational prescribing involves treating each prescription as an exper-
iment and gathering sufcient information to amend it if necessary. There
are some general principles that should be followed, as described below.
Dose titration
Prescribers should generally start with a low dose and titrate this slowly
upwards as necessary. This cautious approach is particularly important if the
patient is likely to be more sensitive to adverse pharmacodynamic effects
(e.g. delirium or postural hypotension in the elderly), if there may be altered
pharmacokinetic handling (e.g. renal or hepatic impairment) and when using
drugs with a low therapeutic index (e.g. benzodiazepines, lithium, digoxin).
However, there are some exceptions. Some drugs must achieve therapeu-
tic concentration quickly because of the clinical circumstance (e.g. antibiot-
ics, glucocorticoids, carbimazole). When early effect is important, but there
may be a delay in achieving steady state because of a drug’s long half-life
(e.g. digoxin, warfarin, amiodarone), an initial loading dose is given prior to
establishing the appropriate maintenance dose (see Fig. 2.4).
2.18 Factors to consider when balancing benets and harms
of drug therapy
Seriousness of the disease or symptom
Efcacy of the drug
Seriousness of potential adverse effects
Likelihood of adverse effects
Efcacy of alternative drugs or non-drug therapies
Safety of alternative drugs or non-drug therapies
52.
30 CLINICALTHERAPEUTICS AND GOOD PRESCRIBING
If adverse effects occur, the dose should be reduced or an alternative
drug prescribed; in some cases, a lower dose may sufce if it can be
combined with another synergistic drug (e.g. the immunosuppressant
azathioprine reduces glucocorticoid requirements in patients with inam-
matory disease). It is important to remember that the shape of the dose–
response curve (see Fig. 2.2) means that higher doses may produce
little added therapeutic effect and might increase the chances of toxicity.
Route
There are many reasons for choosing a particular route of administration
(Box 2.19).
Frequency
Frequency of doses is usually dictated by a manufacturer’s recommen-
dation. Less frequent doses are more convenient for patients, but result
in greater uctuation between peaks and troughs in drug concentration
(see Fig. 2.4). This is relevant if the peaks are associated with adverse
effects (e.g. dizziness with antihypertensives) or the troughs are associ-
ated with troublesome loss of effect (e.g. anti-Parkinsonian drugs). These
problems can be tackled either by splitting the dose or by employing a
modied-release formulation, if available.
Timing
For many drugs the time of administration is unimportant. There are
occasionally pharmacokinetic or therapeutic reasons, however, for giving
drugs at particular times (Box 2.20).
Formulation
For some drugs there is a choice of formulation, some for use by different
routes. Some are easier to ingest, particularly by children (e.g. elixirs). The
formulation is important when writing repeat prescriptions for drugs with
a low therapeutic index that come in different formulations (e.g. lithium,
phenytoin, theophylline). Even if the prescribed dose remains constant,
an alternative formulation may differ in its absorption and bioavailabil-
ity, and hence plasma drug concentration. These are examples of the
small number of drugs that should be prescribed by specic brand name
rather than ‘generic’ international non-proprietary name (INN).
Duration
Some drugs require a single dose (e.g. thrombolysis post-myocardial
infarction), while for others the duration of the course of treatment is cer-
tain at the outset (e.g. antibiotics). For most, the duration will be largely
at the prescriber’s discretion and will depend on response and disease
progression (e.g. analgesics, antidepressants). For many, the treatment
will be long-term (e.g. insulin, antihypertensives, levothyroxine).
Involving the patient
Patients should, whenever possible, be engaged in making choices
about drug therapy. Their beliefs and expectations affect the goals of
therapy and help in judging the acceptable benet/harm balance when
selecting treatments. Very often, patients may wish to defer to the pro-
fessional expertise of the prescriber. Nevertheless, they play key roles in
adherence to therapy and in monitoring treatment, not least by providing
early warning of adverse events. It is important for them to be provided
with the necessary information to understand the choice that has been
made, what to expect from the treatment, and any measurements that
must be undertaken (Box 2.21).
A major drive to include patients has been the recognition that up to
half of the drug doses for chronic preventative therapy are not taken.
This is often termed ‘non-compliance’, but is more appropriately called
‘non-adherence’, to reect a less paternalistic view of the doctor–patient
relationship; it may or may not be intentional. Non-adherence to the dose
regimen reduces the likelihood of benets to the patient and can be costly
in terms of wasted medicines and unnecessary health-care episodes. An
important reason may be lack of concordance with the prescriber about
the goals of treatment. A more open and shared decision-making process
might resolve any misunderstandings at the outset and foster improved
adherence, as well as improved satisfaction with health-care services and
condence in prescribers. Considerable efforts are now made to help
patients to access the reliable information they require to engage more
fully with clinicians. Patient-focused websites and leaets provided by
national services, local health-care providers and charities are increasingly
supplementing the Patient Information Leaet (PIL) approved by the regu-
latory authorities and supplied with all medicines. Fully engaging patients
in shared decision-making is sometimes constrained by various factors,
such as limited consultation time, language barriers and challenges in
communicating complex numerical data.
Writing the prescription
The culmination of the planning described above is writing an accurate
and legible prescription so that the drug will be dispensed and adminis-
tered as planned (see ‘Writing prescriptions’ below).
Monitoring treatment effects
Rational prescribing involves monitoring for the benecial and adverse
effects of treatment so that the balance remains in favour of a positive
outcome (see ‘Monitoring drug therapy’ below).
Stopping drug therapy
It is also important to review long-term treatment at regular intervals
to assess whether continued treatment is required. Elderly patients
are keen to reduce their medication burden and are often prepared to
compromise on the original goals of long-term preventative therapy to
achieve this.
2.19 Factors inuencing the route of drug administration
Reason Example
Only one route possible Dobutamine (IV)
Gliclazide (oral)
Patient adherence Phenothiazines and thioxanthenes (2
weekly IM depot injections rather than
daily tablets, in schizophrenia)
Poor absorption Furosemide (IV rather than oral, in
severe heart failure)
Rapid action Haloperidol (IM rather than oral, in
acute behavioural disturbance)
Vomiting Phenothiazines (PR or buccal rather
than oral, in nausea)
Avoidance of rst-pass
metabolism
Glyceryl trinitrate (SL, in angina
pectoris)
Certainty of effect Amoxicillin (IV rather than oral, in acute
chest infection)
Direct access to the site of
action (avoiding unnecessary
systemic exposure)
Bronchodilators (INH rather than oral,
in asthma)
Local application of drugs to skin,
eyes etc.
Ease of access Diazepam (PR, if IV access is difcult
in status epilepticus)
Adrenaline (epinephrine) (IM, if IV
access is difcult in acute anaphylaxis)
Comfort Morphine (SC rather than IV in terminal
care)
(IM = intramuscular; INH = by inhalation; IV = intravenous; PR = per rectum; SC =
subcutaneous; SL = sublingual)
53.
Prescribing in practice 31
2
Prescribing in special circumstances
Prescribing for patients with renal disease
Patients with renal impairment are identied by a low estimated glomer-
ular ltration rate (eGFR <60mL/min/1.73m2
), based on their serum cre-
atinine, age, sex and ethnic group (see Box 18.1). This group includes
a large proportion of elderly patients. If a drug (or its active metabolites)
is eliminated predominantly by the kidneys, it will tend to accumulate
and so the maintenance dose must be reduced. For some drugs, renal
impairment makes patients more sensitive to their adverse pharmacody-
namic effects. Examples of drugs that require extra caution in patients
with renal disease are listed in Box 2.22.
Prescribing for patients with hepatic disease
The liver has a large capacity for drug metabolism and hepatic insuf-
ciency has to be advanced before drug dosages need to be modi-
ed. Patients who may have impaired metabolism include those with
jaundice, ascites, hypoalbuminaemia, malnutrition or encephalopathy.
Hepatic drug clearance may also be reduced in acute hepatitis, in
hepatic congestion due to cardiac failure and in the presence of intra-
hepatic arteriovenous shunting (e.g. in hepatic cirrhosis). There are no
good tests of hepatic drug-metabolising capacity or of biliary excre-
tion, so dosage should be guided by the therapeutic response and
careful monitoring for adverse effects. The presence of liver disease
also increases the susceptibility to adverse pharmacological effects of
drugs. Some drugs that require extra caution in patients with hepatic
disease are listed in Box 2.22.
Prescribing for older patients
The issues around prescribing in old age are discussed in Box 2.23.
Prescribing for women who are pregnant or
breastfeeding
Prescribing in pregnancy should be avoided if possible to minimise the
risk of adverse effects in the fetus. Drug therapy in pregnancy may,
however, be required either for a pre-existing problem (e.g. epilepsy,
asthma, hypothyroidism) or for problems that arise during pregnancy
(e.g. morning sickness, anaemia, prevention of neural tube defects,
gestational diabetes, hypertension). About 35% of women take drug
2.20 Factors inuencing the timing of drug therapy
Drug Recommended timing Reasons
Diuretics (e.g. furosemide) Once in the morning Night-time diuresis undesirable
Statins (e.g. simvastatin) Once at night HMG CoA reductase activity is greater at night
Antidepressants (e.g. amitriptyline) Once at night Allows adverse effects to occur during sleep
Salbutamol Before exercise Reduces symptoms in exercise-induced asthma
Glyceryl trinitrate When required Relief of acute symptoms only
Paracetamol
Regular nitrate therapy (e.g. isosorbide
mononitrate)
Eccentric dosing regimen (e.g. twice daily at 8 a.m.
and 2 p.m.)
Reduces development of nitrate tolerance by
allowing drug-free period each night
Aspirin With food Minimises gastrotoxic effects
Alendronate Once in the morning before breakfast, sitting upright Minimises risk of oesophageal irritation
Tetracyclines 2 hours before or after food or antacids Divalent and trivalent cations chelate tetracyclines,
preventing absorption
Hypnotics (e.g. temazepam) Once at night Maximises therapeutic effect and minimises daytime
sedation
Antihypertensive drugs (e.g. amlodipine) Once in the morning Blood pressure is higher during the daytime
(HMG CoA = 3-hydroxy-3-methylglutaryl-coenzyme A)
2.21 What patients need to know about their medicines*
Knowledge Comment
The reason for taking the medicine Reinforces the goals of therapy
How the medicine works
How to take the medicine May be important for the effectiveness (e.g. inhaled salbutamol in asthma) and safety (e.g. alendronate for
osteoporosis) of treatment
What benets to expect May help to support adherence or prompt review because of treatment failure
What adverse effects might occur Discuss common and mild effects that may be transient and might not require discontinuation
Mention rare but serious effects that might inuence the patient’s consent
Precautions that improve safety Explain symptoms to report that might allow serious adverse effects to be averted, monitoring that will be required and
potentially important drug–drug interactions
When to return for review This will be important to enable monitoring
*Many medicines are provided with patient information leaets, which the patient should be encouraged to read.
54.
32 CLINICALTHERAPEUTICS AND GOOD PRESCRIBING
therapy at least once during pregnancy and 6% take drug therapy
during the rst trimester (excluding iron, folic acid and vitamins). The
most commonly used drugs are simple analgesics, antibacterial drugs
and antacids. Some considerations when prescribing in pregnancy are
listed in Box 2.24.
Drugs that are excreted in breast milk may cause adverse effects in the
baby. Prescribers should always consult the SPC for each drug or a relia-
ble formulary when treating a pregnant woman or breastfeeding mother.
Writing prescriptions
A prescription is a means by which a prescriber communicates the
intended plan of treatment to the pharmacist who dispenses a med-
icine and to a nurse or patient who administers it. It should be pre-
cise, accurate, clear and legible. The two main kinds of prescription
are those written, dispensed and administered in hospital and those
written in primary care (in the UK by a GP), dispensed at a community
pharmacy and self-administered by the patient. The information sup-
plied must include:
the date
the identication details of the patient
the name of the drug
the formulation
the dose
the frequency of administration
the route and method of administration
the amount to be supplied (primary care only)
instructions for labelling (primary care only)
the prescriber’s signature.
Prescribing in hospital
Although primary care prescribing is increasingly electronic, many hos-
pital prescriptions continue to be based around the prescription and
administration record (the ‘drug chart’) (Fig. 2.8). A variety of charts are
in use and prescribers must familiarise themselves with the local version.
Most contain the following sections:
Basic patient information: will usually include name, age, date of
birth, hospital number and address. These details are often ‘lled
in’ using a sticky addressograph label, but this increases the risk of
serious error.
Previous adverse reactions/allergies: communicates important
patient safety information based on a careful drug history and/or the
medical record.
Other medicines charts: notes any other hospital prescription doc-
uments that contain current prescriptions being received by the
patient (e.g. anticoagulants, insulin, oxygen, uids).
Once-only medications: for prescribing medicines to be used
infrequently, such as single-dose prophylactic antibiotics and other
pre-operative medications.
2.22 Some drugs that require extra caution in patients with
renal or hepatic disease
Kidney disease Liver disease
Pharmacodynamic effects enhanced
ACE inhibitors and ARBs (renal
impairment, hyperkalaemia)
Metformin (lactic acidosis)
Spironolactone (hyperkalaemia)
NSAIDs (impaired renal function)
Sulphonylureas (hypoglycaemia)
Insulin (hypoglycaemia)
Warfarin (increased anticoagulation
because of reduced clotting factor
synthesis)
Metformin (lactic acidosis)
Chloramphenicol (bone marrow
suppression)
NSAIDs (gastrointestinal bleeding,
uid retention)
Sulphonylureas (hypoglycaemia)
Benzodiazepines (coma)
Pharmacokinetic handling altered (reduced clearance)
Aminoglycosides (e.g. gentamicin)
Vancomycin
Other antibiotics (e.g. ciprooxacin)
Digoxin
Lithium
Atenolol
Allopurinol
Cephalosporins
Methotrexate
Opioids (e.g. morphine)
Phenytoin
Rifampicin
Propranolol
Warfarin
Diazepam
Lidocaine
Opioids (e.g. morphine)
(ACE = angiotensin-converting enzyme; ARB = angiotensin receptor blocker; NSAID = non-
steroidal anti-inammatory drug)
2.23 Prescribing in old age
Reduced drug elimination: partly due to impaired renal function.
Increased sensitivity to drug effects: notably in the brain (leading to sedation
or delirium) and as a result of comorbidities.
More drug interactions: largely as a result of polypharmacy.
Lower starting doses and slower dose titration: often required, with careful
monitoring of drug effects.
Drug adherence: may be poor because of cognitive impairment, difculty
swallowing (dry mouth) and complex polypharmacy regimens. Supplying
medicines in pill organisers (e.g. dosette boxes or calendar blister packs),
providing automatic reminders, and regularly reviewing and simplifying the drug
regimen can help.
Some drugs that require extra caution, and their mechanisms
Digoxin: increased sensitivity of Na+
/K+
pump; hypokalaemia due to
diuretics; renal impairment favours accumulation→increased risk of toxicity.
Antihypertensive drugs: reduced baroreceptor function→increased risk of
postural hypotension.
Antidepressants, hypnotics, sedatives, tranquillisers: increased sensitivity of
the brain; reduced metabolism→increased risk of toxicity.
Warfarin: increased tendency to falls and injury and to bleeding from intra-
and extracranial sites; increased sensitivity to inhibition of clotting factor
synthesis→increased risk of bleeding.
Amitriptyline, diltiazem, lidocaine, metoprolol, morphine, propranolol,
theophylline: metabolism reduced→increased risk of toxicity.
Non-steroidal anti-inammatory drugs: poor renal function→increased risk
of renal impairment; susceptibility to gastrotoxicity→increased risk of upper
gastrointestinal bleeding.
2.24 Prescribing in pregnancy
Teratogenesis: a potential risk, especially when drugs are taken between 2
and 8 weeks of gestation (4–10 weeks from last menstrual period). Common
teratogens include retinoids (e.g. isotretinoin), cytotoxic drugs, angiotensin-
converting enzyme inhibitors, antiepileptics (e.g. sodium valproate) and warfarin. If
there is inadvertent exposure, then the timing of conception should be established,
counselling given and investigations undertaken for fetal abnormalities.
Adverse fetal effects in late gestation: e.g. tetracyclines may stain growing
teeth and bones; sulphonamides displace fetal bilirubin from plasma proteins,
potentially causing kernicterus; opioids given during delivery may be associated
with respiratory depression in the neonate.
Altered maternal pharmacokinetics: extracellular uid volume and Vd
increase. Plasma albumin falls but other binding globulins (e.g. for thyroid and
steroid hormones) increase. Glomerular ltration increases by approximately
70%, enhancing renal clearance. Placental metabolism contributes to increased
clearance, e.g. of levothyroxine and glucocorticoids. The overall effect is a fall in
plasma concentration of many drugs.
In practice
Avoid any drugs unless the risk:benet analysis is in favour of treating
(usually the mother).
Use drugs for which there is some record of safety in humans.
Use the lowest dose for the shortest time possible.
Choose the least harmful drug if alternatives are available.
55.
Prescribing in practice 33
2
Regular medications: for prescribing medicines to be taken for a
number of days or continuously, such as a course of antibiotics or
antihypertensive drugs.
‘As required’ medications: for prescribing for symptomatic relief,
usually to be administered at the discretion of the nursing staff (e.g.
antiemetics, analgesics).
Prescribers should be aware of the risks of prescription error (Box 2.25
and see Box 2.14), ensure they have considered the rational basis for their
prescribing decision and then follow the guidance illustrated in Figure 2.8 in
order to write the prescription. It is a basic principle that a prescription will
be followed by a judgement as to its success or failure and any appropriate
changes made (e.g. altered dosage, discontinuation or substitution).
Hospital discharge (‘to take out’) medicines
Most patients will be prescribed a short course of their medicines
at discharge. This prescription is particularly important because it
usually informs future therapy at the point of transfer of prescribing
responsibility to primary care. Great care is required to ensure that this
list is accurate. It is particularly important to ensure that any hospital
medicines that should be stopped are not included and that those
intended to be administered for a short duration only are clearly iden-
tied. It is also important for any signicant ADRs experienced in hos-
pital to be recorded and any specic monitoring or review identied.
Prescribing in primary care
Most of the considerations above are equally applicable to primary care
(GP) prescriptions. In many health-care systems, community prescrib-
ing is electronic, making issues of legibility irrelevant and often providing
basic decision support to limit the range of doses that can be written and
highlight potential drug interactions. Important additional issues more rel-
evant to GP prescribing are:
OTHER MEDICINES CHARTS
CODES FOR NON-ADMINISTRATION OF PRESCRIBED MEDICINE
PREVIOUS ADVERSE REACTIONS
(This must be completed before prescribing on this chart)
Hospital number:
(Attach printed label here)
D.O.B.:
Weight:
Date
Date Time Medicine (approved name) Dose Route
Time
given
Given
by
Prescriber – sign and print
If a dose is not administered as prescribed, intial and enter a code in the column with a circle drawn round the code according to the
reason as shown below. Inform the responsible doctor of the appropriate timescale.
1. Patient refuses
2. Patient not present
3. Medicines not available – CHECK ORDERED
4. Asleep/drowsy
5. Administration route not available – CHECK FOR ALTERNATIVE
6. Vomiting/nausea
7. Time varied on doctor’s instructions
8. Once-only/as-required medicine given
9. Dose withheld on doctor’s instructions
10. Possible adverse reaction/side-effect
Type of chart Medicine Description of reaction Completed by Date
Height:
If rewritten, date:
DISCHARGE PRESCRIPTION
PRESCRIPTION AND ADMINISTRATION RECORD
Standard Chart
ONCE-ONLY MEDICINES
Date completed:– Completed by:–
A
B
Fig. 2.8 Example of a hospital prescription and administration record (‘drug chart’). Front page. The correct identication of the patient is critical to reducing the risk
of an administration error. This page also clearly identies other prescriptions charts in use and previous adverse reactions to drugs to minimise the risk of repeated exposure.
Note also the codes employed by the nursing staff to indicate reasons why drugs may not have been administered. The patient’s name and date of birth should be written on
‘Once-only medicines’
This area is used for prescribing medicines that are unlikely to be repeated on a regular basis. Note that the prescriber has written the names of all drugs legibly in block
capitals. The generic international non-proprietary name (INN) should be used in preference to the brand name (e.g. write ‘SIMVASTATIN’, not ‘ZOCOR’). The only exceptions are
when variation occurs in the properties of alternative branded formulations (e.g. modied-release preparations of drugs such as lithium, theophylline, phenytoin and nifedipine)
or when the drug is a combination product with no generic name (e.g. Kliovance®). The only acceptable abbreviations for drug dose units are ‘g’ and ‘mg’. ‘Units’ (e.g. of
insulin or heparin) and ‘micrograms’ must always be written in full, never as ‘U’ or ‘µg’ (nor ‘mcg’, nor ‘ug’). For liquid preparations write the dose in mg; ‘mL’ can be written
only for a combination product (e.g. Gaviscon liquid) or if the strength is not expressed in weight (e.g. adrenaline (epinephrine) 1 in 1000). Use numbers/gures (e.g. 1 or
‘one’) to denote use of a sachet/enema but avoid prescribing numbers of tablets without specifying their strength. Always include the dose of inhaled drugs in addition to
stating numbers of ‘puffs’, as strengths can vary. Widely accepted abbreviations for route of administration are: intravenous – ‘IV’; intramuscular – ‘IM’; subcutaneous – ‘SC’;
sublingual – ‘SL’; per rectum – ‘PR’; per vaginam – ‘PV’; nasogastric – ‘NG’; inhaled – ‘INH’; and topical – ‘TOP’. ‘ORAL’ is preferred to per oram – ‘PO’. Care should be taken
in specifying ‘RIGHT’ or ‘LEFT’ for eye and ear drops. The prescriber should sign and print their name clearly so that they can be identied by colleagues. The prescription
should be dated and have an administration time. The nurse who administered the prescription has signed to conrm that the dose has been administered.
(Continues overleaf)
56.
34 CLINICALTHERAPEUTICS AND GOOD PRESCRIBING
REGULAR MEDICINES
AS-REQUIRED THERAPY
Drug (approved name)
Dose
Date
Time
6
8
12
14
18
22
6
8
12
14
18
22
6
8
12
14
18
22
Prescriber–sign and print
Notes
Start date
Pharmacy
Route
Drug (approved name)
Dose
Prescriber–sign and print
Notes
Start date
Pharmacy
Route
Drug (approved name)
Dose and frequency
Prescriber–sign and print Start date
Route
Date
Time
Dose
Initials
Date
Time
Dose
Initials
Drug (approved name)
Dose and frequency Route
Date
Time
Dose
Initials
Date
Time
Dose
Initials
Drug (approved name)
Dose
Prescriber–sign and print
Notes
Start date
Pharmacy
Route
C
D
Fig. 2.8, cont’d ‘Regular medicines’. This area is used for prescribing medicines that are going to be given regularly. In addition to the name, dose and route, a
frequency of administration is required for each medicine. Widely accepted Latin abbreviations for dose frequency are: once daily – ‘OD’; twice daily – ‘BD’; 3 times daily –
‘TDS’; 4 times daily – ‘QDS’; as required – ‘PRN’; in the morning – ‘OM’ (omni mane); at night – ‘ON’ (omni nocte); and immediately – ‘stat’. The hospital chart usually
requires specic times to be identied for regular medicines that coincide with nursing drug rounds and these can be circled. If treatment is for a known time period, cross
off subsequent days when the medicine is not required. The ‘notes’ box can be used to communicate additional important information (e.g. whether a medicine should be
taken with food, type of inhaler device used, and anything else that the drug dispenser should know). State here the times for peak/trough plasma levels for drugs requiring
therapeutic monitoring. Prescriptions should be discontinued by drawing a vertical line at the point of discontinuation, horizontal lines through the remaining days on the chart,
and diagonal lines through the drug details and administration boxes. This action should be signed and dated and a supplementary note written to explain it (e.g. describing any
adverse effect). In this example, amlodipine has been discontinued because of ankle oedema. There is room for the ward pharmacist to sign to indicate that the prescription
has been reviewed and that a supply of the medicine is available. The administration boxes allow the nurse to sign to conrm that the dose has been given. Note that these
boxes also allow for recording of reasons for non-administration (in this example ‘2’ indicates that the patient was not present on the ward at the time) and the prevention of
‘As-required medicines’. These prescriptions leave the administration of the drug to the discretion of the nursing staff. The
prescription must describe clearly the indication, frequency, minimal time interval between doses, and maximum dose in any 24-hour period (in this case, the maximum daily
dose of paracetamol is 4g).
Formulation. The prescription needs to carry information about
the formulation for the dispensing pharmacist (e.g. tablets or oral
suspension).
Amount to be supplied. A pharmacist will organise this in hospital.
Elsewhere it must be specied either as the precise number of tab-
lets or as the duration of treatment. Creams and ointments should
be specied in grams and lotions in mL.
Controlled drugs. Prescriptions for ‘controlled’ drugs (e.g. opioid
analgesics, with potential for drug abuse) are subject to additional
legal requirements. In the UK, they must contain the address of the
patient and prescriber (not necessary on most hospital forms), the
form and the strength of the preparation, and the total quantity of
the preparation/number of dose units in both words and gures.
‘Repeat prescriptions’. A large proportion of GP prescribing involves
‘repeat prescriptions’ for chronic medication. These are often gen-
erated automatically, although the prescriber remains responsible
for regular review and for ensuring that the benet-to-harm ratio
remains favourable.
57.
Prescribing in practice 35
2
Monitoring drug therapy
Prescribers should measure the effects of the drug, both benecial and
harmful, to inform decisions about dose titration (up or down), discontinu-
ation or substitution of treatment. Monitoring can be achieved subjectively
by asking the patient about symptoms or, more objectively, by measuring
a clinical effect, which may be possible to assess by clinical examination
(e.g. eczematous rash), physiological measurement (e.g. blood pressure,
pulse oximetry), imaging (e.g. chest X-ray, CT scan) or laboratory tests (e.g.
haemoglobin, INR (International Normalised Ratio)). Alternatively, if the phar-
macodynamic effects of the drug are difcult to assess, the plasma drug
concentration may be measured, if this is closely related to the effect of
the drug (see Fig. 2.2). Rational prescribing ultimately rests on continuous
monitoring of the balance between the risks and benets of drug therapy.
Common examples are ACE inhibitors (negative impact on renal function
versus blood pressure-lowering), cancer chemotherapy (bone marrow
suppression versus radiological tumour regression) and disease-modifying
anti-rheumatic drugs (liver toxicity versus anti-inammatory effect). Advances
in therapeutics have also led to increasing complexity of ongoing monitoring
requirements. The recognition that hospital specialist clinics cannot take on
the responsibility for monitoring of all chronic disease therapy has led to the
development of ‘shared care protocols’ that aim to share monitoring activ-
ities between specialists and primary care physicians with the aim of max-
imising efcient use of health-care resources and convenience for patients.
Clinical and surrogate endpoints
Ideally, clinical endpoints are measured directly and the drug dosage titrated
to achieve the therapeutic goal and avoid toxicity (e.g. control of ventricular
rate in a patient with atrial brillation, monitoring anticoagulation using INR).
Sometimes this is impractical because the clinical endpoint is a future event
(e.g. prevention of myocardial infarction by statins or resolution of a chest
infection with antibiotics); in these circumstances, it may be possible to
select a ‘surrogate’ endpoint that will predict success or failure. This may be
an intermediate step in the pathophysiological process (e.g. serum choles-
terol as a surrogate for risk of myocardial infarction) or a measurement that
follows the pathophysiology, even if it is not a key factor in its progression
(e.g. serum C-reactive protein as a surrogate for resolution of inammation
in chest infection, TSH as a surrogate for adequate replacement of levo-
thyroxine). Such surrogates are sometimes termed ‘biomarkers’.
Plasma drug concentration
The following criteria must be met to justify routine monitoring by plasma
drug concentration:
Clinical endpoints and other pharmacodynamic (surrogate) effects
are difcult to monitor.
The relationship between plasma concentration and clinical effects is
predictable.
The therapeutic index is low. For drugs with a high therapeutic
index, any variability in plasma concentrations is likely to be irrele-
vant clinically.
Some examples of drugs that full these criteria are listed in Box 2.26.
Measurement of plasma concentration may be useful in planning
adjustments of drug dose and frequency of administration; to explain
an inadequate therapeutic response (by identifying subtherapeutic con-
centration or incomplete adherence); to establish whether a suspected
ADR is likely to be caused by the drug; and to assess and avoid potential
drug interactions.
Timing of samples in relation to doses
The concentration of drug rises and falls during the dosage interval
(see Fig. 2.4B). Measurements made during the initial absorption and
2.25 High-risk prescribing moments
Trying to amend an active prescription (e.g. altering the dose/timing) – always
avoid and start again
Writing up drugs in the immediate presence of more than one prescription chart
or set of notes – avoid
Allowing one’s attention to be diverted in the middle of completing a
prescription – avoid
Prescribing ‘high-risk’ drugs (e.g. anticoagulants, opioids, insulin, sedatives) –
ask for help if necessary
Prescribing parenteral drugs – take care
Rushing prescribing (e.g. in the midst of a busy ward round) – avoid
Prescribing unfamiliar drugs – consult the formulary and ask for help if
necessary
Transcribing multiple prescriptions from an expired chart to a new one – take
care to review the rationale for each medicine
Writing prescriptions based on information from another source such as a
referral letter (the list may contain errors and some of the medicines may be the
cause of the patient’s illness) – review the justication for each as if it is a new
prescription
Writing up ‘to take out’ drugs (because these will become the patient’s regular
medication for the immediate future) – take care and seek advice if necessary
Calculating drug doses – ask a colleague to perform an independent calculation
or use approved electronic dose calculators
Prescribing sound-alike or look-alike drugs (e.g. chlorphenamine and
chlorpromazine) – take care
2.26 Drugs commonly monitored by plasma drug concentration
Drug Half-life (hrs)* Comment
Digoxin 36 Steady state takes several days to achieve. Samples should be taken 6hrs post dose. Measurement is useful to conrm
the clinical impression of toxicity or non-adherence but clinical effectiveness is better assessed by ventricular heart rate.
Risk of toxicity increases progressively at concentrations >1.5µg/L, and is likely at concentrations >3.0µg/L (toxicity is
more likely in the presence of hypokalaemia)
Gentamicin 2 Measure pre-dose trough concentration (should be <1mg/L) to ensure that accumulation (and the risk of nephrotoxicity
and ototoxicity) is avoided; see Fig. 6.18
Lithium 24 Steady state takes several days to achieve. Samples should be taken 12hrs post dose. Target range 0.4–1mmol/L
Phenytoin 24 Measure pre-dose trough concentration (should be 10–20mg/L) to ensure that accumulation is avoided. Good
correlation between concentration and toxicity. Concentration may be misleading in the presence of hypoalbuminaemia
Theophylline (oral) 6 Steady state takes 2–3 days to achieve. Samples should be taken 6hrs post dose. Target concentration is 10–20mg/L
but its relationship with bronchodilator effect and adverse effects is variable
Vancomycin 6 Measure pre-dose trough concentration (should be 10–15mg/L) to ensure clinical efcacy and that accumulation and
the risk of nephrotoxicity are avoided
*Half-lives vary considerably with different formulations and between patients.
58.
36 CLINICALTHERAPEUTICS AND GOOD PRESCRIBING
distribution phases are unpredictable because of the rapidly changing
concentration, so samples are usually taken at the end of the dosage
interval (a ‘trough’ or ‘pre-dose’ concentration). This measurement is
normally made in steady state, which usually takes 5 half-lives to achieve
after the drug is introduced or the dose changed (unless a loading dose
has been given).
Interpreting the result
A target range is provided for many drugs, based on average thresh-
olds for therapeutic benet and toxicity. Inter-individual variability means
that these can be used only as a guide. For instance, in a patient who
describes symptoms that could be consistent with toxicity, but has a
drug concentration in the top half of the target range, toxic effects should
still be suspected. Another important consideration is that some drugs
are heavily protein-bound (e.g. phenytoin), but only the unbound drug is
pharmacologically active. Patients with hypoalbuminaemia may, there-
fore, have a therapeutic or even toxic concentration of unbound drug,
despite a low ‘total’ concentration.
Further information
Websites
bnf.org The British National Formulary (BNF) is a key reference resource for UK
NHS prescribers, with a list of licensed drugs, chapters on prescribing in renal
failure, liver disease, pregnancy and during breastfeeding, and appendices on
drug interactions
cochrane.org The Cochrane Collaboration is a leading international body that
provides evidence-based reviews (around 7000 so far)
evidence.nhs.uk NHS Evidence provides a wide range of health information
relevant to delivering quality patient care
icp.org.nz The Interactive Clinical Pharmacology site is designed to increase
understanding of principles in clinical pharmacology
medicines.org.uk/emc/ The electronic medicines compendium (emc) contains
up-to-date, easily accessible information about medicines licensed by the
UK Medicines and Healthcare Products Regulatory Agency (MHRA) and the
European Medicines Agency (EMA)
nice.org.uk The UK National Institute for Health and Care Excellence makes
recommendations to the UK NHS on new and existing medicines, treatments
and procedures
who.int/health-topics/medicines The World Health Organization Essential
Medicines and Pharmaceutical Policies
59.
Multiple Choice Questions
2.1.Which of the following drugs for type 2 diabetes acts on a
transporter protein?
A. Gliclazide
B. Dapagliozin
C. Metformin hydrochloride
D. Pioglitazone
E. Sitagliptin
Answer: B.
Dapagliozin is a sodium-glucose co-transporter 2 inhibitor that is
indicated for the treatment of type 2 diabetes mellitus. Dapagliozin
improves glycaemic control by inhibiting glucose re-absorption in the
proximal tubule of the nephron leading to excretion of excess glucose in
the urine (glycosuria).
2.2. Which of the following is a prodrug that depends on hepatic
metabolism for conversion to its active form?
A. Amlodipine
B. Clopidogrel
C. Metoprolol
D. Simvastatin
E. Tramadol
Answer: B.
Clopidogrel binds specically and irreversibly to the platelet P2RY12
purinergic receptor to inhibit ADP-mediated platelet activation and
aggregation. Clopidogrel is a prodrug that is absorbed in the intestine
and activated in the liver. The conversion of clopidogrel to its active
metabolite involves two sequential oxidation reactions that involve vari-
ous cytochrome P450 isoforms (CYP1A2, CYP2B6, CYP2C9, CYP2C19
and CYP3A4/5).
2.3. A 52-year-old man who is being treated with azathioprine 150mg
orally daily to maintain remission from Crohn’s disease presents
to hospital with severe pancytopenia. Which of his other current
medicines (listed below) is most likely to have interacted with
azathioprine to cause this adverse drug reaction?
A. Allopurinol 200mg orally daily
B. Atorvastain 20mg orally daily
C. Indapamide 2.5mg orally daily
D. Metronidazole 400mg orally three times daily
E. Omeprazole 20mg orally daily
Answer: A.
Allopurinol inhibits the enzyme xanthine oxidase (XO), which is one
of the three enzymes responsible for inactivating 6-mercaptopurine, its
active purine metabolite. Normally, XO is needed to convert the purine
base hypoxanthine to xanthine and then xanthine to uric acid. Allopurinol
is a structural analogue of hypoxanthine and binds to XO, thereby inhib-
iting its effect on purine metabolism.
2.4. Which of the following regular medicines is most likely to require a
dose reduction in a patient who develops chronic renal impairment
(chronic kidney disease stage 3)?
A. Bendroumethiazide 2.5mg orally daily
B. Digoxin 125micrograms orally daily
C. Fluoxetine 20mg orally daily
D. Simvastatin 20mg orally nightly
E. Verapamil hydrochloride m/r 120mg orally daily
Answer: B.
Digoxin is mainly eliminated by renal excretion and there is a risk
of toxicity if the dosage is not reduced as renal function deteriorates.
Bendroumethiazide becomes less effective as renal function deterior-
ates but does not require a dose reduction (unless there is evidence
that dehydration is a contributory factor). Simvastatin should be used
with caution if the eGFR falls below 30mL/min/1.73m2
. Verapamil is
eliminated by hepatic metabolism. Fluoxetine does not require dose
adjustment.
2.5. Which of the following aspects of a new drug is not considered
by drug regulators but considered by heath-care providers (e.g. a
hospital drug and therapeutics committee)?
A. Manufacturing standards
B. Safety prole
C. Efcacy in treating the clinical indication
D. Cost-effectiveness
E. Patient information
Answer: D.
The role of national drug regulatory authorities is to ensure that any
new medicine meets minimum acceptable standards of quality, safety
and efcacy prior to being granted a market authorisation. It is not the
role of the regulator to assess cost-effectiveness which is devolved to
national or local health technology assessments that may compare the
new drug with other available treatment options.
60.
Clinical genetics
3
K Tatton-Brown
Thefundamental principles of genomics 38
The packaging of genes: DNA, chromatin and chromosomes 38
From DNA to protein 38
Non-coding RNA 40
Cell division, differentiation and migration 40
Cell death, apoptosis and senescence 41
Genomics, health and disease 41
Classes of genetic variant 41
Consequences of genomic variation 42
Normal genomic variation 45
Constitutional genetic disease 45
Somatic genetic disease 49
Interrogating the genome: the changing landscape of genomic technologies 50
Looking at chromosomes 50
Looking at genes 50
Genomics and clinical practice 52
Genomics and health care 52
Genomics in infectious disease 55
Treatment of genetic disease 56
Ethics in a genomic age 56
61.
38 CLINICALGENETICS
We have entered a genomic era. Powerful new technologies are driving
forward transformational change in health care. Genetic sequencing has
evolved from the targeted sequencing of a single gene to the parallel
sequencing of multiple genes. In addition to improving the chances of
identifying a genetic cause of rare diseases, these technologies are
increasingly directing therapies and, in the future, are likely to be used in
the diagnosis and prevention of common diseases such as diabetes. In
this chapter we explore the fundamentals of genomics, the basic prin-
ciples underlying these new genomic technologies and how the data
generated can be applied safely for patient benet. We will review the use
of genomic technology across a breadth of medical specialties, including
obstetrics, paediatrics, oncology and infectious disease, and consider
how health care is being transformed by these new genomic technolo-
gies. Finally, we will consider the ethical impact that these technologies
are likely to have, both for the individual and for their wider family.
The fundamental principles of genomics
The packaging of genes: DNA, chromatin and
chromosomes
Genes are functional units encoded in double-stranded deoxyribonucleic
acid (DNA), packaged as chromosomes and located in the nucleus of the
cell: a membrane-bound compartment found in all cells except erythro-
cytes and platelets (Fig. 3.1). DNA consists of a linear sequence of just four
bases: adenine (A,) cytosine (C), thymine (T) and guanine (G.) It forms a
‘double helix’, a twisted ladder-like structure formed from two comple-
mentary strands of DNA joined by hydrogen bonds between bases on
the opposite strand that can form only between a C and a G base and an
A and a T base. It is this feature of DNA that enables faithful DNA replica-
tion and is the basis for many of the technologies designed to interrogate
the genome: when the DNA double helix ‘unzips’, one strand can act as
a template for the creation of an identical strand.
A single copy of the human genome comprises approximately
3.1 billion base pairs of DNA, wound around proteins called histones.
The unit consisting of 147 base pairs wrapped around four different
histone proteins is called the nucleosome. Sequences of nucleosomes
(resembling a string of beads) are wound and packaged to form chroma-
tin: tightly wound, densely packed chromatin is called heterochromatin
and open, less tightly wound chromatin is called euchromatin.
The chromatin is nally packaged into the chromosomes. Humans
are diploid organisms: the nucleus contains two copies of the genome,
visible microscopically as 23 chromosome pairs (known as the karyo-
type). Chromosomes 1 through to 22 are known as the autosomes and
consist of identical chromosome pairs. The 23rd ‘pair’ of chromosomes
are the two sex chromosomes: females have two X chromosomes and
males an X and Y chromosome. A normal female karyotype is therefore
written as 46,XX and a normal male is 46,XY.
From DNA to protein
Genes are functional elements on the chromosome that are capable
of transmitting information from the DNA template via the production
of messenger ribonucleic acid (mRNA) to the production of proteins.
The human genome contains over 20000 genes, although many of
these are inactive or silenced in different cell types, reecting the varia-
ble gene expression responsible for cell-specic characteristics. The
central dogma is the pathway describing the basic steps of protein
production: transcription, splicing, translation and protein modication
(Fig. 3.2). Although this is now recognised as an over-simplication (con-
trary to this linear relationship, a single gene will often encode many different
proteins), it remains a useful starting point to explore protein production.
Transcription: DNA to messenger RNA
Transcription describes the production of ribonucleic acid (RNA) from the
DNA template. For transcription to commence, an enzyme called RNA
polymerase binds to a segment of DNA at the start of the gene: the pro-
moter. Once bound, RNA polymerase moves along one strand of DNA,
producing an RNA molecule complementary to the DNA template. In
protein-coding genes this is known as messenger RNA (mRNA). A DNA
sequence close to the end of the gene, called the polyadenylation signal,
acts as a signal for termination of the RNA transcript (Fig. 3.3).
DNA helix
Histones
Chromatin
Chromosome
Normal female karyotype
Nucleosome
A T G A C G G A T
T A C T G C C T A
Fig. 3.1 The packaging of DNA, genes and chromosomes. From bottom to top:
the double helix and the complementary DNA bases; chromatin; and a normal female
chromosome pattern – the karyotype.
DNA
5’CGATTC3’
3’GCTAAG5’
5’CGAUUC3’
N_ArgPhe_C
RNA
Protein
Transcription
Translation
Fig. 3.2 The central dogma of protein production. Double-stranded DNA as a
template for single-stranded RNA, which codes for the production of a peptide chain
of amino acids. Each of these chains has an orientation. For DNA and RNA, this is 5
to 3 . For peptides, this is N-terminus to C-terminus.
62.
The fundamental principlesof genomics 39
3
RNA differs from DNA in three main ways:
RNA is single-stranded.
The sugar residue within the nucleotide is ribose, rather than
deoxyribose.
It contains uracil (U) in place of thymine (T).
The activity of RNA polymerase is regulated by transcription fac-
tors. These proteins bind to specic DNA sequences at the promoter
or to enhancer elements that may be many thousands of base pairs
away from the promoter; a loop in the chromosomal DNA brings
the enhancer close to the promoter, enabling the bound proteins to
interact. The human genome encodes more than 1200 different tran-
scription factors. Mutations (now more frequently referred to as gene
variants) within transcription factors, promoters and enhancers can
cause disease. For example, the blood disorder alpha-thalassaemia is
usually caused by gene deletions (see Box 3.4). However, it can also
result from a variant in an enhancer located more than 100000 base
pairs (bp) from the α-globin gene promoter, leading to greatly reduced
transcription.
Gene activity, or expression, is inuenced by a number of complex
interacting factors, including the accessibility of the gene promoter to
transcription factors. DNA can be modied by the addition of a methyl
group to cytosine molecules (methylation). If DNA methylation occurs
in promoter regions, transcription is silenced, as methyl cytosines are
usually not available for transcription factor binding. A second mecha-
nism determining promoter accessibility is the structural conguration
of chromatin. In open chromatin, called euchromatin, gene promoters
are accessible to RNA polymerase and transcription factors; therefore it
is transcriptionally active. This contrasts with heterochromatin, which is
densely packed and transcriptionally silent. The chromatin conguration
is determined by modications (such as methylation or acetylation) of
specic amino acid residues of histone protein tails.
Modications of DNA and histone protein tails are termed epigenetic
(‘epi-’ meaning ‘above’ the genome), as they do not alter the primary
sequence of the DNA code but have biological signicance in chromo-
somal function. Abnormal epigenetic changes are increasingly recognised
as important events in the progression of cancer, allowing expression of
normally silenced genes that result in cancer cell de-differentiation and
proliferation. They also afford therapeutic targets. For instance, the his-
tone deacetylase inhibitor vorinostat has been successfully used to treat
cutaneous T-cell lymphoma, due to the re-expression of genes that had
previously been silenced in the tumour. These genes encode transcrip-
tion factors that promote T-cell differentiation as opposed to proliferation,
thereby causing tumour regression.
RNA splicing, editing and degradation
Transcription produces an RNA molecule that is a copy of the whole
gene, termed the primary or nascent transcript. This nascent transcript
then undergoes splicing, whereby regions not required to make pro-
tein (the intronic regions) are removed while those segments that are
necessary for protein production (the exonic regions) are retained and
rejoined.
Splicing is a highly regulated process that is carried out by a multimeric
protein complex called the spliceosome. Following splicing, the mRNA
molecule is exported from the nucleus and used as a template for pro-
tein synthesis. Many genes produce more than one form of mRNA (and
thus protein) by a process termed alternative splicing, in which different
combinations of exons are joined together. Different proteins from the
same gene can have entirely distinct functions. For example, in thyroid C
cells the calcitonin gene produces mRNA encoding the osteoclast inhib-
itor calcitonin, but in neurons the same gene produces an mRNA with
a different complement of exons via alternative splicing that encodes a
neurotransmitter, calcitonin-gene-related peptide.
Protein product
N-term C-term
cap
Messenger RNA
(mRNA)
Messenger RNA
(mRNA)
AAAAA
tRNAs
Ribosome
Nuclear
membrane
Primary RNA
transcript
Spliceosome
RNA export
to cytoplasm
cap
cap
AAAAA
PolyA tail
3'UTR
5'UTR
Nuclear pore
AAAAA
Splicing
Transcription
3'
5'
3'
Sense strand
Enhancer
Transcription
factors
RNA
polymerase II
Exon 1
Intron 2
Intron 1
Exon 2 Exon 3
Exon 1 Exon 2 Exon 3
Promoter
Nucleolus
Nuclear
membrane
Gene A
Gene B
Gene C
Active
gene
RNA
Nucleus
Translation
Fig. 3.3 RNA synthesis and its translation into protein. Gene transcription
involves binding of RNA polymerase II to the promoter of genes being
transcribed with other proteins (transcription factors) that regulate the
transcription rate. The primary RNA transcript is a copy of the whole gene
and includes both introns and exons, but the introns are removed within the
nucleus by splicing and the exons are joined to form the messenger RNA
(mRNA). Prior to export from the nucleus, a methylated guanosine nucleotide
is added to the 5 end of the RNA (‘cap’) and a string of adenine nucleotides
is added to the 3 (‘polyA tail’). This protects the RNA from degradation and
facilitates transport into the cytoplasm. In the cytoplasm, the mRNA binds to
ribosomes and forms a template for protein production. (tRNA = transfer RNA;
UTR = untranslated region)
63.
40 CLINICALGENETICS
Translation and protein production
Following splicing, the segment of mRNA containing the code that
directs synthesis of a protein product is called the open reading frame
(ORF). The inclusion of a particular amino acid in the protein is specied
by a codon composed of three contiguous bases. There are 64 different
codons with some redundancy in the system: 61 codons encode one
of the 20 amino acids, and the remaining three codons – UAA, UAG
and UGA (known as stop codons) – cause termination of the growing
polypeptide chain. ORFs in humans most commonly start with the amino
acid methionine. All mRNA molecules have domains before and after the
ORF called the 5 untranslated region (UTR) and 3 UTR, respectively.
The start of the 5 UTR contains a cap structure that protects mRNA
from enzymatic degradation, and other elements within the 5 UTR are
required for efcient translation. The 3 UTR also contains elements that
regulate efciency of translation and mRNA stability, including a stretch
of adenine bases known as a polyA tail (see Fig. 3.3).
The mRNAs then leave the nucleus via nuclear pores and associate
with ribosomes, the sites of protein production (see Fig. 3.3). Each ribo-
some consists of two subunits (40S and 60S), which comprise non-cod-
ing rRNA molecules (see Fig. 3.9) complexed with proteins. During
translation, a different RNA molecule known as transfer RNA (tRNA)
binds to the ribosome. The tRNAs deliver amino acids to the ribosome
so that the newly synthesised protein can be assembled in a stepwise
fashion. Individual tRNA molecules bind a specic amino acid and ‘read’
the mRNA ORF via an ‘anticodon’ of three nucleotides that is comple-
mentary to the codon in mRNA (see Fig. 3.3). A proportion of ribosomes
is bound to the membrane of the endoplasmic reticulum (ER), a complex
tubular structure that surrounds the nucleus.
Proteins synthesised on these ribosomes are translocated into the
lumen of the ER, where they undergo folding and processing. From here,
the protein may be transferred to the Golgi apparatus, where it under-
goes post-translational modications, such as glycosylation (covalent
attachment of sugar moieties), to form the mature protein that can be
exported into the cytoplasm or packaged into vesicles for secretion. The
clinical importance of post-translational modication of proteins is shown
by the severe developmental, neurological, haemostatic and soft tissue
abnormalities that are associated with the many different congenital dis-
orders of glycosylation. Post-translational modications can also be dis-
rupted by the synthesis of proteins with abnormal amino acid sequences.
For example, the most common CFTR gene variant that causes cystic
brosis (ΔF508) results in an abnormal protein that cannot be exported
from the ER and Golgi (see Box 3.4).
Non-coding RNA
Approximately 4500 genes in humans encode non-coding RNAs
(ncRNA) rather than proteins. There are various categories of ncRNA,
including transfer RNA (tRNA), ribosomal RNA (rRNA), ribozymes and
microRNA (miRNA). The miRNAs, which number over 1000, have a role
in post-translational gene expression: they bind to mRNAs, typically
in the 3 UTR, promoting target mRNA degradation and gene silenc-
ing. Together, miRNAs affect over half of all human genes and have
important roles in normal development, cancer and common degen-
erative disorders. This is the subject of considerable research interest
at present.
Cell division, differentiation and migration
In normal tissues, molecules such as hormones, growth factors and
cytokines provide the signal to activate the cell cycle: a controlled pro-
gramme of biochemical events that culminates in cell division. In all
cells of the body, except the gametes (the sperm and egg cells, also
known as the germ line), mitosis completes cell division, resulting in
two diploid daughter cells. In contrast, the sperm and eggs cells com-
plete cell division with meiosis, resulting in four haploid daughter cells
(Fig. 3.4).
The stages of cell division in the non-germ-line, somatic cells are
shown below:
Cells not committed to mitosis are said to be in G0
Cells committed to mitosis must go through the preparatory phase
of interphase consisting of G1
, S and G2
G1
(rst gap): synthesis of the cellular components necessary to
complete cell division
S (synthesis): DNA replication producing identical copies of each
chromosome called the sister chromatids
Egg
Sperm
Father Mother
Meiotic cell divisions
1st
polar bodies
2nd
polar body
DNA replication
Sister chromatids
Homologous pairing
Swapping of
genetic material
between homologues:
Recombination
Individual chromosome
pair (homologues)
Non-disjunction of
chromosomes is a common
error in human meiosis,
resulting in trisomy of
individual chromosomes
or uniparental disomy
(both chromosomes from
single parent)
Fig. 3.4 Meiosis and gametogenesis: the main chromosomal stages of
meiosis in both males and females. A single homologous pair of chromosomes
is represented in different colours. The nal step is the production of haploid germ
cells. Each round of meiosis in the male results in four sperm cells; in the female,
however, only one egg cell is produced, as the other divisions are sequestered on the
periphery of the mature egg as peripheral polar bodies.
64.
Genomics, health anddisease 41
3
G2
(second gap): repair of any errors in the replicated DNA before
proceeding to mitosis.
Mitosis (M) consists of four phases:
Prophase: the chromosomes condense and become visible,
the centrioles move to opposite ends of the cell and the nuclear
membrane disappears.
Metaphase: the centrioles complete their migration to opposite
ends of the cell and the chromosomes – consisting of two
identical sister chromatids – line up at the equator of the cell.
Anaphase: spindle bres attach to the chromosome and pull the
sister chromatids apart.
Telophase: the chromosomes decondense, the nuclear
membrane reforms and two daughter cells – each with 46
chromosomes – are formed.
The progression from one phase to the next is tightly controlled by
cell-cycle checkpoints. For example, the checkpoint between G2
and
mitosis ensures that all damaged DNA is repaired prior to segregation of
the chromosomes. Failure of these control processes is a crucial driver in
the pathogenesis of cancer, as discussed on page 130.
Meiosis is a special, gamete-specic, form of cell division
(see Fig. 3.4). Like mitosis, meiosis consists of four phases
(prophase, metaphase, anaphase and telophase) but differs from
mitosis in the following ways:
It consists of two separate cell divisions known as meiosis I and
meiosis II.
It reduces the chromosome number from the diploid to the
haploid number via a tetraploid stage, i.e. from 46 to 92 (MI S) to
46 (MI M) to 23 (MII M) chromosomes, so that when a sperm cell
fertilises the egg, the resulting zygote will return to a diploid, 46,
chromosome complement. This reduction to the haploid number
occurs at the end of meiosis II.
The 92 chromosome stage consists of 23 homologous pairs of
sister chromatids, which then swap genetic material, a process
known as recombination. This occurs at the end of MI prophase
and ensures that the chromosome that a parent passes to his
or her offspring is a mix of the chromosomes that the parent
inherited from his or her own mother and father.
The individual steps in meiotic cell division are similar in males and
females. However, the timing of the cell divisions is very different. In
females, meiosis begins in fetal life but does not complete until after ovu-
lation. A single meiotic cell division can thus take more than 40 years
to complete. As women become older, the separation of chromosomes
at meiosis II becomes less efcient. That is why the risk of trisomies
(p. 42) due to non-disjunction grows greater with increasing maternal
age. In males, meiotic division does not begin until puberty and contin-
ues throughout life. In the testes, both meiotic divisions are completed
in a matter of days.
Cell death, apoptosis and senescence
With the exception of stem cells, human cells have only a limited capac-
ity for cell division. The Hayick limit is the number of divisions a cell
population can go through in culture before division stops and enters a
state known as senescence. This ‘biological clock’ is of great interest in
the study of the normal ageing process. Rare human diseases associ-
ated with premature ageing, called progeric syndromes, have been very
helpful in identifying the importance of DNA repair mechanisms in senes-
cence. For example, in Werner syndrome, a DNA helicase (an enzyme
that separates the two DNA strands) is mutated, leading to failure of DNA
repair and premature ageing. A distinct mechanism of cell death is seen
in apoptosis, or programmed cell death.
Apoptosis is an active process that occurs in normal tissues and
plays an important role in development, tissue remodelling and the
immune response. The signal that triggers apoptosis is specic to each
tissue or cell type. This signal activates enzymes, called caspases,
which actively destroy cellular components, including chromosomal
DNA. This degradation results in cell death, but the cellular corpse
contains characteristic vesicles called apoptotic bodies. The corpse
is then recognised and removed by phagocytic cells of the immune
system, such as macrophages, in a manner that does not provoke an
inammatory response.
A third mechanism of cell death is necrosis. This is a pathological pro-
cess in which the cellular environment loses one or more of the compo-
nents necessary for cell viability. Hypoxia is probably the most common
cause of necrosis.
Genomics, health and disease
Classes of genetic variant
There are many different classes of variation in the human genome, cat-
egorised by the size of the DNA segment involved and/or by the mecha-
nism giving rise to the variation.
Nucleotide substitutions
The substitution of one nucleotide for another is the most common type
of genomic variation. This is caused by misincorporation of a nucleotide
during DNA synthesis or by chemical modication of the base. When
these substitutions occur within ORFs of a protein-coding gene, they are
further classied into:
synonymous – resulting in a change in the codon without altering
the amino acid
non-synonymous (also known as a missense variant) – resulting in a
change in the codon and the encoded amino acid
stop gain (or nonsense variant) – introducing a premature stop
codon and resulting in truncation of the protein
splicing – disruption of normal splicing and therefore most frequently
occurring at the junctions between an intron and an exon.
These different types of gene variants are illustrated in Box 3.1 and
examples are shown in Figs. 3.5 and 3.6.
Insertions and deletions
One or more nucleotides may be inserted or lost in a DNA sequence,
resulting in an insertion/deletion (indel) variant (see Box 3.1 and Fig. 3.5).
If a multiple of three nucleotides is involved, this is in-frame. If an indel
change affects one or two nucleotides within the ORF of a protein-coding
gene, this can have serious consequences because the triple nucleotide
sequence of the codons is disrupted, resulting in a frameshift variant. The
effect on the gene is typically severe because the amino acid sequence
is totally disrupted.
Simple tandem repeat variants
Variations in the length of simple tandem repeats of DNA are thought
to arise as the result of slippage of DNA during meiosis and are termed
microsatellite (small) or minisatellite (larger) repeats. These repeats are
unstable and can expand or contract in different generations. This
instability is proportional to the size of the original repeat, in that longer
repeats tend to be more unstable. Many microsatellites and minisat-
ellites occur in introns or in chromosomal regions between genes and
have no obvious adverse effects. However, some genetic diseases
are caused by microsatellite repeats that result in duplication of amino
acids within the affected gene product or affect gene expression
(Box 3.2).
65.
42 CLINICALGENETICS
3.1 Classes of genetic variant
The classes of genetic variant can be illustrated using the sentence
‘THE FAT FOX WAS ILL COS SHE ATE THE OLD CAT’
Synonymous
Silent polymorphism with no
amino acid change
THE FAT FOX WAS ILL COS SHE ATE THE OLD KAT
where the C is replaced with a K but the meaning
remains the same
Non-synonymous
Causing an amino acid
change
THE FAT BOX WAS ILL COS SHE ATE THE OLD CAT
where the F of FOX is replaced by B and the
original meaning of the sentence is lost
Stop-gain (also called a nonsense variant)
Causing the generation of a
premature stop codon
THE CAT
where the F of FAT is replaced by a C, generating a
premature stop codon
Indel
Where the bases are either
inserted or deleted; disruption
of the reading frame is
dependent on the number of
bases inserted or deleted
THE FAT FOX WAS ILL ILL COS SHE ATE THE OLD
CAT
where the insertion of three bases results in
maintenance of the reading frame
THE FAT FOX WAW ASI LLC OSS HEA TET HEO
LDC AT
where the insertion of two bases results in
disruption of the reading frame
Fig. 3.5 Different types of mutation affecting coding exons.
Normal
Silent polymorphism
(no amino acid change)
Missense variant causing
Lys–Gln amino acid change
‘G’ insertion causing
frameshift variant
Nonsense variant causing
premature termination codon
A
B
C
D
E
mosaic distribution) does not occur, but Turner syndrome, in which there
is monosomy for the X chromosome, affects approximately 1 in 2500 live
births (Box 3.3).
Large insertions or deletions of chromosomal DNA also occur and are
usually associated with a learning disability and/or congenital malforma-
tions. Such structural chromosomal anomalies usually arise as the result
of one of two different processes:
non-homologous end-joining
non-allelic homologous recombination.
Random double-stranded breaks in DNA are a necessary process in
meiotic recombination and also occur during mitosis at a predictable rate.
The rate of these breaks is dramatically increased by exposure to ionising
radiation. When such breaks take place, they are usually repaired accu-
rately by DNA repair mechanisms within the cell. However, in a proportion
of breaks, segments of DNA that are not normally contiguous will be joined
(‘non-homologous end-joining’). If the joined fragments are from different
chromosomes, this results in a translocation. If they are from the same
chromosome, this will result in either inversion, duplication or deletion of
a chromosomal fragment (Fig. 3.7). Large insertions and deletions may
be cytogenetically visible as chromosomal deletions or duplications. If the
anomalies are too small to be detected by microscopy, they are termed
microdeletions and microduplications. Many microdeletion syndromes
have been described and most result from non-allelic homologous recom-
bination between repeats of highly similar DNA sequences, which leads to
recurrent chromosome anomalies – and clinical syndromes – occurring in
unrelated individuals (see Fig. 3.7 and Box 3.3).
Consequences of genomic variation
The consequence of an individual genomic variant depends on many
factors, including the variant type, the nature of the gene product and the
position of the variant in the protein. Variants can have profound or subtle
effects on gene and cell function. Variations that have profound effects
are responsible for ‘classical’ genetic diseases, whereas those with sub-
tle effects may contribute to the pathogenesis of common disease where
there is a genetic component, such as diabetes.
Neutral variants have no effect on quality or type of protein
produced.
Copy number variations
Variation in the number of copies of an individual segment of the genome
from the usual diploid (two copies) content can be categorised by the
size of the segment involved. Rarely, individuals may gain (trisomy) or lose
(monosomy) a whole chromosome. Such numerical chromosome anom-
alies most commonly occur by a process known as non-disjunction,
where pairs of homologous chromosomes do not separate at meio-
sis II (p. 40). Common trisomies include Down syndrome (trisomy 21),
Edwards syndrome (trisomy 18) and Patau syndrome (trisomy 13).
Monosomy of the autosomes (present in all the cells, as opposed to in a
66.
Genomics, health anddisease 43
3
Fig. 3.6 Splice-site variants.
result, splicing no longer occurs, leading to read-through of the mRNA into the intron, which contains a premature termination codon downstream of the variant.
Normal
Splice-site variant
Splice donor site
Exon Exon
Intron
Exon Exon
Intron
Intron removed by
splicing
Splice acceptor site
mRNA ‘reads through’ intron
Abnormal protein with
premature stop codon
A
B
3.2 Diseases associated with triplet and other repeat expansions*
Repeat No. of repeats Gene Gene location Inheritance
Normal Mutant
Coding repeat expansion
Huntington’s disease [CAG] 6–34 >35 Huntingtin 4p16 AD
Spinocerebellar ataxia (type 1) [CAG] 6–39 >40 Ataxin 6p22–23 AD
Spinocerebellar ataxia
(types 2, 3, 6, 7)
[CAG] Various Various Various Various AD
Dentatorubral-pallidoluysian atrophy [CAG] 7–25 >49 Atrophin 12p12–13 AD
Machado–Joseph disease [CAG] 12–40 >67 MJD 14q32 AD
Spinobulbar muscular atrophy [CAG] 11–34 >40 Androgen receptor Xq11–12 XL recessive
Non-coding repeat expansion
Myotonic dystrophy [CTG] 5–37 >50 DMPK-3 UTR 19q13 AD
Friedreich’s ataxia [GAA] 7–22 >200 Frataxin-intronic 9q13 AR
Progressive myoclonic epilepsy [CCCCGCCCCGCG]4–8
2–3 >25 Cystatin B-5 UTR 21q AR
Fragile X mental retardation [CGG] 5–52 >200 FMR1–5 UTR Xq27 XL dominant
Fragile site mental retardation 2 (FRAXE) [GCC] 6–35 >200 FMR2 Xq28 XL, probably recessive
*The triplet repeat diseases fall into two major groups: those with disease stemming from expansion of [CAG]n repeats in coding DNA, resulting in multiple adjacent glutamine residues (polyglutamine
tracts), and those with non-coding repeats. The latter tend to be longer. Unaffected parents usually display ‘pre-mutation’ allele lengths that are just above the normal range.
(AD/AR = autosomal dominant/recessive; UTR = untranslated region; XL = X-linked)
Loss-of-function variants result in loss or reduction in the normal
protein function. Whole-gene deletions are the archetypal loss-of-
function variants but stop-gain or indel variants (early in the ORF),
missense variants affecting a critical domain and splice-site variants
can also result in loss of protein function.
Gain-of-function variants result in a gain of protein function. They are
typically non-synonymous variants that alter the protein structure,
leading to activation/alteration of its normal function through causing
either an interaction with a novel substrate or a change in its normal
function.
Dominant negative variants are the result of non-synonymous sub-
stitutions or in-frame deletions/duplications but may also, less fre-
quently, be caused by triplet repeat expansions. Dominant negative
variants are heterozygous changes that result in the production of
an abnormal protein that interferes with the normal functioning of the
wild-type protein.
67.
44 CLINICALGENETICS
3.3 Chromosome and contiguous gene disorders
Disease Locus Incidence Clinical features
Numerical chromosomal abnormalities
Down syndrome (trisomy 21) 47,XY,+21 or 47,XX+21 1 in 800 Characteristic facies, IQ usually <50, congenital heart disease, reduced life
expectancy
Edwards syndrome (trisomy 18) 47,XY,+18 or 47,XX,+18 1 in 6000 Early lethality, characteristic skull and facies, frequent malformations of heart, kidney
and other organs
Patau syndrome (trisomy 13) 47,XY,+13 or 47, XX,+13 1 in 15000 Early lethality, cleft lip and palate, polydactyly, small head, frequent congenital heart
disease
Klinefelter syndrome 47,XXY 1 in 1000 Phenotypic male, infertility, gynaecomastia, small testes
XYY 47,XYY 1 in 1000 Usually asymptomatic, some impulse control problems
Triple X syndrome 47,XXX 1 in 1000 Usually asymptomatic, may have reduced IQ
Turner syndrome 45,X 1 in 5000 Phenotypic female, short stature, webbed neck, coarctation of the aorta, primary
amenorrhoea
Recurrent deletions, microdeletions and contiguous gene defects
Di George/velocardiofacial
syndrome
22q11.2 1 in 4000 Cardiac outow tract defects, distinctive facial appearance, thymic hypoplasia, cleft
palate and hypocalcaemia. Major gene seems to be TBX1 (cardiac defects and cleft
palate)
Prader–Willi syndrome 15q11–q13 1 in 15000 Distinctive facial appearance, hyperphagia, small hands and feet, distinct behavioural
phenotype. Imprinted region, deletions on paternal allele in 70% of cases
Angelman syndrome 15q11–q13 1 in 15000 Distinctive facial appearance, absent speech, electroencephalogram (EEG)
abnormality, characteristic gait. Imprinted region, deletions on maternal allele
encompassing UBE3A
Williams syndrome 7q11.23 1 in 10000 Distinctive facial appearance, supravalvular aortic stenosis, learning disability and
infantile hypercalcaemia. Major gene for supravalvular aortic stenosis is elastin
Smith–Magenis syndrome 17p11.2 1 in 25000 Distinctive facial appearance and behavioural phenotype, self-injury and rapid eye
movement (REM) sleep abnormalities. Major gene seems to be RAI1
Fig. 3.7 Chromosomal analysis and structural chromosomal disorders.
acrocentric if the centromere is at the end. The bands of each chromosome are given a number, starting at the centromere and working out along the short (p) arm and long (q)
arm. Translocations and inversions are balanced structural chromosome anomalies where no genetic material is missing but it is in the wrong order. Translocations can be divided
into reciprocal (direct swap of chromosomal material between non-homologous chromosomes) and Robertsonian (fusion of acrocentric chromosomes). Deletions and duplications
can also occur due to non-allelic homologous recombination (illustrated in part B). Deletions are classied as interstitial if they lie within a chromosome, and terminal if the terminal
region of the chromosome is affected. Duplications can be either in tandem (where the duplicated fragment is inserted next to the region that is duplicated and orientated in the
correct direction) or inverted (where the duplicated fragment is in the wrong direction). (N = normal; A =
allelic homologous recombination, can occur (right panel), resulting in a deletion on one chromosome and a duplication in the homologous chromosome. The error is induced by
tandem repeats in the DNA sequences (green), which can misalign and bind to each other, thereby ‘fooling’ the DNA into thinking the pairing prior to recombination is correct.
Chromosome
9
Cen
p
23
21
12
12
21.1
21.3
21.2
31
33
34.2
q
Cen
p N N A N A N A N A
N A
N A A
13 (sa)
(st)
11.2
12
21
23
24.2
31
32.2
q
Metacentric Acrocentric
Chromosome
14 Reciprocal
translocation
Robertsonian
translocation
Inversions Interstitial Terminal Tandem Inverted
Mechanism underlying recurrent deletions and duplication: non-allelic homologous recombination
How structural chromosomal anomalies are described
Deletions Duplications
B
A
Recombination
Normal pairing Abnormal pairing between DNA repeats
Deletion Duplication
1 1
1
2
1
2
1
2
2
2
2 DNA repeat
Maternal chromosome
Paternal chromosome
1 1
2
1
2
68.
Genomics, health anddisease 45
3
Normal genomic variation
We each have 3–5 million variants in our genome, occurring approxi-
mately every 300 bases. These variants are mostly polymorphisms, aris-
ing in more than 1% of the population; they have no or subtle effects on
gene and cell function, and are not associated with a high risk of disease.
Polymorphisms can occur within exons, introns or the intergenic regions
that comprise 98%–99% of the human genome. Each of the classes
of genetic variant discussed in this chapter (Box 3.1) is present in the
genome as a common polymorphism. However, the most frequent is the
single nucleotide polymorphism, or SNP (pronounced ‘snip’), describing
the substitution of a single base.
Polymorphisms and common disease
The protective and detrimental polymorphisms associated with common
disease have been identied primarily through genome-wide association
studies (GWAS, p. 52) and are the basis for many direct-to-consumer tests
that purport to determine individual risk proles for common diseases or
traits such as cardiovascular disease, diabetes and even male-pattern
baldness! An example is the polymorphism in the gene SLC2A9 that not
only explains a signicant proportion of the normal population variation in
serum urate concentration but also predisposes ‘high-risk’ allele carriers
to the development of gout. However, the current reality is that, until we
have a more comprehensive understanding of the full genomic landscape
and knowledge of the complete set of detrimental and protective poly-
morphisms, we cannot accurately assess risk.
Evolutionary selection
Genetic variants play an important role in evolutionary selection, with
advantageous variants resulting in positive selection via improved
reproductive tness, and variations that decrease reproductive tness
becoming excluded through evolution. Given this simple paradigm, it
would be tempting to assume that common variants are all advan-
tageous and all rare variants are pathogenic. Unfortunately, it is often
difcult to classify any common variant as either advantageous or dele-
terious – or, indeed, neutral. Variants that are advantageous in early life
and thus enhance reproductive tness may be deleterious in later life.
There may be variants that are advantageous for survival in particular
conditions (e.g. famine or pandemic) that may be disadvantageous in
more benign circumstances by causing a predisposition to obesity or
autoimmune disorders.
Constitutional genetic disease
Familial genetic disease is caused by constitutional variants, which are
inherited through the germ line. However, different variants in the same
gene can have different consequences, depending on the genetic mech-
anism underlying that disease. About 1% of the human population car-
ries constitutional pathogenic variants that cause disease.
Constructing a family tree
The family tree – or pedigree – is a fundamental tool of the clinical
geneticist, who will routinely take a three-generation family history,
on both sides of the family, enquiring about details of all medical
conditions in family members, consanguinity, dates of birth and
death, and any history of pregnancy loss or infant death. The basic
symbols and nomenclature used in drawing a pedigree are shown
in Fig. 3.8
Fig. 3.8 Drawing a pedigree and patterns of inheritance.
inheritance (see text for details).
SB
Male
Clinically affected
Deceased
individual
(with age at death)
Separated
Consanguinity
Clinically affected,
several diagnoses
Carrier
Positive pre-
symptomatic test
Monozygotic twins
Dizygotic twins
Stillbirth
(with gestation)
Termination
Miscarriage
(with gestation)
Unknown sex
Female
Partners
Recessive inheritance
Dominant inheritance
Mitochondrial DNA disorder
X-linked recessive inheritance
Transmission to 50% of offspring
independent of gender
Consanguinity
Affected males related through
unaffected females
Both sexes affected but only
inherited through female meiosis
I
II
III
IV
I
II
III
IV
I
II
III
IV
V
I
II
III
IV
1
1
1
2
2 3 4
2 3 4 5 6
1 2 3 4
1
1
1
2
2 3 4
2 3 4 5 6
3 6 7
5 6 7 8
1
1
1
2
2 3 4
2 3 4 5 6
1 2 3 4
1
1
1
2
2 3 4
2 3 4 5 6
1 2 3 4
d. 50 y
30 wk
SB
39 wk
16 wk
A B
69.
46 CLINICALGENETICS
Patterns of disease inheritance
Autosomal dominant inheritance
Take some time to draw out the following pedigree:
Anne is referred to Clinical Genetics to discuss her personal history
of colon cancer (she was diagnosed at the age of 46 years) and family
history of colon/endometrial cancer: her mother was diagnosed with
endometrial cancer at the age of 60 years and her cousin through her
healthy maternal aunt was diagnosed with colon cancer in her fties.
Both her maternal grandmother and grandfather died of ‘old age’.
There is no family history of note on her father’s side of the family. He
has one brother and both his parents died of old age, in their eighties.
Anne has two healthy daughters, aged 12 and 14 years, and a healthy
full sister.
This family history is typical of an autosomal dominant condition (see
Fig. 3.8): in this case, a colon/endometrial cancer susceptibility syn-
drome known as Lynch syndrome, associated with disruption of one of
the mismatch repair genes: MSH2, MSH6, MLH1 and PMS2 (see Ch. 23
and Box 3.11).
Features of an autosomal dominant pedigree include:
There are affected individuals in each generation (unless the patho-
genic variant has arisen de novo, i.e. for the rst time in an affected
individual). However, variable penetrance and expressivity can
inuence the number of affected individuals and the severity of dis-
ease in each generation. Penetrance is dened as the proportion of
individuals bearing a mutated allele who develop the disease phe-
notype. The variant is said to be fully penetrant if all individuals who
inherit a variant develop the disease. Expressivity describes the level
of severity of each aspect of the disease phenotype.
Males and females are usually affected in roughly equal numbers
(unless the clinical presentation of the condition is gender-specic,
such as an inherited susceptibility to breast and/or ovarian cancer).
The offspring risk for an individual affected with an autosomal domi-
nant condition is 1 in 2 (or 50%). This offspring risk is true for each preg-
nancy, since half the affected individual gametes (sperm or egg cells) will
contain the affected chromosome/gene and half will contain the normal
chromosome/gene.
There is a long list of autosomal dominant conditions, some of which
are shown in Box 3.4
Autosomal recessive inheritance
As above, take some time to draw a pedigree representing the following:
Mr and Mrs Kent, a non-consanguineous couple, are referred
because their son, Jamie, had severe neonatal liver disease. Included
among the many investigations that the paediatric hepatologist under-
took was testing for α1
-antitrypsin deciency (Box 3.5). Jamie was
shown to have the PiZZ phenotype. Testing conrmed both parents as
carriers with PiMZ phenotypes. In the family, Jamie has an older sister
who has no medical problems. Mr Kent is one of four children with two
brothers and a sister and Mrs Kent has a younger brother. Both sets
of grandparents are alive and well. There is no family history of α1
-an-
titrypsin deciency.
This family history is characteristic of an autosomal recessive disor-
der (see Fig. 3.8), where both alleles of a gene must be mutated before
the disease is manifest in an individual; an affected individual inherits
one mutant allele from each of their parents, who are therefore healthy
carriers for the condition. An autosomal recessive condition might be
suspected in a family where:
Males and females are affected in roughly equal proportions.
Parents are blood-related; this is known as consanguinity. Where
there is consanguinity, the variant are usually homozygous, i.e. the
same mutant allele is inherited from both parents.
Individuals within one sibship in one generation are affected and so
the condition can appear to have arisen ‘out of the blue’.
3.4 Genetic conditions dealt with by clinicians in other
specialties
Name of condition Gene Reference
Autosomal dominant conditions
Autosomal dominant polycystic kidney
disease (ADPKD)
PKD1 (85%),
PKD2 (15%)
p. 579
Box 18.28
Tuberous sclerosis TSC1
TSC2
p. 1114
Marfan syndrome FBN1 p. 445
Long QT syndrome KCNQ1 p. 418
Brugada syndrome SCN5A p. 418
Neurobromatosis type 1 NF1 p. 1185
Box 28.76
Neurobromatosis type 2 NF2 p. 1185
Box 28.76
Hereditary spherocytosis ANK1 p. 956
Vascular Ehlers–Danlos syndrome (EDS
type 4)
COL3A1 p. 980
Hereditary haemorrhagic telangiectasia ENG, ALK1,
GDF2
p. 980
Osteogenesis imperfecta COL1A1,
COL1A2
p. 1058
Charcot–Marie–Tooth disease PMP22, MPZ,
GJB1
p. 1193
Hereditary neuropathy with liability to
pressure palsies
PMP22
Autosomal recessive conditions
Familial Mediterranean fever MEFV p. 76
Mevalonic aciduria (mevalonate kinase
deciency)
MVK p. 76
Autosomal recessive polycystic kidney
disease (ARPKD)
PKHD1 Box 18.28
Kartagener syndrome (primary ciliary
dyskinesia)
DNAI1 Box 17.30
Cystic brosis CFTR1 p. 510
Box 17.30
p. 854
Pendred syndrome SLC26A4 p. 667
Congenital adrenal hyperplasia-21
hydroxylase deciency
CYP21A p. 689
Box 20.27
Haemochromatosis HFE p. 906
Wilson’s disease ATP7B p. 907
Alpha-1-antitrypsin deciency SERPINA1 p. 908
Gilbert syndrome UGT1A1 p. 908
Benign recurrent intrahepatic cholestasis ATP8B1 p. 912
Alpha-thalassaemia HBA1, HBA2 p. 962
Beta-thalassaemia HBB p. 962
Sickle cell disease HBB p. 960
Spinal muscular atrophy SMN1 p. 1171
X-linked conditions
Alport syndrome COL4A5 Box 18.28
p. 577
Primary agammaglobulinaemia BTK p. 74
Haemophilia A (factor VIII deciency) F8 p. 981
Haemophilia B (factor IX deciency) F9 p. 984
Duchenne muscular dystrophy DMD p. 1196
Box 28.90
70.
Genomics, health anddisease 47
3
Examples of some autosomal recessive conditions, discussed else-
where in this book, are shown in Box 3.4
X-linked inheritance
The following is an exemplar of an X-linked recessive pedigree (see Fig. 3.8):
Edward has a diagnosis of Duchenne muscular dystrophy (DMD,
Box 3.6). His parents had suspected the diagnosis when he was 3 years
old because he was not yet walking and there was a family history of
DMD: Edward’s maternal uncle had been affected and died at the age
of 24 years. Edward’s mother has no additional siblings. After Edward
demonstrated a very high creatinine kinase level, the paediatrician also
requested genetic testing, which identied a deletion of exons 2–8 of the
dystrophin gene. Edward has a younger, healthy sister and grandparents
on both sides of the family are well, although the maternal grandmother
has recently developed a cardiomyopathy. Edward’s father has an older
sister and an older brother who are both well.
Genetic diseases caused by variants on the X chromosome have spe-
cic characteristics:
X-linked diseases are mostly recessive and predominantly affect
males who carry the mutant allele. This is because males have
only one X chromosome, whereas females have two (see Fig. 3.1).
Occasionally, female carriers may exhibit signs of an X-linked dis-
ease due to a phenomenon called skewed X-inactivation. All female
embryos, at about 100 cells in size, stably inactivate one of their
two X chromosomes in each cell. Where this inactivation is random,
approximately 50% of the cells will express the genes from one X
chromosome and 50% of cells will express genes from the other.
Where there is a mutant gene, there is often skewing away from the
associated X chromosome, resulting in an unaffected female carrier.
However, if, by chance, there is a disproportionate inactivation of the
normal X chromosome with skewing towards the mutant allele, then
an affected female carrier may be affected (albeit more mildly than
males).
The gene can be transmitted from female carriers to their sons: in
families with an X-linked recessive condition, there are often a num-
ber of affected males related through unaffected females.
Affected males cannot transmit the condition to their sons (but all
their daughters would be carriers).
The risk of a female carrier having an affected child is 25% or half of
her male offspring.
Mitochondrial inheritance
The mitochondrion is the main site of energy production within the
cell. Mitochondria arose during evolution via the symbiotic associa-
tion with an intracellular bacterium. They have a distinctive structure
with functionally distinct inner and outer membranes. Mitochondria
produce energy in the form of adenosine triphosphate (ATP). ATP
is mostly derived from the metabolism of glucose and fat (Fig. 3.9).
Glucose cannot enter mitochondria directly but is rst metabolised
to pyruvate via glycolysis. Pyruvate is then imported into the mito-
chondrion and metabolised to acetyl-co-enzyme A (acetyl-CoA). Fatty
acids are transported into the mitochondria following conjugation with
carnitine and are sequentially catabolised by a process called β-oxida-
tion to produce acetyl-CoA. The acetyl-CoA from both pyruvate and
fatty acid oxidation is used in the citric acid (Krebs) cycle – a series
of enzymatic reactions that produces CO2
, the reduced form of nico-
tinamide adenine dinucleotide (NADH) and the reduced form of avine
adenine dinucleotide (FADH2
). Both NADH and FADH2
then donate
electrons to the respiratory chain. Here these elections are transferred
via a complex series of reactions, resulting in the formation of a pro-
ton gradient across the inner mitochondrial membrane. The gradient
is used by an inner mitochondrial membrane protein, ATP synthase,
to produce ATP, which is then transported to other parts of the cell.
Dephosphorylation of ATP is used to produce the energy required for
many cellular processes.
3.5 Alpha-1-antitrypsin deciency
Inheritance pattern
Autosomal recessive
Genetic cause
Two common variants in the SERPINA1 gene: p.Glu342Lys and p.Glu264Val
Prevalence
1 in 1500–3000 of European ancestry
Clinical presentation
Variable presentation from neonatal period through to adulthood
Neonatal period: prolonged jaundice with conjugated hyperbilirubinaemia or
(rarely) liver disease
Adulthood: pulmonary emphysema and/or cirrhosis. Rarely, the skin disease,
panniculitis, develops
Disease mechanism
SERPINA1 encodes α1
-antitrypsin, which protects the body from the effects of
neutrophil elastase. The symptoms of α1
-antitrypsin deciency result from the
effects of this enzyme attacking normal tissue
Disease variants
M variant: if an individual has normal SERPINA1 genes and produces normal
levels of α1
-antitrypsin, they are said to have an M variant
S variant: p.Glu264Val variant results in α1
-antitrypsin levels reduced to about
40% of normal
Z variant: p.Glu342Lys variant results in very little α1
-antitrypsin
PiZZ: individuals who are homozygous for the p.Glu342Lys variant are likely to
have α1
-antitrypsin deciency and the associated symptoms
PiZS: individuals who are compound heterozygous for p.Glu342Lys and
p.Glu264Val are likely to be affected, especially if they smoke, but usually to a
milder degree
3.6 Duchenne muscular dystrophy
Inheritance pattern
X-linked recessive
Genetic cause
Deletions/variants encompassing/within the DMD (dystrophin) gene located at
Xp21
Prevalence
1 in 3000–4000 live male births
Clinical presentation
Delayed motor milestones
Speech delay
Grossly elevated creatine kinase levels (in the thousands)
Ambulation is usually lost between the ages of 7 and 13 years
Lifespan is reduced, with a mean age of death, usually from respiratory failure,
in the mid-twenties
Cardiomyopathy affects almost all boys with Duchenne muscular dystrophy and
some female carriers
Disease mechanism
DMD encodes dystrophin, a major structural component of muscle
Dystrophin links the internal cytoskeleton to the extracellular matrix
Disease variants
Becker muscular dystrophy, although a separate disease, is also caused by
variants in the dystrophin gene
In Duchenne muscular dystrophy, there is no dystrophin protein, whereas in
Becker muscular dystrophy there is a reduction in the amount or alteration in
the size of the dystrophin protein
Approximately 1 in 4 children born to carriers of an autosomal reces-
sive condition will be affected. The offspring risk for carrier parents is
therefore 25% and the chances of an unaffected child, with an affected
sibling, being a carrier is 2/3.
71.
48 CLINICALGENETICS
L strand
H strand
Outer
membrane
Inner
membrane
NADH
NAD
I
II
III
Q
Cyt
C
IV
NADH
FADH2
Fatty acid
β-oxidation
cycle
Citric acid
(Krebs)
cycle
H+
e
2e
FADH2
FADH2
Lactate
Pyruvate
PDH
Acetyl-CoA
Glucose
22 tRNAs
NADH dehydrogenase 7
subunits
Cytochrome B/C oxidase 4
subunits
2 ribosomal RNA subunits
2 ATP synthase subunits
Intragenic DNA
Inner
membrane
Cristae
Matrix
Outer
membrane
FFA
CPT I
CPT II
Carnitine
Carnitine-FA ester
C
A B
FFA
FAD
2H+
H2O
O2
ATP
ADP
+ Pi
H+
H+
ATP
synthase
Carnitine
e
e
e
Fig. 3.9 Mitochondria.
The membranes create two compartments: the inter-membrane compartment, which plays a crucial role in the electron transport chain, and the inner compartment (or matrix),
which contains mitochondrial DNA and the enzymes responsible for the citric acid (Krebs) cycle and the fatty acid β
contains several copies of a circular double-stranded DNA molecule, which has a non-coding region, and a coding region that encodes the genes responsible for energy production,
conjugated to carnitine by carnitine-palmityl transferase type 1 (CPT I) and, once inside the matrix, are unconjugated by CPT II to release free fatty acids (FFA). These are broken
down by the β-oxidation cycle to produce acetyl-co-enzyme A (acetyl-CoA). Pyruvate can enter the mitochondrion directly and is metabolised by pyruvate dehydrogenase (PDH) to
produce acetyl-CoA. The acetyl-CoA enters the Krebs cycle, leading to the production of NADH and avine adenine dinucleotide (reduced form) (FADH2
), which are used by proteins
in the electron transport chain to generate a hydrogen ion gradient across the inter-membrane compartment. Reduction of NADH and FADH2
by proteins I and II, respectively,
releases electrons (e), and the energy released is used to pump protons into the inter-membrane compartment. Coenzyme Q10
/ubiquinone (Q) is an intensely hydrophobic electron
carrier that is mobile within the inner membrane. As electrons are exchanged between proteins in the chain, more protons are pumped across the membrane, until the electrons
reach complex IV (cytochrome oxidase), which uses the energy to reduce oxygen to water. The hydrogen ion gradient is used to produce ATP by the enzyme ATP synthase, which
consists of a proton channel and catalytic sites for the synthesis of ATP from ADP. When the channel opens, hydrogen ions enter the matrix down the concentration gradient, and
energy is released that is used to make ATP. (ATP = adenosine triphosphate; NADH = the reduced form of nicotinamide adenine dinucleotide)
energy production, e.g. most forms of Leigh syndrome (although Leigh
syndrome may also be caused by a mitochondrial gene variant).
The inheritance of mtDNA disorders is characterised by transmis-
sion from females, but males and females generally are equally affected
(see Fig. 3.8). Unlike the other inheritance patterns mentioned above,
mitochondrial inheritance has nothing to do with meiosis but reects
the fact that mitochondrial DNA is transmitted by oöcytes: sperm do
not contribute mitochondria to the zygote. Mitochondrial disorders tend
to be variable in penetrance and expressivity within families, and this is
mostly accounted for by the fact that only a proportion of multiple mtDNA
molecules within mitochondria contain the causal variant (the degree of
mtDNA heteroplasmy).
Each mitochondrion contains 2–10 copies of a 16-kilobase (kB) dou-
ble-stranded circular DNA molecule (mtDNA). This mtDNA contains 13
protein-coding genes, all involved in the respiratory chain, and the ncRNA
genes required for protein synthesis within the mitochondria (see Fig. 3.9).
The mutational rate of mtDNA is relatively high due to the lack of protection
by chromatin. Several mtDNA diseases characterised by defects in ATP
production have been described. Mitochondria are most numerous in cells
with high metabolic demands, such as muscle, retina and the basal gan-
glia, and these tissues tend to be the ones most severely affected in mito-
chondrial diseases (Box 3.7). There are many other mitochondrial diseases
that are caused by pathogenic variants in nuclear genes, which encode
proteins that are then imported into the mitochondrion and are critical for
72.
Genomics, health anddisease 49
3
be active while the maternally inherited may be silenced, or vice versa.
Variants within imprinted loci lead to an unusual pattern of inheritance
where the phenotype is manifest only if inherited from the parent who
contributes the transcriptionally active allele. Examples of imprinting dis-
orders are given in Box 3.8
Somatic genetic disease
Somatic variants are not inherited but instead occur during post-zygotic
mitotic cell divisions at any point from embryonic development to late
adult life. An example of this phenomenon is polyostotic brous dysplasia
(McCune–Albright syndrome), in which a somatic variant in the GS
alpha
gene causes constitutive activation of downstream signalling, resulting in
focal lesions in the skeleton and endocrine dysfunction.
The most important example of human disease caused by somatic var-
iants is cancer (see Ch. 7). Here, ‘driver mutations’ occur within genes that
are involved in regulating cell division or apoptosis, resulting in abnormal cell
growth and tumour formation. The two general categories of cancer-caus-
ing variant are gain-of-function variants in growth-promoting genes (onco-
genes) and loss-of-function variants in growth-suppressing genes (tumour
suppressor genes). Whichever mechanism is acting, most tumours require
an initiating variant in a single cell that can then escape from normal growth
controls. This cell replicates more frequently or fails to undergo programmed
death, resulting in clonal expansion. As the size of the clone increases, one
or more cells may acquire additional variants that confer a further growth
advantage, leading to proliferation of these subclones, which may ultimately
result in aggressive metastatic cancer. The cell’s complex self-regulating
machinery means that more than one variant is usually required to produce
a malignant tumour (see Fig. 7.3). For example, if a variant results in acti-
vation of a growth factor gene or receptor, then that cell will replicate more
frequently as a result of autocrine stimulation. However, this mutant cell will
still be subject to normal cell-cycle checkpoints to promote DNA integrity in
its progeny. If additional variants in the same cell result in defective cell-cycle
checkpoints, however, it will rapidly accumulate further genomic variants,
which may allow completely unregulated growth and/or separation from
its matrix and cellular attachments and/or resistance to apoptosis. As cell
growth becomes increasingly dysregulated, cells de-differentiate, lose their
response to normal tissue environment and cease to ensure appropriate
mitotic chromosomal segregation. These processes combine to generate
the classical malignant characteristics of disorganised growth, variable levels
of differentiation and numerical and structural chromosome abnormalities.
An increase in somatic variant rate can occur on exposure to external muta-
gens, such as ultraviolet light or cigarette smoke, or if the cell has defects in
DNA repair systems. Cancer is thus a disease that affects the fundamental
processes of molecular and cell biology.
3.7 The structure of the respiratory chain complexes and
the diseases associated with their dysfunction
Complex Enzyme nDNA
subunits1
mtDNA
subunits2
Diseases
I NADH
dehydrogenase
38 7 MELAS, MERRF bilateral
striatal necrosis,
LHON, myopathy and
exercise intolerance,
parkinsonism, Leigh
syndrome, exercise
myoglobinuria,
leucodystrophy/myoclonic
epilepsy
II Succinate
dehydrogenase
4 0 Phaeochromocytoma,
Leigh syndrome
III Cytochrome
bc1
complex
10 1 Parkinsonism/MELAS,
cardiomyopathy,
myopathy, exercise
myoglobinuria, Leigh
syndrome
IV Cytochrome c
oxidase
10 3 Sideroblastic
anaemia, myoclonic
ataxia, deafness,
myopathy, MELAS,
MERRF mitochondrial
encephalomyopathy,
motor neuron
disease-like, exercise
myoglobinuria, Leigh
syndrome
V ATP synthase 14 2 Leigh syndrome, NARP,
bilateral striatal necrosis
1
nDNA subunits. 2
mtDNA subunits = number of different protein subunits in each complex that
are encoded in the nDNA and mtDNA, respectively.
(ATP = adenosine triphosphate; LHON = Leber hereditary optic neuropathy; MELAS =
myopathy, encephalopathy, lactic acidosis and stroke-like episodes; MERRF = myoclonic
epilepsy and ragged red bres; mtDNA = mitochondrial DNA; NADH = the reduced form of
nicotinamide adenine dinucleotide; NARP = neuropathy, ataxia and retinitis pigmentosa;
nDNA = nuclear DNA)
3.8 Imprinting disorders
Disorder Locus Genes Notes
Beckwith–Wiedemann
syndrome
11p15 CDKN1C,
IGF2, H19
Increased growth, macroglossia, hemihypertrophy, abdominal wall defects, ear lobe pits/creases and
increased susceptibility to developing childhood tumours
Prader–Willi syndrome 15q11–q13 SNRPN,
Necdin and
others
Obesity, hypogonadism and learning disability. Lack of paternal contribution (due to deletion of paternal
15q11–q13, or inheritance of both chromosome 15q11–q13 regions from the mother)
Angelman syndrome (AS) 15q11–q13 UBE3A Severe mental retardation, ataxia, epilepsy and inappropriate laughing bouts. Due to loss-of-function
variants in the maternal UBE3A gene. The neurological phenotype results because most tissues express
both maternal and paternal alleles of UBE3A, whereas the brain expresses predominantly the maternal
allele
Pseudohypoparathyroidism 20q13 GNAS1 Inheritance of the variant from the mother results in hypocalcaemia, hyperphosphataemia, raised
parathyroid hormone (PTH) levels, ectopic calcication, obesity, delayed puberty and shortened 4th and
5th metacarpals (the syndrome known as Albright hereditary osteodystrophy, AHO). When the gene
variant is inherited from the father, PTH, calcium and phosphate levels are normal but the other features
are present. These differences are due to the fact that, in the kidney (the main target organ through
which PTH regulates serum calcium and phosphate), the paternal allele is silenced and the maternal
allele is expressed, whereas both alleles are expressed in other tissues
Imprinting
Several chromosomal regions (loci) have been identied where gene
expression is inherited in a parent-of-origin-specic manner; these are
called imprinted loci. Within these loci the paternally inherited gene may
73.
50 CLINICALGENETICS
Interrogating the genome: the changing
landscape of genomic technologies
Looking at chromosomes
The analysis of metaphase chromosomes by light microscopy was the
mainstay of clinical cytogenetic analysis for decades, the aim being to
detect gain or loss of whole chromosomes (aneuploidy) or large chromo-
somal segments (>4 million bp). More recently, genome-wide microarrays
(array comparative genomic hybridisation or array CGH) have replaced
chromosome analysis, allowing rapid and precise detection of segmental
gain or loss of DNA throughout the genome (see Box 3.3). Microarrays
consist of grids of multiple wells containing short DNA sequences (refer-
ence DNA) that are complementary to known sequences in the genome.
Patient and reference DNA are each labelled with a coloured uorescent
dye (generally, patient DNA is labelled with a green uorescent dye and
reference DNA with a red uorescent dye) and added to the microar-
ray grid. Where there is an equal quantity of patient and reference DNA
bound to the spot, this results in yellow uorescence. Where there is too
much patient DNA (representing a duplication of a chromosome region),
the spot will be greener; it will be more red (appears orange) where there
is 2:1 ratio of the control:patient DNA (representing heterozygous dele-
tion of a chromosome region; Fig. 3.10).
Array CGH and other array-based approaches can detect small chro-
mosomal deletions and duplications. They are also generally more sen-
sitive than conventional karyotyping at detecting mosaicism (where there
are two or more populations of cells, derived from a single fertilised egg,
with different genotypes). However, array-based approaches will not
detect balanced chromosome rearrangements where there is no loss or
gain of genes/chromosome material, such as balanced reciprocal trans-
locations, or a global increase in copy number, such as triploidy.
The widespread use of array-based approaches has brought a num-
ber of challenges for clinical interpretation, including the identication of
copy number variants (CNVs) of uncertain clinical signicance, CNVs of
variable penetrance and incidental ndings. A CNV of uncertain clinical
signicance describes a loss or gain of chromosome material where
there are insufcient data to conclude whether or not it is associated with
a learning disability and/or medical problems. While this uncertainty can
be difcult to prepare families for and can be associated with consider-
able anxiety, it is likely that there will be greater clarity in the future as we
generate larger CNV datasets.
A CNV of variable penetrance, also known as a neurosusceptibility
locus, describes a chromosome deletion or duplication associated with
a lower threshold for manifesting a learning disability or autistic spectrum
disorder. CNVs of variable penetrance are therefore identied at greater
frequencies among individuals with a learning disability and/or autistic
spectrum disorder than in the general population. The current under-
standing is that additional modifying factors (genetic, environmental or
stochastic) must inuence the phenotypic expression of these neurosus-
ceptibility loci.
Finally, an incidental CNV nding describes a deletion or duplication
encompassing a gene or genes that are causative of a phenotype or risk
unrelated to the presenting complaint. For instance, if, through the array
CGH investigation for an intellectual disability, a deletion encompassing
the BRCA1 gene were identied, this would be considered an incidental
nding.
Looking at genes
Gene amplication: polymerase chain reaction
The polymerase chain reaction (PCR) is a fundamental laboratory tech-
nique that amplies targeted sections of the human genome for fur-
ther analyses – most commonly, DNA sequencing. The method utilises
thermal cycling: repeated cycles of heating and cooling allow the initial
separation of double-stranded DNA into two single strands (known as
denaturation), each of which serves as a template during the subsequent
replication step, guided by primers designed to anneal to a specic
genomic region. This cycle of heating/cooling and denaturation/replica-
tion is repeated many times, resulting in the exponential amplication of
DNA between primer sites (Fig. 3.11).
Gene sequencing
In the mid-1970s, a scientist called Fred Sanger pioneered a DNA
sequencing technique (‘Sanger sequencing’) that determined the pre-
cise order and nucleotide type (thymine, cytosine, adenine and guanine)
in a molecule of DNA. Modern Sanger sequencing uses uorescently
labelled, chain-terminating nucleotides that are sequentially incorporated
into the newly synthesised DNA, generating multiple DNA chains of dif-
fering lengths. These DNA chains are subject to capillary electrophoresis,
which separates them by size, allowing the fragments to be ‘read’ by a
laser and producing a sequence chromatogram that corresponds to the
target sequence (Fig. 3.12). Although transformative, Sanger sequencing
was difcult and costly to scale, as exemplied by the Human Genome
Project, which took 12 years to sequence the entire human genome at
a cost approaching 3 billion US$. Recently, DNA sequencing has been
transformed again by a group of technologies collectively known as
‘next-generation sequencing’ (NGS; Fig. 3.13). This refers to a family
of post-Sanger sequencing technologies that utilise the same ve basic
principles:
Library preparation: DNA samples are fragmented (by enzyme
cleavage or ultrasound) and then modied with a custom adapter
sequence.
Amplication: the library fragment is amplied to produce DNA clus-
ters, each originating from a single DNA fragment. Each cluster will
act as a single sequencing reaction.
Capture: if an entire genome is being sequenced, this step will not
be included. The capture step is required if targeted resequencing is
necessary, such as for a panel gene test or an exome (Box 3.9).
CGH
Patient
DNA
Label DNA
with different
fluorescent
dyes
Mix equimolar
amounts of
labelled DNA
Apply DNA mix to
glass slide with
high-density array
of different DNA
probes with known
location in the
human genome
Patient/control
ratio = 0.5:1
→ deletion of
patient DNA
Patient/control
ratio = 1.5:1
→ duplication
of patient DNA
Patient/control
ratio = 1:1
→ normal
Normal control
DNA
Fig. 3.10 Detection of chromosome abnormalities by comparative genomic hybridisation (CGH). Deletions and duplications are detected by looking for deviation
from the 1:1 ratio of patient and control DNA in a microarray. Ratios in excess of 1 indicate duplications, whereas ratios below 1 indicate deletions.
74.
Interrogating the genome:the changing landscape of genomic technologies 51
3
Sequencing: each DNA cluster is simultaneously sequenced and
the data from each captured; this is known as a ‘read’ and is usu-
ally between 50 and 300 bases long (see Box 3.10 for a detailed
description of the three most commonly used sequencing methods:
synthesis, ligation and ion semiconductor sequencing).
Alignment and variant identication: specialised software analyses
read sequences and compare the data to a reference template.
This is known as ‘alignment’ or ‘mapping’ and, although there are
approximately 3 billion bases in the human genome, allows the
remarkably accurate determination of the genomic origin where
a read consists of 25 nucleotides or more. Variants are identied
as differences between the read and the reference genome. For
instance, if there is a different nucleotide in half the reads at a given
position compared to the reference genome, this is likely to repre-
sent a heterozygous base substitution. The number of reads that
align at a given point is called the ‘depth’ or ‘coverage’. The higher
the read depth, the more accurate the variant call. However, in gen-
eral, a depth of 30 or more reads is generally accepted as producing
diagnostic-grade results.
Rather than sequencing only one small section of DNA at a time,
NGS allows the analysis of many hundreds of thousands of DNA
strands in a single experiment and so is also commonly referred to as
multiple parallel sequencing technology. Today’s NGS machines can
sequence the entire human genome in a single day at a cost of less
than 1000 US$.
NGS capture
Although we now have the capability to sequence the entire genome in a
single experiment, whole-genome sequencing is not always the optimal
use of NGS. NGS capture refers to the ‘pull-down’ of a targeted region
of the genome and may constitute several to several hundred genes
associated with a given phenotype (a gene panel), the exons of all known
coding genes (an exome), or the exons of all coding genes known to
be associated with disease (a clinical exome). Each of these targeted
resequencing approaches is associated with a number of advantages
and disadvantages (see Box 3.9). In order for NGS to be used for opti-
mal patient benet, it is essential for the clinician to have a good under-
standing of which test is the best one to request in any given clinical
presentation.
Challenges of NGS technologies
Genomic technologies are transforming the way that we practise med-
icine, and ever faster and cheaper DNA sequencing offers increasing
opportunities to prevent, diagnose and treat disease. However, genomic
technologies are not without their challenges: for instance, storing the
enormous quantities of data generated by NGS. While the A, C, T and G
of our genomic code could be stored on the memory of a smartphone,
huge computers, able to store several petabytes of data (where 1 peta-
byte is 1 million gigabytes of data), are required to store the information
needed to generate each individual’s genome.
Even if we can store and handle these huge datasets successfully,
we then need to be able to sift through the millions of normal variants to
identify the single (or, rarely, several) pathogenic, disease-causing variant.
While this can, to an extent, be achieved through the application of com-
plex algorithms, these take time and considerable expertise to develop
and are not infallible.
Furthermore, even after these data have been sifted by bioinforma-
ticians, it is highly likely that clinicians will be left with some variants for
which there are insufcient data to enable their denitive categorisation
as either pathogenic or non-pathogenic. This may be because we simply
do not know enough about the gene, because the particular variant has
not previously been reported and/or it is identied in an unaffected par-
ent. These variants must be interpreted with caution and, more usually,
their interpretation will require input from a genetics expert in the con-
text of the clinical presentation, where an ‘innocent until proven guilty’
approach is often adopted.
Cool ~60°C
DNA
sample
DNA strands
separate
Primers
bind to
DNA
DNA
replicated
Heat 95°C
Heat 95°C
Cool ~60°C
Repeat cycles
20–30 times
108
107
106
105
104
103
102
10
0 5 10 15
PCR cycles
Exponential amplification
of DNA between primer sites
DNA
molecules
20 25 30
DNA strands
separate
Primers bind
to DNA
Cycle
no. 1
DNA
replicated
Heat ~72°C
Heat ~72°C
Polymerase
+ dNTPs
Primers
Cycle
no. 2
Fig. 3.11 The polymerase chain reaction (PCR). PCR involves adding a tiny amount
of the patient’s DNA to a reaction containing primers (short oligonucleotides 18–21bp
in length, which bind to the DNA anking the region of interest) and deoxynucleotide
phosphates (dATP, dCTP, dGTP, dTTP), which are used to synthesise new DNA and a
heat-stable polymerase. The reaction mix is rst heated to 95°C, which causes the
double-stranded DNA molecules to separate. The reaction is then cooled to 50–60°C,
which allows the primers to bind to the target DNA. The reaction is then heated to
72°C, at which point the polymerase starts making new DNA strands. These cycles
are repeated 20–30 times, resulting in exponential amplication of the DNA fragment
between the primer sites. The resulting PCR products can then be used for further
analysis – most commonly, DNA sequencing (see Fig. 3.12).
75.
52 CLINICALGENETICS
Finally, if we are to interrogate the entire genome or even the exome,
it is foreseeable that we will routinely identify ‘incidental’ or secondary
ndings – in other words, ndings not related to the initial diagnostic
question. Whether or not to report these incidental ndings is a topic of
much debate in the UK, although there is growing support to feedback
incidental ndings where there is an intervention that could slow or halt
development/progress of the associated disease.
Uses of NGS
NGS is now frequently used, within diagnostic laboratories, to identify
base substitutions and indels (although the latter were initially problem-
atic). The current NGS challenge is to detect large deletions or duplica-
tions spanning several hundreds or thousands of bases and therefore
exceeding any single read copy number variants and triplet repeat dis-
orders such as those that cause Huntington’s disease, myotonic dys-
trophy and fragile X syndrome, although techniques to detect both are
improving (see Box 3.2). Increasingly, however, disorders caused by both
copy number variants and triplet repeat expansions are being diagnosed
using sophisticated computational methods. Additional potential uses of
NGS include detection of balanced and unbalanced translocations and
mosaicism: NGS has proved remarkably sensitive at detecting the latter
when there is high read coverage for a given region. Of note, however,
NGS is still not able to interrogate the epigenome and so will not iden-
tify conditions caused by a disruption of imprinting, such as Beckwith–
Wiedemann, Silver–Russell, Angelman and Prader–Willi syndromes (see
Boxes 3.8 and 3.2).
Third-generation sequencing
Increasingly, third-generation or single-molecule sequencing is entering
the diagnostic arena. As with next- or second-generation sequencing,
a number of different platforms are commercially available. One of the
most successful is SMRT technology (single-molecule sequencing in
real time), developed by Pacic Biosciences. This system utilises a sin-
gle-stranded DNA molecule (as compared to the amplied clusters used
in NGS), which acts as a template for the sequential incorporation, using
a polymerase, of uorescently labelled nucleotides. As each complemen-
tary nucleotide is added, the uorescence (and therefore the identity of
the nucleotide) is recorded before it is removed and another nucleotide
is added.
A key advantage of third-generation sequencing is the long length of
the read it generates: in the region of 10–15 kilobases. It is also cheaper
than NGS, as fewer reagents are required. Given these inherent advan-
tages, third-generation sequencing is likely to supersede NGS in the
near future. Given the confusion surrounding the terminology of NGS
and third-generation sequencing, these technologies are increasingly
referred to as ‘massively parallel sequencing’.
Genomics and clinical practice
Genomics and health care
Genomics in rare neurodevelopmental disorders
Although, by denition, the diagnosis of a rare disorder is made infre-
quently, rare diseases, when considered together, affect about 3 million
people in the UK, the majority of whom are children. NGS has trans-
formed the ability to diagnose individuals affected by a rare disease.
Whereas previously, when we were restricted to the sequential analysis
of single genes, a clinician would need to make a clinical diagnosis in
order to target testing, NGS allows the interrogation of multiple genes in
a single experiment. This might be done through a gene panel, a clinical
exome or an exome (see Box 3.9 and p. 51), and has increased the
diagnostic yield in neurodevelopmental disorders to 35%–50%. Not only
does the identication of the genetic cause of a rare disorder potentially
provide families with answers, prognostic information and the opportu-
nity to meet and derive support from other affected families but also it
can provide valuable information for those couples planning further chil-
dren and wishing to consider prenatal testing in the future.
Genomics and common disease
Most common disorders are determined by interactions between a num-
ber of genes and the environment. In this situation, the genetic contribu-
tion to disease is termed polygenic. Until recently, very little progress had
been made in identifying the genetic variants that predispose to com-
mon disorders, but this has been changed by the advent of genome-
wide association studies. A GWAS typically involves genotyping many
(>500000) genetic markers (SNPs) spread across the genome in a large
group of individuals with the disease and in controls. By comparing the
SNP genotypes in cases and controls, it is possible to identify regions of
the genome, and therefore genes, more strongly associated with a given
Capillary
electrophoresis
Largest fragments
migrate slowest
Smallest fastest
Laser
fluorescence
detector
ddTTP
ddCTP
ddATP
ddGTP
DNA
sample
PCR
DNA sequence chromatogram
Key
Fragments detected by laser fluorescence
New DNA molecules terminated by
incorporation of ddNTP
Polymerase
+ ddNTPs
Primers
Fig. 3.12 Sanger sequencing of DNA, which is very widely used in DNA diagnostics. This is performed using polymerase chain reaction (PCR)-amplied fragments
of DNA corresponding to the gene of interest. The sequencing reaction is carried out with a combination dNTP and uorescently labelled di-deoxy-dNTP (ddATP, ddTTP, ddCTP
and ddGTP), which become incorporated into the newly synthesised DNA, causing termination of the chain at that point. The reaction products are then subject to capillary
electrophoresis and the different-sized fragments are detected by a laser, producing a sequence chromatogram that corresponds to the target DNA sequence.
76.
Genomics and clinicalpractice 53
3
SNP prole and therefore more likely to contribute to the disease under
study. Increasingly there is a move to develop disease-specic polygenic
risk scores (PRS) in order to stratify individual risk and determine person-
alised management strategies. Whilst not yet in mainstream practice, it
is likely that PRS will be integrated into clinical management in the next
few years.
Genomics and obstetrics
Prenatal genetic testing may be performed where a pregnancy is consid-
ered at increased risk of being affected with a genetic condition, either
because of the ultrasound/biochemical screening results or because
of the family history. While invasive tests, such as amniocentesis and
chorionic villus sampling, have been the mainstay of prenatal diagnosis
for many years, they are increasingly being superseded by non-invasive
CCGATATCTAGCTTA
ATATCTAGC
CG TAGC
TATCTAGC
CCG TAGCTAGCTTA
1 Library preparation
2 Cluster amplification
Genomic DNA
Fragmentation
Adapter ligation
Flow cell
Amplification
3 Sequencing
Reads
Reference genome
4 Alignment and variant interpretation
G
T
A
C
A
A
Fig. 3.13 Sequencing by synthesis as used in the Illumina system. (1)
Library preparation: DNA is fragmented and specialist adapters are ligated to the
fragmented ends. (2) Cluster amplication: the library is loaded to a ow cell and the
adapters hybridise to the ow-cell surface. Each bound fragment is hybridised. (3)
Sequencing. (4) Alignment and variant interpretation: reads are aligned to a reference
sequence using complex software and differences between reference and case
genomes are identied.
3.9 The advantages and disadvantages of whole-genome
sequencing,whole-exome sequencing and gene panels
Test Advantages Disadvantages
Whole-genome
sequencing (WGS)
The most
comprehensive
analysis of the
genome available
More even coverage
of genes, allowing
better identication of
dosage abnormalities
Will potentially detect
all gene variants,
including intronic
variants
More expensive to
generate and store
Will detect millions of
variants in non-coding
DNA, which can be very
difcult to interpret
Associated with a
greater risk of identifying
incidental ndings
Shallow sequencing (few
reads per gene) and so
less sensitive and less
able to detect mosaicism
Whole-exome
sequencing (WES)
Cheaper than whole-
genome sequencing
Analysis is not
restricted to only
those genes known
to cause a given
condition
Fewer variants
detected than in
WGS and so easier
interpretation
Deeper sequencing
than WGS increases
sensitivity and
detection of
mosaicism
Less even coverage
of the genome and so
dosage abnormalities are
more difcult to detect
Less comprehensive
analysis (1%–2% of the
genome) than WGS
Increased risk of
identifying incidental
ndings over targeted
gene sequencing
Gene panels Cost-effective
Very deep
sequencing,
increasing the
chances of mosaicism
being detected
Fewer variants
detected and so data
easier to interpret
As analysis is
restricted to known
genes, the likelihood
of a variant being
pathogenic is greatly
increased
Will only detect variation
in genes known to cause
a given condition
Difcult to add new genes
to the panel as they are
discovered
testing of cell-free fetal DNA (cffDNA), originating from placental tropho-
blasts and detectable in the maternal circulation from 4–5 weeks’ gesta-
tion; it is present in sufcient quantities for testing by 9 weeks.
Non-invasive prenatal testing (NIPT): the sequencing and quantica-
tion, using NGS, of cffDNA chromosome-specic DNA sequences
to identify trisomy 13, 18 or 21. The accuracy of NIPT in detecting
pregnancy-specic aneuploidy approaches 98%. A false-negative
result can occur when there is too little cffDNA (possibly due to early
gestation or high maternal body mass index) or when aneuploidy
has arisen later in development and is conned to the embryo and
not represented in the placenta. False positives can occur with
conned placental mosaicism (describing aneuploidy in the pla-
centa, not the fetus) or with an alternative cause of aneuploidy in the
maternal circulation, such as cell-free tumour DNA.
77.
54 CLINICALGENETICS
3.11 Inherited cancer predisposition syndromes
Syndrome name Gene Associated cancers Additional clinical features
Birt–Hogg–Dubé syndrome FLCN Renal tumour (oncocytoma, chromophobe (and mixed),
renal cell carcinoma)
Fibrofolliculoma
Trichodiscoma
Pulmonary cysts
Breast/ovarian hereditary
susceptibility
BRCA1
BRCA2
Breast carcinoma
Ovarian carcinoma
Pancreatic carcinoma
Prostate carcinoma
Cowden syndrome PTEN Breast carcinoma
Thyroid carcinoma
Endometrial carcinoma
Macrocephaly
Intellectual disability/autistic spectrum disorder
Trichilemmoma
Acral keratosis
Papillomatous papule
Thyroid cyst
Lipoma
Haemangioma
Intestinal hamartoma
Gorlin syndrome/basal cell
naevus syndrome
PTCH1 Basal cell carcinoma
Medulloblastoma
Odontogenic keratocyst
Palmar or plantar pits
Falx calcication
Rib abnormalities (e.g. bid, fused or missing ribs)
Macrocephaly
Cleft lip/palate
Li–Fraumeni syndrome TP53 Sarcoma (e.g. osteosarcoma, chondrosarcoma,
rhabdomyosarcoma)
Breast carcinoma
Brain cancer (esp. glioblastoma)
Adrenocortical carcinoma
Brain
3.10 Next-generation sequencing methods
Sequencing by synthesis (Fig. 3.13)
The most frequently used NGS method
Used in Illumina systems (commonly used in diagnostic laboratories)
Uses uorescently labelled, terminator nucleotides that are sequentially incorporated into a growing DNA chain
Library DNA samples (fragmented DNA anked by DNA adapter sequences) are anchored to a ow cell by hybridisation of the DNA adapter sequence to probes on the ow-
cell surface
Amplication occurs by washing the ow cell in a mixture containing all four uorescently labelled terminator nucleotides: A, C, T and G
Once the nucleotide, complementary to the rst base of the DNA template, is incorporated, no further nucleotides can be added until the mixture is washed away
The nucleotide terminator is shed and the newly incorporated nucleotide reverts to a regular, non-uorescent nucleotide that can be extended
The process is then repeated with the incorporation of a second base etc.
Sequencing by synthesis is therefore space- and time-dependent: a sensor will detect the order of uorescent emissions for each spot on the plate (representing the cluster)
and determine the sequence for that read
Sequencing by ligation
Used in SOLiD systems
Uses DNA ligase rather than DNA polymerase (as is used in sequencing by synthesis) and short oligonucleotides (as opposed to single nucleotides)
Library DNA samples are washed in a mixture containing oligonucleotide probes representing 4–16 dinucleotide sequences. Only one nucleotide in the probe is uorescently
labelled
The complementary oligo probes will hybridise, using DNA ligase, to the target sequence, initially at a primer annealed to the anchor site and then progressively along the
DNA strand
After incorporation of each probe, uorescence is measured and the dye is cleaved off
Eventually, a new strand is synthesised (composed of a series of the oligo probes)
A new strand is then synthesised but is offset by one nucleotide
The process is repeated a number of times (5 rounds in the SOLiD system), providing overlapping templates that are analysed and a composite of the target sequence
determined
Ion semiconductor sequencing
When a nucleotide is incorporated into a growing DNA strand, a hydrogen ion is released that can be detected by an alteration in the pH of the solution. This hydrogen ion
release forms the basis of ion semiconductor sequencing
Each amplied DNA cluster is located above a semiconductor transistor, capable of detecting differences in the pH of the solution
The DNA cluster is washed in a mixture containing only one type of nucleotide
If the correct nucleotide, complementary to the next base on the DNA template, is in the mixture and incorporated, a hydrogen ion is released and detected
If a homopolymer (sequence of two or more identical nucleotides) is present, this will be detected as a decrease in pH proportionate to the number of identical nucleotides in
the sequence
78.
Genomics and clinicalpractice 55
3
Syndrome name Gene Associated cancers Additional clinical features
Lynch syndrome/hereditary
non-polyposis colon cancer
MLH1
MSH2
MSH6
PMS2
Colorectal carcinoma (majority right-sided)
Endometrial carcinoma
Gastric carcinoma
Cholangiocarcinoma
Ovarian carcinoma (esp. mucinous)
Multiple endocrine neoplasia 1 MEN1 Parathyroid tumour
Endocrine pancreatic tumour
Anterior pituitary tumour
Lipoma
Facial angiobroma
Multiple endocrine neoplasia 2a
and 2b (formerly known as type
2 and 3, respectively)
RET Medullary thyroid tumour
Phaeochromocytoma
Parathyroid tumour
Polyposis, familial adenomatous
(FAP)
APC Colorectal adenocarcinoma (FAP is characterised by
thousands of polyps from the second decade; without
colectomy, malignant transformation of at least one of
these polyps is inevitable)
Duodenal carcinoma
Hepatoblastoma
Desmoid tumour
Congenital hypertrophy of the retinal pigment epithelium
(CHRPE)
Polyposis, MYH-associated MYH
(MUTYH)
Colorectal adenocarcinoma
Duodenal adenocarcinoma
Retinoblastoma, familial RB1 Retinoblastoma
Osteosarcoma
3.11 Inherited cancer predisposition syndromes
Non-invasive prenatal diagnosis (NIPD): the identication of a fetal
single-gene defect that either has been paternally inherited or has
arisen de novo and so is not identiable in the maternal genome.
Examples of conditions that are currently amenable to NIPD include
achondroplasia and the craniosynostoses. Increasingly, however,
NIPD is being used for autosomal recessive conditions such as
cystic brosis, where parents are carriers for different gene variants
and research is under way to perform more agnostic fetal gene
sequencing through whole-exome or whole-genome sequencing.
Where a genetic diagnosis is known in a family, a couple may opt to
undertake pre-implantation genetic diagnosis (PGD). PGD is used as an
adjunct to in vitro fertilisation and involves the genetic testing of a single
cell from a developing embryo, prior to implantation.
Genomics and oncology
Until recently, individuals were stratied to genetic testing if they pre-
sented with a personal and/or family history suggestive of an inherited
cancer predisposition syndrome (Box 3.11). Relevant clinical information
included the age of cancer diagnosis and number/type of tumours. For
example, the diagnosis of bilateral breast cancer in a woman in her thir-
ties with a mother who had ovarian cancer in her forties is suggestive
of BRCA1/2-associated familial breast/ovarian cancer. In many familial
cancer syndromes, somatic gene variants act together with an inherited
variant to cause specic cancers (p. 49). Familial cancer syndromes may
be due to germ-line loss-of-function variants in tumour suppressor genes
encoding DNA repair enzymes or proto-oncogenes. At the cellular level,
loss of one copy of a tumour suppressor gene does not have any func-
tional consequences, as the cell is protected by the remaining normal
copy. However, a somatic variant affecting the normal allele is likely to
occur in one cell at some point during life, resulting in complete loss of
tumour suppressor activity and a tumour developing by clonal expansion
of that cell. This two-hit mechanism (one inherited, one somatic) for can-
cer development is known as the Knudson hypothesis. It explains why
tumours may not develop for many years (or ever) in some members of
these cancer-prone families. In DNA repair diseases, the inherited vari-
ants increase the somatic mutation rate. Autosomal dominant variants
in genes encoding components of specic DNA repair systems are
relatively common causes of familial colon cancer and breast cancer
(e.g. BRCA1).
Increasingly, genetics is moving into the mainstream, becoming inte-
grated into routine oncological care as new gene-specic treatments are
introduced. Testing for a genetic predisposition to cancer is therefore mov-
ing from the domain of clinical genetics, where it has informed diagnosis,
cascade treatment and screening/prophylactic management, to oncology,
where it is informing the immediate management of the patient following
cancer diagnosis. This is exemplied by BRCA1 and BRCA2 (BRCA1/2)-
related breast cancer. Previously, women with a pathogenic variant in
either the BRCA1 or BRCA2 gene would have received similar rst-line
chemotherapy to women with a sporadic breast cancer without a known
genetic association. More recently, it has been shown that BRCA1/2 muta-
tion-positive tumours are sensitive to poly ADP ribose polymerase (PARP)
inhibitors. PARP inhibitors block the single-strand break-repair pathway. In
a BRCA1/2 mutation-positive tumour – with compromised double-strand
break repair – the additional loss of the single-strand break-repair pathway
will drive the cell towards apoptosis. Indeed, PARP inhibitors have been
shown to be so effective at destroying BRCA1/2 mutation-positive tumour
cells, and with such minimal side-effects, that BRCA1/2 gene testing is
increasingly determining patient management. It is likely, with a growing
understanding of the genomic architecture of tumours, increasing acces-
sibility of NGS and an expanding portfolio of gene-directed therapies, that
testing for many of the other inherited cancer susceptibility genes will, in
time, move into the mainstream.
Genomics in infectious disease
NGS technologies are also transforming infectious disease. Given that
a microbial genome can be sequenced within a single day at a current
cost of less than 100US$, microbiologists are able to identify a causa-
tive microorganism and target effective treatment rapidly and accurately.
Moreover, microbial genome sequencing enables the effective surveil-
lance of infections to reduce and prevent transmission. Finally, an under-
standing of the microbial genome will drive the development of vaccines
and antibiotics, essential in an era characterised by increasing microbial
resistance to established antibiotic agents.
79.
56 CLINICALGENETICS
Treatment of genetic disease
Pharmacogenomics
Pharmacogenomics is the science of dissecting the genetic determi-
nants of drug kinetics and effects using information from the human
genome. For more than 50 years it has been appreciated that poly-
morphic variants within genes can affect individual responses to some
drugs, such as loss-of-function variants in CYP2D6 that cause hyper-
sensitivity to debrisoquine, an adrenergic-blocking medication formerly
used for the treatment of hypertension, in 3% of the population. This
gene is part of a large family of highly polymorphic genes encoding
cytochrome P450 proteins, mostly expressed in the liver, which deter-
mine the metabolism of a host of specic drugs. Polymorphisms in
the CYP2D6 gene also determine codeine activation, while those in
the CYP2C9 gene affect warfarin inactivation. Polymorphisms in these
and other drug metabolic genes determine the persistence of drugs
and, therefore, should provide information about dosages and toxicity.
With the increasing use of NGS, genetic testing for assessment of drug
response is seldom employed routinely, but in the near future it may be
possible to predict the best specic drugs and dosages for individual
patients based on genetic proling: so-called ‘personalised medicine’.
An example is the enzyme thiopurine methyltransferase (TPMT), which
catabolises azathioprine, a drug that is used in the treatment of auto-
immune diseases and in cancer chemotherapy. Genetic screening for
polymorphic variants of TPMT can be useful in identifying patients who
have increased sensitivity to the effects of azathioprine and who can be
treated with lower doses than normal.
Gene therapy and genome editing
Replacing or repairing mutated genes (gene therapy) is challenging in
humans. Retroviral-mediated ex vivo replacement of the defective gene
in bone marrow cells for the treatment of severe combined immune de-
ciency syndrome has been successful. The major problems with clinical
use of virally delivered gene therapy have been oncogenic integration of
the exogenous DNA into the genome and severe immune response to
the virus.
Other therapies for genetic disease include PTC124, a compound
that can ‘force’ cells to read through a nonsense variant that results in
a premature termination codon in an ORF with the aim of producing a
near-normal protein product. This therapeutic approach could be applied
to any genetic disease caused by nonsense variants.
The most exciting development in genetics for a generation has been
the discovery of accurate, efcient and specic techniques to enable
editing of the genome in cells and organisms. This technology is known
as CRISPR/Cas9 (clustered regularly interspaced short palindromic
repeats and CRISPR-associated) genome editing. It is likely that ex vivo
correction of genetic disease will become commonplace over the next
few years. In vivo correction is not yet possible and will take much longer
to become part of clinical practice.
Induced pluripotent stem cells and regenerative
medicine
Adult stem-cell therapy has been in wide use for decades in the form of
bone marrow transplantation. The identication of adult stem cells for
other tissues, coupled with the ability to purify and maintain such cells in
vitro, now offers exciting therapeutic potential for other diseases. It was
recently discovered that many different adult cell types can be trans-dif-
ferentiated to form cells (induced pluripotent stem cells or iPS cells) with
almost all the characteristics of embryonal stem cells derived from the
early blastocyst. In mammalian model species, such cells can be taken
and used to regenerate differentiated tissue cells, such as in heart and
brain. They have great potential both for the development of tissue mod-
els of human disease and for regenerative medicine.
Pathway medicine
The ability to manipulate pathways that have been altered in genetic
disease has tremendous therapeutic potential for Mendelian disease, but
a rm understanding of both disease pathogenesis and drug action at
a biochemical level is required. An exciting example has been the dis-
covery that the vascular pathology associated with Marfan syndrome
is due to the defective brillin molecules causing up-regulation of trans-
forming growth factor (TGF)-β signalling in the vessel wall. Losartan is
an antihypertensive drug that is marketed as an angiotensin II receptor
antagonist. However, it also acts as a partial antagonist of TGF-β signalling
and is effective in preventing aortic dilatation in a mouse model of Marfan
syndrome, showing promising effects in early human clinical trials.
Ethics in a genomic age
As genomic technology is increasingly moving into mainstream clinical
practice, it is essential for clinicians from all specialties to appreciate the
complexities of genetic testing and consider whether genetic testing is
the right thing to do in a given clinical scenario. To exemplify the ethical
considerations associated with genetic testing, it may be helpful to think
about them in the context of a clinical scenario. As you read the scenario,
try to think what counselling/ethical issues might arise:
A 32-year-old woman is referred to discuss BRCA2 testing; she is
currently pregnant with her second child (she already has a 2-year-old
daughter) and has an identical twin sister. Her mother, a healthy 65-year-
old with Ashkenazi Jewish ancestry, participated in direct-to-consumer
testing (DCT) for ‘a bit of fun’ and a BRCA2 gene variant – common in
the Ashkenazi Jewish population – was identied. There is no signicant
cancer family history of note.
Consider the following issues:
Pre-symptomatic/predictive testing: this describes testing for a
known familial gene variant in an unaffected individual (compared
with diagnostic testing, where genetic testing is undertaken in
an affected individual). Although this could be considered for the
unaffected patient, in the current scenario any testing would also
have implications for her identical twin sister. This needs to be fully
explored with the patient and her sister prior to testing. There is also
the potential issue of predictive testing in the patient’s rst child.
A fundamental tenet in clinical genetics is that predictive genetic
testing should be avoided in childhood for adult-onset conditions.
This is because, if no benet to the patient is accrued through
childhood testing, it is better to retain the child’s right to decide for
herself, when she is old enough, whether she wishes to participate
in genetic testing or not.
Prenatal testing: the principles behind predictive genetic testing in
childhood can be extended to prenatal testing, i.e. if a pregnancy
is being continued, a baby should not be tested for an adult-on-
set condition that cannot be prevented or treated in childhood.
However, prenatal testing itself is hugely controversial and there
is much debate as to how severe a condition should be to justify
prenatal diagnosis, which would determine ongoing pregnancy
decisions.
DCT: while DCT can be interesting and empowering for individuals
wishing to nd out more about their genetic backgrounds, it also has
several drawbacks. Perhaps the main one is that, unlike face-to-face
genetic counselling (which usually precedes any genetic testing,
certainly where there are serious health implications for the individ-
ual and their family, such as is associated with BRCA1/2 variants),
DCT is undertaken in isolation with no direct access to professional
support. Furthermore, in addition to some (common) single-gene
variants, such as the founder BRCA1/2 variants frequently identied
in the Ashkenazi Jewish population and discussed in this example,
current DCT packages utilise a series of SNPs to determine an
overall risk prole; they evaluate the number of detrimental and
80.
Further information 57
3
protective SNPs for a given disease. However, given that only a
minority of the risk SNPs have so far been characterised, this is
often inaccurate. Individuals may be falsely reassured that they are
not at increased risk of a genetic condition despite a family history
suggesting otherwise, resulting in inadequate surveillance and/or
management.
The ethical considerations listed in this clinical scenario give just a a-
vour of some of the issues frequently encountered in clinical genetics.
They are not meant to be an exhaustive summary and whole textbooks
and meetings are devoted to the discussion of hugely complex ethical
issues in genetics. However, a guiding principle is that, although each
counselling situation will be unique with specic communication and eth-
ical challenges, a genetic result is permanent and has implications for
the whole family, not just the individual. Where possible, therefore, an
informed decision regarding genetic testing should be taken by a com-
petent adult following counselling by an experienced and appropriately
trained clinician.
Further information
Books and journal articles
Alberts B, Bray D, Hopkin K, etal. Essential cell biology, 4th edn. New York:
Garland Science; 2013.
Firth H, Hurst JA. Oxford desk reference: clinical genetics. Oxford: Oxford
University Press; 2005.
Read A, Donnai D. New clinical genetics, 2nd edn. Banbury: Scion; 2010.
Strachan T, Read A. Human molecular genetics, 4th edn. New York: Garland
Science; 2010.
Websites
bsgm.org.uk British Society for Genetic Medicine; has a report on genetic testing
of children.
decipher.sanger.ac.uk Excellent, comprehensive genomic database.
ensembl.org Annotated genome databases from multiple organisms.
futurelearn.com/courses/the-genomics-era Has a Massive Open Online Course
on genomics, for which the author of the current chapter is the lead educator .
genome.ucsc.edu Excellent source of genomic information.
ncbi.nlm.nih.gov Online Mendelian Inheritance in Man (OMIM).
ncbi.nlm.nih.gov/books/NBK1116/ Gene Reviews: excellent US-based source of
information about many rare genetic diseases.
orpha.net/consor/cgi-bin/index.php Orphanet: European-based database on rare
disease.
Multiple Choice Questions
3.1.Which of the following best describes a missense or non-
synonymous variant?
A. A base substitution that results in the introduction of a
premature stop codon
B. A base substitution that results in a different codon and
different amino acid
C. The deletion of two bases that results in a shift of reading frame
and the introduction of a downstream premature stop codon
D. A base substitution that introduces a cryptic splice site
E. A base substitution that results in a different codon but same
amino acid
Answer: B.
The most common type of genomic variation is the substitution of one
nucleotide for another, caused by either misincorporation of a nucleotide
during DNA synthesis or chemical modication of the base. If a nucleo-
tide substitution occurs within the open reading frame of a protein-coding
gene it will affect a codon within that gene. A missense/non-synonymous
variant refers to a nucleotide substitution in which the effect on the codon
in which it occurs is to cause it to encode a different amino acid.
3.2. Approximately how many genes are there in the average human
genome?
A. 2000
B. 20000
C. 200000
D. 2000000
E. 20000000
Answer: B.
The human genome contains over 20 000 genes. Many of these are
inactive or silenced in different cell types, reecting the variable gene
expression responsible for cell-specic characteristics.
3.3. Which of the following is a cardinal feature of an X-linked recessive
disorder?
A. Male-to-male transmission
B. Affected individuals in each generation.
C. Increased frequency in consanguineous unions
D. Predominantly males affected
E. There is characteristically highly variable penetrance and
expressivity in the same family
Answer: D.
In X-linked recessive diseases females who carry a copy of the mutant
allele on one of their X chromosomes (unaffected carriers) will pass the
allele on to 50% of their offspring. Only the male offspring will be affected,
because they have only one X chromosome. The female offspring will be
carriers, as they will inherit a ‘normal’ X chromosome from their fathers.
The overall risk of a female carrier having an affected child is therefore
25%, or half of her male offspring.
Male-to-male transmission of an X-linked recessive condition does not
occur because fathers will pass on their Y chromosome to their sons
(not the X chromosome with mutant allele). The presence of affected
individuals in each generation is a characteristic feature of autosomal
dominant inheritance with full penetrance, in which half of the offspring
of an affected parent will be affected irrespective of their sex. Increased
frequency in consanguineous unions would suggest an autosomal
recessive disorder, because the chance of both the female and the male
parent carrying the same mutant allele would be increased by consan-
guinity. Highly variable penetrance and expressivity in the same family is
a feature of mitochondrial disorders.
3.4. Which type of genomic variant will an array comparative genomic
hybridisation (CGH) best detect?
A. A balanced chromosome translocation
B. A frameshift variant
C. A 2 MB deletion
D. A missense variant
E. An essential splice variant
Answer: C.
Array CGH allows rapid and precise detection of segmental gain or loss
of DNA throughout the genome. Microarrays consist of grids of multiple
wells containing short DNA sequences (reference DNA) that are comple-
mentary to known sequences in the genome. Patient and reference DNA
are each labelled with a coloured uorescent dye (generally, patient DNA
is labelled with a green uorescent dye and reference DNA with a red u-
orescent dye) and added to the microarray grid. Where there is an equal
quantity of patient and reference DNA bound to the spot, this results in
yellow uorescence. Where there is too much patient DNA (representing a
duplication of a chromosome region), the spot will be greener; and it will be
more red (appears orange) where there is a 2:1 ratio of the control:patient
DNA (representing heterozygous deletion of a chromosome region).
Array CGH can detect small chromosomal deletions and duplications and
is also more sensitive than conventional karyotyping at detecting mosaicism.
However, array CGH and other array-based approaches will not detect
balanced chromosome rearrangements where there is no loss or gain of
genes/chromosome material, such as balanced reciprocal translocations, or
a global increase in copy number, such as triploidy. In addition, an array CGH
will not detect small insertions/deletions or single nucleotide substitutions.
3.5. Which of the following is an imprinted condition?
A. Huntington’s disease
B. Patau syndrome
C. Neurobromatosis type 1
D. Pseudohypoparathyroidism
E. Alpha-1-antitrypsin deciency
Answer: D.
Several chromosomal regions (loci) have been identied where gene
expression is inherited in a parent-of-origin-specic manner; these are
called imprinted loci. Within these loci the paternally inherited gene may
be active while the maternally inherited may be silenced, or vice versa.
Variants within imprinted loci lead to an unusual pattern of inheritance
where the phenotype is manifest only if inherited from the parent who
contributes the transcriptionally active allele.
Pseudohypoparathyroidism (PHP) is caused by the maternal inher-
itance of a GNAS1 variant (located at 20q13) and is characterised by
hypocalcaemia, hyperphosphataemia, raised parathyroid hormone
(PTH) levels, ectopic calcication, obesity, delayed puberty and short-
ened 4th and 5th metacarpals (the syndrome also known as Albright
hereditary osteodystrophy, AHO). When the gene variant is inherited from
the father, PTH, calcium and phosphate levels are normal but the other
(skeletal) features are present and the condition is known as pseudpseu-
dohypoparathyroidism (PPHP). GNAS1 encodes Gsα which is primar-
ily expressed from the maternal allele in the kidney, pituitary, thyroid
and gonadal cells (and their disruption is the cause of the biochemical
defects) whereas both alleles are expressed in other tissues (responsible
for the skeletal features). Therefore where a GNAS1 variant is maternally
inherited the individual will have both biochemical and skeletal features
(PHP) but where it is paternally inherited they will not have the biochem-
ical features (PPHP).
83.
Clinical immunology
4
SL Johnston
Functionalanatomy and physiology 60
The innate immune system 60
The adaptive immune system 65
The inammatory response 67
Acute inammation 67
Chronic inammation 69
Laboratory features of inammation 69
Presenting problems in immune disorders 70
Recurrent infections 70
Intermittent fever 71
Anaphylaxis 72
Immune deciency 72
Primary phagocyte deciencies 72
Complement pathway deciencies 73
Primary antibody deciencies 73
Primary T-lymphocyte deciencies 75
Secondary immune deciencies 75
Periodic fever syndromes 76
Amyloidosis 77
Autoimmune disease 78
Allergy 80
Angioedema 82
Pregnancy and the immune system 84
Transplantation and graft rejection 84
Transplant rejection 84
Complications of transplant immunosuppression 85
Organ donation 85
Tumour immunology 85
84.
60 CLINICALIMMUNOLOGY
The immune system has evolved to identify and destroy pathogens
while minimising damage to host tissue. Despite the ancient observa-
tion that recovery from an infectious disease frequently results in pro-
tection against that condition, the existence of the immune system as
a functional entity was not recognised until the end of the 19th century.
More recently, it has become clear that the immune system not only
protects against infection but also regulates tissue repair following injury,
and when dysregulated, governs the responses that can lead to auto-
immune and auto-inammatory diseases. Dysfunction or deciency of
the immune response can lead to a wide variety of diseases that may
potentially involve every organ system in the body.
The aim of this chapter is to provide a general understanding of the
immune system, how it contributes to human disease and how manipu-
lation of the immune system can be put to therapeutic use. A review of
the key components of the immune response is followed by sections that
illustrate the clinical presentation of the most common forms of immune
dysfunction: immune deciency, inammation, autoimmunity and allergy.
More detailed discussion of individual conditions can be found in the
relevant organ-specic, emergency and critical care medicine chapters
Functional anatomy and physiology
The immune system consists of an intricately linked network of lymphoid
organs, cells and proteins that are strategically placed to protect against
infection (Fig. 4.1). Immune defences are normally categorised into the
innate immune response, which provides immediate protection against
an invading pathogen, and the adaptive or acquired immune response,
which takes more time to develop but confers exquisite specicity and
long-lasting protection. Innate and adaptive immunity do not work in iso-
lation, but rather in concert, largely driven by cytokines produced by the
specic immune cell populations.
The innate immune system
Innate defences against infection include anatomical barriers, phagocytic
cells, soluble molecules such as complement and acute phase proteins,
and natural killer cells. The innate immune system recognises generic
microbial structures present on non-mammalian tissue and can be mobi-
lised within minutes. A specic stimulus will elicit essentially identical
responses in different individuals, in contrast with adaptive antibody and
T-cell responses, which vary greatly between individuals.
Physical barriers
The tightly packed keratinised cells of the skin physically limit coloni-
sation by microorganisms. The hydrophobic oils that are secreted by
sebaceous glands further repel water and microorganisms, and micro-
bial growth is inhibited by the skin's low pH and low oxygen tension.
Sweat also contains lysozyme, an enzyme that destroys the structural
integrity of bacterial cell walls; ammonia, which has antibacterial prop-
erties; and several antimicrobial peptides such as defensins. Similarly,
the mucous membranes of the respiratory, gastrointestinal and geni-
tourinary tracts provide a physical barrier to infection. Secreted mucus
traps invading pathogens, and immunoglobulin A (IgA), generated by
the adaptive immune system, prevents bacteria and viruses attaching
to and penetrating epithelial cells. As in the skin, lysozyme and antimi-
crobial peptides within mucosal membranes directly kill invading path-
ogens, and lactoferrin acts to starve invading bacteria of iron. Within
the respiratory tract, cilia directly trap pathogens and contribute to
removal of mucus, assisted by physical manœuvres such as sneez-
ing and coughing. In the gastrointestinal tract, hydrochloric acid and
salivary amylase chemically destroy bacteria, while normal peristal-
sis and induced vomiting or diarrhoea assist clearance of invading
organisms.
Adenoids
Lymph nodes
Tonsils
Thoracic
duct
Spleen
Bone
marrow
Liver
Appendix
Germinal centre
Proliferating B cells after
antigen exposure
Afferent lymph
Paracortex
T cells
Dendritic cells
Cortex
B cells in primary
lymphoid follicles
Efferent lymph
Medulla
Plasma cells
Sinuses with
macrophages
Blood vessels
Capsule
Lymph node section
Lymphatics
Neutrophil
Eosinophil
Cells of the innate immune
system
Natural
killer cell
Basophil Mast cell
Monocyte
Macrophage
T lymphocyte
Cells of the adaptive immune
system
Antigen-presenting cell
B lymphocyte
Thymus
Peyer’s patches in small
intestine
Fig. 4.1 Anatomy of the immune system.
85.
Functional anatomy andphysiology 61
4
The microbiome, which is made up of endogenous commensal bacteria,
provides an additional constitutive defence against infection. Approximately
1014
bacteria normally reside at epithelial surfaces in symbiosis with the
human host. They compete with pathogenic microorganisms for scarce
resources, including space and nutrients, and produce fatty acids and
bactericidins that inhibit the growth of many pathogens. In addition, recent
research has demonstrated that commensal bacteria help to shape the
immune response by inducing specic regulatory T cells within the intestine.
Eradication of the normal ora with broad-spectrum antibiotics commonly
results in opportunistic infection by organisms such as Clostridioides dif-
cile, which rapidly colonise an undefended ecological niche.
These constitutive barriers are highly effective, but if external defences
are breached by a wound or pathogenic organism, the specic soluble
proteins and cells of the innate immune system are activated.
Phagocytes
Phagocytes (‘eating cells’) are specialised cells that ingest and kill
microorganisms, scavenge cellular and infectious debris, and produce
inammatory molecules that regulate other components of the immune
system. They include neutrophils, monocytes and macrophages, and are
particularly important for defence against bacterial and fungal infections.
Phagocytes express a wide range of surface receptors, including pattern
recognition receptors (PRRs), which recognise pathogen-associated
molecular patterns (PAMPs) on invading microorganisms, allowing their
identication. The PRRs include Toll-like receptors, nucleotide oligom-
erisation domain (NOD) protein-like receptors and mannose receptors,
whereas the PAMPs they recognise are molecular motifs not present on
mammalian cells, including bacterial cell wall components, bacterial DNA
and viral double-stranded RNA. Interaction between the PRRs and their
PAMPs leads to activation of nuclear factor kappa B (NFκB), which stim-
ulates expression of pro-inammatory genes, and of nucleotide-binding
oligomerisation domain (NOD)-like receptor proteins (NLRP), which form
complexes with other intracellular proteins to form the inammasome
(Fig. 4.2). The inammasome results in activation of interleukin-1 beta
(IL-1β), which is excreted extracellularly and plays a key pathogenic role
in familial fever syndromes (p. 76) and acute gout (Ch. 26). While phago-
cytes can recognise microorganisms through PRRs alone, engulfment of
microorganisms is greatly enhanced by opsonisation. Opsonins include
acute phase proteins produced by the liver, such as C-reactive protein
and complement component C3b. Antibodies generated by the adaptive
immune system also act as opsonins. They bind both to the pathogen
and to phagocyte receptors, acting as a bridge between the two to facili-
tate phagocytosis (see Fig. 4.2). This is followed by intracellular pathogen
destruction and downstream activation of pro-inammatory genes, result-
ing in the generation of pro-inammatory cytokines as discussed below.
Neutrophils
Neutrophils, also known as polymorphonuclear leucocytes, are derived
from the bone marrow and circulate freely in the blood. They are short-
lived cells with a half-life of 6hours, and are produced at the rate of
1011
cells daily. Their functions are to kill microorganisms, to facilitate
rapid transit of cells through tissues and to amplify the immune response
non-specically. These functions are mediated by enzymes contained
in granules, which also provide an intracellular milieu for the killing and
degradation of microorganisms.
Two main types of granule are recognised: primary or azurophil gran-
ules, and the more numerous secondary or specic granules. Primary
granules contain myeloperoxidase and other enzymes important for
intracellular killing and digestion of ingested microbes. Secondary gran-
ules are smaller and contain lysozyme, collagenase and lactoferrin,
which can be released into the extracellular space. Enzyme production
is increased in response to infection, which is reected by more intense
granule staining on microscopy, known as ‘toxic granulation’.
Microbes
C3b
Antibody
C-reactive
protein
Fc
receptor
Pattern
recognition
receptors
NOD-like
receptors
Microbial DNA
Microbial
RNA
Microbial
proteins
Pathogen-
associated
molecular
patterns
LPS
Peptidoglycans
Crystals
NFκB
NFκB
Lysosome
C3b
receptor
Phagocytic cell
Pro-inflammatory
gene expression Pro-IL-1β IL-1β
NLRP
NLRP
expression
Inflammasome
Fig. 4.2 Phagocytosis and opsonisation. Phagocytosis of microbes can be augmented by several opsonins, such as C-reactive protein, antibodies and complement
fragments like C3b, which enhance the ability of phagocytic cells to engulf microorganisms and destroy them. Phagocytes also recognise components of microbes, such as
lipopolysaccharide, peptidoglycans, DNA and RNA, collectively known as pathogen-associated molecular patterns (PAMPs). These activate pattern recognition receptors (PRRs),
such as Toll-like receptors and nucleotide oligomerisation domain (NOD)-like receptors, which promote inammatory gene expression through the nuclear factor kappa B
(NFκB) pathway. Uric acid and other crystals can also promote inammation through the NOD pathway. (IL = interleukin; LPS = lipopolysaccharide; NLRP = nucleotide-binding
oligomerisation domain (NOD)-like receptor proteins)
86.
62 CLINICALIMMUNOLOGY
Changes in damaged or infected cells trigger local production of
inammatory molecules and cytokines. These cytokines stimulate the
production and maturation of neutrophils in the bone marrow, and their
release into the circulation. Neutrophils are recruited to specic sites of
infection by chemotactic agents, such as interleukin 8 (IL-8), and by acti-
vation of local endothelium. Up-regulation of cellular adhesion molecules
on neutrophils and the endothelium also facilitates neutrophil migration.
The transit of neutrophils through the blood stream is responsible for the
rise in neutrophil count that occurs in early infection. Once present within
infected tissue, activated neutrophils seek out and engulf invading micro-
organisms. These are initially enclosed within membrane-bound vesicles,
which fuse with cytoplasmic granules to form the phagolysosome. Within
this protected compartment, killing of the organism occurs through a
combination of oxidative and non-oxidative killing. Oxidative killing, also
known as the respiratory burst, is mediated by the nicotinamide adenine
dinucleotide phosphate (NADPH)–oxidase enzyme complex, which con-
verts oxygen into reactive oxygen species such as hydrogen peroxide
and superoxide that are lethal to microorganisms. The myeloperoxidase
enzyme within neutrophils produces hypochlorous acid, which is a pow-
erful oxidant and antimicrobial agent. Non-oxidative (oxygen-independ-
ent) killing occurs through the release of bactericidal enzymes into the
phagolysosome. Each enzyme has a distinct antimicrobial spectrum,
providing broad coverage against bacteria and fungi.
An additional, recently identied form of neutrophil-mediated killing is
neutrophil extracellular trap (NET) formation. Activated neutrophils can
release chromatin with granule proteins such as elastase to form an
extracellular matrix that binds to microbial proteins. This can immobilise
or kill microorganisms without requiring phagocytosis. The process of
phagocytosis and NET formation (NETosis) depletes neutrophil glycogen
reserves and is followed by neutrophil death. As the cells die, their con-
tents are released and lysosomal enzymes degrade collagen and other
components of the interstitium, causing liquefaction of closely adjacent
tissue. The accumulation of dead and dying neutrophils results in the
formation of pus, which, if extensive, may lead to abscess formation.
Monocytes and macrophages
Monocytes are the precursors of tissue macrophages. They are pro-
duced in the bone marrow and enter the circulation, where they con-
stitute about 5% of leucocytes. From the blood stream they migrate to
peripheral tissues, where they differentiate into tissue macrophages and
reside for long periods. Specialised populations of tissue macrophages
include Kupffer cells in the liver, alveolar macrophages in the lung, mesan-
gial cells in the kidney, and microglial cells in the brain. Macrophages, like
neutrophils, are capable of phagocytosis and killing of microorganisms
but also play an important role in the amplication and regulation of the
inammatory response (Box 4.1). They are particularly important in tissue
surveillance and constantly survey their immediate surroundings for signs
of tissue damage or invading organisms.
Dendritic cells
Dendritic cells are specialised antigen-presenting cells that are present
in tissues in contact with the external environment, such as the skin and
mucosal membranes. They can also be found in an immature state in
the blood. They sample the environment for foreign particles and, once
activated, carry microbial antigens to regional lymph nodes, where they
interact with T cells and B cells to initiate and shape the adaptive immune
response.
Cytokines
Cytokines are signalling proteins produced by cells of the immune sys-
tem and a variety of other cell types. More than 100 have been identied.
Cytokines have complex and overlapping roles in cellular communication
and regulation of the immune response. Subtle differences in cytokine
production, particularly at the initiation of an immune response, can have
a major impact on outcome. Cytokines bind to specic receptors on
target cells and activate downstream intracellular signalling pathways,
ultimately leading to changes in gene transcription and cellular func-
tion. Two important signalling pathways are illustrated in Figure 4.3. The
nuclear factor kappa B (NFκB) pathway is activated by tumour necrosis
factor (TNF), by other members of the TNF superfamily such as receptor
activator of nuclear kappa B ligand (RANKL), and by the Toll-like recep-
tors and NOD-like receptors (see Fig. 4.2). In the case of TNF superfamily
members, receptor binding causes the inhibitor of kappa B kinase (IKK)
complex of three proteins to be recruited to the receptor by binding TNF
receptor-associated proteins (TRAF). This activates IKK, which in turn
leads to phosphorylation of the inhibitor of nuclear factor kappa B pro-
tein (IκB), causing it to be degraded by the proteasome, allowing NFκB
to translocate to the nucleus and activate gene transcription. The Janus
kinase/signal transducers and activators of transcription (JAK-STAT)
pathway is involved in transducing signals downstream of many cytokine
receptors, including those for IL-2, IL-6 and interferon-gamma (IFN-γ).
On receptor binding, JAK proteins are recruited to the intracellular por-
tion of the receptor and are phosphorylated. These in turn phosphorylate
STAT proteins, which translocate to the nucleus and activate gene tran-
scription, altering cellular function. The function and disease associations
of several important cytokines are shown in Box 4.2. Cytokine inhibitors
are now routinely used in the treatment of autoimmune diseases, most
of which are monoclonal antibodies to cytokines or their receptors. In
addition, small-molecule inhibitors have been developed that inhibit the
intracellular signalling pathways used by cytokines. These include the
Janus kinase inhibitors tofacitinib, baracitinib and upadacitinib and l-
gotinib which are used in the treatment of various inammatory rheu-
matic diseases and inammatory bowel disease, and the tyrosine kinase
inhibitor imatinib, which is used in chronic myeloid leukaemia and other
haematological malignancies.
Integrins
Integrins are transmembrane proteins that play important roles in cell–cell
and cell–matrix interactions. They are expressed on lymphocytes as well
as a variety of other cell types and mediate attachment of cells to the
endothelium and the extracellular matrix, affecting cell migration and sig-
nal transduction. Their role in autoimmune disease has been extensively
studied and therapeutic agents are now being developed which modify
binding of integrins to their targets. For example, natalizumab is a mono-
clonal antibody which targets the a4 integrin which is expressed on lym-
phocytes. Natalizumab prevents α4β1 integrin binding to VCAM-1 and
the α4β7 integrin binding to MAdCAM-1 resulting in an anti-inammatory
4.1 Functions of macrophages
Amplication of the inammatory response
Stimulate the acute phase response (through production of IL-1 and IL-6)
Activate vascular endothelium (IL-1, TNF-α)
Stimulate neutrophil maturation and chemotaxis (IL-1, IL-8)
Stimulate monocyte chemotaxis
Killing of microorganisms
Phagocytosis
Microbial killing through oxidative and non-oxidative mechanisms
Clearance, resolution and repair
Scavenging of necrotic and apoptotic cells
Clearance of toxins and other inorganic debris
Tissue remodelling (elastase, collagenase, matrix proteins)
Down-regulation of inammatory cytokines
Wound healing and scar formation (IL-1, platelet-derived growth factor, broblast
growth factor)
Link between innate and adaptive immune systems
Activate T cells by presenting antigen in a recognisable form
T cell-derived cytokines increase phagocytosis and microbicidal activity of
macrophages in a positive feedback loop
(IL = interleukin; TNF = tumour necrosis factor)
87.
Functional anatomy andphysiology 63
4
effect. It is an effective treatment for multiple sclerosis, which works by
preventing immune cells from entering the central nervous system by
interfering with lymphocyte binding the α4β1 integrin. This treatment is
not without risk, however, since there are reports of progressive multifo-
cal leucoencephalopathy (PML) in MS treated patients, as a result of JC
virus infection of the central nervous system. Similarly, vedolizumab, a
monoclonal antibody directed against the α4/β7 integrin, is an effective
treatment for Crohn’s disease and ulcerative colitis which works selec-
tively by preventing lymphocytes entering gut endothelium by inhibiting
the interaction with MAdCAM-1.
Complement
The complement system comprises a group of more than 20 tightly reg-
ulated, functionally linked proteins that act to promote inammation and
eliminate invading pathogens. Complement proteins are produced in the
liver and are present in inactive form in the circulation. When the comple-
ment system is activated, it sets in motion a rapidly amplied biological
cascade analogous to the coagulation cascade.
There are three mechanisms by which the complement cascade can
be activated (Fig. 4.4):
The alternate pathway is triggered directly by binding of C3 to bac-
terial cell-wall components, such as lipopolysaccharide of Gram-
negative bacteria and teichoic acid of Gram-positive bacteria.
The classical pathway is initiated when two or more IgM or IgG
antibody molecules bind to antigen. The associated conformational
change exposes binding sites on the antibodies for the rst protein
in the classical pathway, C1, which is a multiheaded molecule that
can bind up to six antibody molecules. Once two or more ‘heads’ of
a C1 molecule are bound to antibody, the classical cascade is trig-
gered. An important inhibitor of the classical pathway is C1 inhibitor
(C1inh), as illustrated in Figure 4.4
The lectin pathway is activated when mannose-binding lectin inter-
acts with microbial cell surface carbohydrates. This directly stimu-
lates the classical pathway at the level of C4, bypassing the need for
immune complex formation.
Activation of complement by any of these pathways results in activation
of C3. This in turn activates the nal common pathway, in which the com-
plement proteins C5–C9 assemble to form the membrane attack complex
(MAC). This can puncture the cell wall, leading to osmotic lysis of target cells.
JAK JAK inhibitor
Response
genes
Response
genes
Proteasome
JAK
Cytokines
Cytokine receptor
P
P
P
P
P
STAT
STAT
P P
STAT
STAT
DNA
TNF
TNF receptor
TRAF
IκB
P
IκB
NFκB
NFκB
IKKκ
IKKα IKKβ
IFN-γ
IL-6
IL-2
Fig. 4.3 Cytokines signalling pathways and the immune response. Cytokines regulate the immune response through binding to specic receptors that activate a variety
of intracellular signalling pathways, two of which are shown. Members of the tumour necrosis factor (TNF) superfamily and the Toll-like receptors and NOD-like receptors (see
Fig. 4.2) signal through the nuclear factor kappa B (NFκB) pathway. Several other cytokines, including IL-2, IL-6 and interferons, employ the Janus kinase/signal transducer
and activator of transcription (JAK-STAT) pathway to regulate cellular function (see text for more details). (IFN, interferon; IκB = inhibitor of kappa B; IKK = I kappa B kinase;
IL=interleukin; P = phosphorylation of the signalling protein; TRAF = tumour necrosis factor receptor-associated factor)
Smooth muscle
contraction
Activation of cells
Vascular permeability
Lysis of
bacteria
Membrane
attack
complex
(MAC)
Opsonisation
of bacteria
Direct
activation
Lectin
pathway
Mannose-
binding lectin
Classical
pathway
Antibody–antigen
complexes
Alternate
pathway
C4
C2
C3
C3a
C1inh
C5a
C5
C3b
C5b
C6
C7
C8
C1
C9
Fig. 4.4 The complement pathway. The classical pathway is activated by binding
of antigen–antibody complexes to C1 but is blocked by C1 inhibitor (C1inh), whereas
mannose-binding lectins, which are macromolecules that bind to carbohydrates
on the surface of various microorganisms, activate the pathway by binding C4.
Bacteria can directly activate the pathway through C3, which plays a pivotal role in
complement activation through all three pathways.
88.
64 CLINICALIMMUNOLOGY
There are a number of control proteins within the complement system
that prevent activation on host cells, thereby preventing host cell damage,
some of which are membrane bound, others soluble, such as C1 inhibi-
tor. Factor H is an important example in control of the alternate pathway,
CD59 and decay accelerating factor (DAF) are important in the terminal
complement pathway. Deciencies in factor H can lead to atypical haemo-
lytic uraemic syndrome and failure of binding of CD59 and DAF due to
deciency in the anchoring glycosylphosphatidylinositol on red blood cells
can lead to paroxysmal nocturnal haemoglobulinuria as a result of comple-
ment-mediated haemolysis. A monoclonal antibody directed against the
central complement molecule C5, eculizumab, has been developed for
therapeutic use in these conditions. Invasive infection, including menin-
gococcal sepsis, has been reported with eculizumab therapy, highlighting
the importance of the complement system in preventing such infections.
4.2 Important cytokines in the regulation of the immune response
Cytokine Source Actions Biologic therapies
Interferon-alpha (IFN-α) T cells and macrophages Antiviral activity
Activates NK cells, CD8+
T cells and
macrophages
Recombinant IFN-α used in hepatitis C and
some malignancies
Interferon-gamma (IFN-γ) T cells and NK cells Increases antimicrobial activity of
macrophages
Regulates cytokine production by
T cells and macrophages
Interferon-γ used in chronic
granulomatous disease
Tumour necrosis factor alpha
(TNF-α)
Macrophages, NK cells and others,
including T cells
Pro-inammatory
Increases expression of other
cytokines and adhesion molecules
Causes apoptosis of some target cells
Directly cytotoxic
TNF-α inhibitors used in rheumatoid
arthritis, inammatory bowel disease,
psoriasis and many other inammatory
conditions
Interleukin-1 (IL-1) Macrophages and neutrophils Stimulates neutrophil recruitment,
fever, and T-cell and macrophage
activation as part of the inammatory
response
IL-1 inhibitors used in systemic juvenile
rheumatoid arthritis, periodic fever
syndromes and acute gout
Interleukin-2 (IL-2) CD4+
T cells Stimulates proliferation and
differentiation of antigen-specic
T lymphocytes
IL-2 inhibitors used in the treatment of
transplant rejection
Interleukin-4 (IL-4) CD4+
T cells Stimulates maturation of B and
T cells, and production of IgE antibody
Antibodies to IL-4 receptor used in severe
atopic dermatitis
Interleukin-5 (IL-5) CD4+
T cells, mast cells, eosinophils
and basophils
Growth and differentiation factor for
B cells and eosinophils
Antibodies to IL-5 or the IL-5 receptor
used in treatment refractory eosinophilic
asthma and more recently anti-IL-5
antibodies used in eosinophilic vasculitis
Interleukin-6 (IL-6) Monocytes and macrophages Stimulates neutrophil recruitment,
fever, and T-cell and macrophage
activation as part of the inammatory
response, stimulates maturation of
B cells into plasma cells
Antibodies to IL-6 receptor used in
rheumatoid arthritis and giant cell arteritis
Interleukin-12 (IL-12) Monocytes and macrophages Stimulates IFN-γ and TNF-α release
by T cells
Activates NK cells
Antibody to p40 subunit of IL-12 used in
psoriasis and psoriatic arthritis
Interleukin-17 (IL-17) Th17 cells (T helper), NK cells, NK-T
cells
Pro-inammatory cytokine
Involved in mucosal immunity and
control of extracellular pathogens,
synergy with IL-1 and TNF
Antibody to IL-17 used in psoriasis,
psoriatic arthritis and ankylosing
spondylitis
Interleukin-22 (IL-22) Th17 cells Induction of epithelial cell proliferation
and antimicrobial proteins in
keratinocytes
Interleukin-23 (IL-23) Activated macrophages and dendritic
cells in peripheral tissues
Pro-inammatory cytokine,
proliferation of Th17 T cells with
induction of IL-17
Antibody to IL-23 used in psoriasis,
psoriatic arthritis and inammatory bowel
disease
(IgE = immunoglobulin E; NK = natural killer)
This step is particularly important in the defence against encapsulated
bacteria such as Neisseria spp. and Haemophilus inuenzae.
Complement fragments generated by activation of the cascade can
also act as opsonins, rendering microorganisms more susceptible to
phagocytosis by macrophages and neutrophils (see Fig. 4.2). In addi-
tion, they are chemotactic agents, promoting leucocyte trafcking to
sites of inammation. Some fragments act as anaphylotoxins, binding to
complement receptors on mast cells and triggering release of histamine,
which increases vascular permeability. The products of complement acti-
vation also help to target immune complexes to antigen-presenting cells,
providing a link between the innate and the adaptive immune systems.
Finally, activated complement products dissolve the immune complexes
that triggered the cascade, minimising bystander damage to surrounding
tissues.
89.
Functional anatomy andphysiology 65
4
Mast cells and basophils
Mast cells and basophils are bone marrow-derived cells that play a cen-
tral role in allergic disorders. Mast cells reside predominantly in tissues
exposed to the external environment, such as the skin and gut, while
basophils circulate in peripheral blood and are recruited into tissues
in response to inammation. Both contain large cytoplasmic granules
that enclose vasoactive substances such as histamine (see Fig. 4.14).
Mast cells and basophils express IgE receptors on their cell surface,
which bind IgE antibody. On encounter with specic antigen, the cell
is triggered to release histamine and other mediators present within the
granules and to synthesise additional mediators, including leukotrienes,
prostaglandins and cytokines. An inammatory cascade is initiated that
increases local blood ow and vascular permeability, stimulates smooth
muscle contraction and increases secretion at mucosal surfaces.
Natural killer cells
Natural killer (NK) cells are large granular lymphocytes that play a major
role in defence against tumours and viruses. They exhibit features of both
the adaptive and the innate immune systems in that they are morpho-
logically similar to lymphocytes and recognise similar ligands, but they
are not antigen-specic and cannot generate immunological memory. NK
cells express a variety of cell surface receptors, some of which are stim-
ulatory and others inhibitory. The effects of inhibitory receptors normally
predominate. These recognise human leucocyte antigen (HLA) molecules
that are expressed on normal nucleated cells, preventing NK cell-medi-
ated attack, whereas the stimulatory receptors recognise molecules that
are expressed primarily when cells are damaged. This allows NK cells
to remain tolerant to healthy cells but not to damaged ones. When cells
become infected by viruses or undergo malignant change, expression of
HLA class I molecules on the cell surface can be down-regulated. This
is an important mechanism by which these cells then evade adaptive
T-lymphocyte responses. In this circumstance, however, NK cell defences
becomes important, as down-regulation of HLA class I abrogates the
inhibitory signals that normally prevent NK activation. The net result is NK
attack on the abnormal target cell. NK cells can also be activated by bind-
ing of antigen–antibody complexes to surface receptors. This physically
links the NK cell to its target in a manner analogous to opsonisation and is
known as antibody-dependent cellular cytotoxicity (ADCC).
Activated NK cells can kill their targets in various ways. They secrete
pore-forming proteins such as perforin into the membrane of the target
cell, and proteolytic enzymes called granzymes into the target cell, which
cause apoptosis. In addition, NK cells produce a variety of cytokines such
as TNF-α and IFN-γ, which have direct antiviral and anti-tumour effects.
The adaptive immune system
If the innate immune system fails to provide effective protection against
an invading pathogen, the adaptive immune system is mobilised (see
Fig. 4.1). This has three key characteristics:
It has exquisite specicity and can discriminate between very small
differences in molecular structure.
It is highly adaptive and can respond to an almost unlimited number
of molecules.
It possesses immunological memory, and changes consequent
to initial activation by an antigen allow a more effective immune
response on subsequent encounters.
There are two major arms of the adaptive immune response. Humoral
immunity involves the production of antibodies by B lymphocytes, and
cellular immunity involves the activation of T lymphocytes, which syn-
thesise and release cytokines that affect other cells, as well as directly
killing target cells. These interact closely with each other and with the
components of the innate immune system to maximise effectiveness of
the immune response.
Lymphoid organs
The primary lymphoid organs are involved in lymphocyte development.
They include the bone marrow, where T and B lymphocytes differentiate
from haematopoietic stem cells and where B lymphocytes also mature,
and the thymus, the site of T-cell maturation (see Fig. 4.1). After matu-
ration, lymphocytes migrate to the secondary lymphoid organs. These
include the spleen, lymph nodes and mucosa-associated lymphoid tis-
sue. These trap and concentrate foreign substances and are the major
sites of interaction between naïve lymphocytes and microorganisms.
The thymus
The thymus is a bi-lobed structure in the anterior mediastinum, and is
organised into cortical and medullary areas. The cortex is densely pop-
ulated with immature T cells, which migrate to the medulla to undergo
selection and maturation. The thymus is most active in the fetal and neo-
natal period, and involutes after puberty. Failure of thymic development is
associated with profound T-cell immune deciency but surgical removal
of the thymus in childhood (usually during major cardiac surgery) is not
associated with signicant immune dysfunction.
The spleen
The spleen is the largest of the secondary lymphoid organs. It is highly
effective at ltering blood and is an important site of phagocytosis of
senescent erythrocytes, bacteria, immune complexes and other debris,
and of antibody synthesis. It is important for defence against encap-
sulated bacteria, and asplenic individuals are at risk of overwhelming
Streptococcus pneumoniae and H. inuenzae infection (see Box 4.5).
Lymph nodes
These are positioned to maximise exposure to lymph draining from sites
of external contact, and are highly organised (see Fig. 4.1).
The cortex contains primary lymphoid follicles, which are the site of
B-lymphocyte interactions. When B cells encounter antigen, they
undergo intense proliferation, forming germinal centres.
The paracortex is rich in T lymphocytes and dendritic cells.
The medulla is the major site of antibody-secreting plasma cells.
Within the medulla there are many sinuses, which contain large
numbers of macrophages.
Mucosa-associated lymphoid tissue
Mucosa-associated lymphoid tissue (MALT) consists of diffusely distrib-
uted lymphoid cells and follicles present along mucosal surfaces. It has a
similar function to the more organised, encapsulated lymph nodes. They
include the tonsils, adenoids and Peyer's patches in the small intestine.
Lymphatics
Lymphoid tissue is connected by a network of lymphatics, with three
major functions: lymphatics provide access to lymph nodes, return inter-
stitial uid to the venous system and transport fat from the small intestine
to the blood stream (see Fig. 19.12). The lymphatics begin as blind-
ending capillaries, which come together to form lymphatic ducts, enter-
ing and leaving regional lymph nodes as afferent and efferent ducts,
respectively. They eventually coalesce and drain into the thoracic duct
and left subclavian vein. Lymphatics may be either deep or supercial,
and follow the distribution of major blood vessels.
Humoral immunity
Humoral immunity is mediated by B lymphocytes, which differentiate from
haematopoietic stem cells in the bone marrow. Their major functions are
to produce antibody and interact with T cells, but they are also involved
in antigen presentation. Mature B lymphocytes can be found in the bone
marrow, lymphoid tissue, spleen and, to a lesser extent, the blood stream.
They express a unique immunoglobulin receptor on their cell surface,
the B-cell receptor, which binds to soluble antigen targets (Fig. 4.5).
90.
66 CLINICALIMMUNOLOGY
Encounters with antigen usually occur within lymph nodes. If provided with
appropriate cytokines and other signals from nearby T lymphocytes, anti-
gen-specic B cells respond by rapidly proliferating in a process known as
clonal expansion (see Fig. 4.5). This is accompanied by a highly complex
series of genetic rearrangements known as somatic hypermutation, which
generates B-cell populations that express receptors with greater afnity
for antigen than the original. These cells differentiate into either long-lived
memory cells, which reside in the lymph nodes, or plasma cells, which pro-
duce antibody. Memory cells allow production of a more rapid and more
effective response on subsequent exposure to that pathogen.
Immunoglobulins
Immunoglobulins (Ig) play a central role in humoral immunity. They are solu-
ble proteins produced by plasma cells and are made up of two heavy and
two light chains (Fig. 4.6). The heavy chain determines the antibody class
or isotype, such as IgG, IgA, IgM, IgE or IgD. Subclasses of IgG and IgA
also occur. The antigen is recognised by the antigen-binding regions (Fab
)
of both heavy and light chains, while the consequences of antibody bind-
ing are determined by the constant region of the heavy chain (Fc
) (Box4.3).
Antibodies have several functions. They facilitate phagocytosis by acting
as opsonins (see Fig. 4.2) and facilitate cell killing by cytotoxic cells, par-
ticularly NK cells by antibody-dependent cellular cytotoxicity. Binding of
antibodies to antigen can trigger activation of the classical complement
pathway (see Fig. 4.4). In addition, antibodies can directly neutralise the
biological activity of their antigen target. This is a particularly important
feature of IgA antibodies, which act predominantly at mucosal surfaces.
The humoral immune response is characterised by immunological
memory, in which the antibody response to successive exposures to an
antigen is qualitatively and quantitatively improved from the rst expo-
sure. When a previously unstimulated or ‘naïve’ B lymphocyte is acti-
vated by antigen, the rst antibody to be produced is IgM, which appears
in the serum after 5–10 days. Depending on additional stimuli provided
by T lymphocytes, other antibody classes (IgG, IgA and IgE) are pro-
duced 1–2 weeks later. If the memory B cell is subsequently re-exposed
to the same antigen, the lag time between exposure and production of
antibody is decreased to 2–3 days, the amount of antibody produced is
increased, and the response is dominated by IgG antibodies of high afn-
ity. Furthermore, in contrast to the initial antibody response, secondary
antibody responses do not require additional input from T lymphocytes.
This allows the rapid generation of highly specic responses on re-expo-
sure to a pathogen and is an important mechanism in vaccine efcacy.
Cellular immunity
Cellular immunity is mediated by T lymphocytes, which play important
roles in defence against viruses, fungi and intracellular bacteria. They
also play an important immunoregulatory role, by orchestrating and
regulating the responses of other components of the immune system.
T-lymphocyte precursors differentiate from haematopoietic stem cells in
the bone marrow and are exported to the thymus when they are still
immature (see Fig. 4.1). Individual T cells express a unique receptor that
is highly specic for a single antigen. Within the thymus T cells undergo
a process of stringent selection to ensure that autoreactive cells are
destroyed. Mature T lymphocytes leave the thymus and expand to pop-
ulate other organs of the immune system. It has been estimated that
an individual possesses 107
–109
T-cell clones, each with a unique T-cell
receptor, ensuring at least partial coverage for any antigen encountered.
Unlike B cells, T cells cannot recognise intact protein antigens in their
native form. Instead, the protein must be broken down into component pep-
tides by antigen-presenting cells for presentation to T lymphocytes in asso-
ciation with HLA molecules on the antigen-presenting cell surface (Fig. 4.7).
This process is known as antigen processing and presentation, and it is the
complex of peptide and HLA together that is recognised by individual T cells.
The structure of HLA molecules varies widely between individuals. Since
each HLA molecule has the capacity to present a subtly different peptide
repertoire to T lymphocytes, this ensures enormous diversity in recognition
of antigens by the T-cell population. All nucleated cells have the capacity to
process and present antigens, but cells with specialised antigen-presenting
functions include dendritic cells, macrophages and B lymphocytes. These
cells carry additional co-stimulatory molecules, such as CD80 and CD86,
providing the necessary ‘second signal’ for full T-cell activation (Fig. 4.8).
Mechanisms also exist to inhibit activation of T-cells. One of the most impor-
tant is the programmed cell death 1 (PD-1) pathway. The PD1 receptor is
expressed on lymphocytes and inhibits lymphocyte activation when bound
by its ligand (PDL-1) which is a molecule expressed on antigen-present-
ing cells, endothelial; cells and come tumour cells (Fig. 4.8). Monoclonal
antibodies which inhibit this interaction are important treatments for certain
types of cancer (see tumour immunology later in this chapter).
T lymphocytes can be divided into two subgroups on the basis of
function and recognition of HLA molecules. These are designated CD4+
and CD8+
T cells, according to the ‘cluster of differentiation’ (CD) antigen
number of key proteins expressed on their cell surface.
CD8+
T lymphocytes
These cells recognise antigenic peptides in association with HLA class
I molecules (HLA-A, HLA-B, HLA-C). They kill infected cells directly
Variable
region (Fab)
Constant
region (Fc)
Light chain
Heavy chain
Fig. 4.6 The structure of an immunoglobulin (antibody) molecule. The variable
region is responsible for antigen binding, whereas the constant region can interact
with immunoglobulin receptors expressed on immune cells.
T-helper cell Immunoglobulin
receptor
Antigen
CD40L
CD40 TCR
HLA
B-cell activation
B cell
Clonal
expansion
Plasma cells
Antibodies
Memory B cells
IL-4
IL-5
Fig. 4.5 B-cell activation. Activation of B cells is initiated through binding of
an antigen with the immunoglobulin receptor on the cell surface. For activation
to proceed, an interaction with T-helper cells is also required, providing additional
signals through binding of CD40 ligand (CD40L) to CD40; an interaction between
the T-cell receptor (TCR) and processed antigenic peptides presented by human
leucocyte antigen (HLA) molecules on the B-cell surface; and cytokines released by
the T-helper cells. Fully activated B cells undergo clonal expansion with differentiation
towards plasma cells that produce antibody. Following activation, memory cells are
generated that allow rapid antibody responses when the same antigen is encountered
on a second occasion. (IL = interleukin)
91.
The inflammatory response 67
4
through the production of pore-forming molecules such as perforin and
release of digesting enzymes triggering apoptosis of the target cell, and
are particularly important in defence against viral infection.
CD4+
T lymphocytes
These cells recognise peptides presented on HLA class II molecules
(HLA-DR, HLA-DP and HLA-DQ) and have mainly immunoregulatory
functions. They produce cytokines and provide co-stimulatory signals
that support the activation of CD8+
T lymphocytes and assist the pro-
duction of mature antibody by B cells. In addition, their close interaction
with phagocytes determines cytokine production by both cell types.
CD4+
lymphocytes can be further subdivided into subsets on the basis
of the cytokines they produce:
Th1 (T-helper) cells typically produce IL-2, IFN-γ and TNF-α,
and support the development of delayed-type hypersensitivity
responses.
Th2 cells typically produce IL-4, IL-5, IL-10 and IL-13, and promote
allergic responses.
T-regulatory cells (T-regs) are a further subset of specialised CD4+
lymphocytes that are important in actively suppressing activation
of other cells and preventing autoimmune disease. They produce
cytokines such as TGF-β and IL-10.
Th17 cells are pro-inammatory cells dened by their production of
IL-17. They are related to regulatory T cells. Th17 cells have a key
role in defence against extracellular bacteria and fungi. They also
have a role in the development of autoimmune disease.
T-cell activation is regulated by a balance between co-stimulatory mol-
ecules, the second signal required for activation, and inhibitory molecules
that down-regulate T-cell activity. One such inhibitory molecule, CTLA4,
has been harnessed therapeutically in the form of abatacept, which is a
fusion protein comprised of the Fc fragment of immunoglobulin linked to
CTLA4. This is used to inhibit T-cell activation in rheumatoid arthritis and
solid organ transplantation.
The inammatory response
Inammation is the response of tissues to injury or infection, and is nec-
essary for normal repair and healing. This section focuses on the general
principles of the inammatory response and its multisystem manifesta-
tions. The role of inammation in specic diseases is discussed in many
other chapters of this book.
Acute inammation
Acute inammation is the result of rapid and complex interplay between
the cells and soluble molecules of the innate immune system. The clas-
sical external signs include heat, redness, pain and swelling (Fig. 4.9).
The inammatory process is initiated by local tissue injury or infection.
Damaged epithelial cells produce cytokines and antimicrobial peptides,
causing early inltration of phagocytic cells. Production of leukotrienes,
4.3 Classes and properties of antibody
Antibody Concentration in adult
serum
Complement
activation*
Opsonisation Presence in external
secretions
Other properties
IgG 6.0–16.0g/L IgG1 +++
IgG2 +
IgG3 +++
IgG1 ++
IgG3 ++
++ Four subclasses: IgG1, IgG2, IgG3, IgG4
Distributed equally between blood and
extracellular uid, and transported across
placenta
IgG2 is particularly important in defence
against polysaccharides antigens
IgA 1.5–4.0g/L – – ++++ Two subclasses: IgA1, IgA2
Highly effective at neutralising toxins
Particularly important at mucosal surfaces
IgM 0.5–2.0g/L ++++ + Highly effective at agglutinating pathogens
IgE 0.003–0.04g/L – – – Majority of IgE is bound to mast cells,
basophils and eosinophils
Important in allergic disease and defence
against parasite infection
IgD Not detected – – – Function in B-cell development
*Activation of the classical pathway, also called ‘complement xation’.
(IgG = immunoglobulin)
CD80/86
CD28
T cell
T-cell
proliferation
T-cell effector
function
Pathogen PAMP
PAMP internalised
by PRR and degraded
in lysosome
PRR
APC
CD4/CD8
TCR
Processed
antigenic peptide
HLA
Antigenic peptide
IL-2
positive
feedback
Co-stimulation
T-cell
activation
Fig. 4.7 Antigen presentation. For T-cell activation, antigen has to be processed
via antigen-presenting cells (APC). Pathogens, bearing pathogen-associated
molecular patterns (PAMPs) on their surface, are recognised by pattern recognition
receptors (PRRs) on the APC. Following endocytosis, the pathogen is broken down
intracellularly. Antigenic peptides are then loaded onto human leucocyte antigens
(HLA), also known as major histocompatibility complexes (MHC), and presented at
the APC surface. The combination of peptide and HLA is then recognised by the
T-cell receptor (TCR). For T-cell activation, a second, co-stimulatory signal is required
through binding of CD80 or CD86 on the APC to CD28 on the T cell (see also
Fig.4.8). (CD28 and CD80/86 = co-stimulatory molecules; IL = interleukin)
92.
68 CLINICALIMMUNOLOGY
prostaglandins, histamine, kinins, anaphylotoxins and inducible nitric
oxide synthase also occurs within inamed tissue. These mediators
cause vasodilatation and increased vascular permeability, causing traf-
cking of uid and cells into the affected tissue. In addition, pro-inam-
matory cytokines, such as IL-1, TNF-α and IL-6 produced at the site
of injury, are released systemically and act on the hypothalamus to cause
fever, and on the liver to stimulate production of acute phase proteins.
The acute phase response
The acute phase response refers to the production of a variety of proteins
by the liver in response to inammatory stimuli. These proteins have a
wide range of activities. Circulating levels of C-reactive protein (CRP) and
serum amyloid A may be increased 100- to 1000-fold, contributing to
host defence and stimulating repair and regeneration. Fibrinogen plays an
essential role in wound healing, and α1
-antitrypsin and α1
-antichymotrypsin
control the pro-inammatory cascade by neutralising the enzymes pro-
duced by activated neutrophils, preventing widespread tissue destruction.
In addition, antioxidants such as haptoglobin and manganese superoxide
dismutase scavenge for oxygen free radicals, while increased levels of iron-
binding proteins such as ferritin and lactoferrin decrease the iron available
for uptake by bacteria. Immunoglobulins are not acute phase proteins but
are often increased in chronic inammation.
Septic shock
Septic shock is the clinical manifestation of overwhelming inammation.
It is characterised by excessive production of pro-inammatory cytokines
by macrophages, causing hypotension, hypovolaemia and tissue
oedema. In addition, uncontrolled neutrophil activation causes release
of proteases and oxygen free radicals within blood vessels, damaging
the vascular endothelium and further increasing capillary permeability.
Direct activation of the coagulation pathway combines with endothelial
cell disruption to form clots within the damaged vessels. The clinical con-
sequences include cardiovascular collapse, acute respiratory distress
syndrome, disseminated intravascular coagulation, multi-organ failure
and often death. Septic shock most frequently results from infection with
Gram-negative bacteria, because lipopolysaccharide produced by these
organisms is particularly effective at activating the inammatory cascade.
Early recognition and appropriate early intervention can improve patient
outcome. More details on the diagnosis and management of septic
shock are provided in Chapter 9.
Resolution of inammation
Resolution of an inammatory response is crucial for normal healing.
This involves active down-modulation of inammatory stimuli and repair
CD80/CD86
CD80/CD86
CD4+
Th2 cells
B-cell activation
IL-4, IL-5, IL-10,
IL-13
Th1 cells
Pro-inflammatory
TNF-α, IFN-γ
Th17 cells
Mucosal immunity
Pro-inflammatory
IL-17
Regulatory
T cells
Memory
T cells
Fas ligand
TNF-α, IFN-γ
Direct cell killing
CD8+
Anti-inflammatory
IL-10, TGF-β
PD-L1
Tumour cell
Proliferation
HLA
TCR
T-cell
activation
Antigenic
peptide
PD-1
CTLA4
CD28
CD4/CD8
Antigen-presenting cell
IL-2
Fig. 4.8 T-cell activation. Activation of T cells is initiated when an antigenic peptide bound to a human leucocyte antigen (HLA) molecule on antigen-presenting cells interacts
with the T-cell receptor expressed by T lymphocytes. Additional signals are required for T-cell activation, however. These include binding of the co-stimulatory molecules
CD80 and CD86 with CD28 on the T cell, and interleukin 2 (IL-2), which is produced in an autocrine manner by T cells that are undergoing activation. Other molecules are
present that can inhibit T-cell activation, however, including cytotoxic T-lymphocyte-associated protein 4 (CTLA4), which competes with CD28 for binding to CD80 and CD86;
and PD-1, which, by binding PD-L1, is also inhibitory. Following activation, T cells proliferate and, depending on their subtype, have various functions with distinct patterns of
cytokine production, as indicated. Memory cells are also generated that can mount a rapid immune response on encountering the same antigen. (IFN-γ = interferon-gamma;
IL = interleukin; PD-1 = programmed cell death 1; PD-L1 = programmed death ligand 1; TCR = T-cell receptor; TGF-β = transforming growth factor beta; TNF-α = tumour
necrosis factor alpha)
93.
The inflammatory response 69
4
of bystander damage to local tissues. Extravasated neutrophils undergo
apoptosis and are phagocytosed by macrophages, along with the remains
of microorganisms. Macrophages also synthesise collagenase and
elastase, which break down local connective tissue and aid in the removal
of debris. Normal tissue homeostasis is also associated with reversion of
parenchymal cells to a non-inammatory phenotype. Macrophage-derived
cytokines, including transforming growth factor-beta (TGF-β) and plate-
let-derived growth factor, stimulate broblasts and promote the synthesis
of new collagen, while angiogenic factors stimulate new vessel formation.
Chronic inammation
In most instances, the development of an active immune response
results in clearance and control of the inammatory stimulus and res-
olution of tissue damage. Failure of this process may result in chronic
inammation, with signicant associated bystander damage, known as
hypersensitivity responses. Persistence of microorganisms can result in
ongoing accumulation of neutrophils, macrophages and activated T lym-
phocytes within the lesion. If this is associated with local deposition of
brous tissue, a granuloma may form. Granulomas are characteristic of
tuberculosis and leprosy (Hansen's disease), in which the microorgan-
ism is protected by a robust cell wall that shields it from killing, despite
phagocytosis.
Laboratory features of inammation
Inammation is associated with changes in many laboratory investi-
gations. Leucocytosis is common, and reects the transit of activated
neutrophils and monocytes to the site of infection. The platelet count
may also be increased. The most widely used laboratory measure of
acute inammation is CRP. Circulating levels of many other acute phase
reactants, including brinogen, ferritin and complement components, are
also increased in response to acute inammation, while albumin levels
are reduced. Chronic inammation is frequently associated with a nor-
mocytic normochromic anaemia.
C-reactive protein
C-reactive protein (CRP) is an acute phase reactant synthesised by the liver,
which opsonises invading pathogens. Circulating concentrations of CRP
increase within 6hours of the start of an inammatory stimulus. Serum con-
centrations of CRP provide a direct biomarker of acute inammation and,
because the serum half-life of CRP is 18hours, levels fall promptly once
the inammatory stimulus is removed. Sequential measurements are useful
in monitoring disease activity (Box 4.4). For reasons that remain unclear,
some diseases are associated with only minor elevations of CRP despite
Hypothalamus:
Change in temperature set point
Fever
Sweating
Neuro-endocrine and autonomic stress responses
Flushing
↑Respiratory rate
↑Heart rate, flow murmur
Adrenal release of glucocorticoids
and catecholamines
Release of insulin
from pancreas
Bone marrow:
↑Production and mobilisation
of neutrophils
Vasodilatation
↑Local vascular
permeability
Neutrophils
+
Macrophages
Inflammatory mediators
and cytokines
Tissue damage
Bacteria
Local infection
Skin rupture
Phagocytosis
Cytokine production
Vasodilatation
↑Local vascular
permeability
↑Leucocyte influx
Headache
Delirium
Anorexia
Low blood pressure
Liver:
↑Synthesis of acute
phase proteins
Enlarged draining
lymph nodes
Ascending
lymphangitis
Local cellulitis
Pain
Redness
Swelling
Nail
Fig. 4.9 Clinical features of acute inammation. In this example, the response is to a penetrating injury and infection of the foot.
94.
70 CLINICALIMMUNOLOGY
unequivocal evidence of active inammation. These include systemic lupus
erythematosus (SLE), systemic sclerosis, ulcerative colitis and leukaemia.
An important practical point is that if the CRP is raised in these conditions,
it suggests intercurrent infection rather than disease activity. Since the CRP
is a more sensitive early indicator of the acute phase response, it is gener-
ally used in preference to the erythrocyte sedimentation rate (ESR). If both
ESR and CRP are used, any discrepancy should be resolved by assessing
the individual determinants of the ESR, which are discussed below.
Erythrocyte sedimentation rate
The ESR is an indirect measure of inammation. It measures how fast
erythrocytes fall through plasma, which is determined by the composi-
tion of plasma proteins and the morphology of circulating erythrocytes.
These factors govern the propensity of red cells to aggregate, the major
determinant of the ESR. Erythrocytes are inherently negatively charged,
which prevents them from clumping together in the blood stream. Since
plasma proteins such as brinogen and immunoglobulins are posi-
tively charged, increased concentrations of these proteins neutralise
an increase in plasma protein concentrations neutralises the negative
charge of erythrocytes, overcoming their inherent repulsive forces and
causing them aggregate, resulting in rouleaux formation. Rouleaux have
a higher mass-to-surface area ratio than single red cells, and therefore
sediment faster. The most common reason for an increased ESR is an
acute phase response, which causes an increase in the concentration of
acute phase proteins, including CRP. However, other conditions that do
not affect acute phase proteins may alter the composition and concen-
tration of other plasma proteins (see Box 4.4). For example, immuno-
globulins comprise a signicant proportion of plasma proteins but do not
participate in the acute phase response. Thus any condition that causes
an increase in serum immunoglobulins will increase the ESR without a
corresponding increase in CRP. In addition, abnormal red cell morphol-
ogy can make rouleaux formation impossible. For these reasons, an
inappropriately low ESR occurs in spherocytosis and sickle-cell anaemia.
Plasma viscosity
Plasma viscosity is another surrogate measure of plasma protein concen-
tration. Like the ESR, it is affected by the concentration of large plasma
proteins, including brinogen and immunoglobulins. It is not affected by
properties of erythrocytes and is generally considered to be more reliable
than the ESR as a marker of inammation.
Presenting problems in immune disorders
Recurrent infections
Infections can occur in otherwise healthy individuals but recurrent infec-
tion raises suspicion of an immune deciency. Depending on the compo-
nent of the immune system affected, the infections may involve bacteria,
viruses, fungi or protozoa, as summarised in Box 4.5. T-cell deciencies
can involve pathogens from all groups.
Aetiology
Infections secondary to immune deciency occur because of defects in
the number or function of phagocytes, B cells, T cells or complement, as
described later in this chapter.
Clinical assessment
Clinical features that may indicate immune deciency are listed in Box
4.6. Frequent or severe infections, or ones caused by unusual organisms
or at unusual sites, are typical of immune deciency.
Investigations
Initial investigations should include full blood count and white cell differ-
ential, CRP, renal and liver function tests, urine dipstick, serum immu-
noglobulins with protein electrophoresis, and HIV testing. Additional
microbiological tests, virology and imaging are required to identify the
causal organism and localise the site of infection, as outlined in Box 4.7.
If primary immune deciency is suspected on the basis of initial investi-
gations, more specialised tests should be considered, as summarised
in Box 4.8
Management
If an immune deciency is suspected but has not yet been formally char-
acterised, patients should not receive live vaccines because of the risk of
vaccine-induced disease. Further management depends on the underly-
ing cause and details are provided later.
4.4 Conditions commonly associated with abnormal C-reactive protein (CRP) and/or erythrocyte sedimentation rate (ESR)
Condition Consequence Effect on CRP1
Effect on ESR2
Acute bacterial, fungal or viral
infection
Stimulates acute phase response Increased (range 50–150mg/L; in
severe infections may be > 300mg/L)
Increased
Necrotising bacterial infection Stimulates profound acute
inammatory response
Greatly increased (may be
> 300mg/L)
Increased
Chronic bacterial or fungal infection
Localised abscess, bacterial
endocarditis or tuberculosis
Stimulates acute and chronic
inammatory response with polyclonal
increase in immunoglobulins, as well
as increased acute phase proteins
Increased (range 50–150mg/L) Increased disproportionately to CRP
Acute inammatory diseases
Crohn's disease, polymyalgia
rheumatica, inammatory arthritis
Stimulates acute phase response Increased (range 50–150mg/L) Increased
Systemic lupus erythematosus,
Sjögren syndrome, ulcerative colitis
Chronic inammatory response Normal Increased
Multiple myeloma Monoclonal increase in serum
immunoglobulin without acute
inammation
Normal Increased
Pregnancy, old age, end-stage renal
disease
Increased brinogen Normal Moderately increased
1
Reference range <5mg/L. 2
Reference range: adult males <10mm/hr, adult females <3–15mm/hr.
72 CLINICALIMMUNOLOGY
family history and early age at onset, then the likelihood of a familial
fever syndrome is increased.
Investigations
Blood should be taken for a full blood count, measurement of ESR and
CRP, and assessment of renal and liver function. Serum ferritin should
be checked, as very high levels support the diagnosis of Still's disease.
Blood and urine cultures should also be performed, along with an auto-
immune screen that includes measurement of antinuclear antibodies
and consideration of antineutrophil cytoplasmic antibodies to check for
evidence of SLE or vasculitis, respectively. Imaging may be required to
exclude occult infection. If these investigations provide no evidence of
infection or another cause, then genetic analysis should be considered
to conrm the diagnosis of a familial fever syndrome. Negative genetic
testing does not, however, entirely exclude a periodic fever syndrome.
Management
Symptomatic management with non-steroidal anti-inammatory drugs
(NSAIDs) should be initiated, pending the results of investigations. If the
response to NSAIDs is inadequate, glucocorticoids can be tried, pro-
vided that infection has been excluded. If a familial fever syndrome is
conrmed, then denitive therapy should be initiated, depending on the
underlying diagnosis, as discussed later in this chapter (p. 76).
Anaphylaxis
Anaphylaxis is a potentially life-threatening, systemic allergic reaction
characterised by circulatory collapse, bronchospasm, laryngeal stridor,
often associated with angioedema, and urticaria. The risk of death is
increased in patients with pre-existing asthma, particularly if this is poorly
controlled, and in situations where treatment with adrenaline (epineph-
rine) is delayed. Further details are provided in the ‘Presenting problems
in acute medicine’ section in Chapter 9
Immune deciency
The consequences of immune deciency include recurrent infection,
autoimmunity as a result of immune dysregulation, and increased sus-
ceptibility to malignancy, especially malignancy driven by viral infec-
tions such as Epstein–Barr virus. Immune deciency may arise through
intrinsic defects in immune function but is much more commonly due
to secondary causes, including infection, drug therapy, malignancy
and ageing. This section gives an overview of primary immune de-
ciencies. More than a hundred such deciencies have been described,
most of which are genetically determined and present in childhood
or adolescence. The presentation of immune deciency depends on
the component of the immune system that is defective (see Box 4.5).
There is considerable overlap and redundancy in the immune network,
however, and some diseases do not fall easily into this classication.
Primary phagocyte deciencies
Primary phagocyte deciencies typically present with recurrent bacterial
and fungal infections, which may involve unusual sites. Affected patients
require aggressive management of infections, including intravenous anti-
biotics and surgical drainage of abscesses, and long-term prophylaxis
with antibacterial and antifungal agents. The most important examples
are illustrated in Figure 4.10 and discussed below.
Chronic granulomatous disease
This is caused by mutations in genes that encode NADPH oxidase
enzymes, which results in failure of oxidative killing. The defect leads
to susceptibility to catalase-positive organisms such as Staphylococcus
aureus, Burkholderia cenocepacia and Aspergillus. Intracellular killing of
mycobacteria in macrophages is also impaired. Infections most com-
monly involve the lungs, lymph nodes, soft tissues, bone, skin and uri-
nary tract, and are characterised histologically by granuloma formation.
Most cases are X-linked
Leucocyte adhesion deciencies
These very rare disorders of phagocyte migration occur because of fail-
ure to express adhesion molecules on the surface of leucocytes, result-
ing in their inability to exit the blood stream. The most common cause is
loss-of-function mutations affecting the ITGB2 gene, which encodes the
integrin β-2 chain, a component of the adhesion molecule LFA1. They
are characterised by recurrent bacterial infections but sites of infection
lack evidence of neutrophil inltration, such as pus formation. Peripheral
blood neutrophil counts may be very high during acute infection because
of the failure of mobilised neutrophils to exit blood vessels. Specialised
tests show reduced or absent expression of adhesion molecules on
neutrophils.
4.8 Specialist investigations in suspected immune deciency
Test Value Comment
Complement (C3/C4/CH50/AP50) Investigation of recurrent pyogenic bacterial
infection
Inherited complement deciency likely to give low/
absent results on functional assays
Test vaccination Determination of functional humoral immune
response
Helpful in patients with borderline low or normal
immunoglobulins but conrmed recurrent infection
Neutrophil function Investigation of recurrent invasive bacterial and
fungal infection, especially with catalase-positive
organisms
Respiratory burst low/absent in chronic
granulomatous disease
Investigation of leucocyte adhesion deciency Leucocytosis with absent CD11a, b, c expression
Lymphocyte immunophenotyping (by ow
cytometry)
Determination of specic lymphocyte subsets, T cell,
B cell, NK cell
May dene specic primary immune deciency, e.g.
absent B cells in X-linked agammaglobulinaemia
Lymphocyte proliferation Determination of lymphocyte proliferation in
response to mitogenic stimulation
Poor responses seen in certain T-cell immune
deciencies
Cytokine production To determine T-cell immune function in response to
antigen stimulation; limited availability, not routine
Can be helpful, for example, in investigation of
atypical mycobacterial infection
Genetic testing Under specialist supervision when specic primary
immune deciency suspected
May conrm genetic cause, with implications for
family members and future antenatal testing
(NK = natural killer)
97.
Immune deficiency 73
4
Defects in cytokines and cytokine receptors
Mutations of the genes encoding cytokines such as IFN-γ, IL-12, IL-23 or
their receptors result in failure of intracellular killing by macrophages, and
affected individuals are particularly susceptible to mycobacterial infections.
Complement pathway deciencies
Loss-of-function mutations have been identied in almost all the com-
plement pathway proteins (see Fig. 4.4). While most complement de-
ciencies are rare, mannose-binding lectin deciency is common and
affects about 5% of the northern European population, many of whom
are asymptomatic (see below).
Clinical features
Patients with deciency in complement proteins can present in differ-
ent ways. In some cases, the presenting feature is recurrent infection
with encapsulated bacteria, particularly Neisseria spp., reecting the
importance of the membrane attack complex in defence against these
organisms. However, genetic deciencies of the classical complement
pathway (C1, C2 and C4) also present with an increased risk of auto-
immune disease, particularly SLE. Individuals with mannose-binding
lectin deciency have an increased incidence of bacterial infections if
subjected to an additional cause of immune compromise, such as pre-
mature birth or chemotherapy. The signicance of this condition has
been debated, however, since population studies have shown no overall
increase in infectious disease or mortality in patients with this disorder.
Deciency of the regulatory protein Cl inhibitor is not associated with
recurrent infection but causes recurrent angioedema.
Investigations
Screening for complement deciencies usually involves specialised func-
tional tests of complement-mediated haemolysis. These are known as
the CH50 (classical haemolytic pathway 50) and AP50 (alternative path-
way 50) tests. If abnormal, haemolytic tests are followed by measure-
ment of individual complement components.
Management
Patients with complement deciencies should be vaccinated with
meningococcal, pneumococcal and H. inuenzae B vaccines to boost
their adaptive immune responses. Lifelong prophylactic penicillin to
prevent meningococcal infection is recommended, as is early access
to acute medical assessment in the event of infection. Patients should
also carry a MedicAlert or similar. At-risk family members should be
screened for complement deciencies with functional complement
assays. The management of C1 esterase deciency is discussed
elsewhere.
Primary antibody deciencies
Primary antibody deciencies occur as the result of abnormalities in
B-cell function, as summarised in Figure 4.11. They are characterised
by recurrent bacterial infections, particularly of the respiratory and gas-
trointestinal tract. The most common causative organisms are encap-
sulated bacteria such as Streptococcus pneumoniae and H. inuenzae.
These disorders usually present in infancy, when the protective benet
of placental transfer of maternal immunoglobulin has waned. The most
important causes are discussed in more detail below.
Neutrophils traverse endothelium
through binding of LFA1 to ICAM1
Normal
Primary phagocyte deficiency
Leucocyte adhesion deficiency
Neutrophils cannot traverse
endothelium due to defects in
ITGB2, a component of LFA1
Chronic granulomatous
disease
Cytokines activate
macrophages
Destruction of microorganisms through
NADPH oxidase-mediated killing
Cytokine defects
LFA1
IL-23
IL-12
IFN-γ
Phagocytes cannot be
activated due to defects in
cytokines or their receptors
Microorganisms cannot be
destroyed in lysosomes due
to NADPH oxidase deficiency
IL-23
IL-12
IFN-γ
IL-23
IL-12
IFN-γ
ICAM1
Fig. 4.10 Normal phagocyte function and mechanisms of primary phagocyte deciency. Under normal circumstances, neutrophils traverse the endothelium to enter
tissues by the cell surface molecule lymphocyte function-associated antigen 1 (LFA1), which binds to intercellular adhesion molecule 1 (ICAM1) on endothelium. In order for
macrophages to engulf and kill microorganisms, they need to be activated by cytokines and also require nicotinamide adenine dinucleotide phosphate (NADPH) oxidase to
generate free radicals. Primary phagocyte deciencies can occur as the result of leucocytes being unable to traverse endothelium due to defects in LFA1, because of mutations
in cytokines or their receptors, or because of defects in NADPH oxidase. (IFN-γ = interferon-gamma; IL = interleukin)
98.
74 CLINICALIMMUNOLOGY
X-linked agammaglobulinaemia
This rare X-linked disorder is caused by mutations in the BTK gene, which
encodes Bruton tyrosine kinase, a signalling protein that is required for
B-cell development. Affected males present with severe bacterial infec-
tions during infancy. There is a marked reduction in B-cell numbers and
immunoglobulin levels are low or undetectable. Management is with
immunoglobulin replacement therapy and antibiotics to treat infections.
Selective IgA deciency
This is the most common primary antibody deciency, affecting 1:600
northern Europeans. Although IgA deciency is usually asymptomatic
with no clinical sequelae, about 30% of individuals experience recurrent
mild respiratory and gastrointestinal infections. The diagnosis can be
conrmed by measurement of IgA levels, which are low or undetectable
(< 0.05g/L). In some patients, there is a compensatory increase in serum
IgG levels. Specic treatment is generally not required.
Common variable immune deciency
Common variable immune deciency (CVID) is characterised by low
serum IgG levels and failure to make antibody responses to exoge-
nous pathogens. It is a heterogeneous adult-onset primary immune
deciency, the underlying cause is unknown in most cases, although
genetic mutations have been identied in a minority of patients. The
presentation is with recurrent infections, and bronchiectasis is a recog-
nised complication. Paradoxically, antibody-mediated autoimmune dis-
eases, such as idiopathic thrombocytopenic purpura and autoimmune
haemolytic anaemia, are common in CVID. It is also associated with an
increased risk of malignancy, particularly lymphoproliferative disease.
Specic antibody deciency
This is a poorly characterised condition resulting in defective anti-
body responses to polysaccharide antigens. Some patients are also
decient in the antibody subclasses IgG2 and IgG4, and this con-
dition was previously called IgG subclass deciency. There is over-
lap between specic antibody deciency, IgA deciency and CVID,
and some patients may progress to a more global antibody deciency
over time.
Investigations
Serum immunoglobulins (Box 4.9) should be measured in conjunction
with protein and urine electrophoresis to exclude secondary causes of
hypogammaglobulinaemia, and B- and T-lymphocyte subsets should
be measured. Specic antibody responses to known pathogens should
be assessed by measuring IgG antibodies against tetanus, H. inuen-
zae and Strep. pneumoniae (most patients will have been exposed to
these antigens through infection or immunisation). If specic antibody
levels are low, immunisation with the appropriate killed vaccine should
Failure of lymphocyte precursors:
Severe combined immune deficiency
Stem cells
Lymphoid
progenitors
Bone marrow
Failure of production of IgG antibodies:
Common variable immune deficiency
Specific antibody deficiency
IgM-producing
B cells
Failure of B-cell maturation:
X-linked agammaglobulinaemia
Immature
B cells
IgG
IgE
IgA
Plasma cells
Failure of IgA production:
Selective IgA deficiency
Fig. 4.11 B lymphocytes and primary antibody deciencies (green boxes). (Ig = immunoglobulin)
4.9 Investigation of primary antibody deciencies
Selective IgA
deciency
Normal Often elevated Absent Normal Normal Normal Not applicable*
Common
variable
immune
deciency
Normal or low Low Low or absent Low or absent Variable Variable No antibody
response
Specic
antibody
deciency
Normal Normal Normal Normal Normal Normal No antibody
response to
polysaccharide
antigens
*Test immunisation is not usually performed in IgA deciency but some patients may have impaired responses.
99.
Immune deficiency 75
4
be followed by repeat antibody measurement 6–8 weeks later; failure
to mount a response indicates a signicant defect in antibody produc-
tion. These functional tests have generally superseded IgG subclass
quantitation.
Management
The mainstay of treatment in most patients with antibody deciency is
immunoglobulin replacement therapy. Human normal immunoglobulin
(Box 4.10) is derived from plasma from hundreds of donors and contains
IgG antibodies to a wide variety of common organisms. Replacement
immunoglobulin may be administered either intravenously or subcutane-
ously, with the aim of maintaining trough IgG levels (the IgG level just prior
to an infusion) within the normal range. This has been shown to mini-
mise progression of end-organ damage and improve clinical outcome.
Patients with antibody deciencies also require aggressive treatment of
infections when they occur; prophylactic antibiotics may be indicated.
Treatment may be self-administered and is life-long. Benets of immu-
nisation are limited because of the defect in IgG antibody production,
but patients may derive some T cell benet from vaccination. As with all
primary immune deciencies, live vaccines should be avoided.
Primary T-lymphocyte deciencies
These are a group of diseases characterised by recurrent viral, proto-
zoal and fungal infections (see Box 4.5). Many T-cell deciencies are also
associated with defective antibody production because of the impor-
tance of T cells in providing help for B cells. These disorders generally
present in childhood. Several causes of T-cell deciency are recognised.
These are summarised in Figure 4.12 and discussed in more detail below.
DiGeorge syndrome
This results from failure of development of the third and fourth phary-
ngeal pouches, and is usually caused by a deletion of chromosome
22q11. The immune deciency is accounted for by failure of thymic
development; however, the immune deciency can be very heterogene-
ous. Affected patients can have very low numbers of circulating T cells
despite normal development in the bone marrow. It is associated with
multiple developmental anomalies, including congenital heart disease,
hypoparathyroidism, tracheo-oesophageal stulae, cleft lip and palate.
Bare lymphocyte syndromes
These very rare disorders are caused by mutations in a variety of genes
that regulate expression of HLA molecules or their transport to the cell
surface. If HLA class I molecules are affected, CD8+
lymphocytes fail
to develop normally, while absent expression of HLA class II molecules
affects CD4+
lymphocyte maturation. In addition to recurrent infections,
failure to express HLA class I is associated with systemic vasculitis
caused by uncontrolled activation of NK cells.
Severe combined immune deciency
Severe combined immune deciency (SCID) results from mutations in
a number of genes that regulate lymphocyte development, with fail-
ure of T-cell maturation, with or without accompanying B- and NK-cell
maturation. The most common cause is X-linked SCID, resulting from
loss-of-function mutations in the interleukin-2 receptor gamma (IL2RG)
gene. The gene product is a component of several interleukin receptors,
including those for IL-2, IL-7 and IL-15, which are absolutely required
for T-cell and NK development. This results in T-cell-negative, NK-cell-
negative, B-cell-positive SCID. Another cause is deciency of the
enzyme adenosine deaminase (ADA), which causes lymphocyte death
due to accumulation of toxic purine metabolites intracellularly, resulting in
T-cell-negative, B-cell-negative and NK-cell-negative SCID.
The absence of an effective adaptive immune response causes recur-
rent bacterial, fungal and viral infections soon after birth. Stem cell trans-
plantation (SCT) is the treatment option of rst choice. Gene therapy has
been approved for treatment of ADA deciency when there is no suitable
donor for SCT, has been used successfully in X-linked SCID and is under
investigation for a number of other causes of SCID.
Investigations
The principal tests for T-lymphocyte deciencies are a total lymphocyte
count and quantitation of individual lymphocyte subpopulations. Serum
immunoglobulins should also be measured. Second-line, functional tests
of T-cell activation and proliferation may be indicated. Patients in whom
T-lymphocyte deciencies are suspected should be tested for HIV infec-
tion. Genetic testing to identify the underlying cause is usually under-
taken following specialist referral.
Management
PatientswithT-celldecienciesshouldbeconsideredforanti-Pneumocystis
and antifungal prophylaxis, and require aggressive management of infec-
tions when they occur. Immunoglobulin replacement is indicated for asso-
ciated defective antibody production. Stem cell transplantation or gene
therapy may be appropriate in some disorders. Where a family history is
known and antenatal testing conrms a specic defect, stem cell therapy
prior to recurrent invasive infection can improve outcome.
Autoimmune lymphoproliferative syndrome
This rare disorder is caused by failure of normal lymphocyte apoptosis,
most commonly due to mutations in the FAS gene, which encodes Fas,
a signalling protein that regulates programmed cell death in lymphocytes.
This results in massive accumulation of autoreactive T cells, which cause
autoimmune-mediated anaemia, thrombocytopenia and neutropenia.
Other features include lymphadenopathy, splenomegaly and a variety
of other autoimmune diseases. Susceptibility to infection is increased
because of the neutropenia.
Secondary immune deciencies
Secondary immune deciencies are much more common than primary
immune deciencies and occur when the immune system is compromised
by external factors (Box 4.11). Common causes include infections, such
as HIV and measles, and cytotoxic and immunosuppressive drugs, par-
ticularly those used in the management of transplantation, autoimmunity
and cancer. Physiological immune deciency occurs at the extremes of
4.10 Types of immunoglobulin preparations for therapeutic use
Normal human Immunoglobulin
Derived from multiple plasma donors and provides passive immunity to
those unable to produce immunoglobulin, be that as a result of primary or
secondary antibody deciency. Immunoglobulin replacement therapy provides
short-term protection, lasting for the duration of the passively transferred
antibodies, IgG having a half-life of approximately 21 days. Treatment is
therefore required on an ongoing basis
Hyperimmune globulin
Concentrated antibody products, initially derived from animal sources
(diphtheria and tetanus anti-toxins), but more recently of human origin,
developed for the prevention of a number of infectious diseases, including
rabies as post-exposure treatment, hepatitis B and varicella zoster virus as
post-exposure prophylaxis and cytomegalovirus as prophylaxis in patients on
immune suppressive therapy, particularly transplant recipients
Convalescent plasma
Convalescent plasma derived from individual donors with high antibody
levels against certain pathogens following recovery from natural infection,
recently under investigation for the treatment of epidemic/pandemic
infection, such as Ebola virus infection and more recently COVID-19, where
clinical trials of ABO-compatible convalescent plasma for severe COVID-19
pneumonia have to date proved disappointing, possibly as a result of such
intervention being given too late in the course of infection.
100.
76 CLINICALIMMUNOLOGY
life; the decline of the immune response in the elderly is known as immune
senescence (Box 4.12). Management of secondary immune deciency is
described in the relevant chapters on infectious diseases (Ch. 13), HIV (Ch.
14), haematological disorders (Ch. 25) and oncology (Ch. 7).
Periodic fever syndromes
These rare disorders are characterised by recurrent episodes of fever and
organ inammation, associated with an elevated acute phase response.
Familial Mediterranean fever
Familial Mediterranean fever (FMF) is the most common of the familial
periodic fevers, predominantly affecting Mediterranean people, including
Arabs, Turks, Sephardic Jews and Armenians. It results from mutations of
the MEFV gene, which encodes a protein called pyrin that regulates neutro-
phil-mediated inammation by indirectly suppressing the production of IL-1.
FMF is characterised by recurrent painful attacks of fever associated with
peritonitis, pleuritis and arthritis, which last for a few hours to 4 days and are
associated with markedly increased CRP levels. Symptoms resolve com-
pletely between episodes. Most individuals have their rst attack before the
age of 20. The major complication of FMF is AA amyloidosis (see below).
Colchicine signicantly reduces the number of febrile episodes in 90% of
patients but is ineffective during acute attacks. Anti-cytokine therapy block-
ing IL-1, for example anakinra or the monoclonal antibody canakinumab,
can be effective for patients with colchicine refractory symptoms.
Mevalonate kinase deciency
Mevalonate kinase deciency, previously known as hyper-IgD syndrome,
is an autosomal recessive disorder that causes recurrent attacks of fever,
abdominal pain, diarrhoea, lymphadenopathy, arthralgia, skin lesions and
aphthous ulceration. Most patients are from Western Europe, particularly
the Netherlands and northern France. It is caused by loss-of-function
mutations in the gene encoding mevalonate kinase, which is involved
in the metabolism of cholesterol. It remains unclear why this causes an
inammatory periodic fever. Serum IgD and IgA levels may be persistently
elevated, and CRP levels are increased during acute attacks. Standard
anti-inammatory drugs, including colchicine and glucocorticoids, are
ineffective in suppressing the attacks but IL-1 inhibitors, such as ana-
kinra, and TNF inhibitors, such as etanercept, may improve symptoms
and can induce complete remission in some patients.
TNF receptor-associated periodic syndrome
TNF receptor-associated periodic syndrome (TRAPS) also known as
Hibernian fever, is an autosomal dominant syndrome caused by muta-
tions in the TNFRSF1A gene. The presentation is with recurrent attacks of
Failure of lymphocyte
precursors:
Severe combined
immune deficiency
Stem
cells
Lymphoid progenitors
Bone
marrow
Failure of expression
of HLA molecules:
Bare lymphocyte
syndromes
Failure of thymic
development:
DiGeorge syndrome
Proliferation and
maturation of thymocytes
Export of mature
T lymphocytes
to periphery
T-lymphocyte activation
and effector function
Apoptotic cell death
Failure of apoptosis:
Autoimmune lymphoproliferative
syndromes
Thymus
Failure of cytokine production:
Cytokine deficiencies
Fig. 4.12 T-lymphocyte function and dysfunction (green boxes). (HLA = human leucocyte antigen)
4.11 Causes of secondary immune deciency
Physiological
Ageing
Prematurity
Pregnancy
Infection
HIV infection
Measles
Mycobacterial infection
Iatrogenic
Immunosuppressive therapy
Antineoplastic agents
Glucocorticoids
Stem cell transplantation
Radiation injury
Antiepileptic agents
Malignancy
B-cell malignancies including
leukaemia, lymphoma and
myeloma
Solid tumours
Thymoma
Biochemical and nutritional disorders
Malnutrition
Renal insufciency/dialysis
Diabetes mellitus
Specic mineral deciencies (iron,
zinc)
Other conditions
Burns Asplenia/hyposplenism
101.
Amyloidosis 77
4
fever,arthralgia, myalgia, serositis and rashes. Attacks may be prolonged
for 1 week or more. During a typical attack, laboratory ndings include
neutrophilia, increased CRP and elevated IgA levels. The diagnosis can
be conrmed by low serum levels of the soluble type 1 TNF receptor and
by mutation screening of the TNFRSF1A gene. As in FMF, the major com-
plication is amyloidosis, and regular screening for proteinuria is advised.
Acute episodes respond to systemic glucocorticoids. Therapy with IL-1
inhibitors, such as anakinra, can be effective in preventing attacks.
Cryopyrin-associated periodic syndrome (CAPS)
This disorder includes three phenotypes: familial cold auto-inammatory
syndrome, Muckle–Wells syndrome and neonatal-onset multisystem
inammatory disease. CAPS results from gain of function mutation of
the NLRP3 gene coding cryopyrin, which forms part of the inammas-
ome. Defects lead to overproduction of IL-1, resulting in the inammatory
manifestations. Treatments are now targeted at the IL-1 pathway.
Amyloidosis
Amyloidosis is the name given to a group of acquired and hereditary dis-
orders characterised by the extracellular deposition of insoluble proteins.
Pathophysiology
Amyloidosis is caused by deposits consisting of brils of the specic pro-
tein involved, linked to glycosaminoglycans, proteoglycans and serum
amyloid P. Protein accumulation may be localised or systemic, and the
clinical manifestations depend on the organ(s) affected. Amyloid diseases
are classied by the aetiology and type of protein deposited (Box 4.13).
Clinical features
The clinical presentation may be with nephrotic syndrome, cardiomyopa-
thy or peripheral neuropathy. Amyloidosis should always be considered
as a potential diagnosis in patients with these disorders when the cause
is unclear.
Investigations
The diagnosis is established by biopsy, which may be of an affected
organ, rectum or subcutaneous fat. The pathognomonic histological fea-
ture is apple-green birefringence of amyloid deposits when stained with
Congo red dye and viewed under polarised light. Immunohistochemical
staining can identify the type of amyloid bril present. Quantitative scin-
tigraphy with radiolabelled serum amyloid P is a valuable tool in deter-
mining the overall load and distribution of amyloid deposits.
Management
The aims of treatment are to support the function of affected organs and,
in acquired amyloidosis, to prevent further amyloid deposition through
treatment of the primary cause. When the latter is possible, regression of
existing amyloid deposits may occur.
4.12 Immune senescence
T-cell responses: decline, with reduced delayed-type hypersensitivity responses.
Antibody production: decreased for many exogenous antigens. Although
autoantibodies are frequently detected, autoimmune disease is less common.
Response to vaccination: reduced; 30% of healthy older people may not
develop protective immunity after inuenza vaccination.
Allergic disorders and transplant rejection: less common.
Susceptibility to infection: increased; community-acquired pneumonia by
threefold and urinary tract infection by 20-fold. Latent infections, including
tuberculosis and herpes zoster, may be reactivated.
Manifestations of inammation: may be absent, with lack of pyrexia or
leucocytosis.
Secondary immune deciency: common.
4.13 Causes of amyloidosis
Disorder Pathological
basis
Predisposing
conditions
Other features
Acquired systemic amyloidosis
Reactive (AA)
amyloidosis
Increased
production of
serum amyloid
A as part of
prolonged or
recurrent acute
inammatory
response
Chronic infection
(tuberculosis,
bronchiectasis,
chronic abscess,
osteomyelitis)
Chronic
inammatory
diseases
(untreated
rheumatoid
arthritis, familial
Mediterranean
fever)
90% of patients
present with
non-selective
proteinuria
or nephrotic
syndrome
Light chain
amyloidosis (AL)
Increased
production of
monoclonal light
chain
Monoclonal
gammopathies,
including
myeloma,
benign
gammopathies
and
plasmacytoma
Restrictive
cardiomyopathy,
peripheral and
autonomic
neuropathy,
carpal tunnel
syndrome,
proteinuria,
spontaneous
purpura, amyloid
nodules and
plaques
Macroglossia
occurs
rarely but is
pathognomonic
Prognosis is
poor
Dialysis-
associated
(Aβ2
M)
amyloidosis
Accumulation
of circulating
β2
-microglobulin
due to failure of
renal catabolism
in kidney failure
Renal dialysis Carpal tunnel
syndrome,
chronic
arthropathy and
pathological
fractures
secondary to
amyloid bone
cyst formation
Manifestations
occur 5–10
years after the
start of dialysis
Senile systemic
amyloidosis
Normal
transthyretin
protein
deposited in
tissues
Age > 70 years Feature of
normal ageing
(affects > 90%
of 90-year-olds)
Usually
asymptomatic
Hereditary systemic amyloidosis
> 20 forms
of hereditary
systemic
amyloidosis
Production of
protein with
an abnormal
structure that
predisposes to
amyloid bril
formation. Most
commonly due
to mutations in
transthyretin
gene
Autosomal
dominant
inheritance
Peripheral and
autonomic
neuropathy,
cardiomyopathy
Renal
involvement
unusual
10% of gene
carriers are
asymptomatic
throughout life
FCPS Single Best Question
102.
78 CLINICALIMMUNOLOGY
Autoimmune disease
Autoimmunity can be dened as the presence of immune responses
against self-tissue. This may be a harmless phenomenon, identied
only by the presence of low-titre autoantibodies or autoreactive T cells.
However, if these responses cause signicant organ damage, autoim-
mune diseases occur. These are a major cause of chronic morbidity and
disability, affecting up to 1 in 30 adults at some point during life.
Pathophysiology
Autoimmune diseases result from the failure of immune tolerance, the pro-
cess by which the immune system recognises and accepts self-tissue.
Central immune tolerance occurs during lymphocyte development, when
T and B lymphocytes that recognise self-antigens are eliminated before
they develop into fully immunocompetent cells. This process is most active
in fetal life but continues throughout life as immature lymphocytes are gen-
erated. Some autoreactive cells inevitably evade deletion and escape into
the circulation, however, and are controlled through peripheral tolerance
mechanisms. Peripheral immune tolerance mechanisms include the sup-
pression of autoreactive cells by regulatory T cells; the generation of func-
tional hyporesponsiveness (anergy) in lymphocytes that encounter antigen
in the absence of the co-stimulatory signals that accompany inammation;
and cell death by apoptosis. Autoimmune diseases develop when self-re-
active lymphocytes escape from these tolerance mechanisms.
Multiple genetic and environmental factors contribute to the devel-
opment of autoimmune disease. Autoimmune diseases are much more
common in women than in men, for reasons that remain unclear. Many
are associated with genetic variations in the HLA loci, reecting the impor-
tance of HLA genes in shaping lymphocyte responses. Other important
susceptibility genes include those determining cytokine activity, co-stimu-
lation (the expression of second signals required for full T-cell activation; see
Figs. 4.7 and 4.8) and cell death. Many of the same gene variants under-
lie multiple autoimmune disorders, reecting their common pathogenesis
(Box 4.14). Even though some of these associations are the strongest that
have been identied in complex genetic diseases, they have very limited
predictive value and are generally not useful in determining management of
individual patients. Several environmental factors may be associated with
autoimmunity in genetically predisposed individuals, including infection,
cigarette smoking and hormone levels. The most widely studied of these is
infection, as occurs in acute rheumatic fever following streptococcal infec-
tion or reactive arthritis following bacterial infection. Several mechanisms
have been invoked to explain the autoimmunity that occurs after an infec-
tious trigger. These include cross-reactivity between proteins expressed
by the pathogen and the host (molecular mimicry), such as Guillain–Barré
syndrome and Campylobacter infection; release of sequestered antigens
from tissues that are damaged during infections that are not usually visi-
ble to the immune system; and production of inammatory cytokines that
overwhelm the normal control mechanisms that prevent bystander dam-
age. Occasionally, autoimmune disease may be an adverse effect of drug
treatment. For example, metabolic products of the anaesthetic agent hal-
othane can bind to liver enzymes, resulting in a structurally novel protein
that is recognised as a foreign antigen by the immune system. This can
provoke the development of autoantibodies and activated T cells, which
can cause hepatic necrosis.
Clinical features
The clinical presentation of autoimmune disease is highly variable.
Autoimmune diseases can be classied by organ involvement or by the
predominant mechanism responsible for tissue damage. The Gell and
Coombs classication of hypersensitivity is the most widely used, and
distinguishes four types of immune response that result in tissue damage
(Box 4.15).
Type I hypersensitivity is relevant in allergy but is not associated with
autoimmune disease.
Type II hypersensitivity causes injury to a single tissue or organ and
is mediated by specic autoantibodies.
Type III hypersensitivity results from deposition of immune complexes,
which initiates activation of the classical complement cascade, as well
as recruitment and activation of phagocytes and CD4+
lymphocytes.
The site of immune complex deposition is determined by the relative
amount of antibody, size of the immune complexes, nature of the
4.14 Association of specic gene polymorphisms with
autoimmune diseases
Gene Function Diseases
HLA complex Key determinants of
antigen presentation to
T cells
Most autoimmune
diseases
PTPN22 Regulation of T- and
B-cell receptor
signalling
Rheumatoid arthritis,
type 1 diabetes,
systemic lupus
erythematosus
CTLA4 Important co-
stimulatory molecule
that transmits inhibitory
signals to T cells
Rheumatoid arthritis,
type 1 diabetes
IL23R Cytokine-mediated
control of T cells
Inammatory bowel
disease, psoriasis,
ankylosing spondylitis
TNFRSF1A Control of tumour
necrosis factor network
Multiple sclerosis
ATG5 Autophagy Systemic lupus
erythematosus
4.15 Gell and Coombs classication of hypersensitivity
diseases
Type Mechanism Example of
disease in
response to
exogenous
agent
Example of
autoimmune
disease
Type I
Immediate
hypersensitivity
IgE-mediated
mast cell
degranulation
Allergic disease None described
Type II
Antibody-
mediated
Binding of
cytotoxic IgG or
IgM antibodies
to antigens on
cell surface
causes cell
killing
ABO blood
transfusion
reaction
Hyperacute
transplant
rejection
Autoimmune
haemolytic
anaemia
Idiopathic
thrombocytopenic
purpura
Goodpasture's
disease
Type III
Immune
complex-
mediated
IgG or IgM
antibodies
bind soluble
antigen to
form immune
complexes that
trigger classical
complement
pathway
activation
Serum sickness
Farmer's lung
Systemic lupus
erythematosus
Cryoglobulinaemia
Type IV
Delayed type
Activated T
cells, and
phagocytes
Acute cellular
transplant
rejection
Nickel
hypersensitivity
Type 1 diabetes
Hashimoto's
thyroiditis
103.
Autoimmune disease 79
4
antigen and local haemodynamics. Generalised deposition of immune
complexes gives rise to systemic diseases such as SLE.
Type IV hypersensitivity is mediated by activated T cells and mac-
rophages, which together cause tissue damage.
Investigations
Autoantibodies
Many autoantibodies have been identied and are used in the diagnosis
and monitoring of autoimmune diseases, as discussed elsewhere in this
book. Antibodies can be quantied either by titre (the maximum dilution
of the serum at which the antibody can be detected) or by concentra-
tion in standardised units using an enzyme-linked immunosorbent assay
(ELISA) in which the antigen is used to coat microtitre plates to which
the patient's serum is added (Fig. 4.13A). Immunoblotting (Fig. 4.13B)
can also be employed for autoantibody detection as well as qualitative
tests in which the pattern of immunouorescence staining is recorded
(Fig. 4.13C). Antibody testing can also be performed using Luminex
technology. In this case multiple test antigens are individually bound
to red and infra-red uorescently labelled polystyrene or paramagnetic
beads. Each antigen-coated bead is coated with a unique proportion of
red and infrared dyes. The patient sample is incubated with these beads
and if antibodies are present in the sample these will bind to the test anti-
gens coating the beads. After washing to remove unbound antibodies,
a third uorescent dye is added and the sample is processed through a
dual laser Luminex analyser, which can detect the unique spectral sig-
natures arising from beads coated with different test antigens allowing
detection of multiple antibodies present in a single patient sample.
Complement
Measurement of complement components can be useful in the evalu-
ation of immune complex-mediated diseases. Classical complement
pathway activation leads to a decrease in circulating C4 levels and is
A
B
C
Antibodies bind to target
Target antigen
Wash
Detection of bound antibody Quantitate on plate reader
Target antigen
Wash
Nucleolar Homogenous Speckled P-ANCA
Fig. 4.13 Autoantibody testing.
plates to which patient serum is added. If autoantibodies are present, these bind to the target antigen on the microtitre plate. The amount of bound antibody is quantitated
autoantibodies by immunoblotting. Test strips are coated with puried antigens in parallel lines. The strips are incubated with patient serum or plasma and controls. After
washing to remove unbound antibody, strips are incubated with enzyme-conjugated alkaline phosphatase-labelled anti-human IgG. After washing and addition of a chromogenic
substrate, an enzyme-mediated colour reaction develops. After a nal wash, the strips are dried and any observed banding pattern is scanned for band intensity against an
electronic template. The black bars on the strip separate the antigens being tested. Positive results are seen as vertical grey lines below the test antigen and the intensity
of staining corresponds to the concentration of antibody in the patient’s serum. The strength of staining is expressed both as a number and as a grade from + (weak) to +++
(strong). Note that all strips contain an internal quality control (marked as Ko or Co), to ensure the assay has worked. In the example shown, the patient sample was positive for
of indirect immunouorescence staining. In this assay, patient serum is added to cell substrate and a secondary antibody is added with a uorescent label to detect any bound
antibody. If antibodies are present, they are detected as bright green staining using a uorescence microscope. Different antinuclear antibody patterns may be seen in different
types of connective tissue disease using a HEp2 or HEp2000 (human epithelial cell line) as substrate (see Ch. 26). Immunouorescence can be undertaken using different
substrates, according to the autoantibody under investigation. An antinucleolar ANA can be seen in systemic sclerosis, a homogenous ANA can be seen in SLE and a speckled
ANA can be seen in SLE or Sjögren syndrome. In the context of small vessel vasculitis, the tissue substrate is ethanol-xed neutrophils. Two main staining patterns of anti-
neutrophil cytoplasmic antibodies (ANCA) are clinically relevant, cytoplasmic and perinuclear, seen in granulomatosis with polyangiitis and microspcopic polyangiitis (see Ch. 26),
the autoantibodies recognising proteinase 3 and myeloperoxidase respectively. (B and C, Nucleolar and Homogenous) Courtesy of Juliet Dunphy, Biomedical Scientist, Royal
United Hospital Bath, UK; (C, Speckled and P-ANCA) Courtesy of Mr Richard Brown, Clinical Scientist in Immunology, Southwest Pathology Services, UK
104.
80 CLINICALIMMUNOLOGY
often also associated with decreased C3 levels. Serial measurement of
C3 and C4 can be helpful as a surrogate measure of disease activity in
conditions such as SLE.
Cryoglobulins
Cryoglobulins are antibodies that can be directed against other immuno-
globulins, which form immune complexes that precipitate in the cold. They
can lead to type III hypersensitivity reactions, with typical clinical manifes-
tations including purpuric rash, often of the lower extremities, arthralgia
and peripheral neuropathy. Cryoglobulins are classied into three types,
depending on the properties of the immunoglobulin involved (Box 4.16).
Testing for cryoglobulins requires the transport of a serum specimen to
the laboratory at 37°C. Cryoglobulins should not be confused with cold
agglutinins; the latter are autoantibodies specically directed against the I/i
antigen on the surface of red cells, which can cause intravascular haemol-
ysis in the cold (p. 958).
Management
The management of autoimmune disease depends on the organ sys-
tem involved and further details are provided elsewhere in this book. In
general, treatment of autoimmune diseases involves the use of gluco-
corticoids and immunosuppressive agents, which are increasingly used
in combination with biologic agents targeting disease-specic cytokines
and their receptors. Not all conditions require immune suppression, how-
ever. For example, the management of coeliac disease involves dietary
gluten withdrawal, while autoimmune hypothyroidism requires appropri-
ate thyroxine supplementation.
Allergy
Allergic diseases are a common and increasing cause of illness, affecting
between 15% and 20% of the population at some time. They comprise a
range of disorders from mild to life-threatening and affect many organs.
Atopy is the tendency to produce an exaggerated IgE immune response
to otherwise harmless environmental substances, while an allergic dis-
ease can be dened as the clinical manifestation of this inappropriate IgE
immune response.
Pathophysiology
The immune system does not normally respond to the many environ-
mental substances to which it is exposed on a daily basis. In allergic indi-
viduals, however, an initial exposure to a normally harmless exogenous
substance (known as an allergen) triggers the production of specic IgE
antibodies by activated B cells. These bind to high-afnity IgE receptors
on the surface of mast cells, a step that is not itself associated with clin-
ical sequelae. However, re-exposure to the allergen binds to and cross-
links membrane-bound IgE, which activates the mast cells, releasing a
variety of vasoactive mediators (the early phase response; Fig. 4.14 and
see Chapter 9). This type I hypersensitivity reaction forms the basis of
an allergic reaction, which can range from sneezing and rhinorrhea to
anaphylaxis (Box 4.17). In some individuals, the early phase response
is followed by persistent activation of mast cells, manifest by ongoing
swelling and local inammation. This is known as the late phase reac-
tion and is mediated by mast cell metabolites, basophils, eosinophils
and macrophages. Long-standing or recurrent allergic inammation may
give rise to a chronic inammatory response characterised by a complex
inltrate of macrophages, eosinophils and T lymphocytes, in addition to
mast cells and basophils. Once this has been established, inhibition of
mast cell mediators with antihistamines is clinically ineffective in isolation.
Mast cell activation may also be non-specically triggered through other
signals, such as neuropeptides, anaphylotoxins and bacterial peptides.
The increasing incidence of allergic diseases is largely unexplained
but one widely held theory is the ‘hygiene hypothesis’. This proposes
that infections in early life are critically important in maturation of the
immune response and bias the immune system against the development
of allergies; the high prevalence of allergic disease is the penalty for the
decreased exposure to infection that has resulted from improvements
in sanitation and health care. Genetic factors also contribute strongly to
the development of allergic diseases. A positive family history is com-
mon in patients with allergy, and genetic association studies have iden-
tied a wide variety of predisposing variants in genes controlling innate
immune responses, cytokine production, IgE levels and the ability of the
epithelial barrier to protect against environmental agents. The expression
of a genetic predisposition is complex; it is governed by environmental
factors, such as pollutants and cigarette smoke, and the incidence of
bacterial and viral infection.
4.16 Classication of cryoglobulins
Type I Type II Type III
Immunoglobulin (Ig) isotype and
specicity
Isolated monoclonal IgM paraprotein
with no particular specicity
Immune complexes formed by
monoclonal IgM paraprotein
directed towards constant region
of IgG
Immune complexes formed by polyclonal
IgM or IgG directed towards constant region
of IgG
Prevalence 25% 25% 50%
Disease association Lymphoproliferative disease,
especially Waldenström
macroglobulinaemia
Infection, particularly hepatitis C;
lymphoproliferative disease
Infection, particularly hepatitis C;
autoimmune disease, including rheumatoid
arthritis and systemic lupus erythematosus
Symptoms Hyperviscosity:
Raynaud's phenomenon
Acrocyanosis
Retinal vessel occlusion
Arterial and venous thrombosis
Small-vessel vasculitis:
Purpuric rash
Arthralgia
Neuropathy
Cutaneous ulceration,
hepatosplenomegaly,
glomerulonephritis, Raynaud's
phenomenon
Small-vessel vasculitis:
Purpuric rash, arthralgia
Cutaneous ulceration hepatosplenomegaly,
glomerulonephritis
Raynaud's phenomenon
Protein electrophoresis Monoclonal IgM paraprotein Monoclonal IgM paraprotein No monoclonal paraprotein
Rheumatoid factor Negative Strongly positive Strongly positive
Complement Usually normal Decreased C4 Decreased C4
Serum viscosity Raised Normal Normal
105.
Allergy 81
4
Clinicalfeatures
Common presentations of allergic disease are shown in Box 4.17. Those
that affect the respiratory system and skin are discussed in more detail
in Chapters 17 and 27, respectively. Here we focus on general principles
of the approach to the allergic patient and some specic allergies. The
management of acute anaphylaxis is discussed in Chapter 9
Insect venom allergy
Local non-IgE-mediated reactions to insect stings are common and may
cause extensive swelling around the site lasting up to 7 days. These
usually do not require specic treatment. Toxic reactions to venom after
multiple (50–100) simultaneous stings may mimic anaphylaxis. In addi-
tion, exposure to large amounts of insect venom frequently stimulates
the production of IgE antibodies, and thus may be followed by allergic
reactions to single stings. Allergic IgE-mediated reactions vary from mild
to life-threatening. Antigen-specic immunotherapy (desensitisation; see
below) with bee or wasp venom can reduce the incidence of recurrent
anaphylaxis from 50% to 60% to approximately 10%, but requires 3–5
years of treatment or more.
Peanut allergy
Peanut allergy is the most common food-related allergy. More than 50%
of patients present before the age of 3 years and some individuals react
to their rst known exposure to peanuts, thought to result from sensi-
tisation to arachis oil in topical creams. Peanuts are ubiquitous in the
Western diet, and every year up to 25% of peanut-allergic individuals
experience a reaction as a result of inadvertent exposure.
Birch oral allergy syndrome
This syndrome is characterised by the combination of birch pollen hay
fever and local oral symptoms, including itch and angioedema, after con-
tact with certain raw fruits, raw vegetables and nuts. Cooked fruits and
vegetables are tolerated without difculty. It is due to shared or cross-
reactive allergens that are destroyed by cooking or digestion, and can be
conrmed by skin-prick testing using fresh fruit. Severe allergic reactions
are unusual.
Diagnosis
When assessing a patient with a complaint of allergy, it is important to
identify what the patient means by the term, as up to 20% of the UK
population describe themselves as having a food allergy; in fact, less
than 1% have true allergy, as dened by an IgE-mediated hypersensi-
tivity reaction conrmed on double-blind challenge. The nature of the
symptoms should be established and specic triggers identied, along
with the predictability of a reaction, and the time lag between exposure to
a potential allergen and onset of symptoms. An allergic reaction usually
occurs within minutes of exposure and provokes predictable, reproduc-
ible symptoms such as angioedema, urticaria and wheezing. Specic
enquiry should be made about other allergic symptoms, past and pres-
ent, and about a family history of allergic disease. Potential allergens in
the home and workplace should be identied. A detailed drug history
should always be taken, including details of adherence to medication,
possible adverse effects and the use of over-the-counter or complemen-
tary therapies.
Investigations
Skin-prick tests
Skin-prick testing is a key investigation in the assessment of patients
suspected of having allergy. A droplet of diluted standardised allergen
is placed on the forearm and the skin is supercially punctured through
the droplet with a sterile lancet. Positive and negative control mate-
rial must be included in the assessment. After 15minutes, a positive
response is indicated by a local weal and are response 2mm or more
larger than the negative control. A major advantage of skin-prick test-
ing is that the patient can clearly see the results, which may be useful
in gaining adherence to avoidance measures. Disadvantages include
the remote risk of a severe allergic reaction, so resuscitation facilities
should be available. Results are unreliable in patients with extensive
skin disease. Antihistamines inhibit the magnitude of the response and
should be discontinued for at least 3 days before testing; low-dose glu-
cocorticoids do not inuence test results. A number of other prescribed
medicines can also lead to false-negative results, including amitriptyline
and risperidone.
B
B
B
T
T B
B
B
B
Allergen
T and B cells IgE antibody
IgE receptor
Mast cell
Histamine, tryptase and
vasoactive peptides
A B C
Fig. 4.14 Type I (immediate) hypersensitivity response.
mast cells. This cross-linking of the IgE triggers mast cell activation with release of vasoactive mediators.
4.17 Clinical manifestations of allergy
Dermatological
Urticaria
Atopic eczema if chronic
Allergic contact eczema
Angioedema
Respiratory
Asthma Atopic rhinitis
Ophthalmological
Allergic conjunctivitis
Gastrointestinal
Food allergy
Other
Anaphylaxis
Drug allergy
Allergy to insect venom
106.
82 CLINICALIMMUNOLOGY
Specic IgE tests
An alternative to skin-prick testing is the quantitation of IgE directed
against the suspected allergen. The sensitivity and specicity of specic
IgE tests (previously known as radioallergosorbent tests, RAST) are lower
than those of skin-prick tests. However, IgE tests may be very useful if
skin testing is inappropriate, such as in patients taking antihistamines
or those with severe skin disease or dermatographism. They can also
be used to test for cross-reactivity – for example, with multiple insect
venoms, where component-resolved diagnostics, using recombinant
allergens, is now increasingly used rather than crude allergen extract.
Component resolved diagnostics (CRD) is a more recent development
in allergic investigation. CRD uses puried native or recombinant aller-
gens to detect specic IgE directed against individual allergenic mole-
cules. CRD can discriminate genuine sensitisation from sensitisation due
to cross reactivity and in some cases can be used in risk stratication. For
example, in patients with hazelnut allergy, the clinical features can be mild
and most consistent with the oral allergy syndrome rather than primary
nut allergy, in which case CRD may conrm specic IgE to Cor a1, a birch
pollen homologue, hence the association with the oral allergy syndrome,
whereas patients with primary hazelnut allergy positive for Cor a9 or Cor
a14 tend to have more severe allergy. Severity of reaction in peanut allergy
can also be associated with specic Ara h allergens, which are present in
peanuts. Such risk stratication can impact on patient management, for
example, identifying patients who may require an adrenaline auto-injector.
Supervised exposure to allergen
Tests involving supervised exposure to an allergen (allergen challenge)
are usually performed in specialist centres on carefully selected patients,
and include bronchial provocation testing, nasal challenge, and food or
drug challenge. These may be particularly useful in the investigation of
occupational asthma or food allergy. Patients can be considered for chal-
lenge testing when skin tests and/or IgE tests are negative, as they can
be helpful in ruling out allergic disease.
Mast cell tryptase
Measurement of serum mast cell tryptase is extremely useful in investi-
gating a possible anaphylactic event. Ideally, measurements should be
made at the time of the reaction following appropriate resuscitation, and
3hours and 24 hours later. The basis of the test is the fact that circulat-
ing levels of mast cell degranulation products rise dramatically to peak
1–2hours after a systemic allergic reaction. Tryptase is the most stable
of these and is easily measured in serum.
Serum total IgE
Serum total IgE measurements are not routinely indicated in the investi-
gation of allergic disease, other than to aid in the interpretation of specic
IgE results, as false-positive specic IgEs are common in patients with
atopy, who often have a high total IgE level. Although atopy is the most
common cause of an elevated total IgE in developed countries, there
are many other causes, including parasitic and helminth infections, lym-
phoma, drug reactions and eosinophilic granulomatosis with polyangiitis
(previously known as Churg–Strauss vasculitis). Normal total IgE levels
do not exclude allergic disease.
Eosinophilia
Peripheral blood eosinophilia is common in atopic individuals but lacks
specicity. Eosinophilia of more than 20% or an absolute eosinophil count
over 1.5×109
/L should initiate a search for a non-atopic cause, such as
eosinophilic granulomatosis with polyangiitis or parasitic infection.
Management
Several approaches can be deployed in the management of allergic indi-
viduals, as discussed below.
Avoidance of the allergen
This is indicated in all cases and should be rigorously attempted, with the
advice of specialist dietitians and occupational physicians as necessary.
Antihistamines
Antihistamines are useful in the management of allergy as they inhibit the
effects of histamine on tissue H1
receptors. Long-acting, non-sedating
preparations are particularly useful for prophylaxis.
Glucocorticoids
These are highly effective in allergic disease, and if used topically, adverse
effects can be minimised.
Sodium cromoglicate
Sodium cromoglicate stabilises the mast cell membrane, inhibiting
release of vasoactive mediators. It is effective as a prophylactic agent
in asthma and allergic rhinitis but has no role in management of acute
attacks. It is poorly absorbed and therefore generally ineffective in the
management of food allergies.
Antigen-specic immunotherapy
This involves the sequential administration of increasing doses of aller-
gen extract over a prolonged period of time. The mechanism of action
is not fully understood but it is highly effective in the prevention of insect
venom anaphylaxis and of allergic rhinitis secondary to grass pollen
allergy. The traditional route of administration is by subcutaneous injec-
tion, which carries a risk of anaphylaxis and should be performed only
in specialised centres. Sublingual immunotherapy is also increasingly
used. Clinical studies to date do not support the use of allergen immu-
notherapy for food hypersensitivity, although this is an area of active
investigation.
Omalizumab
Omalizumab is a monoclonal antibody directed against IgE; it inhibits the
binding of IgE to mast cells and basophils. It is licensed for treatment of
refractory chronic spontaneous urticaria and also for severe persistent
allergic asthma that has failed to respond to standard therapy. The dose
and frequency are determined by baseline IgE (measured before the start
of treatment) and body weight. It is under investigation for allergic rhinitis
but not yet approved for this indication.
Adrenaline (epinephrine)
Adrenaline given by injection in the form of a pre-loaded self-injectable device
can be life-saving in the acute management of anaphylaxis (see Ch. 9).
Angioedema
Angioedema is an episodic, localised, non-pitting swelling of submucous
or subcutaneous tissues.
Pathophysiology
The causes of angioedema are summarised in Box 4.18. It may be a
manifestation of allergy or non-allergic degranulation of mast cells in
response to drugs and toxins. In these conditions the main cause is mast
cell degranulation with release of histamine and other vasoactive media-
tors. In hereditary angioedema, the cause is C1 inhibitor deciency, which
leads to increased local release of bradykinin. Angiotensin-converting
enzyme (ACE) inhibitor-induced angioedema also occurs as the result of
increased bradykinin levels due to inhibition of its breakdown.
Clinical features
Angioedema is characterised by soft-tissue swelling that most frequently
affects the face (Fig. 4.15) but can also affect the extremities and gen-
italia. Involvement of the larynx or tongue may cause life-threatening
respiratory tract obstruction, and oedema of the intestinal mucosa may
cause abdominal pain and distension.
Investigations
Differentiating the mechanism of angioedema is important in determin-
ing the most appropriate treatment. A clinical history of allergy or drug
107.
Angioedema 83
4
exposurecan give clues to the underlying diagnosis. If no obvious trigger
can be identied, measurement of complement C4 is useful in differenti-
ating hereditary and acquired angioedema from other causes. If C4 levels
are low, further investigations should be initiated to look for evidence of
C1 inhibitor deciency.
Management
Management depends on the underlying cause. Angioedema associated
with allergen exposure generally responds to antihistamines and gluco-
corticoids. Following acute management of angioedema secondary to
drug therapy, drug withdrawal should prevent further attacks, although
ACE inhibitor-induced angioedema can continue for a limited period post
drug withdrawal. Management of angioedema associated with C1 inhib-
itor deciency is discussed below.
Hereditary angioedema
Hereditary angioedema (HAE), also known as inherited C1 inhibitor de-
ciency, is an autosomal dominant disorder caused by decreased produc-
tion or activity of C1 inhibitor protein. This complement regulatory protein
inhibits spontaneous activation of the classical complement pathway (see
Fig. 4.4). It also acts as an inhibitor of the kinin cascade, activation of which
increases local bradykinin levels, giving rise to local pain and swelling.
Clinical features
The angioedema in HAE may be spontaneous or triggered by local
trauma or infection. Multiple parts of the body may be involved, especially
the face, extremities, upper airway and gastrointestinal tract. Oedema of
the intestinal wall causes severe abdominal pain and many patients with
undiagnosed HAE undergo exploratory laparotomy. The most important
complication is laryngeal obstruction, often associated with minor dental
procedures, which can be fatal. Episodes of angioedema are self-limiting
and usually resolve within 48 hours. Patients with HAE generally present
in adolescence but may go undiagnosed for many years. A family history
can be identied in 80% of cases. HAE is not associated with allergic
diseases and is specically not associated with urticaria.
Investigations
Acute episodes are accompanied by low C4 levels; a low C4 during an
episode of angioedema should therefore trigger further investigation. The
diagnosis can be conrmed by measurement of C1 inhibitor levels and
function.
Management
Severe acute attacks should be treated with plasma-derived or recombi-
nant C1 inhibitor or the bradykinin receptor antagonist icatibant. Anabolic
4.18 Types of angioedema
Allergic reaction to specic
trigger
Idiopathic angioedema Hereditary angioedema ACE-inhibitor associated
angioedema
Pathogenesis IgE-mediated degradation of
mast cells
Non-IgE-mediated
degranulation of mast cells
C1 inhibitor deciency, with
resulting increased local
bradykinin concentration
Inhibition of breakdown of
bradykinin
Key mediator Histamine Histamine Bradykinin Bradykinin
Prevalence Common Common Rare autosomal dominant
disorder
0.1%–0.2% of patients
treated with ACE inhibitors
Clinical features Usually associated with
urticaria
History of other allergies
common
Follows exposure to specic
allergen, in food, animal
dander or insect venom
Usually associated with
urticaria
May be triggered by physical
stimuli such as heat, pressure
or exercise
Dermatographism common
Occasionally associated with
underlying infection or thyroid
disease
Not associated with urticaria
or other features of allergy
Does not cause anaphylaxis
May cause life-threatening
respiratory tract obstruction
Can cause severe abdominal
pain
Not associated with urticaria
Does not cause anaphylaxis
Usually affects the head and
neck, and may cause life-
threatening respiratory tract
obstruction
Can occur years after the
start of treatment
Investigations Specic IgE tests or skin-
prick tests
Specic IgE tests and skin-
prick tests often negative
Hypothyroidism should be
excluded
Complement C4 (invariably
low in acute attacks)
C1 inhibitor levels
No specic investigations
Treatment Allergen avoidance
Antihistamines
Antihistamines are mainstay
of treatment and prophylaxis
Unresponsive to
antihistamines
Anabolic steroids
C1 inhibitor concentrate or
icatibant for acute attacks
ACE inhibitor should be
discontinued
ARBs should be avoided if
possible unless there is a
strong indication
Associated drug reactions Specic drug allergies NSAIDs
Opioids, radiocontrast media
ACE inhibitors, ARBs
(ACE = angiotensin-converting enzyme; ARBs = angiotensin II receptor blockers; IgE = immunoglobulin E; NSAIDs = non-steroidal anti-inammatory drugs)
B
A
Fig. 4.15 Angioedema.
From Helbert M. Flesh and bones of
immunology. Edinburgh: Churchill Livingstone, Elsevier Ltd; 2006.
108.
84 CLINICALIMMUNOLOGY
steroids, such as danazol, can be used to prevent attacks and act by
increasing endogenous production of complement proteins, but is limited
by treatment toxicity. Tranexamic acid can be helpful as prophylaxis in
some patients. C1 inhibitor concentrate can also be used as prophylaxis,
for example for surgical or dental intervention. Patients can be taught to
self-administer therapy and should be advised to carry a MedicAlert or
similar. More recently, a humanised monoclonal antibody which inhibits
plasma kallikrein activity, limiting the production of bradykinin, has been
developed for the prevention of recurrent attacks in patients with C1
inhibitor deciency. This is not licensed for acute attacks.
Acquired C1 inhibitor deciency
This rare disorder is clinically indistinguishable from HAE but presents
in late adulthood. It is associated with autoimmune and lymphoprolif-
erative diseases. Most cases are due to the development of autoan-
tibodies to C1 inhibitor, but the condition can also be caused by
autoantibodies that activate C1. Treatment of the underlying disorder
may induce remission of angioedema. As with HAE, a low C4 is seen
during acute episodes.
Pregnancy and the immune system
Major adaptations occur in the immune system during pregnancy so
that the mother does not mount an immune response to the developing
fetus. These adaptations can inuence the risk and severity of certain
infectious diseases such as varicella pneumonia (see p. 1271) as well
as the activity of some autoimmune diseases. Some considerations for
the development and management of immunological diseases during
pregnancy and breastfeeding are summarised in Box 4.19
Transplantation and graft rejection
Transplantation provides the opportunity for denitive treatment of end-
stage organ disease. The major complications are graft rejection, drug
toxicity and infection consequent to immunosuppression. Transplant
survival continues to improve, as a result of the introduction of less toxic
immunosuppressive agents and increased understanding of the pro-
cesses of transplant rejection. Stem cell transplantation and its compli-
cations are discussed in more detail in Chapter 25
Transplant rejection
Solid organ transplantation inevitably stimulates an aggressive immune
response by the recipient, unless the transplant is between monozygotic
twins. The type and severity of the rejection response is determined by
the genetic disparity between the donor and recipient, the immune sta-
tus of the host and the nature of the tissue transplanted (Box 4.20). The
most important genetic determinant is the difference between donor and
recipient HLA proteins. The extensive polymorphism of these proteins
means that donor HLA antigens are almost invariably recognised as for-
eign by the recipient immune system, unless an active attempt has been
made to minimise incompatibility.
Hyperacute rejection results in rapid and irreversible destruction
of the graft (see Box 4.20). It is mediated by pre-existing recipient
antibodies against donor HLA antigens, which arise as a result of
previous exposure through transplantation, blood transfusion or
pregnancy. It is very rarely seen in clinical practice, as the use of
screening for anti-HLA antibodies and pre-transplant cross-match-
ing ensures the prior identication of such recipient–donor
incompatibility.
Acute cellular rejection is the most common form of graft rejection. It
is mediated by activated T lymphocytes and results in deterioration
in graft function. If allowed to progress, it may cause fever, pain and
tenderness over the graft. It is usually amenable to increased immu-
nosuppressive therapy.
Acute vascular rejection is mediated by antibody formed de novo
after transplantation. It is more curtailed than the hyperacute
response because of the use of intercurrent immunosuppression but
it is also associated with reduced graft survival. Aggressive immu-
nosuppressive therapy is indicated and physical removal of antibody
through plasmapheresis may be indicated in severe causes. Not
all post-transplant anti-donor antibodies cause graft damage; their
consequences are determined by specicity and ability to trigger
other immune components, such as the complement cascade.
4.19 Immunological diseases in pregnancy
Allergic disease
Maternal dietary restrictions during pregnancy or lactation: current
evidence does not support these for prevention of allergic disease.
Breastfeeding for at least 4 months: prevents or delays the occurrence
of atopic dermatitis, cow's milk allergy and wheezing in early childhood, as
compared with feeding formula milk containing intact cow's milk protein.
Autoimmune disease
Suppressed T-cell-mediated immune responses in pregnancy: may suddenly
reactivate post-partum. Some autoimmune diseases may improve during pregnancy
but are immediately after delivery. Systemic lupus erythematosus (SLE) is an
exception, however, as it is prone to exacerbation in pregnancy or the puerperium.
Passive transfer of maternal antibodies: can mediate autoimmune disease in
the fetus and newborn, including SLE, Graves' disease and myasthenia gravis.
Antiphospholipid syndrome (p. 987): an important cause of fetal loss,
intrauterine growth restriction and pre-eclampsia.
HIV in pregnancy: see p. 367.
4.20 Classication of transplant rejection
Type Time Pathological ndings Mechanism Treatment
Hyperacute rejection Minutes to hours Thrombosis, necrosis Pre-formed antibody to
donor antigens results in
complement activation (type II
hypersensitivity)
None – irreversible graft loss
Acute cellular rejection 5–30 days Cellular inltration CD4+
and CD8+
T cells (type
IV hypersensitivity)
Increase immunosuppression
Acute vascular rejection 5–30 days Vasculitis Antibody and complement
activation
Increase immunosuppression
Chronic allograft failure > 30 days Fibrosis, scarring Immune and non-immune
mechanisms
Minimise drug toxicity,
control hypertension and
hyperlipidaemia
109.
Tumour immunology 85
4
Chronic allograft failure, also known as chronic rejection, is a major
cause of graft loss. It is associated with proliferation of transplant
vascular smooth muscle, interstitial brosis and scarring. The
pathogenesis is poorly understood but contributing factors include
immunological damage caused by subacute rejection, hypertension,
hyperlipidaemia and chronic drug toxicity.
Investigations
Pre-transplantation testing
HLA typing determines an individual's HLA polymorphisms and facili-
tates donor–recipient matching. Potential transplant recipients are also
screened for the presence of anti-HLA antibodies. The recipient is
excluded from receiving a transplant that carries these alleles.
Donor–recipient cross-matching is a functional assay that directly
tests whether serum from a recipient (which potentially contains anti-do-
nor antibodies) is able to bind and/or kill donor lymphocytes. It is specic
to a prospective donor–recipient pair and is done immediately prior to
transplantation. A positive cross-match is a contraindication to trans-
plantation because of the risk of hyperacute rejection.
Post-transplant biopsy: C4d staining
C4d is a fragment of the complement protein C4 (see Fig. 4.4).
Deposition of C4d in graft capillaries indicates local activation of the
classical complement pathway and provides evidence of antibody-me-
diated damage. This is useful in the early diagnosis of vascular rejection.
Complications of transplant immunosuppression
Transplant recipients require indenite treatment with immunosuppres-
sive agents. In general, two or more immunosuppressive drugs are
used in synergistic combination in order to minimise adverse effects
(Box 4.21). The major complications of long-term immunosuppression
are infection and malignancy. The risk of some opportunistic infections
may be minimised through the use of prophylactic medication, such as
ganciclovir for cytomegalovirus prophylaxis and trimethoprim–sulfameth-
oxazole for Pneumocystis prophylaxis. Immunisation with killed vaccines
is appropriate, although the immune response may be curtailed. Live
vaccines should not be given.
The increased risk of malignancy arises because T-cell suppression
results in failure to control viral infections associated with malignant
transformation. Virus-associated tumours include lymphoma (associated
with Epstein–Barr virus), Kaposi's sarcoma (associated with human her-
pesvirus 8) and skin tumours (associated with human papillomavirus).
Immunosuppression is also linked with a small increase in the incidence
of common cancers not associated with viral infection (such as lung,
breast and colon cancer), reecting the importance of T cells in anti-can-
cer surveillance.
Organ donation
The major problem in transplantation is the shortage of organ donors.
Cadaveric organ donors are usually previously healthy individuals who
experience brainstem death, frequently as a result of road trafc acci-
dents or cerebrovascular events. Even if organs were obtained from all
potential cadaveric donors, though, their numbers would be insufcient
to meet current demands. An alternative is the use of living donors.
Altruistic living donation, usually from close relatives, is widely used in
renal transplantation. Living organ donation is inevitably associated with
some risk to the donor and it is highly regulated to ensure appropriate
appreciation of the risks involved. Because of concerns about coercion
and exploitation, non-altruistic organ donation (the sale of organs) is ille-
gal in most countries.
Tumour immunology
Surveillance by the immune system is critically important in monitor-
ing and removing damaged and mutated cells as they arise. The abil-
ity of the immune system to kill cancer cells effectively is inuenced
by tumour immunogenicity and specicity. Many cancer antigens are
poorly expressed and specic antigens can mutate, either sponta-
neously or in response to treatment, which can result in evasion of
immune responses. In addition, the inhibitory pathways that are used
to maintain self-tolerance and limit collateral tissue damage during anti-
microbial immune responses can be co-opted by cancerous cells to
evade immune destruction. Recognition and understanding of these
immune checkpoint pathways has led to the development of a number
of new treatments for cancers that are otherwise refractory to treat-
ment. Immune checkpoint blockade enhances anti-tumour immunity
by blocking down-regulators of immune activation. Immune check-
point inhibitors targeting CTLA-4, PD1 and PD-L1, such as ipilimimab,
nivolumab, and pembrolizumab, have shown benet in a number of
tumour types, including melanoma, non-small cell lung cancer, urothe-
lial cancers, colorectal malignancy and classic Hodgkin’s lymphoma.
These agents can, however, have serious inammatory side effects,
with immune-related adverse events most commonly involving the
skin, liver, endocrine and gastrointestinal tracts, which may be treat-
ment limiting. The effects of the different agents vary, with lung and
thyroid involvement being more common with anti-PD1 therapy, colitis
and hypophysitis being more common with anti-CTLA-4 therapy, with
4.21 Immunosuppressive drugs used in transplantation
Drug Mechanism of action Major adverse effects
Anti-proliferative
agents
Azathioprine,
mycophenolate mofetil
Inhibit lymphocyte
proliferation by
blocking DNA synthesis
May be directly
cytotoxic at high doses
Increased susceptibility
to infection
Leucopenia
Hepatotoxicity
Calcineurin inhibitors
Ciclosporin, tacrolimus
Inhibit T-cell signalling;
prevent lymphocyte
activation; block
cytokine transcription
Increased susceptibility
to infection
Hypertension
Nephrotoxicity
Diabetogenic
(especially tacrolimus)
Gingival hypertrophy,
hirsutism (ciclosporin)
Glucocorticoids Decrease phagocytosis
and release of
proteolytic enzymes;
decrease lymphocyte
activation and
proliferation; decrease
cytokine production;
decrease antibody
production
Increased susceptibility
to infection
Multiple other
complications
Anti-thymocyte
globulin (ATG)
Antibodies to cell
surface proteins
deplete or block T cells
Profound non-specic
immunosuppression
Increased susceptibility
to infection
Basiliximab Monoclonal antibody
directed against
CD25 (IL-2Rα chain),
expressed on activated
T cells
Increased susceptibility
to infection
Gastrointestinal side-
effects
Belatacept Selectively inhibits
T-cell activation
through blockade of
CD80/CD86
Increased susceptibility
to infection and
malignancy
Gastrointestinal side-
effects
Hypertension
Anaemia/leucopenia
110.
86 CLINICALIMMUNOLOGY
anti-CTLA-4 therapy-related events often being more severe. However,
patients who have had a favourable response to immune checkpoint
blockade but discontinue as a result of immune-related adverse events
may maintain their anti-tumour response. Glucocorticoids are consid-
ered rst line therapy for these side effects, with additional immune
suppression if required. The development of autoimmunity reects the
importance of these pathways in the control of self-tolerance.
Another recent advance is CAR-T cell therapy, in which genetically
engineered, chimaeric antigen receptor T cells are specically developed
for an individual patient, with effective reprogramming of the patient’s
immune cells, which are then used to target their cancer. This has been
applied to certain otherwise treatment-refractory haematological malig-
nancies (Ch. 25). Not only is it a very expensive treatment, but cytokine
storm and subsequent antibody deciency are predictable side effects of
the CAR T-cell therapies directed at B-cell antigens, requiring manage-
ment in their own right.
Further information
Websites
allergy.org.au An Australasian site providing information on allergy, asthma and
immune diseases.
allergyuk.org UK site for patients and health-care professionals.
anaphylaxis.org.uk Provides information and support for patients with severe
allergies.
info4pi.org A US site managed by the non-prot Jeffrey Modell Foundation, which
provides extensive information about primary immune deciencies .
niaid.nih.gov National Institute of Allergy and Infectious Diseases: provides useful
information on a variety of allergic diseases, immune deciency syndromes and
autoimmune diseases.
111.
Multiple Choice Questions
4.1.In the investigation of allergy, which of the following statements are
correct?
A. Mast cell tryptase is labile in serum and therefore not a useful
biomarker of mast cell activation
B. Measurement of total IgE is not useful
C. Component resolved diagnostics can have predictive value in
some food allergies
D. Skin testing is not affected by antihistamine medication
E. Some antidepressant medications can lead to false-negative
skin test results
Answer: C and E.
Mast cell tryptase is stable in serum, making it a particularly con-
venient biomarker of mast cell activation. A total IgE can be helpful in
atopic patients as a high total level can be associated with false-positive
specic IgEs. Component resolved diagnostics uses puried native or
recombinant allergens to detect specic IgE directed against individual
allergenic molecules. CRD can discriminate genuine sensitisation from
sensitisation due to cross reactivity and in some cases can be used in
risk stratication, having predictive value, such as in peanut and some
nut allergy. A number of medications, including certain antidepressant
classes, can have antihistamine properties and thereby interfere with skin
testing. Patients should be advised to discontinue interacting medica-
tions in advance of testing.
4.2. Which of the following are required for naïve T-cell activation?
A. Antigen processing by antigen-presenting cells
B. Antigenic peptide presentation by pattern recognition
receptors
C. HLA class 1 for CD4 T cells
D. Co-stimulatory molecules
E. Intracellular T-cell signalling
Answer: A, D and E.
Unlike B cells, which recognise native antigen, T cells require antigen
processing through professional antigen-presenting cells, with presenta-
tion of antigenic peptide by self-HLA molecules expressed at the APC
surface. The Ag–HLA complex then interacts with the T-cell receptor.
Pattern recognition receptors are expressed by phagocytic cells and
recognise pathogen-associated molecular patterns on invading micro-
organisms. They do not present antigen to T cells. CD4 T cells recognise
antigenic peptide presented by HLA class II molecules; CD8 T cells rec-
ognise antigenic peptide presented by HLA class I molecules. A second
signal, known as co-stimulation, is required for naïve T-cell activation.
Downstream intracellular T cells signalling then drives T-cell proliferation.
4.3. Which of the following statements are correct regarding T-cell
populations?
A. CD4 T cells kill virally infected cells through production of pore-
forming molecules such as perforin and release of enzymes
triggering apoptosis of the target cell
B. CD8 T cells are important in defence against viral infection
C. Th1 (T-helper) cells typically produce IL-2, IFN γ and TNF-α
D. T regs are regulatory CD4 T cells that promote activation of
other cells and augment autoimmune disease
E. Th-17 cells are pro-inammatory cells that produce IL-17 and
have a key role in defence against extracellular bacteria and
fungi
Answer: B, C and E.
CD8 T cells, not CD4 T cells, kill infected cells directly through the
production of pore-forming molecules such as perforin and release of
enzymes triggering apoptosis of the target cell. CD8 T cells are particu-
larly important in defence against viral infection. Th1 (T-helper) cells typ-
ically produce IL-2, IFN-γ and TNF-α, and support the development of
delayed-type hypersensitivity responses. T-regulatory cells (T regs) are a
subset of specialised CD4+ lymphocytes important in actively suppress-
ing activation of other cells and preventing autoimmune disease. They
produce cytokines such as TGF-beta and IL-10. Th17 cells are pro-in-
ammatory cells dened by their production of IL-17. They have a key
role in defence against extracellular bacteria and fungi. They also have a
role in the development of autoimmune disease.
4.4. In tumour immunology, which of the following statements are
correct?
A. NK cells have an important role in immune surveillance
B. Tumour cells reliably express HLA molecules to allow immune
recognition by T cells
C. Immune checkpoint blockade includes anti-CTLA and
anti-PD1 pathways
D. Autoimmune disease is a recognised complication of immune
checkpoint blockade in tumour therapy
E. Tumour progression occurs rapidly if immune checkpoint
blockade is withdrawn
Answer: A, C and D.
NK cells have an important role in tumour surveillance especially as
tumour cells lose their HLA expression, thereby allowing NK cell activa-
tion. Loss of HLA expression is one of the mechanisms by which tumours
can evade T-cell immunity. Immune checkpoint blockade includes anti-
bodies directed at the normally downregulatory pathways, thereby allow-
ing immune cells to be active against the tumour. Some patients maintain
the anti-tumour effect of immune checkpoint blockade despite treatment
withdrawal in the event of drug toxicity.
4.5. In the context of organ transplantation, which of the following
statements are correct?
A. The major complications are graft rejection, drug toxicity and
infection
B. HLA incompatibility does not have a bearing on transplant
outcome
C. Acute cellular rejection is predominantly mediated by activated
B cells
D. Post-transplantation, failure to control viral infections
associated with malignant transformation leads to an increased
risk of malignancy
E. Co-stimulatory blockade has no role in post-transplant immune
suppression
Answer: A and D.
The major complications of transplantation are graft rejection, drug
toxicity and infection consequent to immunosuppression. Solid organ
transplantation stimulates an aggressive immune response by the recip-
ient, unless the transplant is between monozygotic twins. The most
important genetic determinant is the difference between donor and
recipient HLA proteins. The polymorphism of these proteins means that
donor HLA antigens are almost invariably recognised as foreign by the
recipient immune system, unless an active attempt has been made to
minimise incompatibility. Acute cellular rejection is mediated by activated
T lymphocytes and results in deterioration in graft function. The risk of
112.
post-transplant malignancy arisesbecause T-cell suppression results in
failure to control viral infections associated with malignant transforma-
tion, e.g. lymphoma associated with Epstein–Barr virus, Kaposi's sar-
coma associated with human herpesvirus 8 and skin tumours associated
with human papillomavirus. Co-stimulatory blockade, using the CTLA-4
fusion protein belatacept, with high afnity for CD80/86 on T cells, selec-
tively inhibits T-cell activation and has a role in post-transplant immune
suppression.
113.
Population health and
epidemiology
5
HCampbell
DA McAllister
Global burden of disease and underlying risk factors 88
Life expectancy 88
Global causes of death and disability 88
Risk factors underlying disease 88
Social determinants of health 89
The hierarchy of systems – from molecules to ecologies 89
The life course 89
Preventive medicine 89
Principles of screening and immunisation 90
Screening 90
Immunisation 91
Epidemiology 91
Understanding causes and effect 91
Mendelian randomisation 91
Health data/informatics 94
Management of epidemics 95
114.
88 POPULATIONHEALTH AND EPIDEMIOLOGY
The UK Faculty of Public Heath denes public health as ‘The science
and art of promoting and protecting health and well-being, preventing
ill-health and prolonging life through the organised efforts of society’.
This recognises that there is a collective responsibility for the health of
the population which requires partnerships between government, health
services and other partners to promote and protect health and prevent
disease. Population health has been dened as ‘the health outcomes of
a group of individuals, including the distribution of such outcomes within
the group’. Medical doctors can play a role in all these efforts to improve
health both as part of their clinical work but also through supporting
broader actions to improve public health.
Global burden of disease and underlying risk
factors
The Global Burden of Disease (GBD) exercise was initiated by the World
Bank in 1992, with rst estimates appearing in the World Development
Report in 1993. Regular updated estimates have been published since
that time together with projections of future disease burden. The aim of
the exercise was to produce reliable and internally consistent estimates
of disease burden for all diseases and injuries and to assess their physi-
ological, behavioural and social risk factors so that this information could
be made available to health workers, researchers and policy-makers.
The GBD exercise adopted the metric ‘disability life year’ or DALY to
describe population health. This combines information about prema-
ture mortality in a population (measured as Years of Life Lost from an
‘expected’ life expectancy) and years of life lived with disability (Years
of Life lived with Disability (YLD), which is weighted by a severity factor).
The International Classication of Disease (ICD) rules, which assign one
cause to each death, are followed. All estimates are presented by age
and sex groups and by regions of the world. Many countries now also
report their own national burden of disease data.
Life expectancy
Global life expectancy at birth increased from 61.7 years in 1980 to 73.0
years in 2017, an increase of about 0.3 years per calendar year. This
change is due to a substantial fall in child mortality (mainly due to com-
mon infections) partly offset by rises in mortality from adult conditions
such as diabetes and chronic kidney disease. Some areas have not
shown these increases in life expectancy in men, often due to war and
interpersonal violence.
Global causes of death and disability
Box 5.1 shows a ranked list of the major causes of global deaths in 2019.
Communicable, maternal, neonatal and nutritional causes accounted for
about one-quarter of deaths worldwide – down from about one-third in
1990. In contrast, deaths from non-communicable diseases are increas-
ing in importance and now account for about two-thirds of all deaths
globally,: including about 18.5 million from cardiovascular disease (ischae-
mic heart disease and stroke), 10 million from cancer and about 4 million
from chronic respiratory diseases. The age standardised death rates for
most diseases globally are falling. However, despite this, the numbers of
deaths from many diseases are rising due to global population growth
and the change in age structure of the population to older ages and
this is placing an increasing burden on health systems. For a few con-
ditions (e.g. HIV/AIDS, diabetes mellitus and chronic kidney disease)
age-standardised death rates continue to rise. Within this overall pattern,
signicant regional variations exist – for example, communicable, mater-
nal, neonatal and nutritional causes still account for about two-thirds of
premature mortality in sub-Saharan Africa.
GBD also provides estimates of disability from disease (Box 5.2). This
has raised awareness of the importance of conditions like depression
and other common mental health conditions, low back and neck pain
and other musculo-skeletal conditions, and asthma, which account for a
relatively large disease burden but relatively few deaths. This in turn has
resulted in greater health policy priority given to these conditions. Since
the policy focus in national health systems is increasingly on keeping
people healthy rather than only on reducing premature deaths it is impor-
tant to have measures of these health outcomes.
It is important to recognise that although these estimates represent
the best overall picture of burden of disease globally, they are based on
limited and imperfect data. Nevertheless, the quality of data underlying
the estimates and the modelling processes are improving steadily over
time and provide an increasingly robust basis for evidence-based health
planning and priority setting.
Risk factors underlying disease
Box 5.3 shows a ranked list of the main risk factors underlying GBD
in 2019 and how this ranking has changed over the past 29 years. A
number of key insights have been identied in this, the most recent, GBD
exercise:
Socio-demographic development has been progressing steadily
since 1990 but it has increased faster in countries with the highest
socio-demographic development index and thus gaps have been
widening.
5.1 Global causes of death – top 15 ranked causes 2019
[rank in 1990]
1. Cardiovascular disease [1]
2. Neoplasms [2]
3. Chronic respiratory [6]
4. Respiratory infections and TB [3]
5. Diabetes and CKD [10]
6. Digestive diseases [8]
7. Neurological disorders [15]
8. Maternal and neonatal [15]
9. Unintentional injuries [9]
10. Enteric infections [5]
11. Transport injuries [13]
12. Self harm and violence [12]
13. Other non-communicable diseases [11]
14. HIV/AIDS and STIs [17]
15. NTDs and malaria [14]
(CKD = chronic kidney disease; TB = tuberculosis; STIs = sexually transmitted infections;
NTDs = neglected tropical diseases)
From GBD 2019. https://vizhub.healthdata.org/gbd-compare/.
5.2 Global disability – top 15 ranked causes 2019 [rank in 1990]*
1. Musculoskeletal disorders [1]
2. Mental disorders [2]
3. Other non-communicable diseases [3]
4. Sense organ diseases [5]
5. Neurological disorders [4]
6. Diabetes and CKD [12]
7. Skin diseases [7]
8. Unintentional injuries [8]
9. Nutritional deciencies [6]
10. Cardiovascular diseases [11]
11. Chronic respiratory diseases [9]
12. Substance use [13]
13. Maternal and neonatal conditions [17]
14. Transport injuries [16]
15. Digestive diseases [15]
*By years of life lived with disability (YLD).
(CKD = chronic kidney disease)
From GBD 2019. https://vizhub.healthdata.org/gbd-compare/.
115.
Social determinants ofhealth 89
5
5.4 ‘Hierarchy of systems’ applied to ischaemic heart disease
Level in the hierarchy Example of effect
Molecular ApoB mutation causing hypercholesterolaemia
Cellular Foam cells accumulate in vessel wall
Tissue Atheroma and thrombosis of coronary artery
Organ Ischaemia and infarction of myocardium
System Cardiac failure
Person Limited exercise capacity, impact on employment
Family Passive smoking, diet
Community Shops and leisure opportunities
Population Prevalence of obesity
Society Policies on smoking, screening for risk factors
Ecology Agriculture inuencing fat content in diet
Health systems need to transform to be better able to respond to
the changing pattern of NCDs and disabilities.
The Millennium Development Goal (MDG) programme from 2000 to
2015 has led to faster progress in reducing deaths from maternal,
child and neonatal conditions/TB/HIV/malaria but this level of atten-
tion now needs to be directed at NCDs.
Public health is not giving sufcient priority to important global risk
factors which are increasing over time, such as high blood pressure,
high fasting glucose, high BMI, ambient particulate matter pollution
and drug and alcohol use.
There are many challenges resulting from the change in global
population pyramid structures, which have become inverted over
recent decades and now pose many health, nancial and political
challenges.
Social determinants of health
Health emerges from a highly complex interaction between a person’s
genetic background and environmental factors (aspects of the physical,
biological (microbes), built and social environments and also distant inu-
ences such as the global ecosystem) (Fig. 5.1).
The hierarchy of systems – from molecules to
ecologies
Inuences on health exist at many levels and extend beyond the individual
to include the family, community, population and ecology. Box 5.4 shows
an example of this for determinants of coronary heart disease and demon-
strates the importance of considering not only the disease process in a
patient but also its context. Health care is not the only determinant – and is
usually not the major determinant – of health status in the population. The
concept of ‘global health’ recognises the global dimension of health prob-
lems, whether these be, for example, emerging or pandemic infections or
global economic inuences on health internationally.
The life course
The determinants of health operate over the whole lifespan. Values and
behaviours acquired during childhood and adolescence have a pro-
found inuence on educational outcomes, job prospects and risk of
disease. These can have a strong inuence, for example, on whether
a young person takes up a damaging behaviour like smoking, risky
sexual activity and drug misuse. Inuences on health can even operate
before birth. Low birth weight can lead to higher risk hypertension and
type 2 diabetes in young adults and of cardiovascular disease in mid-
dle age. It has been suggested that under-nutrition during middle to
late gestation permanently ‘programmes’ cardiovascular and metabolic
responses.
This ‘life course’ perspective highlights the cumulative effect (through
each stage of life) on health of exposures to illness, adverse environmen-
tal conditions and behaviours that damage health.
Preventive medicine
The complexity of the interactions between physical, social and eco-
nomic determinants of health means that successful prevention is often
difcult. Moreover, the life course perspective illustrates that it may be
necessary to intervene early in life or even before birth, to prevent impor-
tant disease in later life. Successful prevention is likely to require many
interventions across the life course and at several levels in the hierarchy
of systems. The examples below illustrate this principle.
5.3 Global risk factors – top 10 ranked causes 2019 [rank in
1990]*
1. High blood pressure [7]
2. Smoking/second hand smoke exposure [5]
3. High fasting blood glucose [11]
4. Low birth weight [2]
5. High BMI [16]
6. Short gestation [3]
7. Ambient particulate matter pollution [13]
8. High LDL cholesterol [14]
9. Alcohol use [15]
10. Household air pollution [4]
From GBD 2019 Diseases and Injuries Collaborators. Global burden of 369 diseases and
injuries in 204 countries and territories, 1990–2019: a systematic analysis for the Global
Burden of Disease Study 2019. Lancet 2020; 396:1204–1222.
M
acro-econom
y,
politics, culture,
global forces
Other
neighbourhoods,
other regions
People
Age, sex and
hereditary
factors
L festyle
W
o
r
k
p
l
a
y
D
i
e
t
p
h
y
s
i
c
a
l
a
c
tivity
S
o
c
a
l
c
a
p
i
t
a
l
Community
N
e
t
w
o
r
k
s
W
e
a
t
h
c
r
e
a
t
i
o
n
Loca economy
M
a
r
k
e
t
s
B
u
d
n
g
s
p
l
a
c
e
s
Environment
A
i
r
w
a
t
er
l
a
n
d
C
m
a
t
e
c
h
a
n
g
e
Globa ecosystem
B
o
d
v
e
r
s
i
t
y
Fig. 5.1 Hierarchy of systems that inuence population health. Adapted from
an original model by Whitehead M, Dahlgren G. What can be done about inequalities
in health? Lancet 1991; 338:1059–1063
*Risk factors ranked by % of burden of disease they cause.
(BMI = body mass index; LDL = low density lipoprotein)
116.
90 POPULATIONHEALTH AND EPIDEMIOLOGY
Alcohol
Alcohol use is an increasingly important risk factor underlying global bur-
den of disease (see Box 5.3). Reasons for increasing rates of alcohol-
related harm vary by place and time but include the falling price of alcohol
(in real terms), increased availability and cultural change fostering higher
levels of consumption. Public, professional and governmental concern
has now led to a minimum price being charged for a unit of alcohol,
tightening of licensing regulations and curtailment of some promotional
activity in many countries. However, even more aggressive public health
measures will be needed to reverse the levels of harm in the population.
The approach for individual patients suffering adverse effects of alcohol
is described on pages 892 and 1240.
Smoking
Smoking is also one of the top three risk factors underlying global bur-
den of disease (see Box 5.3). It is responsible for a substantial major-
ity of cases of lung cancer and chronic obstructive pulmonary disease
(COPD), and most smokers die either from these respiratory diseases or
from ischaemic heart disease. Smoking also causes cancers of the upper
respiratory and gastrointestinal tracts, pancreas, bladder and kidney, and
increases risks of peripheral vascular disease, stroke and peptic ulcer-
ation. Maternal smoking is an important cause of fetal growth retarda-
tion. Moreover, there is increasing evidence that passive (‘second hand’)
smoking has adverse effects on cardiovascular and respiratory health.
The decline in smoking rates in many high-income countries has been
achieved not only by warning people of the health risks but also increased
taxation of tobacco, banning of advertising, banning of smoking in public
places and support for smoking cessation to maintain a decline in smok-
ing rates. However, smoking rates remain high in many poorer areas and
are increasing amongst young women. In many low-income countries,
tobacco companies have found new markets and rates are rising.
There is a complex hierarchy of systems that interact to cause smokers
to initiate and maintain their habit. At the molecular and cellular levels, nic-
otine acts on the nervous system to create dependence and acts to main-
tain the smoking habit. There are also strong inuences at the personal and
social level, such as young female smokers being motivated to ‘stay thin’
or ‘look cool’ and peer pressure. Other important inuences in the wider
environment include cigarette advertising, with the advertising budget of
the tobacco industry being much greater than that of health services.
Strategies to help individuals quit smoking (such as nicotine replacement
therapy, anti-smoking advice and behavioural support) are cost-effective
and form an important part of the overall anti-tobacco strategy.
Obesity
Obesity is an increasingly important risk factor underlying global burden
of disease (see Box 5.3). The weight distribution of almost the whole
population is shifting upwards – the slim are becoming less slim while the
overweight and obese are becoming more so. In the UK, this translates
into a 1-kilogram increase in weight per adult per year (on average over
the adult population). The current obesity epidemic cannot be explained
simply by individual behaviour and poor choice but also requires an
understanding of the obesogenic environment that encourages people
to eat more and exercise less. This includes the availability of cheap
and heavily marketed energy-rich foods, the increase in labour-saving
devices (e.g. elevators and remote controls) and the increase in passive
transport (cars as opposed to walking, cycling, or walking to public trans-
port hubs). To combat the health impact of obesity, therefore, we need to
help those who are already obese but also develop strategies that impact
on the whole population and reverse the obesogenic environment.
Poverty and afuence
The adverse health and social consequences of poverty are well doc-
umented: high birth rates, high death rates and short life expectancy.
Typically, with industrialisation, the pattern changes: low birth rates, low
death rates and longer life expectancy. Instead of infections, chronic
conditions such as heart disease dominate in an older population.
Adverse health consequences of excessive afuence are also becoming
apparent. Despite experiencing sustained economic growth for the last
50 years, people in many high-income countries are not growing any
happier and the litany of socioeconomic problems – crime, congestion,
inequality, mental health problems – persists.
Many countries are now experiencing a ‘double burden’. They have
large populations still living in poverty who are suffering from problems
such as diarrhoea and malnutrition, alongside afuent populations (often
in cities) who suffer from chronic illness such as diabetes and heart
disease.
Atmospheric pollution
Emissions from industry, power plants and motor vehicles of sulphur
oxides, nitrogen oxides, respirable particles and metals are severely pol-
luting cities and towns in Asia, Africa, Latin America and Eastern Europe.
Increased death rates from respiratory and cardiovascular disease occur
in vulnerable adults, such as those with established respiratory disease
and older people, while children experience an increase in bronchitic
symptoms. Low-income countries also suffer high rates of respiratory
disease as a result of indoor pollution caused mainly by heating and
cooking combustion.
Carbon dioxide and global warming
Climate change is arguably the world’s most important environmental
health issue. A combination of increased production of carbon dioxide
and habitat destruction, both caused primarily by human activity, seems
to be the main cause. The temperature of the globe is rising, climate is
being affected, and if the trend continues, sea levels will rise and rainfall
patterns will be altered so that both droughts and oods will become
more common. These have already claimed millions of lives during the
past 20 years and have adversely affected the lives of many more. The
economic costs of property damage and the impact on agriculture, food
supplies and prosperity have also been substantial. The health impacts
of global warming will also include changes in the geographical range
of some vector-borne infectious diseases. Currently, politicians can-
not agree on an effective framework of actions to tackle the problem.
Meanwhile, the industrialised world continues with lifestyles and levels of
waste that are beyond the planet’s ability to sustain.
Principles of screening and immunisation
Screening
Screening is the application of a screening test to a large number of
asymptomatic people with the aim of reducing morbidity or mortality
from a disease. WHO have identied a set of (Wilson and Jungner) cri-
teria to guide health systems in deciding when it is appropriate to imple-
ment screening programmes. The essential criteria are:
Is the disease an important public health problem?
Is there a suitable screening test available?
Is there a recognisable latent or early stage?
Is there effective treatment for the disease at this stage which
improves prognosis?
A suitable screening test is one that is cheap, acceptable, easy to
perform, safe and gives a valid result in terms of sensitivity and specic-
ity. Screening programmes should always be evaluated in trials so that
robust evidence is provided in favour of their adoption. These evaluations
are prone to several biases – self-selection bias, lead-time bias and length
117.
Epidemiology 91
5
bias– and these need to be accounted for in the analysis. Examples of
large-scale screening programmes in the UK include breast, colorectal
and cervical cancer national screening programmes (https://www.gov.
uk/topic/population-screening-programmes) and a number of screening
tests carried out in pregnancy and in the newborn, such as the:
diabetic eye screening programme
fetal anomaly screening programme
infectious diseases in pregnancy screening programme
newborn and infant physical examination screening programme
newborn blood spot screening programme
newborn hearing screening programme
sickle-cell anaemia and thalassaemia screening programme.
These are illustrated in Figure 5.2
Problems with screening include:
over-diagnosis (of a disease that would not have come to clinical
attention on its own or would not have led to death)
false reassurance
diversion of resources from investments that could control the dis-
ease more cost-effectively.
Immunisation
Immunisation can confer immunity to specic infectious diseases and
be either passive (through injected antibodies, such as the monoclo-
nal palivizimab against respiratory syncytial virus (RSV) infection given
to premature infants) or active (through administration of a vaccine).
Immunisation invokes antibody and/or cell-mediated immunity and
can lead to both short- and longer-term protection in the person who
is vaccinated. Immunisation has also been used to eradicate a dis-
ease such as occurred in the smallpox eradication programme and
is currently being targeted in the polio eradication programme. As
well as direct effects of vaccination a number of indirect effects can
occur – such as protection of individuals who are vaccinated through
altering disease transmission leading to ‘herd immunity’; or reduction
of antibiotic resistance through selective reduction of pneumococcal
serogroups that are associated with antibiotic resistance. The UK
immunisation schedule is described in detail and regularly updated in
the UK government publication ‘Immunisation against infectious dis-
ease’ (Green Book).
Epidemiology
Epidemiologists study disease in free-living humans, seeking to describe
patterns of health and disease and to understand how different expo-
sures cause or prevent disease (Box 5.5). Chronic diseases and risk
factors (e.g. smoking, obesity etc.) are often described in terms of their
prevalence. A prevalence is simply a proportion, for example the preva-
lence of diabetes among people aged 80 and older in developed coun-
tries is around 10%.
Events such as deaths, hospitalisations and rst occurrences of a dis-
ease are described using incidence rates, so, for example, if there are
100 new cases of a disease in a single year in a population of 1000,
the incidence rate is 105 per 1000 person-years. The rate is 105 rather
than 100 because the denominator is person-time, the sum of the total
‘exposed’ time for the population, which in this example is 950 per-
son-years. Person-time is the sum of the total ‘exposed’ time for the
population and in this example is 950 person-years. The reason the per-
son-time is less than 1000 is that 100 people experienced the event.
These 100 people are assumed to have had an event, on average, half-
way through the time-period, removing 100 ×0.5 person-years from the
exposure-time (as it is not possible to have a rst occurrence of a disease
twice).
A similar measure to the incidence rate is the cumulative incidence
or risk, which is the number of new cases as a proportion of the total
people at risk at the beginning of the exposure time. If in the example
above the same 1000 people were observed for a year (i.e. with no one
joining or leaving the group) then the one-year risk is 10% (100/1000).
The time-period should always be specied for risks.
These rates and proportions are used to describe how diseases (and
risk factors) vary according to time, person and place. Temporal varia-
tion may occur seasonally; for example, malaria occurs in the wet sea-
son but not the dry, or as longer-term ‘secular’ trends, e.g. malaria may
re-emerge due to drug resistance. Person comparisons include age, sex,
socio-economic status, employment, and lifestyle characteristics. Place
comparisons include the local environment (e.g. urban versus rural) and
international comparisons.
Understanding causes and effect
Epidemiological research complements that based on animal, cell and
tissue models, the ndings of which do not always translate to humans.
For example, only a minority of drug discoveries from laboratory research
are found to be effective when tested in people.
However, differentiating causes from mere non-causal associations is
a considerable challenge for epidemiology. This is because while labora-
tory researchers can directly manipulate conditions to isolate and under-
stand causes, such approaches are impossible in free-living populations.
Epidemiologists have developed a different approach, based around a
number of study designs (Box 5.6). Of these, the clinical trial is closest to
the laboratory experiment. An early example of a clinical trial is shown in
Figure 5.3, along with ‘effect measures’ which are used to quantify the
difference in rates and risks.
In clinical trials, patients are usually randomly allocated to treatments
so that, on average, groups are similar apart from the intervention of
interest. Nevertheless, for any particular trial, especially a small trial, the
laws of probability mean that differences can and do occur by chance.
Poorly designed or executed trials can also limit comparability between
groups. Allocation may not truly be random (e.g. because of inadequate
concealment of the randomisation sequence), and there may be system-
atic differences (biases) in the way people allocated to different groups
are treated or studied.
Such biases also occur in observational epidemiological study
designs, such as cohort, case–control and cross-sectional studies (see
Box 5.6). These designs are also much more subject to the problem of
confounding than are randomised trials.
Confounding is where the relationship between an exposure and out-
come of interest is confused by the presence of some other causal factor.
For example, coffee consumption may be associated with lung cancer
because smoking is commoner among coffee-drinkers. Here, smoking is
said to confound the association between coffee and lung cancer.
Despite these limitations, for most causes of diseases, randomised
controlled trials are not feasible because of ethical, or more often prac-
tical, considerations. Epidemiologists therefore seek to minimise bias
and confounding by good study design and analysis. Epidemiologists
subsequently make causal inferences by balancing the probability that
an observed association has been caused by chance, bias and/or con-
founding against the alternative probability that the relationship is causal.
This weighing-up requires an understanding of the frequency and impor-
tance of different sources of bias and confounding as well as the scien-
tic rationale of the putative causal relationship. It was this approach,
collectively and over a number of years, that settled the fact that smoking
causes lung cancer, and, subsequently, heart disease.
Mendelian randomisation
Mendelian randomisation (MR) is a method to study whether the rela-
tionship between a (modiable) risk factor and a disease may be causal.
It uses genetic variation in a gene that inuences the level of the risk
factor under consideration and studies the impact of this variation on
118.
92 POPULATIONHEALTH AND EPIDEMIOLOGY
Blood for sickle cell
and thalassaemia
Commence
folic acid
Pre-conception
Antenatal
Newborn
Blood for haemoglobin,
group, rhesus and
antibodies as early as
possible, or as soon
as a woman arrives for
care, including labour
Blood for syphilis, hepatitis B, HIV and
rubella susceptibility as early as possible,
or at any stage of the pregnancy,
including labour
Re-offer screening for
infectious diseases if
initially declined
For babies of hepatitis B-positive
mothers, give hepatitis B vaccination
± immunoglobulin within 24 hrs*
Repeat
haemoglobin
and antibodies
0
1
2
3
4
5
6
7
8
9
10
11
12
13
14
15
16
17
18
19
20
21
22
23
24
25
26
27
28
29
30
31
32
33
34
35
36
+1
+2
+3
+4
+5
+6
Birth
Key
Week
Blood for T21,
T18 and T13
(combined test)
Blood for T21
(quadruple test)
Newborn
physical
examination
by 72 hrs Newborn blood spot screens
(ideally on day 5) for:
sickle cell disease (SCD),
cystic fibrosis (CF), congenital
hypothyroidism (CHT) and inherited
metabolic diseases (PKU, MCADD,
MSUD, IVA, GA1 and HCU)
Note that babies who missed
the screen can be tested up to
1 year (except CF offered up
to 8 weeks)
Early pregnancy scan to
support T21, T18 and
T13 screening
Women with type 1 or type 2
diabetes are offered diabetic
eye (DE) screening annually.
In pregnancy women with
type 1 or type 2 diabetes are
offered a DE screen when
they first present for care
Give screening
information as
soon as possible
Give and discuss
newborn screening
information
Follow-up DE screen for
women with type 1 or 2
diabetes found to have
diabetic retinopathy
Further DE screen for
women with type 1 or 2
diabetes
Detailed ultrasound
scan for structural
abnormalities, including
T18 and T13
Newborn
hearing
screen
Infant physical
examination
at 6–8 weeks
Fetal anomaly (Down syndrome/T21,
Edwards syndrome/T18, Patau
syndrome/T13, and fetal anomaly
ultrasound)
Sickle cell and thalassaemia
Newborn and infant
physical examination
Newborn blood spot
Infectious diseases
in pregnancy
Diabetic eye
Newborn hearing
Fig. 5.2 UK NHS pregnancy and newborn screening programmes. Antenatal and newborn screening timeline. *To stop mother-to-baby transmission of infection follow
up all infection screens in pregnancy that are positive: carry out paediatric assessment and follow-up of mothers who are found to be HIV-positive or had syphilis treatment in
pregnancy; and if mothers are found to be susceptible to rubella then offer the mother MMR vaccination postnatally and refer to GP for second dose. (GA1 = glutaric aciduria
type 1; HCU = homocystinuria; IVA = isovaleric acidaemia; MCADD = medium-chain acyl-CoA dehydrogenase deciency; MSUD = maple syrup urine deciency;
PKU = phenylketonuria) Based on Version 8.4, January 2019. Gateway Ref: 20144696. www.gov.uk/phe/screening.
119.
Epidemiology 93
5
diseaserisk (see Fig. 5.4 for an example). The genetic variant (or mul-
tiple variants or genetic risk score) is used as an instrumental variable
under certain assumptions. MR investigates the effect of differences
in the risk factor level through the life course which have been deter-
mined by the genetic variants. This approach uses observational data
to test a proposed causal relationship and to estimate the size of effect.
Guidelines such as STROBE-MR have been published for the proper
conduct of these studies and analytic software packages are now avail-
able which contain a range of methods and tools. The selection of the
most appropriate method depends on the research question and the
data structure.
The MR approach requires very large sample sizes to have sufcient
power and very large databases of genetic and health data such as UK
Biobank are often used. MR can be conducted using either individual
level data or summary data from genome-wide association studies
(GWAS); and data from one study (one sample) or two studies with the
variant–risk factor association measure from one and the risk factor–
outcome association from the other (two sample).
Correct interpretation of MR results is challenging and multiple ana-
lytic methods are often employed. This includes methods to detect and
adjust for pleiotropy (having more than one effect) which is a common
problem in data interpretation. The strength of the conclusions depends
on the degree to which instrumental variable assumptions are met and
the level of consistency of ndings across different methods. MR has
proven useful in both identifying new causal relationships or conrming
trial results but also in redirecting research interest away from relation-
ships that have been shown not to be causal. MR can be considered
5.5 Calculation of risk using descriptive epidemiology
Prevalence
The ratio of the number of people with a longer-term disease or condition at a
specied time, to the number of people in the population who are at risk
Incidence
The number of events (new cases or episodes) occurring in the population at
risk during a dened period of time
Attributable risk
The difference between the risk (or incidence) of disease in exposed and non-
exposed populations
Attributable fraction
The ratio of the attributable risk to the incidence
Relative risk
The ratio of the risk (or incidence) in the exposed population to the risk (or
incidence) in the non-exposed population
5.6 Epidemiological study designs
Design Description Example
Clinical trial Enrols a sample from
a population and
compares outcomes
after randomly
allocating patients to an
intervention
The Medical Research
Council (MRC) streptomycin
trial – demonstrated
effectiveness of streptomycin
in tuberculosis
Cohort Enrols a sample from
a population and
compares outcomes
according to exposures
The Framingham Study –
identied risk factors for
cardiovascular disease
Case–control Enrols cases with an
outcome of interest and
controls without that
outcome, and compares
exposures between the
groups
Doll and Hill’s study on
smoking and carcinoma
of the lung (BMJ 1950, 2)
demonstrated that smoking
caused lung cancer
Cross-sectional Enrols a cross-section
(sample) of people
from the population of
interest. Obtains data on
exposures and outcomes
World Health Organization
Demographic and Health
Survey. Captures risk factor
data in a uniform way across
many countries
Enrolled 107 patients
with tuberculosis
Effect measures
Risk ratio (relative risk, RR)
Odds ratio (OR)
Absolute risk reduction (ARR)
Relative risk reduction (RRR)
Number needed to treat to prevent
one death (NNT= 1/ARR)
Random allocation
Streptomycin
55 patients
Bed rest
52 patients
Follow-up and count deaths
Events 4
Risk 7.3%
Odds 0.068
Events 15
Risk 28.8%
Odds 0.224
0.25
0.30
21.6%
74.8%
4.6
Fig. 5.3 An example of a clinical trial: streptomycin versus bed rest in
tuberculosis. Both prevalences and risks are, in fact, proportions and are therefore
frequently expressed as odds. The reasons for doing so are beyond the scope of
this text.
Genotype group B
Population
LDL-C lower LDL-C unchanged
CV event rate lower CV event rate unchanged
Random allocation of alleles
Genotype group A
Fig. 5.4 Mendelian randomisation. An example showing comparison of a
conventional trial with a Mendelian randomisation study. (CV = cardiovascular;
LDL-C = low-density lipoprotein cholesterol) Adapted with permission from
Bennett DA, Holmes MV. Heart 2017; 103:1400–1407.
120.
94 POPULATIONHEALTH AND EPIDEMIOLOGY
to provide further evidence for or against a causal relationship but care
should be taken in interpretation of the size of the expected impact from
an intervention.
Health data/informatics
As patients pass through health and social care systems, data are
recorded concerning their family background, lifestyle and disease
states, which is of potential interest to health-care organisations seek-
ing to deliver services, policy-makers concerned with improving health,
scientic researchers seeking to understand health, and also to phar-
maceutical and other commercial organisations seeking to identify
markets.
There is a long tradition of maintaining health information systems.
In most countries, the registration of births and deaths is required by
law, and in the majority, the cause of death is also recorded (Fig. 5.5).
There are numerous challenges in ensuring such data are useful, espe-
cially for making comparisons across time and place. First, a system of
standard terminologies is needed, such as the World Health Organization
International Classication of diseases, which provides a list of diagnos-
tic codes attempting to cover every diagnostic entity. Secondly, these
terms must be understood to refer to the same, or at least similar dis-
eases in different places. Thirdly, access to diagnostic skill and facilities
is required, fourthly standard protocols for assigning clinical diagnoses
to ICD-10 codes are needed and fthly, robust quality control processes
are needed to maintain some level of data completeness and accuracy.
Many countries employ similar systems for hospitalisations, either to
allow recovery of healthcare utilisation costs, or to manage and plan ser-
vices. Similar data are, however, rarely collected for community-based
healthcare. Nor are detailed data on health-care process generally
included in national data systems.
Consequently, there has been considerable interest in using data from
information technology systems used to deliver care – such as electronic
patient records, drug-dispensing databases, radiological software, and
clinical laboratory information systems.
Data from such systems are, of course, much less structured than
those obtained from vital registrations. Moreover, the completeness of
such data depends greatly on local patterns of healthcare utilisation as
well as how clinicians and others use IT systems within different settings.
As such, deriving useful unbiased information from such data is a con-
siderable challenge.
Much of the discipline of health informatics is concerned with address-
ing this challenge. One approach has been to develop comprehensive
standard classication systems such as SNOMED-CT ‘a standardised,
multilingual vocabulary of terms relating to the care of the individual’
which has been designed for electronic health-care records. An alterna-
tive has been to use statistical methods such as natural language pro-
cessing to automatically derive information from free text (such as culling
diagnoses from radiological reports), or to employ ‘machine learning’, in
which software algorithms are applied to data in order to derive useful
insights. Such approaches are suited to large, messy data where the
costs of systematisation would be prohibitive. It is likely that such inno-
vations will over the coming years provide useful information to com-
plement that obtained from more traditional health information systems.
Cause of death Approximate
interval between
onset and death
I
Disease or condition directly
leading to death*
INTERNATIONAL FORM OF MEDICAL CERTIFICATE OF CAUSE OF DEATH
due to (or as a consequence of)
due to (or as a consequence of)
due to (or as a consequence of)
(a)
(b)
(c)
(d)
I21.9
E78.0
J47
*This does not mean the mode of dying, e.g. heart failure, respiratory failure.
It means the disease, injury, or complication that caused death.
Antecedent causes
Morbid conditions, if any,
giving rise to the above cause,
stating the underlying
condition last
II
Other significant conditions
contributing to the death, but
not related to the disease or
condition causing it
Fig. 5.5 Completed death certicate. International Classication of Diseases 10 (ICD-10) codes are appended in red. Based on World Health Organization, ICD-10, vol. 2.
Geneva: WHO; 1990. Form retrieved from https://commons.m.wikimedia.org/wiki/File:International_form_of_medical_certicate_of_cause_of_death.png.
121.
Further information 95
5
Management of epidemics
An epidemic, as dened by the World Health Organization, occurs when
‘in a community or region [the] cases of an illness, specic health-related
behaviour, or other health-related events [are] clearly in excess of normal
expectancy’. Epidemics that are small-scale or conned to a small geo-
graphic area are informally referred to as ‘outbreaks’.
Epidemics are regularly caused where preventative measures break
down; examples include breaches of food safety procedures in restau-
rants resulting in outbreaks of enteropathogenic E. coli infection, failure
to adequately maintain cooling towers causing Legionella infections and
falls in vaccination coverage resulting in measles epidemics.
Epidemics can also arise from novel infectious agents such as the
SARS-CoV-1, MERS, SARS-CoV-2 and H1N1 viruses, for which new
preventative measures require to be developed. Where epidemics from
novel agents are not controlled locally, but instead spread beyond inter-
national borders, these are termed pandemics, of which SARS-CoV-2
and H1N1 are examples.
To detect epidemics early, public health agencies undertake surveil-
lance. Surveillance involves the collection and review of cases that have
been identied via statutory notications, or health information systems
(such as microbiology reporting systems). For some infections such as
E. coli and Legionella, one challenge for public health agencies is to
distinguish epidemics from ‘sporadic’ cases at an early stage. This is
done using human judgement, for example, if a large number of cases of
E. coli occur in individuals who ate a specic food, although this task is
sometimes supported using computer algorithms.
Epidemics require an incident management team, some of whose
members have legal powers to impose measures to control infection.
The team establish a formal case denition for conrmed, probable and
possible cases (based on epidemiological features as well as clinical and/
or microbiological ndings), interview individuals with whom cases have
had signicant contact (termed contact tracing), test potential sources
of infections (e.g. foodstuffs or industrial cooling towers), and implement
control measures. The latter might include requiring infected individuals
to isolate for a time period (e.g. until they are asymptomatic or until they
test negative depending on the infectious agent), and/or ordering tem-
porary closure of businesses thought to be sources of infection (e.g. a
restaurant with poor food hygiene practices).
Epidemic curves are crucial to the monitoring and management of
epidemics. Figure 5.6 shows a notional epidemic curve for an epidemic
caused by a single source, which is eliminated after 31 days. However,
they are also used in more complex epidemics where there is per-
son-to-person spread. In the latter, epidemic curves are used to esti-
mate key statistics such as the basic reproduction number (R0) in the
population, as well as to make projections about future infections. Where
the source of an epidemic is not apparent, comparative epidemiological
methods can be used to identify the likely source, for example, a case–
control design may identify a higher than expected consumption of a
given food – for example uncooked legumes – among cases compared
to controls prompting testing of foods, an education campaign and/or
product recalls.
The principles for managing pandemics are similar to those for man-
aging epidemics but operate on a national or international scale. The
universality of pandemics, however, means that they pose special prob-
lems. First, they pose risks to the society-wide infrastructure needed
to deal with diseases, including, but not limited to health-care facilities.
Secondly, pandemics, unlike local and regional epidemics are rare, so
agencies have little direct experience with their management. For this
reason, most settings develop pandemic plans and undertake regu-
lar table-top simulations to test and improve their preparedness. The
COVID-19 pandemic, caused by SARS-CoV-2, is a key example and is
discussed in detail in Chapter 13
Further information
Kindig D, Stoddart G. What is population health? Am J Publ Health 2003;93:
380–383.
UK Faculty of Public Health. What is public health? http://www.fph.org.uk/what_
is_public_health
Burden of disease
GBD Diseases and Injuries Collaborators. Global burden of 369 diseases and
injuries in 204 countries and territories, 1990–2019: a systematic analysis for
the Global Burden of Disease Study 2019. Lancet 2019; 2020(396):
1204–1222.
GBD Risk Factors Collaborators. Global burden of 87 risk factors in 204
countries and territories, 1990–2019: a systematic analysis for the Global
Burden of Disease Study 2019. Lancet 2019; 2020(396):1223–1249.
GBD Viewpoint Collaborators. Five insights from the Global Burden of Disease
Study 2019. Lancet 2019; 2020(396):1135–1159.
Screening
Gov.UK. Population screening programmes. https://www.gov.uk/topic/population-
screening-programmes Detailed information on 11 NHS population screening
programmes.
Immunisation
Gov.UK. Immunisation against infectious disease. https://www.gov.uk/
government/collections/immunisation-against-infectious-disease-the-green-
book The Green Book has the latest information on vaccines and vaccination
procedures, for vaccine-preventable infectious diseases in the UK.
Epidemiology
Burgess S, Davey Smith G, Davies NM, et al. Guidelines for performing Mendelian
randomization investigations. Wellcome Open Res 2020;4:186.
STROBE-MR Steering Group: STROBE-MR: Guidelines for strengthening the
reporting of Mendelian randomization studies. PeerJ https://peerj.com/
preprints/27857/
Days
New
cases
1
0
5
10
15
6 11 16 21 26 31 26 41
Fig. 5.6 Epidemic curve for a notional outbreak within a single ongoing
source. Red line indicates time point where source of infection was removed (e.g.
closure of a restaurant).
Multiple Choice Questions
5.1.Which of the following are examples of large-scale screening
programmes across all of the UK in 2021?
A. Cervical cancer national screening programme in women
B. Prostate cancer national screening programme in men
C. Diabetic eye screening programme in both men and women
D. Newborn hearing screening programme
E. Abdominal aortic aneurysm screening programme in both men
and women
Answer: A, C and D.
Prostate cancer screening has not proven to be cost-effective and
thus is not approved by the UK National Screening Committee. An
abdominal aortic aneurysm screening programme operates only in men
65 years of age or over but not in women.
5.2. Which of the following statements describe the current newborn
blood spot screening programme in the UK?
A. Ideally, blood spots should be collected on day 14
B. Babies who missed the screen can be tested up to 1 month
of age
C. Blood spots are screened for sickle-cell disease
D. Blood spots are screened for cystic brosis
E. Blood spots are screened for congenital hypothyroidism
Answer: C, D and E.
Ideally, blood spots should be collected on day 5. Babies who missed
the screen can be tested up to 1 year of age. Newborn blood spot
screens for: sickle-cell disease, cystic brosis, congenital hypothyroid-
ism and inherited metabolic diseases (PKU, MCADD, MSUD, IVA, GA1
and HCU – see Fig. 5.2).
124.
Principles of infectiousdisease
6
JAT Sandoe
DH Dockrell
Agents causing infection 98
Normal microbial ora 100
Host–pathogen interactions 101
Investigation of infection 102
Direct detection of pathogens 103
Culture 104
Indirect detection of pathogens 105
Antimicrobial susceptibility testing 106
Epidemiology of infection 106
Infection prevention and control 108
Health care-associated infection 109
Outbreaks of infection 109
Immunisation 111
Antimicrobial stewardship 112
Treatment of infectious diseases 113
Principles of antimicrobial therapy 113
Antibacterial agents 116
Antimycobacterial agents 121
Antifungal agents 121
Antiviral agents 122
Antiparasitic agents 124
125.
98 PRINCIPLESOF INFECTIOUS DISEASE
‘Infection’ in its strict sense describes the situation where microorganisms
or other infectious agents become established in the host organism’s
cells or tissues, replicate, induce a host response, and result in pathologic
changes in tissues. If a microorganism survives and replicates on an epithe-
lial surface without causing pathological change the host is said to be ‘colo-
nised’ by that organism. If a microorganism survives and lies dormant after
invading host cells or tissues, infection is said to be ‘latent’. When a micro-
organism, or the host response to it, is sufcient to cause illness or harm,
then the process is said to have caused infection, which may manifest by
symptoms or signs. In milder cases there can be asymptomatic infection
that may be identied by detection of a pathogen or the host response
to the pathogen. Most pathogens (agents that can cause infection) are
microorganisms but some are multicellular organisms. The manifestations
of disease may aid pathogen dissemination (e.g. diarrhoea or coughing).
The term ‘infection’ is often used interchangeably with ‘infectious dis-
ease’ but not all infections are ‘infectious’, i.e. transmissible from person
to person, and not all infections result in symptomatic disease. Infectious
diseases caused by pathogens that are transmitted between hosts can
also be called ‘communicable diseases’, whereas infection caused by
organisms that are already colonising the host are described as ‘endog-
enous’. The distinction is blurred in some situations, including health
care-associated infections such as meticillin-resistant Staphylococcus
aureus (MRSA) or Clostridioides (formerly Clostridium) difcile infection
(CDI), in which colonisation precedes infection but the colonising bac-
teria may have been recently transmitted between patients. The chain
of infection (Fig. 6.1) describes elements for communicable disease
transmission.
Despite dramatic advances in hygiene, immunisation and antimicrobial
therapy, infections still cause a massive burden of disease worldwide.
Key challenges remain in tackling infection in resource-poor countries.
Microorganisms are continually mutating and evolving; the emergence
of new infectious agents (e.g. SARS-CoV-2, see Ch. 13) and antimi-
crobial-resistant microorganisms is therefore inevitable. This chapter
describes the biological and epidemiological principles of infectious dis-
eases and the general approach to their prevention, diagnosis and treat-
ment. Specic infectious diseases are described in Chapters 13–15 and
many of the organ-based chapters.
Agents causing infection
The concept of an infectious agent was established by Robert Koch in
the 19th century (Box 6.1). Although fullment of ‘Koch’s postulates’
became the standard for conrming the cause of an infection, many
infectious agents do not full Koch’s postulates (e.g. uncultivable organ-
isms and the causes of endogenous infections), and the postulates are
now of mainly historical interest. The groups of infectious agents that are
now recognised are described in the following sections.
A
B C
Fig. 6.1 The chain of infection can be linear (A) and cyclical (B and C). Campylobacter enteritis) require a portal of exit from
respiratory virus infection) require a mode of exit from the infected person, a mode of transmission (e.g. contaminated respiratory droplets or transmission via the surface of an
susceptible human host and a mode of transmission back to the insect vector. Many infections involve a combination of these modes of transmission (e.g. Lyme disease, which
has both an animal host and an insect vector) or several different modes of transmission.
6.1 Denition of an infectious agent – Koch’s postulates
1. The same organism must be present in every case of the disease.
2. The organism must be isolated from the diseased host and grown in pure
culture.
3. The isolate must cause the disease, when inoculated into a healthy, susceptible
animal.
4. The organism must be re-isolated from the inoculated, diseased animal.
126.
Agents causing infection 99
6
Viruses
Viruses are incapable of independent replication. Instead, they subvert
host cellular processes to ensure synthesis of their nucleic acids and
proteins. Viruses’ genetic material (the genome) consists of single- or
double-stranded DNA or RNA. Retroviruses transcribe their RNA into
DNA in the host cell by reverse transcription. An antigenically unique pro-
tein coat (capsid) encloses the genome, and together these form the
nucleocapsid. In many viruses, the nucleocapsid is packaged within a
lipid envelope. Enveloped viruses are less able to survive in the envi-
ronment and are spread by respiratory, sexual or blood-borne routes,
including arthropod-based transmission. Non-enveloped viruses survive
better in the environment and are predominantly transmitted by faecal–
oral or, less often, respiratory routes. A generic virus life cycle is shown
in Figure 6.2. A virus that infects a bacterium is a bacteriophage (phage).
Prokaryotes: bacteria (including mycobacteria and
actinomycetes)
Prokaryotic cells can synthesise their own proteins and nucleic acids, and
are able to reproduce autonomously, although they lack a nucleus. The
bacterial cell membrane is bounded by a peptidoglycan cell wall, which is
thick (20–80nm) in Gram-positive organisms and thin (5–10nm) in Gram-
negative ones. The Gram-negative cell wall is surrounded by an outer mem-
brane containing lipopolysaccharide. Genetic information is contained within
a chromosome but bacteria may also contain rings of extra-chromosomal
DNA, known as plasmids, which can be transferred between organisms,
without cells having to divide. Bacteria may be embedded in a polysaccha-
ride capsule, and motile bacteria are equipped with agella. Although many
prokaryotes are capable of independent existence, some (e.g. Chlamydia tra-
chomatis, Coxiella burnetii) are obligate intracellular organisms. Bacteria that
can grow in articial culture media are classied and identied using a range
of characteristics (Box 6.2); examples are shown in Figures 6.3 and 6.4
Eukaryotes: fungi, protozoa and helminths
Eukaryotic cells contain membrane-bound organelles, including nuclei,
mitochondria and Golgi apparatus. Pathogenic eukaryotes are unicellular
(e.g. yeasts, protozoa) or complex multicellular organisms (e.g. nema-
todes, trematodes and cestodes).
Host cell
2 Penetration
Receptor-mediated endocytosis
or, in some enveloped viruses,
membrane fusion (shown here)
3
Uncoating
Nucleic acid is liberated from the
phagosome (if endocytosed)
and/or capsid by complex
enzymatic and/or receptor-mediated
processes
Interaction between host receptor
molecule and virus ligand
(determines host-specificity of the virus)
Adsorption
1
Lipid envelope
Capsid
Nucleic acid
Virus
Assembly 5
Assembly of virus components
is mediated by host and/or
viral enzymes
Release 6
Complete virus particles are
released by budding of host cell
membrane (shown here) or
disintegration of host cell
4
Synthesis
Nucleic acid and protein synthesis is mediated by
host and/or viral enzymes. This takes place in nucleus
or cytoplasm, depending on the specific virus
Fig. 6.2 A generic virus life cycle. Life cycle components common to most viruses are host cell attachment and penetration, virus uncoating, nucleic acid and protein
synthesis, virus assembly and release. Virus release is achieved either by budding, as illustrated, or by lysis of the cell membrane. Life cycles vary between viruses.
6.2 How bacteria are identied
Gram stain reaction (see Fig. 6.3)
Gram-positive (thick peptidoglycan layer), Gram-negative (thin peptidoglycan) or
unstainable
Microscopic morphology
Cocci (round cells) or bacilli (elongated cells)
Presence or absence of capsule
Cell association
Association in clusters, chains or pairs
Colonial characteristics
Colony size, shape or colour
Effect on culture media (e.g. β-haemolysis of blood agar in haemolytic
streptococci; see Fig. 6.4)
Atmospheric requirements
Strictly aerobic (requires O2
), strictly anaerobic (requires absence of O2
), facultatively
aerobic (grows with or without O2
) or micro-aerophilic (requires reduced O2
)
Biochemical reactions
Expression of enzymes (oxidase, catalase, coagulase)
Ability to ferment or hydrolyse various biochemical substrates
Motility
Motile or non-motile
Antibiotic susceptibility
Identies organisms with invariable susceptibility (e.g. to optochin in
Streptococcus pneumoniae or metronidazole in obligate anaerobes)
Matrix-assisted laser desorption/ionisation time-of-ight mass spectrometry
(MALDI-TOF-MS)
A rapid technique that identies bacteria and some fungi from their specic
molecular composition
Sequencing bacterial 16S ribosomal RNA gene
A highly specic test for identication of organisms in pure culture and in
samples from normally sterile sites
Whole-genome sequencing
Although not yet in routine use, whole-genome sequencing (WGS) offers the potential
to provide rapid and simultaneous identication, sensitivity testing and typing of
organisms from pure culture and/or directly from clinical samples.As such,WGS is
likely to replace many of the technologies described above over the next few years
127.
100 PRINCIPLESOF INFECTIOUS DISEASE
Gram-positive
bacilli
Colony morphology, growth
characteristics (e.g. growth
in anaerobic atmosphere),
Gram stain appearance,
MALDI-TOF-MS identification
Colony morphology, growth
characteristics, oxidase
reaction, sugar
fermentation/MALDI-TOF-MS
identification
Colony morphology, growth
characteristics, lactose
fermentation, oxidase
reaction, MALDI-TOF-MS
identification
Colony morphology (e.g.
haemolysis), Gram stain
appearance, agglutination
reactions, coagulase test,
catalase
Examples
Actinomycetes
Arcanobacterium haemo-
lyticum
Bacillus spp.
Corynebacterium diphtheriae
Lactobacillus spp.
Listeria monocytogenes
Nocardia spp.
Clostridium spp.
or
Examples
Neisseria meningitidis
Neisseria gonorrhoeae
Moraxella catarrhalis
Examples
Escherichia coli
Klebsiella pneumoniae
Proteus spp.
Enterobacter spp.
Serratia spp.
Salmonella spp.
Shigella spp.
Yersinia spp.
Vibrio spp.
Pseudomonas aeruginosa
Gram-positive cocci–clusters
Examples
Staphylococcus
aureus
Coagulase-negative
staphylococci
Gram-negative
cocci
Gram stain
Gram-negative
bacilli
Gram-positive
cocci
Gram-positive cocci–chains
Examples
Oral streptococci
Streptococcus
pneumoniae (often
pairs)
-haemolytic strepto-
cocci
Enterococci (short chains)
Fig. 6.3 Flow chart for bacterial identication, including Gram lm appearances on light microscopy (×100). (MALDI-TOF-MS = matrix-assisted laser desorption/
ionisation time-of-ight mass spectroscopy).
B
A
Fig. 6.4 β-haemolytic streptococci (A) and α-haemolytic streptococci (B)
spread on each half of a blood agar plate (backlit). This image is half life-size.
× 0.5. ß-haemolysis renders the agar transparent around the colonies (A) and
α-haemolysis imparts a green tinge to the agar (B).
Fungi exist as either moulds (lamentous fungi) or yeasts. Dimorphic
fungi exist in either form, depending on environmental conditions (see
Fig. 13.62). The fungal plasma membrane differs from the human cell
membrane in that it contains the sterol, ergosterol. Fungi have a cell
wall made up of polysaccharides, chitin and mannoproteins. In most
fungi, the main structural component of the cell wall is β-1,3-D-glucan, a
polysaccharide. These differences from mammalian cells are important
because they offer useful therapeutic targets.
Protozoa and helminths are often referred to as parasites. Many par-
asites have complex multistage life cycles, which involve animal and/or
plant hosts in addition to humans.
Prions
Although prions are transmissible and have some of the characteristics
of infectious agents, they are not microorganisms, and are discussed on
page 1181.
Normal microbial ora
The human body is colonised by large numbers of microorganisms (collec-
tively termed the human microbiota). Colonising bacteria, also referred to
as the ‘normal bacterial ora’, are able to survive and replicate on epithelial
128.
Host–pathogen interactions 101
6
surfaces, e.g. skin and mucosal surfaces. The gastrointestinal tract and
the mouth are the two most heavily colonised sites in the body and they
have distinct microbiota, in both composition and function. Knowledge of
non-sterile body sites and their normal bacterial ora is required to inform
microbiological sampling strategies and interpret culture results (Fig. 6.5).
The microbiota is the total burden of microorganisms, their genes and
their environmental interactions, and is now recognised to have a profound
inuence on human health and disease. Maintenance of the normal ora is
benecial to normal immune function. Other functions include: lower gas-
trointestinal tract bacteria synthesise and excrete vitamins (e.g. vitamins K
and B12
); colonisation with normal ora confers ‘colonisation resistance’ to
infection with pathogenic organisms by altering the local environment (e.g.
lowering pH), producing antibacterial agents (e.g. bacteriocins (small anti-
microbial peptides/proteins), fatty acids and metabolic waste products),
and inducing host antibodies that cross-react with pathogenic organisms.
Conversely, some body sites are either sterile or contain very low num-
bers of colonising bacteria. For example, the submucosal tissues, blood
stream, peritoneal and pleural cavities are maintained as sterile by physi-
cal separation from the external environment; and the lower airways and
bladder, sites that were formerly believed to be sterile but are now known
to support limited microbiota, are protected from excessive contamina-
tion by the mucociliary escalator and urethral sphincter respectively, as
well as local immune responses.
Members of the normal ora can cause (endogenous) infection by
‘translocation’ from their normal habitat to other body sites or by exces-
sive growth at the ‘normal’ site (overgrowth). Overgrowth is exemplied
by dental caries and ‘blind loop’ syndrome (p. 821). Translocation results
from spread along a surface or penetration through a colonised surface,
e.g. urinary tract infection caused by perineal/enteric ora, and surgical
site infections, particularly of prosthetic materials, caused by skin ora
such as staphylococci. Normal ora also contribute to disease by
cross-infection; organisms colonising one individual can cause disease
when transferred to another, more susceptible, individual.
The importance of limiting antimicrobial-induced perturbations of
microbiota is increasingly recognised. ‘Probiotics’ are proprietary
microbes or mixtures of microbes administered with the aim of restor-
ing a benecial prole of gastrointestinal normal ora. Faecal microora
transplantation (FMT) has the same aim, by giving gastrointestinal micro-
biota from healthy people (ltered extract of faeces) to a patient. Although
the clinical effectiveness of probiotics remains a subject of debate, FMT
has proven benet in recurrent C. difcile infection.
Host–pathogen interactions
A ‘pathogen’ is a microorganism that can cause infection. The manifes-
tations of infection, including a pathogen’s ability to cause severe disease
in a previously healthy host, are affected by its ‘virulence’. Virulence is
determined by the number and type of disease-causing proteins and
other factors that it can produce (‘virulence factors’).
Primary pathogens cause disease in a proportion of individuals to
whom they are exposed, regardless of the host’s immunological
status.
Opportunistic pathogens cause disease only in individuals whose
host defences are compromised, e.g. by an intravascular catheter,
or when the immune system is compromised, by genetic suscepti-
bility or immunosuppressive therapy.
Pharynx
Haemophilus spp.
Moraxella catarrhalis
Neisseria spp. (including N. meningitidis)
Staph. aureus
Strep. pneumoniae
Strep. pyogenes (group A)
Oral streptococci (α-haemolytic)
Oral cavity
Oral streptococci (α-haemolytic)
Anaerobic Gram-positive bacilli
(including Actinomyces spp.)
Anaerobic Gram-negative bacilli
Prevotella spp.
Fusobacterium spp.
Candida spp.
Small bowel
Distally, progressively increasing
numbers of large bowel bacteria
Candida spp.
Large bowel
Enterobacterales
Escherichia coli
Klebsiella spp.
Enterobacter spp.
Proteus spp.
Enterococci
E. faecalis
E. faecium
Streptococcus anginosus group
Strep. anginosus
Strep. intermedius
Strep. constellatus
Anaerobic Gram-positive bacilli
Clostridium spp.
Anaerobic Gram-negative bacilli
Bacteroides spp.
Prevotella spp.
Candida spp.
Scalp
As for skin
Nares
Staph. aureus
Coagulase-negative staphylococci
Skin
Coagulase-negative staphylococci
Staph. aureus
Corynebacterium spp.
Propionibacterium spp.
Malassezia spp.
Hands
Resident: as for skin
Transient: skin flora (including
meticillin-resistant and other
Staph. aureus), bowel flora
(including Clostridioides difficile,
Candida spp. and Enterobacterales)
Vagina
Lactobacillus spp.
Staph. aureus
Candida spp.
Enterobacterales
Strep. agalactiae (group B)
Perineum
As for skin
As for large bowel
Fig. 6.5 Human non-sterile sites and normal ora in health.
129.
102 PRINCIPLESOF INFECTIOUS DISEASE
Determinants of virulence
For a primary pathogen to cause infection in a healthy host it must
compete with colonising ora to reach target host cells. It can do this
in various ways, including sequestration of nutrients, adapting metab-
olism to exploit metabolites not used by commensal ora, production
of bacteriocins, and using motility to ‘swim’ to the site of infection.
Many microorganisms, including viruses, use ‘adhesins’ to initiate their
attachment to host cells. Some pathogens can invade through tis-
sues. Many bacteria and fungi multiply after initial adhesion to a host
surface to form ‘biolms’. These are complex three-dimensional struc-
tures surrounded by a matrix of host and bacterial products, which
afford protection to the colony and limit the effectiveness of antimicro-
bials. Biolm-related infections on man-made medical devices such as
vascular catheters or grafts can be particularly difcult to treat.
Pathogens may produce toxins, microbial molecules that cause
adverse effects on host cells, either at the site of infection, or remotely
following carriage through the blood stream. Endotoxin is the lipid
component of Gram-negative bacterial outer membrane lipopoly-
saccharide. It is released when bacterial cells are damaged and has
generalised inflammatory effects. Exotoxins are proteins released by
living bacteria, which often have specific effects on target organs
(Box 6.3).
Intracellular pathogens, including viruses, bacteria (e.g. Salmonella
spp., Listeria monocytogenes and Mycobacterium tuberculosis), para-
sites (e.g. Leishmania spp.) and fungi (e.g. Histoplasma capsulatum), are
able to survive in intracellular environments, including after phagocytosis
by macrophages. Pathogenic bacteria express different genes, depend-
ing on environmental stress (pH, iron starvation, O2
starvation etc.) and
anatomical location.
Genetic diversity enhances the pathogenic capacity of bacteria. Some
virulence factor genes are found on plasmids or in phages and are
exchanged between different strains or species. The ability to acquire
genes from the gene pool of all strains of the species increases diver-
sity and the potential for pathogenicity. Viruses exploit their rapid repro-
duction and potential to exchange nucleic acid with other strains of the
virus to enhance diversity. Once a new strain acquires sufcient virulence
genes, including those enhancing infectivity, it may become an epidemic
or pandemic strain, resulting in regional or global transmission, respec-
tively. This phenomenon accounts for inuenza and COVID-19 pandem-
ics (see Box 6.10 and Ch. 13).
The host response
Innate and adaptive immune and inammatory responses, which humans
use to control the normal ora and respond to pathogens, are reviewed
in Chapter 4
Pathogenesis of infectious disease
The severity of an infection is determined by the virulence of the pathogen
and the host response. Whilst an intact host response protects against
infection or reduces its severity, an excessive response can be damaging.
Both the host immune response and pathogen-produced factors can
contribute to tissue injury and systemic manifestations of infection (see
‘Sepsis’, p. 198). The contribution of the immune response to disease
manifestations is exemplied by the immune reconstitution inammatory
syndrome (IRIS), which can be seen in human immunodeciency virus
(HIV) infection, post-transplantation neutropenia or tuberculosis (which
causes suppression of T-cell function): there is a paradoxical worsening
of the clinical condition as the immune dysfunction is corrected, caused
by an exuberant but dysregulated inammatory response.
Clinical manifestations of infection
The clinical manifestations of infection can be localised to the site of
infection or generalised. Examples of local manifestations include the
inammation of cellulitis, facial pain of sinusitis or neck stiffness of men-
ingitis (Ch. 13). Generalised manifestations include sweats, chills (feeling
very cold, even with extra clothes/blankets), rigors, fevers, anorexia, leth-
argy and generalised aches, and many of these result from the immune
response to infection. While the presence of infection may be clinically
clear in some settings, often it is not. Identifying the responsible patho-
gen in patients with infection is usually not possible on clinical grounds,
neither is prediction of pathogen susceptibility and resistance to antimi-
crobial agents. Hence there is a need to carefully investigate suspected
infections to optimise, or avoid unnecessary, antimicrobial therapy.
Rigors are a clinical symptom (or sign if they are witnessed) characterised
by feeling very cold (‘chills’) and uncontrollable shivering, usually followed
by fever and sweating. Rigors occur when the thermoregulatory centre
attempts to correct a core temperature to a higher level by stimulating
skeletal muscle activity and shaking.
Thermoregulation can be altered during infection, causing both hyper-
thermia (fever) and hypothermia. Fever is mediated mainly by ‘pyrogenic
cytokines’ (e.g. interleukins IL-1 and IL-6, and tumour necrosis factor
(TNF)), which are released in response to various stimuli. including acti-
vation of pattern recognition receptors (PRRs) by microbial products (e.g.
lipopolysaccharide) and factors released by injured cells (Ch. 4). This
induces prostaglandin E2
production, which binds to specic receptors
in the preoptic nucleus of the hypothalamus (thermoregulatory centre),
causing the core temperature to rise.
Investigation of infection
The aims of investigating a patient with suspected infection are: to con-
rm the presence of infection; identify the specic pathogen(s); and, where
appropriate, identify its susceptibility to specic antimicrobial agents in order
to optimise therapy. Pathogens may be detected directly (e.g. by culturing a
normally sterile body site) or their presence may be inferred by identifying the
host response to the organism (‘indirect detection’, Box 6.4), e.g. C-reactive
protein or procalcitonin as part of the acute phase response (p. 68), although
these are activated to varying extents by other inammatory stimuli. Careful
sampling increases the likelihood of diagnosis (Box 6.5). Culture results must
be interpreted in the context of the normal ora at the sampled site (see Fig.
6.5). The extent to which a microbiological test result supports or excludes
a particular diagnosis depends on its statistical performance (e.g. sensitivity,
specicity, positive and negative predictive value). Sensitivity and specicity
vary according to the type of test, sampling and processing techniques, and
time between infection and testing. Positive and negative predictive values
depend on the prevalence of the condition in the test population. The com-
plexity of test interpretation is illustrated in Figure 6.8 below, which shows
the ‘windows of opportunity’ afforded by various testing methods. Given
this complexity, coordinated thought and action ensures appropriate test
application and timing; effective communication between clinicians and the
microbiologists facilitates optimal results and interpretation.
6.3 Exotoxin-mediated bacterial diseases
Disease Organism
Antibiotic-associated diarrhoea/
pseudomembranous colitis
Clostridioides difcile
Botulism Clostridium botulinum
Cholera Vibrio cholerae
Diphtheria Corynebacterium diphtheriae
Haemolytic uraemic syndrome Enterohaemorrhagic Escherichia coli
(E. coli O157 and other strains)
Necrotising pneumonia Staphylococcus aureus
Tetanus Clostridium tetani
Toxic shock syndrome Staph. aureus
Streptococcus pyogenes
130.
Investigation of infection 103
6
Detection of whole organisms
Whole organisms are detected by examination of biological uids or tis-
sue using a microscope.
Bright eld microscopy (in which the test sample is interposed
between the light source and the objective lens) uses stains to
enhance visual contrast between the organism and its background.
Examples include Gram staining of bacteria and Ziehl–Neelsen or
auramine staining of acid- and alcohol-fast bacilli (AAFB) in tubercu-
losis (the latter requires an ultraviolet light source). In histopatholog-
ical examination of tissue samples, stains are used to demonstrate
not only the presence of microorganisms but also features of dis-
ease pathology.
Dark eld microscopy (in which light is scattered to make organisms
appear bright on a dark background) is used, for example, to exam-
ine genital chancre uid in suspected syphilis.
Electron microscopy may be used to examine stool and vesicle uid
to detect enteric and herpesviruses, respectively, but its use has
largely been supplanted by nucleic acid detection (see below).
Flow cytometry can be used to analyse liquid samples (e.g. urine) for
the presence of particles based on properties such as size, imped-
ance and light scatter. This technique can detect bacteria but may
misidentify other particles as bacteria too.
Detection of components of organisms
Components of microorganisms detected for diagnostic purposes include
nucleic acids, cell wall molecules, toxins and other antigens. Commonly
used examples include SARS-CoV-2 antigen in respiratory secretions,
Legionella pneumophila serogroup 1 antigen in urine and cryptococcal
polysaccharide antigen in cerebrospinal uid (CSF). Most antigen detec-
tion methods are based on in vitro binding of specic antigen/antibody
and are therefore described with serological tests below. Other methods
may be used, such as tissue culture cytotoxicity assay for C. difcile toxin.
In toxin-mediated disease, detection of toxin may be of greater relevance
than identication of the organism itself (e.g. stool C. difcile toxin).
Nucleic acid amplication tests
In a nucleic acid amplication test (NAAT), specic sequences of micro-
bial DNA or RNA are identied using a nucleic acid primer that is ampli-
ed exponentially by enzymes to generate multiple copies of a target
nucleotide sequence. The most commonly used NAAT is the polymerase
chain reaction (PCR; see Fig. 3.11). Reverse transcription PCR (RT-PCR)
is used to detect RNA from RNA viruses. The use of uorescent labels
in the reaction enables ‘real-time’ detection of amplied DNA; quanti-
cation is based on the principle that the time taken to reach the detec-
tion threshold is proportional to the initial number of copies of the target
nucleic acid sequence. In ‘broad range’ (bacterial) PCR the primers are
targeted to parts of the gene that encode 16S ribosomal RNA (rRNA)
that have shared DNA sequences across most bacteria. Between these
shared DNA sequences, the 16S rRNA gene varies between species;
so, using PCR, nucleotide sequencing of the product and comparison of
the DNA sequence information with large databases, bacterial detection
and species identication can be achieved. In multiplex PCR, multiple
primer pairs are used to enable detection of several different organisms
in a single reaction.
Determination of nucleotide sequences in a target gene(s) can be used
to assign microorganisms to specic strains, which may be relevant to
treatment and/or prognosis (e.g. in hepatitis C infection). Genes that are
relevant to virulence (such as toxin genes) or antimicrobial resistance can
also be detected; for example, the mecA gene can be used to screen
for MRSA.
NAATs are the most sensitive direct detection methods and are also
relatively rapid. They are used widely in virology, where the possibility
of false-positive results from colonising or contaminating organisms is
6.4 Tests used to diagnose infection
Non-specic markers of inammation/infection
e.g. White cell count in blood sample (WCC), plasma C-reactive protein (CRP),
procalcitonin (PCT)*, serum lactate, cell counts in urine or cerebrospinal uid
(CSF), CSF protein and glucose
Direct detection of organisms or organism components
Microscopy
Detection of organism components (e.g. antigen, toxin)
Nucleic acid amplication tests (e.g. polymerase chain reaction)
Culture of organisms
± Antimicrobial susceptibility testing
Tests of the host’s specic immune response
Antibody detection
Interferon-gamma release assays (IGRA)
*Although PCT is used increasingly in clinical practice, further evidence is required to establish
its precise role in distinguishing bacterial infections from other causes of inammation.
6.5 How to provide samples for microbiological sampling
Communicate with the laboratory
Discuss samples that require processing urgently or that may contain hazardous
or unusual pathogens with laboratory staff before collection
Communication is key to optimising microbiological diagnosis. If there is doubt about
any aspect of sampling, it is far better to discuss it with laboratory staff beforehand
than to risk diagnostic delay by inappropriate sampling or sample handling
Take samples based on a clinical diagnosis
Sampling in the absence of clinical evidence of infection is rarely appropriate
(e.g. collecting urine, or sputum for culture)
Use the correct container
Certain tests (e.g. nucleic acid and antigen detection tests) require proprietary
sample collection equipment
Follow sample collection procedures
Failure to follow sample collection instructions precisely can result in false-
positive (e.g. contamination of blood culture samples) or false-negative (e.g.
collection of insufcient blood for culture) results
Label sample and request form correctly
Label sample containers and request forms according to local policies, with
demographic identiers, specimen type and time/date collected
Include clinical details on request forms
Identify samples carrying a high risk of infection (e.g. blood liable to contain a
blood-borne virus) with a hazard label
Use appropriate packaging
Close sample containers tightly and package securely (usually in sealed plastic
bags)
Attach request forms to samples but not in the same compartment (to avoid
contamination, should leakage occur)
Manage storage and transport
Transport samples to the microbiology laboratory quickly
Consider pre-transport storage, conditions (e.g. refrigeration, incubation,
storage at room temperature) vary with sample type
Notify the receiving laboratory prior to arrival of unusual or urgent samples, to
ensure timely processing
Direct detection of pathogens
Some direct detection methods provide rapid results and enable detec-
tion of organisms that cannot be grown easily on articial culture media,
such as Chlamydia spp.; they can also provide information on antimicro-
bial susceptibility, e.g. M. tuberculosis.
131.
104 PRINCIPLESOF INFECTIOUS DISEASE
remote, and are applied to blood, respiratory samples, stool and urine.
In bacteriology, PCR is used to examine CSF, blood, tissue and genital
samples, and multiplex PCR is being developed for use in faeces. PCR
is particularly helpful for microorganisms that cannot be readily cultured,
e.g. Tropheryma whipplei, and is being used increasingly in mycology
and parasitology.
Culture
Microorganisms may be both detected and further characterised by cul-
ture from clinical samples (e.g. tissue, swabs and body uids).
Ex vivo culture (tissue or cell culture) was widely used in the isolation
of viruses but has been largely supplanted by NAATs.
In vitro culture (in articial culture media) of bacteria and fungi is
used to conrm the presence of pathogens, allow identication, test
antimicrobial susceptibility and subtype the organism for epidemio-
logical purposes.
Culture has its limitations: results are not immediate, even for organ-
isms that are easy to grow, and negative cultures rarely exclude infection.
Organisms such as M. tuberculosis are slow-growing, typically taking at
least 2 weeks, even in rapid-culture systems. Certain organisms, such
as Mycobacterium leprae and Tropheryma whipplei, cannot be cultivated
on articial media, and others (e.g. Chlamydia spp. and viruses) grow
only in culture systems, which are slow and labour-intensive.
Blood culture
The terms ‘bacteraemia’ and ‘fungaemia’ describe the presence of bac-
teria and fungi in the blood without implication of clinical signicance,
while the term ‘blood stream infection’ means bacteraemia or fungae-
mia are present together with symptoms or signs infection; this is dis-
cussed on p. 270. Bacteraemia/fungaemia is identied by inoculating a
liquid culture medium with freshly drawn blood, which is then incubated
in a system that monitors it constantly for growth of microorganisms
(e.g. by detecting products of microbial respiration using uorescence;
Fig. 6.6). If growth is detected, organisms are identied and sensitivity
testing is performed. Traditionally, identication has been achieved by
Gram stain appearance and biochemical reactions. However, matrix-
assisted laser desorption/ionisation time-of-ight mass spectroscopy
(MALDI-TOF-MS; see Box 6.2) is being used increasingly to identify
organisms. MALDI-TOF-MS produces a prole of proteins of different
sizes from the target microorganism and uses databases of such
Department of
Microbiology
1 Patient sampling
Contamination minimised by
aseptic technique. Maximise
sensitivity by sampling correct
volume
2 Sample handling
Follow local instructions for safety,
labelling, and numbers of samples and
bottles required
3 Specimen transport
Transport samples to laboratory as quickly
as possible. Follow manufacturer’s
instructions for the blood culture system
used if temporary storage is required
4 Incubation
Incubate at 35–37°C for 5–7 days.
Microbial growth is usually detected
by constant automatic monitoring of
CO2. If no growth, specimen is
negative and discarded
5 Growth detection
Time to positivity (TTP) is usually
12–24 hrs in significant bacteraemia,
but may be shorter in overwhelming
sepsis or longer with fastidious
organisms (e.g. Brucella spp.)
6 Preliminary results*
A Gram film of the blood culture medium is examined and
results are communicated immediately to the clinician
to guide antibiotic therapy
7 Incubation
A small amount of the medium
is incubated on a range of
culture media. Preliminary
susceptibility testing may be
carried out
8 Culture results*
Preliminary
susceptibility results
are communicated to
the clinician
9 Definitive results
Further overnight incubation
is often required for definitive
identification of organisms (by
biochemical testing) and additional
susceptibility testing; identification
by MALDI-TOF-MS (Fig. 6.7) is
more rapid
Overnight incubation required
Urgent communication required
10 Reporting A final summary is released when all testing is complete. For
clinical care, communication of interim results (Gram film,
preliminary identification and susceptibility) is usually more
important than the final report. Effective clinical–laboratory
communication is vital
Fig. 6.6 An overview of the processing of blood cultures. *In laboratories equipped with MALDI-TOF-MS (see Fig. 6.7), rapid denitive organism identication may be
achieved at stage 6 and/or stage 8.
132.
Investigation of infection 105
6
proles to identify the organism (Fig. 6.7). It is rapid and accurate.
Taking multiple blood samples for culture at different times allows differ-
entiation of transient (one positive sample) and persistent (majority are
positive) bacteraemia. This can be clinically important in identifying the
source of infection.
Indirect detection of pathogens
Tests may be used to detect the host’s immune (antibody) response
to a specic microorganism, and can enable the diagnosis of infection
with organisms that are difcult to detect by other methods or are no
longer present in the host. The term ‘serology’ describes tests carried
out on serum and includes both antigen (direct) and antibody (indirect)
detection. Antigen detection tests used on other uids (e.g. CSF and
respiratory secretions) are also described in this section, as they largely
share the same methodology as serological tests.
Antibody detection
Organism-specic antibody detection is applied mainly to blood
(Fig. 6.8). Results are typically expressed as titres: that is, the reciprocal
of the highest dilution of the serum at which antibody is detectable (e.g.
detection at serum dilution of 1:64 gives a titre of 64). ‘Seroconversion’
is dened as either a change from negative to positive detection or a
fourfold rise in titre between acute and convalescent serum samples.
An acute sample is usually taken during the rst week of disease and
the convalescent sample 2–4 weeks later. Earlier diagnosis can be
achieved by detection of immunoglobulin M (IgM) antibodies, which are
produced early in infection. A limitation of these tests is that antibody
production requires a fully functional host immune system, so there may
be false-negative results in immunocompromised patients. Also, other
than in chronic infections and with IgM detection, antibody tests usually
provide a retrospective diagnosis. False-positive results can occur when
there is cross-reactivity of the test reagents with other molecules or when
patients have been given blood products containing other people’s anti-
bodies. Serological testing methods are described below.
Enzyme-linked immunosorbent assay
The principles of the enzyme-linked immunosorbent assay (ELISA, EIA)
are illustrated in Figure 6.9. These assays rely on linking an antibody with
an enzyme that generates a colour change on exposure to a chromoge-
nic substrate. Various congurations allow detection of antigens or spe-
cic subclasses of immunoglobulin (e.g. IgG, IgM, IgA). ELISA may also
be adapted to detect PCR products, using immobilised oligonucleotide
hybridisation probes and various detection systems.
Immunoblot (Western blot)
Microbial proteins are separated according to molecular weight by poly-
acrylamide gel electrophoresis (PAGE) and transferred (blotted) on to a
nitrocellulose membrane, which is incubated with patient serum. Binding
of specic antibody is detected with an enzyme–anti-immunoglobulin
conjugate similar to that used in ELISA, and specicity is conrmed by
its location on the membrane. Immunoblotting is a highly specic test,
which may be used to conrm the results of less specic tests such as
ELISA (e.g. in Lyme disease).
Immunouorescence assays
Indirect immunouorescence assays (IFAs) detect antibodies by incubat-
ing a serum sample with immobilised antigen (e.g. virus-infected cells
on a glass slide); any virus-specic antibody present in the serum binds
to antigen and is then detected by uorescence microscopy using a u-
orescent-labelled anti-human immunoglobulin (‘secondary’ antibody).
This method can also detect organisms in clinical samples (usually tissue
or centrifuged cells) using a specic antibody in place of immobilised
antigen to achieve capture.
Complement xation test (CFT)
In a CFT, patient serum is heat-treated to inactivate complement and
mixed with test antigen. Any specic antibody in the serum will com-
plex with the antigen. Complement is then added to the reaction. If
antigen–antibody complexes are present, the complement will be ‘xed’
(consumed). Sheep erythrocytes, coated with an anti-erythrocyte anti-
body, are added. The degree of erythrocyte lysis reects the remaining
complement and is inversely proportional to the quantity of the specic
antigen–antibody complex present.
Agglutination tests
When antigens are present on the surface of particles (e.g. cells, latex
particles or microorganisms) and cross-linked with antibodies, visible
clumping (or ‘agglutination’) occurs.
Lighter
m/z
Mass spectrum
Detector
Flight tube
Laser
Sample plate
Voltage grid
Intensity
Heavier
Separation region
(electric field-free)
Fig. 6.7 The workings of matrix-assisted laser desorption/ionisation time-
of-ight mass spectrometry (MALDI-TOF MS). Adapted from Sobin K, Hameer D,
Ruparel T. Digital genotyping using molecular afnity and mass spectrometry. Nature
Rev Genet 2003; 4:1001–1008.
Acute sample
Nucleic acid (NA)
detection
Antigen (Ag)
detection
IgM
NA
Ag
IgG
Limit
of detection
Antibody detection: IgM
Antibody detection: IgG (seroconversion)
Antibody detection: IgG
(fourfold rise in titre)
Convalescent sample
Windows
of
diagnostic
opportunity
Serum
levels
Fig. 6.8 Detection of antigen, nucleic acid and antibody in infectious
disease. The acute sample is usually taken during the rst week of illness, and the
convalescent sample 2–4 weeks later. Detection limits and duration of detectability
vary between tests and diseases, although in most diseases immunoglobulin M (IgM)
is detectable within the rst 1–2 weeks.
133.
106 PRINCIPLESOF INFECTIOUS DISEASE
In direct agglutination, patient serum is added to a suspension of
organisms that express the test antigen. The Widal agglutination test
uses a suspension of Salmonella typhi and S. paratyphi ‘A’ and ‘B’,
treated to retain only ‘O’ and ‘H’ antigens. These antigens are kept
to detect corresponding antibodies in serum from a patient sus-
pected of having typhoid fever.
In indirect (passive) agglutination, specic antigen is attached to the
surface of carrier particles, which agglutinate when incubated with
patient samples that contain specic antibodies.
In reverse passive agglutination (an antigen detection test), the car-
rier particle is coated with antibody rather than antigen.
Immunodiffusion
Immunodiffusion involves antibodies and antigen migrating through gels
and forming insoluble complexes where they meet. The complexes are
seen on staining as ‘precipitin bands’. Immunodiffusion is used in the
diagnosis of dimorphic fungi and some forms of aspergillosis.
Lateral ow immunochromatography
Lateral ow (LF) immunochromatography is mainly used to detect anti-
gens, and often in uids other then blood (e.g. respiratory secretions,
urine). The system consists of a porous test strip (e.g. a nitrocellulose mem-
brane), at one end of which there is target-specic antibody, complexed
with coloured microparticles. Further specic antibody is immobilised in
a transverse narrow line some distance along the strip. Test material is
added to the antibody–particle complexes, which then migrate along the
strip by capillary action. If these are complexed with antigen, they will be
immobilised by the specic antibody and visualised as a transverse line
across the strip. If the test is negative, the antibody–particle complexes will
bind to a line of immobilised anti-immunoglobulin antibody placed further
along the strip, which acts as a negative control. Immunochromatographic
tests are rapid and relatively cheap to perform, and are appropriate for
point-of-care testing, e.g. in HIV-1, COVID-19 and malaria.
Antibody-independent specic immunological tests
Interferon-gamma release assays (IGRA) are used to diagnose latent
tuberculosis infection. The principle behind IGRA is illustrated in
Fig. 17.42. IGRA cannot distinguish between latent and active tubercu-
losis infection and is therefore appropriate for use only in regions where
the background incidence of tuberculosis is low.
Antimicrobial susceptibility testing
If growth of microorganisms in culture is inhibited by the addition of an
antimicrobial agent, the organism is considered to be susceptible to
that antimicrobial. Bacteriostatic agents cause reversible inhibition of
replication and bactericidal agents cause cell death; the terms fungistatic/
fungicidal are equivalent for antifungal agents, and virustatic/virucidal for
antiviral agents. The lowest concentration of the antimicrobial agent at
which growth is inhibited is the minimum inhibitory concentration (MIC),
and the lowest concentration that causes cell death is the minimum bac-
tericidal concentration (MBC). If the MIC is less than or equal to a prede-
termined breakpoint threshold, the organism is considered susceptible,
and if the MIC is greater than the breakpoint, it is resistant. Breakpoints
are determined for antimicrobial agents using a combination of pharma-
cokinetic and clinical data. The relationship between in vitro antimicrobial
susceptibility and clinical response is complex, as response also depends
on severity of illness, site of infection, pharmacokinetics, immune status,
comorbidities and antibiotic dosing. Thus, although treating a patient
according to the results of susceptibility testing increases the likelihood of
recovery, it does not guarantee therapeutic success.
Susceptibility testing is often carried out by disc diffusion (Fig. 6.10).
Antibiotic-impregnated lter paper discs are placed on agar plates con-
taining bacteria; antibiotic diffuses into the agar, resulting in a concen-
tration gradient centred on the disc. Bacteria are unable to grow where
the antibiotic concentration exceeds the MIC, which may therefore be
inferred from the size of the zone of inhibition. The MIC is commonly
measured in diagnostic laboratories using ‘diffusion strips’.
Epidemiology of infection
The communicability of many infections means that, once a clini-
cian has diagnosed an infectious disease, potential exposure of other
patients must also be considered. Measures to control spread may be
required at a patient level (e.g. separation from other patients (‘isola-
tion’)), at an organisation or institutional levels (e.g. in a nursing home
(Ch. 5) or hospital) at a national level, or, in the case of a pandemic, at
an international level. The approach will be specic to the microorganism
involved (Chs. 11–13) but the principles are outlined below.
Geographical and temporal patterns of infection
Endemic disease
Endemic disease has a constant presence within a given geographical
area or population. The infectious agent may have a reservoir, vector
or intermediate host that is geographically restricted, or may itself have
restrictive environmental requirements (e.g. temperature range, humid-
ity). The population affected may be geographically isolated or the dis-
ease may be limited to unvaccinated populations. Factors that alter
geographical restriction include:
expansion of an animal reservoir (e.g. Lyme disease from reforestation)
vector escape (e.g. airport malaria)
Antibody capture
ELISA
Patient Ab
Ig subclass-specific Ab
Ab specific to Ag from
the disease-causing
organism
Specific Ag
Chromogenic substrate
Antibody–enzyme
conjugate
Competitive antibody
detection ELISA
Double antibody sandwich
ELISA (for antigen detection)
Antibody detection
ELISA
A B C D
Fig. 6.9 Antibody (Ab) and antigen (Ag) detection by enzyme-linked immunosorbent assay (ELISA).
antibody–enzyme conjugate and chromogenic substrate. In A, the conjugate Ab is specic for human immunoglobulin. In B–D, it is specic for Ag from the disease-causing organism.
134.
Epidemiology of infection 107
6
extension of host range (e.g. schistosomiasis from dam
construction)
importation of foods
human migration (e.g. carbapenemase-producing Klebsiella
pneumoniae)
public health service breakdown (e.g. diphtheria in unvaccinated
areas)
climate change (e.g. dengue virus and Rift Valley fever).
Emerging and re-emerging disease
An emerging infectious disease is one that has newly appeared in a pop-
ulation, or has been known for some time but is increasing in incidence
or geographical range. If the disease was previously known and thought
to have been controlled or eradicated, it is considered to be re-emerging.
Many emerging diseases are caused by organisms that infect animals
and have undergone adaptations that enable them to infect humans.
This is exemplied by HIV-1, which originated in higher primates in Africa,
and SARS-CoV-2, from bats potentially via intermediate hosts. The geo-
graphical pattern of some recent emerging and re-emerging infections is
shown in Figure 6.11
Reservoirs of infection
The US Centers for Disease Control (CDC) dene a reservoir of infec-
tion as any person, other living organism, environment or combination
of these in which the infectious agent lives and replicates and on which
the infectious agent is dependent for its survival. The infectious agent is
transmitted from this reservoir to a susceptible host.
Human reservoirs
Both colonised and infected individuals can act as human reservoirs, and
infected human reservoirs may be asymptomatic. The organism must be
long-lasting in at least a proportion of those affected, to facilitate onward
transmission. Infections in which humans act as reservoirs include tuber-
culosis, MRSA, HIV and COVID-19. For some infections (e.g. measles)
humans are the only known reservoir.
Animal reservoirs
The World Health Organization (WHO) denes a zoonosis as ‘a disease
or infection that is naturally transmissible from vertebrate animals to
Zone of
inhibition
Zone of
inhibition
1 2
A B
C
D
E
F
A B
C
D
E
F
3 4
5
Fig. 6.10 Antimicrobial susceptibility testing by disc diffusion (panels
1–4) and minimum inhibitory concentration (MIC, panel 5). 1 The test
organism is spread over the surface of an agar plate. 2 Antimicrobial-
impregnated discs (A–F) are placed on the surface and the plate is incubated
(e.g. overnight). 3–4 After incubation, zones of growth inhibition may be seen.
The organism is considered susceptible if the diameter of the zone of inhibition
exceeds a predetermined threshold. 5 In a ‘diffusion strip’ test, the strip is
impregnated with antimicrobial at a concentration gradient that decreases
steadily from top to bottom.The system is designed so that the MIC value is the
point at which the ellipse cuts a scale on the strip (arrow). (4) Kindly supplied by
Charlotte Symes.
CPE
Ebola virus
disease
Cholera
Cholera
Cholera
Cryptococcus
gattii
Cryptococcus
gattii
Zika virus Zika virus
Cyclospora
Chikungunya
virus
Chikungunya
virus
XDR-TB
MERS-Co-V
Anthrax
MDR-TB
Fig. 6.11 Geographical locations of some infectious disease outbreaks, with examples of emerging and re-emerging diseases. (CPE = carbapenemase-producing
Enterobacterales; MDR-TB = multidrug-resistant tuberculosis; MERS-Co-V = Middle East respiratory syndrome coronavirus; XDR-TB = extensively drug-resistant tuberculosis)
135.
108 PRINCIPLESOF INFECTIOUS DISEASE
humans’. Infected animals may be asymptomatic. Zoonotic agents may
be transmitted via any of the routes described below. Primary infection
with zoonoses may be transmitted onward between humans, causing
secondary disease (e.g. Q fever, brucellosis, Ebola virus disease).
Environmental reservoirs
Some pathogens are acquired from an environmental source (e.g.
Pseudomonas aeruginosa from hospital water supplies). However, some
of these are maintained in human or animal reservoirs, with the environ-
ment acting only as a conduit for infection.
Transmission of infection
Communicable diseases may be transmitted by one or more of the fol-
lowing routes:
Respiratory route: airborne/droplet spread (see p. 293).
Faecal–oral route: ingestion of material originating from faeces.
Sexually transmitted infections: direct contact between mucous
membranes.
Blood-borne infections: direct inoculation of blood.
Direct contact: very few organisms are capable of causing infec-
tion by direct contact with intact skin. Most infection by this route
requires contact with damaged skin (e.g. surgical wound).
Via a vector or fomite: the vector/fomite bridges the gap between
the infected host or reservoir and the uninfected host. Vectors are
animate, and include mosquitoes in malaria and dengue virus infec-
tion and humans in MRSA. Fomites are inanimate objects such as
door handles, water taps and ultrasound probes, which are particu-
larly associated with health care-associated infection (HCAI).
The basic reproduction number (R0
) is a measure of the propensity
for a communicable disease to spread between people; it is the aver-
age number of people one person with an infection is likely to pass
on infection to. R0
is calculated by mathematical models that take
into account infection rates, the period of infectivity (known or esti-
mated), opportunities for transmission and susceptibility to infection. R0
assumes everyone is susceptible, whereas ‘effective R’ (Re
) takes into
account the development of herd immunity within the population (e.g.
from infection or vaccination) and therefore varies with time. R0
and
Re
are calculated for whole populations, so will hide local variations in
transmission.
The likelihood of infection following transmission of a pathogen
depends on the virulence of the organism and the susceptibility of the
host. The incubation period is the time between exposure and devel-
opment of symptoms, and the period of infectivity is the period after
exposure during which the patient is infectious to others. Knowledge of
incubation periods and periods of infectivity is important in controlling
the spread of disease, although for many diseases these estimates are
imprecise (Boxes 6.6 and 6.7).
Deliberate release
Deliberate release of pathogens with the intention of causing disease
is known as biological warfare or bioterrorism. Deliberate release inci-
dents have included a 750-person outbreak of Salmonella typhimurium
caused by contamination of salads in 1984 (Oregon, USA) and 22 cases
of anthrax (ve fatal) from the mailing of nely powdered (weaponised)
anthrax spores in 2001 (New Jersey, USA). Diseases with high potential
for deliberate release include anthrax, plague, tularaemia, smallpox and
botulism (through toxin release).
Infection prevention and control
Infection prevention and control (IPC) describes the measures applied to
populations with the aim of breaking the chain of infection (see Fig. 6.1).
6.6 Incubation periods of important infections1
Infection Incubation period
Short incubation periods
Anthrax, cutaneous2
Anthrax, inhalational2
Bacillary dysentery4
Cholera2
Dengue haemorrhagic fever5
Diphtheria5
Gonorrhoea6
Inuenza4
Meningococcaemia2
Norovirus
SARS2
Scarlet fever4
9hrs to 2 weeks
2 days3
1–6 days
2hrs to 5 days
3–14 days
1–10 days
2–10 days
1–3 days
2–10 days
1–3 days
2–7 days3
2–4 days
Intermediate incubation periods
Amoebiasis5
Brucellosis6
Chickenpox4
COVID-19 (SARS-CoV-2)
Lassa fever2
Malaria²
Measles4
Mumps4
Poliomyelitis5
Psittacosis6
Rubella4
Typhoid4
Whooping cough4
1–4 weeks
5–30 days
11–20 days
5–6 days2
3–21 days
10–15 days
6–19 days
15–24 days
3–35 days
1–4 weeks
15–20 days
5–31 days
5–21 days
Long incubation periods
Hepatitis A4
Hepatitis B6
Leishmaniasis, cutaneous5
Leishmaniasis, visceral5
Leprosy (Hansen’s disease)2
Rabies6
Trypanosoma brucei gambiense
infection5
Tuberculosis4
3–7 weeks
6 weeks to 6 months
Weeks to months
Months to years
5–20 years
3–12 weeks3
Months to years
1–12 months
1
Incubation periods are approximate and may differ from local or national guidance. 2
World Health
Organization. 3
Longer incubation periods have been reported. 4
Richardson M, et al. Paediatr Infect
Dis J 2001; 20:380–88. 5
Centers for Disease Control, USA. 6
Public Health England.
(SARS = severe acute respiratory syndrome)
6.7 Periods of infectivity in common childhood infectious
diseases
Disease Infectious period
Chickenpox1
From 4 days before until 5 days after appearance of
the rash (transmission before 48hrs prior to the onset
of rash is rare)4
Measles2
From 4 days before onset to 4 days after onset of the rash
Mumps3
From 2–3 days before to 5 days after disease onset5
Rubella3
From 10 days before until 15 days after the onset of
the rash, but most infectious during prodromal illness4
Scarlet fever1
Unknown6
Whooping cough1
Unknown6,7
1
From Richardson M, Elliman D, Maguire H, et al. Pediatr Infect Dis J 2001; 20:380–388. 2
Centers
for Disease Control, USA; cdc.gov/measles/hcp/. 3
Bennett JE, Dolin R, Blaser MJ. Mandell, Douglas
and Bennett’s Principles and practice of infectious diseases, 8th edn. Philadelphia: Elsevier; 2015.
4–6
Exclude from contact with non-immune and immunocompromised people for 5 days from
4
onset of rash 5
onset of parotitis, or 6
start of antibiotic treatment. 7
Exclude for 3 weeks if untreated.
Durations are approximate and vary between information sources, and these recommendations
may differ from local or national guidance.
136.
Infection prevention andcontrol 109
6
Health care-associated infection
The risk of developing infection following admission to a health-care
facility (health care-associated infection, HCAI) in the developed world
is about 10%. Many nosocomial bacterial infections are caused by
organisms that are resistant to numerous antibiotics (multi-resistant
bacteria), including MRSA, extended-spectrum β-lactamases (ESBLs)
and carbapenemase-producing Enterobacterales (CPE), and glycopep-
tide-resistant enterococci (GRE). Other infections of particular concern in
hospitals include C. difcile and norovirus. Some examples are shown in
Figure 6.12
IPC measures are described in Box 6.8. The most important is
maintenance of good hand hygiene (Fig. 6.13). Hand decontamination
(e.g. using alcohol gel or washing) is mandatory before and after every
patient contact. Decontamination with alcohol gel is usually adequate
but hand-washing (with hot water, liquid soap and complete drying) is
required after any procedure that involves more than casual physical
contact, or if hands are visibly soiled. In situations where the prevalence
of C. difcile is high (e.g. a local outbreak), alcohol gel decontamina-
tion between patient contacts is inadequate as it does not kill C. difcile
spores, and hands must be washed.
Some infections necessitate additional measures to prevent cross-in-
fection (Box 6.9) and sometimes these are combined, e.g. both droplet
and contact precautions in the case of SARS-CoV-2. To minimise risk
of infection, invasive procedures must be performed using strict aseptic
technique.
Outbreaks of infection
Descriptive terms for infectious disease outbreaks are dened in
Box 6.10. Conrmation of an infectious disease outbreak usually
requires evidence from ‘typing’ that the causal organisms have identical
Temporary central venous
catheter infection
Staphylococcus aureus (incl. MRSA)
Coagulase-negative staphylococci
Coliforms
Candida
Prosthetic joint infection
Coagulase-negative staphylococci
Staphylococcus aureus
Streptococci
Coliforms
Cutibacterium (formerly Propionibacterium)
acnes
Staphylococcus aureus
-haemolytic streptococci
Coliforms
Anaerobes
Cuffed/tunnelled central venous
catheter infection
Coagulase-negative staphylococci
Staphylococcus aureus (incl. MRSA)
Coliforms
Candida
Pseudomonas spp.
Enterococcus spp.
External ventricular drain and
Coagulase-negative staphylococci
Staphylococcus aureus
Diphtheroids
Pseudomonas aeruginosa
Peritoneal dialysis-related peritoniti
Staphylococcus aureus
Coagulase-negative staphylococci
Coliforms
Pseudomonas spp.
Breast implant infection
Staphylococcus aureus
Coagulase-negative staphylococci
Fig. 6.12 Commonly encountered health care-associated infections (HCAIs) and the factors that predispose to them.
6.8 Measures used in infection prevention and control (IPC)
Organisational measures
Handling, storage and disposal of clinical waste
Containment and safe removal of spilled blood and body uids
Cleanliness of environment and medical equipment
Specialised ventilation (e.g. laminar ow, air ltration, controlled pressure gradients)
Sterilisation and disinfection of instruments and equipment
Food hygiene
Laundry management
Health-care staff interventions
Education
Hand hygiene, including hand-washing (see Fig. 6.13)
Sharps management and disposal
Use of personal protective equipment (PPE, e.g. masks, sterile and non-sterile
gloves, gowns and aprons)
Screening health workers for disease (e.g. tuberculosis, hepatitis B virus, MRSA)
Immunisation and post-exposure prophylaxis
Clinical practice
Antibiotic stewardship
Aseptic technique
Perioperative antimicrobial prophylaxis
Screening patients for colonisation or infection (e.g. MRSA, GRE, CPE)
Response to infections
Surveillance to detect alert organism (see text) outbreaks and antimicrobial resistance
Antibiotic chemoprophylaxis in infectious disease contacts, if indicated (see Box 6.19)
Isolation (see Box 6.9)
Reservoir control
Vector control
Population measures
See Box 6.12 and p. 293
(CPE = carbapenemase-producing Enterobacterales; GRE = glycopeptide-resistant
enterococci; MRSA = meticillin-resistant Staphylococcus aureus)
Infection prevention andcontrol 111
6
6.10 Terminology in outbreaks of infection
Term Denition
Classication of related cases of infectious disease*
Cluster An aggregation of cases of a disease that are closely
grouped in time and place, and may or may not exceed
the expected number
Epidemic The occurrence of more cases of disease than expected
in a given area or among a specic group of people over
a particular period of time
Outbreak Synonymous with epidemic. Alternatively, a localised, as
opposed to generalised, epidemic
Pandemic An epidemic occurring over a very wide area (several
countries or continents) and usually affecting a large
proportion of the population
Classication of affected patients (cases)
Index case The rst case identied in an outbreak
Primary cases Cases acquired from a specic source of infection
Secondary
cases
Cases acquired from primary cases
Types of outbreak
Common source
outbreak
Exposure to a common source of infection (e.g. water-
cooling tower, medical staff member shedding MRSA).
New primary cases will arise until the source is no longer
present
Point source
outbreak
Exposure to a single source of infection at a specic
point in time (e.g. contaminated food at a party). Primary
cases will develop disease synchronously
Person-to-
person spread
Outbreak with both primary and secondary cases. May
complicate point source or common source outbreak
*Adapted from cdc.gov. (MRSA = meticillin-resistant Staphylococcus aureus)
systems are used nationally; many countries publish lists of organisms
and diseases, which, if detected (or suspected), must be reported to
public health authorities. Reasons why a disease might be reportable are
shown in Box 6.11
A pandemic is the most extensive form of outbreak, in which the dis-
ease spreads over a wide area and may affect a large proportion of the
population. The most recent pandemic was the COVID-19 pandemic of
2019 onwards. The measures that may be put in place in an attempt to
control a pandemic are shown in Box 6.12
Immunisation
Immunisation may be passive or active. Passive immunisation is achieved
by administering antibodies targeting a specic pathogen. Antibodies are
obtained from blood, so confer some of the infection risks associated
with blood products. The protection afforded by passive immunisation
is immediate but of short duration (a few weeks or months); it is used to
prevent or attenuate infection before or after exposure (Box 6.13).
Vaccination
Active immunisation is achieved by vaccination with whole organisms,
organism components or nucleic acids (DNA/RNA) (Box 6.14).
Types of vaccine
Whole-cell vaccines consist of live or inactivated (killed) microorganisms.
Component vaccines contain only extracted or synthesised components
of microorganisms (e.g. polysaccharides or proteins). Live vaccines con-
tain organisms with attenuated (reduced) virulence, which cause only
6.11 Reasons for including an infectious disease on a regional/
national list of reportable diseases*
Reason for inclusion Examples
Endemic/local disease with the
potential to spread and/or cause
outbreaks
Inuenza, Salmonella,
tuberculosis
Imported disease with the propensity
to spread and/or cause outbreaks
Typhoid, cholera (depending on
local epidemiology)
Evidence of a possible breakdown
in health protection/public health
functions
Legionella, Cryptosporidium
Evidence of a possible breakdown in
food safety practices
Botulism, verotoxigenic
Escherichia coli
Evidence of a possible failure of a
vaccination programme
Measles, poliomyelitis, pertussis
Disease with the potential to be a novel
or increasing threat to human health
COVID-19, MERS-CoV, multi-
resistant bacteria
Evidence of expansion of the range of
a reservoir/vector
Lyme disease, rabies, West Nile
encephalitis
Evidence of possible deliberate
release
Anthrax, tularaemia, plague,
smallpox, botulism
*Given the different geographical ranges of individual diseases and wide national variations
in public health services, vaccination programmes and availability of resources, reporting
regulations vary between regions, states and countries. Many diseases are reportable for more
than one reason.
(MERS-CoV = Middle East respiratory syndrome)
6.12 Pandemic response*
International travel restrictions
National entry screening (e.g. testing for fever or pathogen) and quarantining
new arrivals from areas with high infection rates
Isolation of disease contacts
Social isolation
Shielding (strict separation of the vulnerable)
Social distancing (maintaining physical separation between people and
restricting public gatherings)
Encouraging home working
Restricting commuting and work-related travel to ‘key workers’ (workers
required to maintain a functioning society)
Home education
Curfews
Closure or reduction in public transport
Requirement for PPE use in public areas (e.g. face coverings, eye protection)
Litigation and imposition of legal sanctions to enforce the above responses
*Pandemic control measures vary widely by jurisdiction and depend on the stage and extent
of the outbreak and political considerations such as the acceptability of restriction of personal
freedoms and the economic consequences of the interventions. See also p. 293.
mild symptoms but induce T-lymphocyte and humoral responses and
are therefore more immunogenic than inactivated whole-cell vaccines.
The use of live vaccines in immunocompromised individuals is not gener-
ally recommended, but they may be used by specialists following a risk/
benet assessment.
Component vaccines consisting only of polysaccharides, such as
the pneumococcal polysaccharide vaccine (PPV), are poor activators
of T lymphocytes and produce a short-lived antibody response without
long-lasting memory. Conjugation of polysaccharide to a protein, as in
the Haemophilus inuenzae type B (Hib) vaccine and the protein con-
jugate pneumococcal vaccine (PCV), activates T lymphocytes, which
results in a sustained response and immunological memory. Toxoids are
bacterial toxins that have been modied to reduce toxicity but maintain
antigenicity. Vaccine response can be improved by co-administration
with mildly pro-inammatory adjuvants, such as aluminium hydroxide.
139.
112 PRINCIPLESOF INFECTIOUS DISEASE
Another approach is to use viral vectors in which an unrelated and mod-
ied virus expresses the antigen of interest. These vectors include pox
viruses (e.g. modied vaccinia virus Ankara or canary pox virus), adenovi-
rus or adeno-associated viruses. The vector may replicate or be non-repli-
cating. These vaccines work on the same basis as DNA vaccines and are
safe and stable. Pre-existing immunity can inuence efcacy but use of
rare human or related animal strains or a strategy of priming with a non-vi-
ral DNA vaccine can help overcome this (‘prime-boost’ strategy). Nucleic
acid vaccines are administered intramuscularly or intradermally while viral
vector vaccines can be given by these routes but also intranasally or orally.
The rst viral vector vaccine to receive clinical approval was for dengue
virus, while to date DNA vaccines have only been used in veterinary set-
tings. However, both strategies are employed in studies investigating a
range of approaches against Ebola virus and pandemic infections. For
COVID-19, vaccines in use or development include viral vectored, RNA,
DNA, live attenuated, inactivated (killed) virus, subunit and virus-like particle
(lacking genetic material) vaccines. The rst types to be used widely were
mRNA and adenovirus vector RNA vaccines (see Box 13.38).
Use of vaccines
Vaccination may be applied to entire populations or to subpopulations
at specic risk through travel, occupation or other activities. In ring vac-
cination, the population immediately surrounding a case or outbreak of
infectious disease is vaccinated to curtail further spread. This strategy
has been used for Ebola in West Africa. Vaccination is aimed mainly at
preventing infectious disease. However, vaccination against human pap-
illomavirus (HPV) was introduced to prevent cervical and other cancers
that complicate HPV infection. Vaccination guidelines for individuals are
shown in Box 6.15
Vaccination becomes successful for a population once the number
of susceptible hosts falls below the level required to sustain continued
transmission of the target organism, i.e. when Re
is less than 1 ( herd
immunity’). Naturally acquired smallpox was declared to have been erad-
icated worldwide in 1980 through mass vaccination. In 1988, the WHO
resolved to eradicate poliomyelitis by vaccination; the number of cases
of wild polio virus infection worldwide has since fallen from approximately
350000 per annum to 176 in 2019. Recommended vaccination sched-
ules, including catch-up schedules for people who join vaccination pro-
grammes late, vary between countries.
Antimicrobial stewardship
Antimicrobial stewardship (AMS) refers to the systems and processes
applied to a population to optimise the use of antimicrobial agents.
The populations referred to here may be a nation, region, hospital, or a
unit within a health-care organisation (e.g. ward or clinic). AMS aims to
improve patient outcomes and reduce antimicrobial resistance (AMR).
IPC and AMS complement each other (Fig. 6.14). Elements of AMS
include treatment guidelines, antimicrobial formularies and ward rounds
by infection specialists.
6.13 Indications for post-exposure prophylaxis with
immunoglobulins
Human normal immunoglobulin (pooled immunoglobulin)
Hepatitis A (unvaccinated contacts*)
Measles (exposed child with heart or lung disease)
Human specic immunoglobulin
Hepatitis B (sexual partners, inoculation injuries, infants born to infected
mothers)
Tetanus (high-risk wounds or incomplete or unknown immunisation status)
Rabies
Chickenpox (immunosuppressed children and adults, pregnant women)
*Active immunisation is preferred if contact is with a patient who is within 1 week of onset of
jaundice.
6.14 Vaccines in current clinical use
Live attenuated vaccines
Measles, mumps, rubella (MMR)
Oral poliomyelitis (OPV, not used in UK)
Rotavirus
Tuberculosis (bacille Calmette–Guérin, BCG)
Typhoid (oral typhoid vaccine)
Varicella zoster virus
Inactivated (killed) whole-cell vaccines
Cholera
Hepatitis A
Inuenza
Poliomyelitis (inactivated polio virus, IPV)
Rabies
Component vaccines
Anthrax (adsorbed extracted antigens)
COVID-19
Diphtheria (adsorbed toxoid)
Hepatitis B (adsorbed recombinant hepatitis B surface antigen, HBsAg)
Haemophilus inuenzae type B (conjugated capsular polysaccharide)
Human papillomavirus (recombinant capsid proteins)
Meningococcal, quadrivalent A, C, Y, W135 (conjugated capsular polysaccharide)
Meningococcal, serogroup C (conjugated capsular polysaccharide)
Pertussis (adsorbed extracted antigens)
Pneumococcal conjugate (PCV; conjugated capsular polysaccharide, 13 serotypes)
Pneumococcal polysaccharide (PPV; puried capsular polysaccharide, 23 serotypes)
Tetanus (adsorbed toxoid)
Typhoid (puried Vi capsular polysaccharide)
Viral vector vaccines
Dengue virus (containing Yellow fever 17D vaccine strain with dengue virus genes)
Ebola virus (vesicular stomatitis virus expressing Ebola virus glycoproteins)
COVID-19
Nucleic acid vaccines
COVID-19 (RNA vaccines)
Ebola virus (DNA in clinical trials)
Recent and emerging vaccine approaches include nucleic acid (DNA or
RNA)-based vaccines, in which the vaccine nucleic acid encodes the micro-
bial component of interest; when the vaccine enters the vaccinee’s cells
they are induced to express viral antigens. They have several advantages
over component vaccines: antigen is expressed for a period of time and
with the modications that the host cell would normally produce; antigen is
presented by both major histocompatibility class I and II and induces broad
B- and T-lymphocyte responses; vaccines are stable and cost-effective;
production can be at large scale. DNA and RNA vaccines can be admin-
istered directly or may be added with other components to aid cell entry,
e.g. liposomses for DNA vaccines or lipid nanoparticles for RNA vaccines.
Long-term persistence and chromosomal integration with the potential for
mutagenesis remain theoretical concerns for DNA vaccines.
6.15 Guidelines for vaccination against infectious disease
The principal contraindication to inactivated vaccines is an anaphylactic reaction
to a previous dose or a vaccine component
Live vaccines should not be given during an acute infection, to pregnant women
or to the immunosuppressed, unless the immunosuppression is mild and the
benets outweigh the risks
If two live vaccines are required, they should be given either simultaneously in
opposite arms or 4 weeks apart
Live vaccines should not be given for 3 months after an injection of human
normal immunoglobulin (HNI)
HNI should not be given for 2 weeks after a live vaccine
Hay fever, asthma, eczema, sickle-cell disease, topical glucocorticoid therapy,
antibiotic therapy, prematurity and chronic heart and lung diseases, including
tuberculosis, are not contraindications to vaccination
140.
Treatment of infectiousdiseases 113
6
Treatment of infectious diseases
Key components of treating infection are:
prompt initiation of antimicrobial therapy in severe infections, e.g.
sepsis, meningitis
optimising antimicrobial therapy while minimising selection for anti-
microbial resistance and the impact on the normal microbial ora
addressing predisposing factors, e.g. glycaemic control in diabetes
mellitus; viral load control in HIV-1 infection
achieving source control, e.g. removal of an infected medical device,
pus or necrotic tissue
managing complications, e.g. sepsis and acute kidney injury.
For communicable disease, treatment must also take into account
contacts of the infected patient, and may include IPC interventions such
as isolation, antimicrobial prophylaxis, vaccination and contact tracing.
Principles of antimicrobial therapy
In some situations (e.g. pneumonia, meningitis, sepsis) it is important to
start appropriate antimicrobial therapy promptly, whereas in others prior
conrmation of the diagnosis and pathogen is preferred. The principles
underlying the choice of antimicrobial agent(s) are discussed below. The
WHO ‘World Antibiotic Awareness Week’ campaign is a yearly event
aimed at highlighting the importance of prudent antimicrobial prescribing
(see ‘Further information’).
Antimicrobial action and spectrum
Antimicrobial agents may kill or inhibit microorganisms by targeting
essential and non-essential cellular processes, respectively. The range,
or spectrum, of microorganisms that is killed or inhibited by a particu-
lar antimicrobial agent needs consideration when selecting therapy.
Mechanisms of action of the major classes of antibacterial agent are
listed in Box 6.16 and appropriate agents for some common infecting
organisms are shown in Box 6.17. In severe infections and/or immu-
nocompromised patients, it is customary to use bactericidal agents in
preference to bacteriostatic agents.
Empiric versus targeted therapy
Empiric antimicrobial therapy is selected to treat a suspected infection
(e.g. meningitis) before the microbiological cause is known. Targeted or
‘directed’ therapy can be prescribed when the pathogen(s) is known.
Empirical antimicrobial regimens need to have activity against the range
of pathogens potentially causing the infection; because broad-spec-
trum agents affect a wide range of bacteria they select for antimicrobial
resistance. ‘Start Smart – Then Focus’ (Fig. 6.15) describes the prin-
ciple of converting from empiric therapy to narrow-spectrum targeted
therapy. Optimum empiric therapy depends on the site of infection,
patient characteristics and local antimicrobial resistance patterns.
National or local guidelines should inform antimicrobial prescribing
decisions.
Combination therapy
It is sometimes appropriate to combine antimicrobial agents:
when there is a need to increase clinical effectiveness (e.g. biolm
infections)
when no single agent’s spectrum covers all potential pathogens (e.g.
polymicrobial infection)
when there is a need to reduce development of antimicrobial resist-
ance in the target pathogen, as the organism would need to develop
resistance to multiple agents simultaneously (e.g. antituberculous
chemotherapy and antiretroviral therapy (ART) for HIV.
Antimicrobial resistance
Microorganisms have evolved in the presence of naturally occur-
ring antibiotics and have therefore developed resistance mechanisms
to all classes of antimicrobial agent (antibiotics and their derivatives)
(Fig. 6.16). Intrinsic resistance is an innate property of a microorganism,
whereas acquired resistance arises by spontaneous mutation or hori-
zontal transfer of genetic material from another organism, usually via a
plasmid. Plasmids can be easily transferred between bacteria (especially
Enterobacterales) and often encode resistance to multiple antibiotics.
Penicillin-binding proteins (PBP) are enzymes involved in bacterial cell
wall synthesis. The mecA gene encodes a PBP, which has a low afn-
ity for penicillins and therefore confers resistance to β-lactam antibiotics
in staphylococci. Extended-spectrum β-lactamases (ESBLs) are bacte-
rial-produced enzymes that break down β-lactam antibiotics, and are
frequently encoded on plasmids in Enterobacterales. Plasmid-encoded
carbapenemases have been detected in strains of Klebsiella pneumo-
niae (e.g. New Delhi metallo-β-lactamase 1, NDM-1). Strains of MRSA
have been described that also have reduced susceptibility to glycopep-
tides through the development of a relatively impermeable cell wall.
Antimicrobial
stewardship
Infection
prevention
and control
Effective antimicrobial
stewardship reduces
health care-associated
infections
Effective infection
prevention and control
reduces the need for
antimicrobials
Fig. 6.14 The relationship between infection prevention and control (IPC) and
antimicrobial stewardship (AMS).
6.16 Target and mechanism of action of common
antibacterial agents
Aminoglycosides, chloramphenicol, macrolides, lincosamides, oxazolidinones
Inhibition of bacterial protein synthesis by binding to subunits of bacterial ribosomes
Tetracyclines
Inhibition of protein synthesis by preventing transfer RNA binding to ribosomes
ß-lactams
Inhibition of cell wall peptidoglycan synthesis by competitive inhibition of
transpeptidases (‘penicillin-binding proteins’)
Cyclic lipopeptide (daptomycin)
Insertion of lipophilic tail into plasma membrane causing depolarisation
and cell death
Fluoroquinolones
Inhibition of DNA replication by binding to DNA topoisomerases (DNA gyrase
and topoisomerase IV), preventing supercoiling and uncoiling of DNA
Glycopeptides
Inhibition of cell wall peptidoglycan synthesis by forming complexes with
D-alanine residues on peptidoglycan precursors
Nitroimidazoles
The reduced form of the drug causes strand breaks in DNA
Rifamycins
Inhibition of RNA synthesis by inhibiting DNA-dependent RNA polymerase
Sulphonamides and trimethoprim
Inhibition of folate synthesis by dihydropteroate synthase (sulphonamides) and
dihydrofolate reductase (trimethoprim) inhibition
141.
114 PRINCIPLESOF INFECTIOUS DISEASE
6.17 Antimicrobial options for common infecting bacteria
Organism Antimicrobial options*
Gram-positive organisms
Enterococcus faecalis Ampicillin, vancomycin/teicoplanin
Enterococcus faecium Vancomycin/teicoplanin, linezolid
Glycopeptide-resistant enterococci Linezolid, tigecycline, daptomycin
MRSA Clindamycin, vancomycin, rifampicin (never used as monotherapy), linezolid, daptomycin,
tetracyclines, tigecycline, co-trimoxazole
Staphylococus aureus Flucloxacillin, clindamycin
Streptococcus pyogenes Penicillin, clindamycin, vancomycin
Streptococcus pneumoniae Penicillin, cephalosporins, levooxacin, vancomycin
Gram-negative organisms
E. coli, ‘coliforms’ (enteric Gram-negative bacilli) Amoxicillin, trimethoprim, cefuroxime, ciprooxacin, co-amoxiclav
Enterobacter spp., Citrobacter spp. Ciprooxacin, meropenem, ertapenem, aminoglycosides
ESBL-producing Enterobacterales Ciprooxacin, meropenem, ertapenem (if sensitive), temocillin, aminoglycosides
Carbapenemase-producing Enterobacterales Ciprooxacin, aminoglycosides, tigecycline, colistin
Haemophilus inuenzae Amoxicillin, co-amoxiclav, macrolides, cefuroxime, cefotaxime, ciprooxacin
Legionella pneumophila Azithromycin, levooxacin, doxycycline
Neisseria gonorrhoeae Ceftriaxone/cexime, spectinomycin
Neisseria meningitidis Penicillin, cefotaxime/ceftriaxone, chloramphenicol
Pseudomonas aeruginosa Ciprooxacin, piperacillin–tazobactam, aztreonam, meropenem, aminoglycosides, ceftazidime/
cefepime
Salmonella typhi Ceftriaxone, azithromycin (uncomplicated typhoid), chloramphenicol (resistance common)
Strict anaerobes
Bacteroides spp. Metronidazole, clindamycin, co-amoxiclav, piperacillin–tazobactam, meropenem
Clostridioides difcile Metronidazole, vancomycin (oral), daxomicin
Clostridium spp. Penicillin, metronidazole, clindamycin
Fusobacterium spp. Penicillin, metronidazole, clindamycin
Other organisms
Chlamydia trachomatis Azithromycin, doxycycline
Treponema pallidum Penicillin, doxycycline
*Antibiotic selection depends on multiple factors, including local susceptibility patterns, which vary enormously between geographical areas. There are many appropriate alternatives to those listed.
(ESBL= extended-spectrum β-lactamase; MRSA = meticillin-resistant Staphylococcus aureus)
1 Empiric therapy
Based on:
Predicted susceptibility
of likely pathogens
Local antimicrobial
policies
2 Targeted therapy
Based on:
Predicted susceptibility
of infecting organism(s)
Local antimicrobial policies
3 Susceptibility-guided therapy
Based on:
Susceptibility testing results
Antimicrobial
susceptibility results
Clinical diagnosis Information available:
Organ system involved
Endogenous or
exogenous infection
Likely pathogens
Infecting organism(s)
Likely antimicrobial
susceptibility
Level of
knowledge
of infecting
organism(s)
Antimicrobial
spectrum of
agent(s)
used
Antimicrobial
susceptibility
of infecting
organism(s)
Laboratory investigations:
microbiological diagnosis
Fig. 6.15 Stages in the selection and renement of antimicrobial therapy: ‘Start Smart – Then Focus’.
142.
Treatment of infectiousdiseases 115
6
Factors promoting antimicrobial resistance include the inappropriate
use of antibiotics (e.g. to treat viral infections), inadequate dosage or
unnecessarily prolonged treatment, and use of antimicrobials as growth
promoters in agriculture. However, any antimicrobial use exerts a selec-
tion pressure that favours the development of resistance. Combination
antimicrobial therapy may reduce the emergence of resistance in the
target pathogen but not in the normal ora that it also affects. Despite
use of combination therapy for M. tuberculosis, multi-resistant tubercu-
losis (MDR-TB) and extremely drug-resistant tuberculosis (XDR-TB) have
been reported worldwide and are increasing in incidence (see p. 524).
The term ‘post-antibiotic era’ was coined to describe a future in which
widespread antimicrobial resistance will render antimicrobials useless.
At present there is a gradual but inexorable progression of resistance
globally, necessitating the use of more expensive antimicrobials or older
antimicrobials with signicant toxicity.
Duration of therapy
Treatment duration reects the severity of infection and accessibility of
the infected site to antimicrobial agents. For most infections, there is
limited evidence available to support a specic duration of treatment
(Box 6.18). Depending on the indication, initial intravenous therapy can
often be switched to oral as soon as the patient is apyrexial and improv-
ing. In the absence of specic guidance, antimicrobial therapy should
be stopped when there is no longer any clinical evidence of infection.
Pharmacokinetics and pharmacodynamics
Pharmacokinetics of antimicrobial agents is the study of how antibiotics
are absorbed, distributed and excreted from the body. Septic patients
often have poor gastrointestinal absorption, so the preferred initial route
of therapy is intravenous. Knowledge of anticipated antimicrobial drug
concentrations at sites of infection is critical. For example, achieving a
‘therapeutic’ blood level of gentamicin is of little practical use in treating
meningitis, as CSF penetration of the drug is poor. Knowledge of routes
of antimicrobial elimination is also critical; for instance, urinary tract infec-
tion is ideally treated with a drug that is excreted unchanged in the urine.
Pharmacodynamics describes the complex relationship between anti-
microbialconcentrationsandmicrobialkillinginthebody.Formanyagents,
antimicrobial effect can be categorised as ‘concentration-dependent’
or ‘time-dependent’. The concentration of antimicrobial achieved after
a single dose is illustrated in Figure 6.17. The maximum concentration
achieved is Cmax
and the measure of overall exposure is the area under
the curve (AUC). The efcacy of antimicrobial agents whose killing is con-
centration-dependent (e.g. aminoglycosides) increases with the amount
by which Cmax
exceeds the minimum inhibitory concentration (Cmax
:MIC
ratio). For this reason, it has become customary to administer amino-
glycosides (e.g. gentamicin) infrequently at high doses (e.g. 7mg/kg)
rather than frequently at low doses. This has the added advantage of
minimising toxicity by reducing the likelihood of drug accumulation.
Conversely, the β-lactam antibiotics and vancomycin exhibit time-de-
pendent killing, and their efcacy depends on Cmax
exceeding the MIC for
a certain time (which is different for each class of agent). This is reected
in the dosing interval of benzylpenicillin, which is usually given every
4 hours in severe infection (e.g. meningococcal meningitis), and may be
administered by continuous infusion. For other antimicrobial agents, the
pharmacodynamic relationships are more complex and often less well
understood. With some agents, bacterial inhibition persists after anti-
microbial exposure (post-antibiotic and post-antibiotic sub-MIC effects).
Therapeutic drug monitoring
Therapeutic drug monitoring is used to conrm that levels of antimicro-
bial agents with a low therapeutic index (e.g. aminoglycosides) are not
Impermeability/reduced permeability
Carbapenem resistance in Pseudomonas spp.
Aminoglycoside resistance in anaerobes
(uptake requires O2-dependent transport
mechanism)
Antimicrobial target Antimicrobial agent
Active efflux of antimicrobial agent
Tetracycline resistance in Gram-positive
and Gram-negative bacteria
Fluconazole resistance in Candida spp.
Target modification
β-lactam resistance in MRSA – altered
penicillin-binding protein
Glycopeptide resistance in enterococci – altered
peptidoglycan amino acid sequence
Rifampicin resistance in M. tuberculosis – RNA
polymerase mutation
Ciprofloxacin resistance in Enterobacterales – DNA
gyrase mutation
Linezolid resistance in staphylococci and
enterococci – 23S rRNA methylation
Enzymatic degradation of agent
β-lactam resistance in many organisms
(penicillinase in Staph. aureus; ESBLs, ampC and
NDM-1 in Enterobacteriales)
Chloramphenicol resistance in staphylococci (CAT)
Fig. 6.16 Examples of mechanisms of antimicrobial resistance. (CAT = chloramphenicol acetyltransferase; ESBLs = extended-spectrum β-lactamases; MRSA =
meticillin-resistant Staph. aureus; NDM-1 = New Delhi metallo-β-lactamase 1).
143.
116 PRINCIPLESOF INFECTIOUS DISEASE
6.18 Duration of antimicrobial therapy for some
common infections*
Infection Duration of therapy
Viral infections
Herpes simplex encephalitis 2–3 weeks
Bacterial infections
Gonorrhoea Single dose
Infective endocarditis
(streptococcal, native valve)
4 weeks ± gentamicin for rst 2
weeks
Infective endocarditis
(prosthetic valve)
6 weeks
Osteomyelitis 6 weeks
Pneumonia (community-acquired,
severe)
7–10 days (no organism identied),
14–21 days (Staphylococcus aureus
or Legionella spp.)
Septic arthritis 2–4 weeks
Urinary tract infection (male) 1–2 weeks depending on severity
Urinary tract infection, upper tract,
uncomplicated (female)
7 days
Urinary tract infection, lower
(female)
3 days
Mycobacterial infections
Tuberculosis (meningeal) 12 months
Tuberculosis (pulmonary) 6 months
Fungal infections
Invasive pulmonary aspergillosis Until clinical/radiological resolution and
reversal of predisposition
Candidaemia (acute disseminated) 2 weeks after last positive blood
culture and resolution of signs and
symptoms
*All recommendations are indicative. Actual duration takes into account predisposing factors,
specic organisms and antimicrobial susceptibility, source control, current guidelines and clinical
response.
Time after dose
Time above MIC
Peak
concentration
(Cmax)
Minimum
inhibitory
concentration
(MIC)
Area under the
curve (AUC)
Concentration
Fig. 6.17 Antimicrobial pharmacodynamics. The curve represents drug
concentrations after a single dose of an antimicrobial agent. Factors that determine
microbial killing are Cmax
:MIC ratio (concentration-dependent killing), time above MIC
(time-dependent killing) and AUC:MIC ratio.
excessive, and that levels of agents with marked pharmacokinetic var-
iability (e.g. vancomycin) are adequate. Specic recommendations for
monitoring depend on individual clinical circumstances; for instance,
different pre- and post-dose levels of gentamicin are recommended,
6.19 Recommendations for antimicrobial prophylaxis
in adults*
Infection risk Recommended antimicrobial
Bacterial
Diphtheria (prevention of secondary
cases)
Erythromycin
Gas gangrene (after high
amputation or major trauma)
Penicillin or metronidazole
Lower gastrointestinal tract surgery Cefuroxime + metronidazole,
gentamicin + metronidazole, or co-
amoxiclav (single dose only)
Meningococcal disease (prevention
of secondary cases)
Rifampicin or ciprooxacin
Rheumatic fever (prevention of
recurrence)
Phenoxymethylpenicillin or sulfadiazine
Tuberculosis (prevention of
secondary cases)
Isoniazid ± rifampicin
Whooping cough (prevention of
secondary cases)
Erythromycin
Viral
HIV, occupational exposure
(sharps injury)
Combination tenofovir/emtricitabine
and raltegravir. Modied if index case’s
virus known to be resistant
Inuenza A (prevention of secondary
cases in adults with chronic
respiratory, cardiovascular or renal
disease, immunosuppression or
diabetes mellitus)
Oseltamivir
Fungal
Aspergillosis (in high-risk
haematology patients)
Posaconazole (voriconazole or
itraconazole alternatives if intolerant)
Pneumocystis pneumonia
(prevention in HIV and other
immunosuppressed states)
Co-trimoxazole, pentamidine or
dapsone
Protozoal
Malaria (prevention of travel-
associated disease)
Specic antimalarials depend on travel
itinerary. Specialist guidance should be
consulted
*These are based on current UK practice. Recommendations may vary locally or nationally.
Antimicrobial prophylaxis for infective endocarditis during dental procedures is not currently
recommended in the UK.
depending on whether it is being used in traditional divided doses, once
daily or for synergy in infective endocarditis (p. 462).
Antimicrobial prophylaxis
Antimicrobial prophylaxis is the use of antimicrobial agents to prevent infec-
tion. Primary prophylaxis is used to reduce the risk of infection following
certain medical procedures (e.g. colonic resection or prosthetic hip inser-
tion), following exposure to a specic pathogen (e.g. Bordetella pertussis)
or in specic situations such as post-splenectomy (Box 6.19). Antimicrobial
prophylaxis should be chosen to have minimal adverse effects and based
on robust evidence. In the case of exposure, it may be combined with pas-
sive immunisation (see Box 6.13). Secondary prophylaxis is used in patients
who have been treated successfully for an infection but remain predisposed
to it. It is used in haemato-oncology patients in the context of fungal infec-
tion and in HIV-positive individuals with an opportunistic infection until a
dened level of immune reconstitution is achieved.
Antibacterial agents
For details of antibacterial usage in pregnancy and old age, see
Boxes 6.20 and 6.21.
144.
Treatment of infectiousdiseases 117
6
infectious mononucleosis may develop a rash if given aminopenicillins;
this does not imply lasting allergy. The relationship between allergy to
penicillin and allergy to cephalosporins depends on the specic cephalo-
sporin used. Avoidance of cephalosporins, however, is recommended in
patients who have IgE-mediated penicillin allergy (p. 80). Cross-reactivity
between penicillin and carbapenems is rare (approximately 1% by skin
testing) and carbapenems may be administered if there are no suitable
alternatives and appropriate resuscitation facilities are available.
Gastrointestinal upset and diarrhoea are common, and a mild reversible
hepatitis is recognised with many β-lactams. More severe forms of hep-
atitis can be observed with ucloxacillin and co-amoxiclav. Leucopenia,
thrombocytopenia, coagulation deciencies, interstitial nephritis and
potentiation of aminoglycoside-mediated kidney damage are also rec-
ognised. Seizures and encephalopathy have been reported, particularly
with high doses in the presence of renal insufciency. Thrombophlebitis
occurs in up to 5% of patients receiving parenteral β-lactams.
Drug interactions
Synergism occurs in combination with aminoglycosides in vitro. Ampicillin
decreases the biological effect of oral contraceptives and the whole class
is signicantly affected by concurrent administration of probenecid, pro-
ducing a 2–4-fold increase in the peak serum concentration.
Penicillins
Natural penicillins are primarily effective against Gram-positive organ-
isms (except staphylococci, most of which produce a penicillinase) and
anaerobic organisms. Streptococcus pyogenes has remained sensitive
to natural penicillins worldwide. According to the European Antimicrobial
Resistance Surveillance Network (EARS-Net), the prevalence of non-sus-
ceptibility to penicillin in Streptococcus pneumoniae in Europe in 2018
varied widely from 0.1% (Belgium) to 40% (Romania).
Penicillinase-resistant penicillins are the mainstay of treatment for
infections with Staph. aureus, other than MRSA. However, EARS-Net
data from 2018 indicate that MRSA rates in Europe vary widely from 0%
(Iceland) to 43% (Romania).
Aminopenicillins have the same spectrum of activity as the natural pen-
icillins, with additional Gram-negative cover against Enterobacterales.
Amoxicillin has better oral absorption than ampicillin. Unfortunately,
resistance to these agents is widespread (Escherichia coli Europe-wide
in 2018, range 35.3%–67.6%), so they are no longer appropriate for
empiric use in Gram-negative infections. In many organisms, resistance
is due to β-lactamase production, which can be overcome by the addi-
tion of β-lactamase inhibitors (clavulanic acid or sulbactam).
Carboxypenicillins (e.g. ticarcillin) and ureidopenicillins (e.g. piperacil-
lin) are particularly active against Gram-negative organisms, especially
Pseudomonas spp., which are resistant to the aminopenicillins. ß-lactamase
inhibitors may be added to extend their spectrum of activity (e.g. piperacillin–
tazobactam). Temocillin is derived from ticarcillin; it has good activity against
Enterobacterales, including those that produce ESBL enzymes, but poor
activity against P. aeruginosa and Gram-positive bacteria.
6.21 Problems with antimicrobial therapy in old age
Clostridioides difcile infection: all antibiotics predispose to some extent, but
second- and third-generation cephalosporins, co-amoxiclav and clindamycin
especially so.
Hypersensitivity reactions: rise in incidence due to increased previous exposure.
Renal impairment: may be signicant in old age, despite creatinine levels
being within the reference range.
Nephrotoxicity: more likely, e.g. aminoglycosides.
Accumulation of β-lactam antibiotics: may result in myoclonus, seizures or coma.
Reduced gastric acid production: gastric pH is higher, which causes
increased penicillin absorption.
Reduced hepatic metabolism: results in a higher risk of isoniazid-related
hepatotoxicity.
Quinolones: associated with delirium and may increase the risk of seizures.
ß-lactam antibiotics
These antibiotics have a β-lactam ring structure and exert a bactericidal action
by inhibiting PBPs and cell wall synthesis. They are classied in Box 6.22
Pharmacokinetics
Good drug levels are achieved in lung, kidney, bone, muscle and
liver, and in pleural, synovial, pericardial and peritoneal uids.
CSF levels are low, except when meninges are inamed.
‘Inoculum effect’: the activity of ß-lactams is reduced in the pres-
ence of a high organism burden, although the clinical relevance of
this effect is a subject of debate.
Generally safe in pregnancy (except imipenem/cilastatin).
Adverse effects
Immediate (IgE-mediated) allergic reactions are rare but life-threaten-
ing. Approximately 90% of patients who report a penicillin allergy do not
have a true IgE-mediated allergy, highlighting the importance of careful
allergy histories and documentation of reactions. Other reactions, such
as rashes, fever and haematological effects (e.g. low white cell count),
usually follow more prolonged therapy (more than 2 weeks). Patients with
6.22 ß-lactam antibiotics
Penicillins
Natural penicillins: benzylpenicillin, phenoxymethylpenicillin
Penicillinase-resistant penicillins: methicillin*, ucloxacillin, nafcillin, oxacillin
Aminopenicillins: ampicillin, amoxicillin
Carboxy- and ureido-penicillins: ticarcillin, piperacillin, temocillin
Cephalosporins
See Box 6.23
Monobactams
Aztreonam
Carbapenems
Imipenem, meropenem, ertapenem, doripenem
*Not used for treatment.
6.20 Antimicrobial agents in pregnancy1
Contraindicated
Chloramphenicol:neonatal‘grey baby’ syndrome – collapse,hypotension and cyanosis
Fluconazole: teratogenic in high doses
Quinolones: arthropathy in animal studies
Sulphonamides: neonatal haemolysis and methaemoglobinaemia
Tetracyclines, glycylcyclines: skeletal abnormalities in animals in rst trimester;
fetal dental discoloration and maternal hepatotoxicity with large parenteral
doses in second or third trimesters
Trimethoprim: teratogenic in rst trimester
Macrolides2
: major malformations in rst trimester and genital malformations
any trimester
Relatively contraindicated
Aminoglycosides: potential damage to fetal auditory and vestibular nerves in
second and third trimesters
Metronidazole: avoidance of high dosages is recommended3
Not known to be harmful; use only when necessary
Aciclovir
Penicillins and cephalosporins
Clindamycin
Glycopeptides
Linezolid
Meropenem
1
Data extracted from Joint Formulary Committee. British National Formulary (online). London:
BMJ Group and Pharmaceutical Press; (medicinescomplete.com) [accessed 16 March 2013].
2
Fan H, Gilbert R, O’Callaghan F, Li L. Associations between macrolide antibiotics prescribing
during pregnancy and adverse child outcomes in the UK: population based cohort study. BMJ
2020; 368:m331. 3
Theoretical risk of teratogenicity, not supported by available clinical evidence.
145.
118 PRINCIPLESOF INFECTIOUS DISEASE
Cephalosporins and cephamycins
Cephalosporins are broad-spectrum agents. Unfortunately, their use is
associated with C. difcile infection. With the exception of ceftobiprole,
the group has no activity against enterococci. Only the cephamycins
have anti-anaerobic activity. All cephalosporins are inactivated by ESBL.
Cephalosporins are arranged in ‘generations’ (Box 6.23).
First-generation compounds have excellent activity against Gram-
positive organisms and some activity against Gram-negatives.
Second-generation drugs retain Gram-positive activity but have
extended Gram-negative activity. Cephamycins (e.g. cefoxitin),
included in this group, are active against anaerobic Gram-
negative bacilli.
Third-generation agents have improved anti-Gram-negative cover-
age. For some (e.g. ceftazidime), this is extended to include
Pseudomonas spp. Cefotaxime and ceftriaxone have excellent
Gram-negative activity and retain good activity against Strep. pneu-
moniae and β-haemolytic streptococci. Ceftriaxone is administered
once daily and is therefore a suitable agent for outpatient intraven-
ous (parenteral) antimicrobial therapy (OPAT).
Fourth-generation agents, e.g. cefepime, have a broad spectrum of
activity, including streptococci and some Gram-negatives, including
P. aeruginosa
Fifth-generation agents, such as ceftobiprole and ceftaroline, have
an enhanced spectrum of Gram-positive activity that includes
MRSA, and also have activity against Gram-negative bacteria;
some, such as ceftobiprole, are active against P. aeruginosa
Cederocol is a novel siderophore cephalosporin, which is also
active in the presence of ESBLs and carbapenamases.
The spectrum of cephalosporins has also been enhanced by add-
ing β-lactamase inhibitors, e.g. ceftazidime/avibactam and ceftolazone/
tazobactam.
Monobactams
Aztreonam is the only available monobactam. It is active against Gram-
negative bacteria, except ESBL-producing organisms, but inactive
against Gram-positive organisms or anaerobes. It is a parenteral-only
agent and may be used safely in most penicillin-allergic patients other
than those with an allergy to ceftazidime, which shares a common side
chain.
Carbapenems
These intravenous agents have the broadest antibiotic activity of the
β-lactam antibiotics, covering most clinically signicant bacteria, includ-
ing anaerobes (e.g. imipenem, meropenem, ertapenem). Carbapenems
are also being combined with β-lactamase inhibitors in response to
emergence of carbapenemase enzymes that inactivate this class (e.g.
meropenem-vaborbactam).
Macrolide and lincosamide antibiotics
Macrolides (e.g. erythromycin, clarithromycin and azithromycin) and lin-
cosamides (e.g. clindamycin) are bacteriostatic agents. Both classes
bind to the same component of the ribosome, so they are not admin-
istered together. Macrolides are used for Legionella, Mycoplasma,
Chlamydia and Bordetella infections. Azithromycin is employed for sin-
gle-dose/short-course therapy for genitourinary Chlamydia/Mycoplasma
spp. infections. Clindamycin is used primarily for skin, soft tissue, bone
and joint infections.
Pharmacokinetics
Macrolides
Variable bioavailability (intravenous and oral preparations available).
Frequency of administration: erythromycin is administered 4 times
daily, clarithromycin twice daily, azithromycin once daily.
High protein binding.
Excellent intracellular accumulation.
Lincosamides (e.g. clindamycin)
Good oral bioavailability.
Food has no effect on absorption.
Good bone/joint penetration; limited CSF penetration.
Adverse effects
Gastrointestinal upset, especially in young adults (erythromycin 30%).
Cholestatic jaundice with erythromycin estolate.
Prolongation of QT interval can cause torsades de pointes (p. 418).
Clindamycin predisposes to CDI.
Aminoglycosides and spectinomycin
Aminoglycosides are effective mainly in Gram-negative infections and
are therefore commonly used in regimens for intra-abdominal infection.
Some aminoglycosides, e.g. amikacin, are important components of
therapy for MDR-TB. Because they act synergistically with β-lactam anti-
biotics they are used in combinations to treat biolm infections, including
infective endocarditis and orthopaedic implant infections. They cause
very little local irritation at injection sites and negligible allergic responses.
Oto- and nephrotoxicity must be avoided by monitoring of renal function
and drug levels and by use of short treatment regimens. Aminoglycosides
are not subject to an inoculum effect and they all exhibit a post-antibiotic
effect. Resistance is mediated mainly through inactivation by aminogly-
coside-modifying enzymes (AMEs). Plazomicin is a novel aminoglycoside
that is not yet inactivated by AMEs.
Pharmacokinetics
Negligible oral absorption.
Hydrophilic, so excellent penetration to extracellular uid in body
cavities and serosal uids, but poor penetration into adipose tissue.
Very poor intracellular penetration (except hair cells in cochlea and
renal cortical cells).
Negligible CSF and corneal penetration.
Peak plasma levels 30 minutes after infusion.
Exhibit a post-antibiotic effect.
Monitoring of therapeutic levels required.
Gentamicin dosing
Gentamicin should be dosed according to actual body weight or
ideal body weight if obese.
Except in certain forms of endocarditis, pregnancy, severe burns,
end-stage renal disease and paediatric patients, gentamicin can be
administered at 7mg/kg body weight. The appropriate dose interval
6.23 Cephalosporins
First generation
Cefalexin, cefradine (oral) Cefazolin (IV)
Second generation
Cefuroxime (oral/IV)
Cefaclor (oral)
Cefoxitin (IV)
Third generation
Cexime (oral)
Cefotaxime (IV)
Ceftriaxone (IV)
Ceftazidime (IV)
Fourth generation
Cefepime (IV)
Fifth generation (also referred to as ‘next generation’)
Ceftobiprole (IV) Ceftaroline (IV)
(IV = intravenous)
146.
Treatment of infectiousdiseases 119
6
depends on drug clearance and is determined by reference to the
Hartford nomogram (Fig. 6.18).
In most other situations, gentamicin is usually administered twice or
3 times daily at 3–5mg/kg/day with target pre- and post-dose levels
of <1mg/L and 5–10mg/L.
In streptococcal and enterococcal endocarditis, gentamicin is used
with a cell wall active agent (usually a β-lactam), to provide synergy.
Lower doses and target levels are used in this situation.
For all aminoglycoside dosing local guidance should be consulted.
Adverse effects
Renal toxicity (usually reversible) accentuated by other nephrotoxic
agents.
Cochlear toxicity (permanent) more likely in older people and those
with a predisposing mitochondrial gene mutation.
Neuromuscular blockade after rapid intravenous infusion (poten-
tiated by calcium channel blockers, myasthenia gravis and
hypomagnesaemia).
Spectinomycin
Chemically similar to the aminoglycosides and given intramuscularly,
spectinomycin was developed to treat strains of Neisseria gonor-
rhoeae resistant to β-lactam antibiotics. Unfortunately, resistance to
spectinomycin is very common. Its only indication is the treatment of
gonococcal urethritis in pregnancy or in patients allergic to β-lactam
antibiotics.
Quinolones and uoroquinolones
These are effective and generally well-tolerated bactericidal agents.
The quinolones have purely anti-Gram-negative activity, whereas the
uoroquinolones are broad-spectrum agents (Box 6.24). Ciprooxacin
has anti-pseudomonal activity but resistance emerges rapidly. In 2018,
European surveillance showed that resistance to uoroquinolones in E.
coli ranged between 8.4% (Finland) and 42.4% (Cyprus). Quinolones and
uoroquinolones are used for a variety of infections, including pyelone-
phritis, osteomyelitis and less common problems like MDR-TB.
Pharmacokinetics
Well absorbed after oral administration but delayed by food, ant-
acids, ferrous sulphate and multivitamins.
Wide volume of distribution; tissue concentrations twice those
in serum.
Good intracellular penetration, concentrating in phagocytes.
Adverse effects
Gastrointestinal side-effects in 1%–5%.
Rare skin reactions (phototoxicity).
Tendinitis and Achilles tendon rupture, especially in older people.
Central nervous system effects (delirium, tremor, dizziness and
occasional seizures in 5%–12%), especially in older people.
Reduces clearance of xanthines and theophyllines, potentially
inducing insomnia and increased seizure potential.
Prolongation of QT interval on ECG, cardiac arrhythmias.
Ciprooxacin use is associated with acquisition of MRSA and strains
of C. difcile
Glycopeptides
Glycopeptides (vancomycin and teicoplanin) are effective against Gram-
positive organisms only, and are used mainly against staphylococci
(including MRSA) and enterococci. Some staphylococci and enterococci
are resistant, and glycopeptide use should be restricted to limit the emer-
gence of resistance. Glycopeptides are not absorbed after oral adminis-
tration but vancomycin is used orally to treat CDI.
Dalbavancin and telavancin are semisynthetic derivatives of glyco-
peptide (lipoglycopeptides). Dalbavancin is used primarily for skin and
skin structure infections. Telavancin has a dual mechanism of action,
inhibiting both cell wall synthesis and membrane polarisation; it is
reserved for hospital-acquired pneumonia when other agents cannot
be used.
Pharmacokinetics
Vancomycin is administered by slow intravenous infusion. It has
good tissue distribution and a short half-life, and enters the CSF only
in the presence of inammation. Therapeutic drug monitoring is rec-
ommended, to maintain pre-dose levels of >10mg/L (15–20mg/L in
serious staphylococcal infections).
Teicoplanin is administered intravenously or intramuscularly. Its long
half-life allows once-daily dosing
Telavancin and dalbavancin are administered intravenously.
Telavancin has a long half-life that allows once-daily dosing; dalba-
vancin has a very long half-life, allowing once-weekly dosing.
Dose every
48 hours
Dose every
36 hours
Dose every
24 hours
Hours since administration
Concentration
in
plasma
(µg/mL)
2
6 7 8 9 10 11 12 13 14
4
6
8
10
12
14
Fig. 6.18 Dosing of aminoglycosides using the Hartford nomogram. The
nomogram is used to determine the dose interval for 7mg/kg doses of gentamicin
or tobramycin, using measurements of drug levels in plasma 6–14 hours after a
single dose.
6.24 Quinolones and uoroquinolones
Agent
Route of
administration Typical antimicrobial spectrum
Quinolones
Nalidixic acid Oral Enteric Gram-negative bacilli
(not Pseudomonas aeruginosa)
Fluoroquinolones
Ciprooxacin
Noroxacin
Ooxacin
IV/oral
Oral
IV/oral/topical
Enteric Gram-negative bacilli,
P. aeruginosa, Haemophilus
spp., ‘atypical’ respiratory
pathogens*
Levooxacin
(L-isomer of ooxacin)
IV/oral Haemophilus spp.,
Streptococcus
pneumoniae, ‘atypical’
respiratory pathogens*
Moxioxacin Oral Strep pneumoniae.
Staphylococcus aureus,
‘atypical’ respiratory
pathogens,* mycobacteria and
anaerobes
*‘Atypical’ pathogens include Mycoplasma pneumoniae and Legionella spp. Fluoroquinolones
have variable activity against Mycobacterium tuberculosis and other mycobacteria.
}
147.
120 PRINCIPLESOF INFECTIOUS DISEASE
Adverse effects
Rapid infusion of vancomycin can cause histamine release (‘red
man’ syndrome), although this is rare with modern preparations.
Vancomycin and teicoplanin are associated with nephrotoxicity and
ototoxicity.
Vancomycin and teicoplanin can cause DRESS (drug reaction with
eosinophilia and systemic symptoms, see Box 27.35).
Lipopeptides
Daptomycin is a cyclic lipopeptide with bactericidal activity against Gram-
positive organisms only, including MRSA and glycopeptide-resistant
enterococci. It is not absorbed orally, and is used intravenously to treat
Gram-positive infections, such as soft tissue infections and Staph. aureus
infective endocarditis. Daptomycin is inactivated by pulmonary surfactant
and is not effective for pneumonia. Treatment can be associated with
increased levels of creatine kinase and eosinophilic pneumonitis.
Polymyxins
Colistin is a polymyxin antibiotic that binds and disrupts the outer cell mem-
brane of Gram-negative bacteria, including P. aeruginosa and Acinetobacter
baumannii. Its use has increased with the emergence and spread of mul-
ti-resistant Gram-negative bacteria, including CPEs. It can be administered
by oral, intravenous and nebulised routes. Signicant adverse effects include
neurotoxicity, including encephalopathy, and nephrotoxicity.
Folate antagonists
These are bacteriostatic antibacterials. A combination of a sulphona-
mide and either trimethoprim or pyrimethamine is most commonly used,
which interferes with two consecutive steps in the folate metabolic path-
way. Available combinations include trimethoprim/sulfamethoxazole (co-
trimoxazole) and pyrimethamine with either sulfadoxine (used to treat
malaria) or sulfadiazine (used in toxoplasmosis). Co-trimoxazole is the
rst-line drug for Pneumocystis jirovecii infection, the second-line drug
for treatment and prevention of B. pertussis infection, and is also used for
a variety of other infections, including Staph. aureus. Dapsone is used to
treat leprosy (Hansen’s disease) and to prevent toxoplasmosis and pneu-
mocystis when patients are intolerant of other medications. Folinic acid
should be given to prevent myelosuppression if these drugs are used
long-term or unavoidably in early pregnancy.
Pharmacokinetics
Well absorbed orally.
Sulphonamides are hydrophilic, distributing well to the
extracellular uid.
Trimethoprim is lipophilic with high tissue concentrations.
Adverse effects
Trimethoprim is generally well tolerated, with few adverse effects.
Sulphonamides and dapsone may cause haemolysis in
glucose-6-phosphate dehydrogenase deciency.
Sulphonamides and dapsone cause skin and mucocutaneous
reactions, including Stevens–Johnson syndrome and ‘dapsone
syndrome’ (rash, fever and lymphadenopathy).
Dapsone causes methaemoglobinaemia and peripheral neuropathy.
Tetracyclines and glycylcyclines
Tetracyclines
Of this mainly bacteriostatic class, the newer drugs doxycycline and
minocycline show better absorption and distribution than older ones.
Resistance is common in streptococci and Gram-negative bacteria.
Tetracyclines are indicated for infections caused by Mycoplasma spp.,
Chlamydia spp., Rickettsia spp., Coxiella spp., Bartonella spp., Borrelia
spp., Helicobacter pylori, Treponema pallidum and atypical mycobac-
teria. Tetracyclines can also be used for malaria prevention.
Pharmacokinetics
Best oral absorption is in the fasting state (doxycycline is 100%
absorbed unless gastric pH rises) and absorption is inhibited by cati-
ons, e.g. calcium or iron, which should not be administered at the
same time.
Adverse effects
All tetracyclines except doxycycline are contraindicated in renal
failure.
Dizziness with minocycline.
Binding to metallic ions in bones and teeth causes discoloration
(avoid in children and pregnancy) and enamel hypoplasia.
Oesophagitis/oesophageal ulcers with doxycycline.
Phototoxic skin reactions.
Glycylcyclines (tigecycline)
Chemical modication of tetracycline has produced tigecycline, a
broad-spectrum, parenteral-only antibiotic with activity against resist-
ant Gram-positive and Gram-negative pathogens, such as MRSA and
ESBL (but excluding Pseudomonas spp.). Re-analysis of trial data has
shown that there was excess mortality following tigecycline treatment as
opposed to comparator antibiotics, so tigecycline should be used only
when there has been adequate assessment of risk versus benet.
Nitroimidazoles
Nitroimidazoles (e.g. metronidazole) are highly active against strictly
anaerobic bacteria, especially Bacteroides fragilis, C. difcile and
Clostridium spp. Both metronidazole and tinidazole have signicant anti-
protozoal activity against amoebae and Giardia lamblia. Nitroimidazoles
are almost completely absorbed after oral administration (60% after rec-
tal administration) and well distributed, especially to brain and CSF. They
can be used in pregnancy after a risk assessment.
Adverse effects include: metallic taste (dose-dependent), severe vom-
iting if taken with alcohol (‘Antabuse effect’) and peripheral neuropathy
with prolonged use.
Phenicols (chloramphenicol)
Chloramphenicol use is best reserved for severe and life-threatening
infections when other antibiotics are either unavailable or impractical,
due to toxicity. It is bacteriostatic to most organisms and has a very
broad spectrum of activity against aerobic and anaerobic organisms. It
competes with macrolides and lincosamides for ribosomal binding sites,
so should not be used in combination with these agents. Signicant
adverse effects are ‘grey baby’ syndrome in infants (cyanosis and circu-
latory collapse due to inability to conjugate drug and excrete the active
form in urine); reversible dose-dependent bone marrow depression in
adults receiving high cumulative doses; and severe aplastic anaemia in
1 in 25000–40000 exposures (unrelated to dose, duration of therapy or
route of administration).
Oxazolidinones
Linezolid and tedizolid are examples and have good activity against
Gram-positive organisms. They are primarily used in infection caused
by resistant Gram-positive bacteria, including MRSA and GRE, but also
for resistant Mycobacterium tuberculosis. Administration can be intra-
venous or oral. Common linezolid adverse effects include mild gastroin-
testinal upset and tongue discoloration. Myelodysplasia and peripheral
and optic neuropathy can occur with prolonged use. Linezolid has mon-
oamine oxidase inhibitor (MAOI) activity, and co-administration with other
MAOIs or serotonin re-uptake inhibitors should be avoided, as this may
precipitate a ‘serotonin syndrome’.
148.
Treatment of infectiousdiseases 121
6
Other antibacterial agents
Fusidic acid
This antibiotic, active against Gram-positive bacteria, is available in intra-
venous, oral or topical formulations. It is lipid-soluble and distributes well
to tissues. Its antibacterial activity is, however, unpredictable. Fusidic
acid is used in combination, typically with antistaphylococcal penicillins,
or for MRSA with clindamycin or rifampicin. It interacts with coumarin
derivatives and oral contraceptives.
Nitrofurantoin
This drug has very rapid renal elimination and is active against aerobic
Gram-negative and Gram-positive bacteria, including enterococci. It is
used only for treatment of lower urinary tract infection, being generally
safe in pregnancy and childhood. With prolonged treatment, however, it
can cause eosinophilic lung inltrates, fever, pulmonary brosis, periph-
eral neuropathy, hepatitis and haemolytic anaemia so its use must be
carefully balanced against risks.
Fidaxomicin
Fidaxomicin is an RNA synthesis inhibitor, and was introduced for the
treatment of CDI in 2012. The registration trial found daxomicin was
non-inferior to oral vancomycin in non-severe CDI and was associated
with a lower recurrence rate. Its effectiveness for severe CDI has not
been assessed in trials.
Fosfomycin
Fosfomycin acts by inhibiting cell wall synthesis. It has activity against
Gram-negative and Gram-positive bacteria and can demonstrate in vitro
synergy against MRSA when combined with other antimicrobials. It is
used for treatment of urinary tract infections but can be employed in
other situations against multi-resistant bacteria.
Lefamulin
Lefamulin is a pleuromutilin antibiotic that inhibits the 50S ribosome and
inhibits protein synthesis. It has recently been licensed for use in com-
munity-acquired pneumonia and is available orally and intravenously. It
should not be used in pregnancy and prolongs the QT interval.
Antimycobacterial agents
Isoniazid
Isoniazid is bactericidal for replicating bacteria and bacteriostatic for
non-replicating bacteria. It is activated by mycobacterial catalase-perox-
idase (KatG) and inhibits the InhA gene product, a reductase involved in
mycolic acid synthesis. Mutations in KatG or InhA result in isoniazid resist-
ance, which was reported in 15% of cases of M. tuberculosis infection glob-
ally in 2013. Isoniazid is well absorbed orally and metabolised by acetylation
in the liver. The major side-effects are hepatitis, neuropathy (ameliorated by
co-administration of pyridoxine) and hypersensitivity reactions.
Rifampicin
Rifampicin inhibits DNA-dependent RNA polymerase and is bacteri-
cidal against replicating bacteria. It is also active in necrotic foci, where
mycobacteria have low levels of replication, and is therefore important in
sterilisation and sputum conversion. Resistance most often involves the
β-subunit of RNA polymerase and most often occurs with isoniazid-re-
sistant MDR-TB. Rifampicin is well absorbed orally. It is metabolised
by the liver via the microsomal cytochrome P450 system and is one of
the most potent inducers of multiple P450 isoenzymes, so is subject to
extensive drug–drug interactions. Common side-effects include hepati-
tis, inuenza-like symptoms and hypersensitivity reactions. Orange dis-
coloration of urine and body secretions is expected.
Pyrazinamide
The mechanism of action of pyrazinamide is incompletely dened but
includes inhibition of fatty acid synthase and ribosomal trans-translation,
a quality control system for mRNA and protein synthesis. Pyrazinamide
is often bacteriostatic but can be bactericidal and is active against semi-
dormant bacteria in a low-pH environment. Primary resistance is rare
but MDR-TB strains are frequently pyrazinamide-resistant and intrinsic
resistance is a feature of Mycobacterium bovis strains. Pyrazinamide is
well absorbed orally and metabolised by the liver. Side-effects include
nausea, hepatitis, asymptomatic elevation of uric acid and myalgia.
Ethambutol
Ethambutol is a bacteriostatic agent. It inhibits arabinosyl transferase,
which is involved in the synthesis of arabinogalactan, a component of
the mycobacterial cell wall. Resistance is usually seen when resistance to
other antimycobacterial agents is also present, e.g. in MDR-TB strains.
It is orally absorbed but, in contrast to the rst-line agents described
above, it achieves poor CSF penetration and is renally excreted. The
major side-effect is optic neuritis with loss of red–green colour discrimi-
nation and impaired visual acuity.
Streptomycin
Streptomycin is an aminoglycoside whose mechanism of action and
side-effects are similar to those of other aminoglycosides. It is adminis-
tered intramuscularly.
Other antituberculous agents
Second-line agents used in MDR or XDR strains (p. 524) include amino-
glycosides (amikacin, capreomycin or kanamycin) and uoroquinolones
(moxioxacin or levooxacin), discussed above. Other established sec-
ond-line agents administered orally are cycloserine (which causes neuro-
logical side-effects); ethionamide or prothionamide (which are not active
with InhA-gene-mediated resistance but have reasonable CSF penetration;
their side-effect prole includes gastrointestinal disturbance, hepatotoxicity
and neurotoxicity); and paraminosalicylic acid (which causes rashes and
gastrointestinal upset). Linezolid may also be used and has good CSF pen-
etration, while meropenem with co-amoxiclav is occasionally chosen. New
drugs developed for XDR-TB include delamanid and bedaquiline (which
targets the ATP synthase); their adverse effects include QT prolongation
and cardiac arrhythmias. Their co-administration with other agents with a
similar side-effect prole (e.g. uoroquinolones) therefore requires careful
risk assessment. Pretomanid is used in combinations with bedaquiline for
drug-resistant strains and improves the bactericidal activity of combination
therapy which may enable shorter durations. Its side-effects include nerve
damage, headaches, gastrointestinal upset, abnormal liver function tests,
low glucose and skin rash.
Clofazimine
Clofazimine is used against M. leprae and resistant strains of M. tubercu-
losis. Its mode of action may involve induction of oxidative stress and it
is weakly bactericidal. Oral absorption is variable and it is excreted in the
bile. Side-effects include gastrointestinal upset, dry eyes and skin, and
skin pigmentation, especially in those with pigmented skin.
Antifungal agents
See Box 6.25.
Azole antifungals
The azoles (imidazoles and triazoles) inhibit synthesis of ergosterol, a
constituent of the fungal cell membrane. Side-effects vary but include
gastrointestinal upset, hepatitis and rash. Azoles are inhibitors of
cytochrome P450 enzymes, so tend to increase exposure to cytochrome
P450-metabolised drugs.
Imidazoles
Miconazole, econazole, clotrimazole and ketoconazole are relatively
toxic and therefore administered topically. Clotrimazole is used exten-
sively to treat supercial fungal infections. Triazoles are used for systemic
treatment because they are less toxic.
149.
122 PRINCIPLESOF INFECTIOUS DISEASE
Polyenes
Amphotericin B (AmB) deoxycholate causes cell death by binding to
ergosterol and damaging the fungal cytoplasmic membrane. Its use in
resource-rich countries has been largely supplanted by less toxic agents.
Its long half-life enables once-daily administration. CSF penetration is poor.
Adverse effects include immediate anaphylaxis, other infusion-related
reactions and nephrotoxicity. Nephrotoxicity may be sufcient to require
dialysis and occurs in most patients who are adequately dosed. It may
be ameliorated by concomitant infusion of normal saline. Irreversible
nephrotoxicity occurs with large cumulative doses of AmB.
Nystatin has a similar spectrum of antifungal activity to AmB. Its toxicity
limits it to topical use, e.g. in oral and vaginal candidiasis.
Lipid formulations of amphotericin B
Lipid formulations of AmB have been developed to reduce AmB toxicity
and have replaced AmB deoxycholate in many regions. They consist of
AmB encapsulated in liposomes (liposomal AmB, L-AmB) or complexed
with phospholipids (AmB lipid complex, ABLC). The drug becomes active
on dissociating from its lipid component. Adverse effects are similar to,
but considerably less frequent than, those with AmB deoxycholate, and
efcacy is similar. Lipid formulations of AmB are used in invasive fungal
disease, as empirical therapy in patients with neutropenic fever and also
in visceral leishmaniasis.
Other antifungal agents
Flucytosine
Flucytosine (5-uorocytosine) has particular activity against yeasts. When
it is used as monotherapy, acquired resistance develops rapidly, so it
should be given in combination with another antifungal agent. Adverse
effects include myelosuppression, gastrointestinal upset and hepatitis.
Griseofulvin
Griseofulvin has been largely superseded by terbinane and itraconazole
for treatment of dermatophyte infections, except in children, for whom
these agents remain largely unlicensed. It is deposited in keratin precur-
sor cells, which become resistant to fungal invasion.
Terbinane
Terbinane distributes with high concentration to sebum and skin, with
a half-life of more than 1 week. It is used topically for dermatophyte skin
infections and orally for onychomycosis. The major adverse reaction is
hepatic toxicity (approximately 1:50000 cases). Terbinane is not recom-
mended for breastfeeding mothers.
Antiviral agents
Most viral infections in immunocompetent individuals resolve without
intervention. Antiviral therapy is available for a limited number of infec-
tions only (Box 6.26).
Antiretroviral agents
These agents, used predominantly against HIV, are discussed on page
366.
Anti-herpesvirus agents
Aciclovir, valaciclovir, penciclovir and famciclovir
These antivirals are acyclic analogues of guanosine, which inhibit viral
DNA polymerase after being phosphorylated by virus-derived thymi-
dine kinase (TK). Aciclovir is poorly absorbed after oral dosing; better
levels are achieved intravenously or by use of the prodrug valaciclovir.
Famciclovir is the prodrug of penciclovir. Resistance is mediated by viral
TK or polymerase mutations.
6.25 Antifungal agents
Agent Usual
route(s) of
administration
Clinically relevant antifungal
spectrum
Imidazoles
Miconazole
Econazole
Clotrimazole
Topical
Candida spp., dermatophytes
Ketoconazole Topical, oral Malassezia spp., dermatophytes,
agents of eumycetoma
Triazoles
Fluconazole Oral, IV Yeasts (Candida and
Cryptococcus spp.)
Itraconazole Oral, IV Yeasts, dermatophytes, dimorphic
fungi (p. 342), Aspergillus spp.
Voriconazole Oral, IV Yeasts and most lamentous
fungi (excluding mucoraceous
moulds)
Posaconazole Oral, IV Yeasts and many lamentous
fungi (including most
mucoraceous moulds)
Isavuconazole Oral, IV Yeasts and many lamentous
fungi (variable activity against
mucoraceous moulds)
Echinocandins
Anidulafungin
Caspofungin
Micafungin
IV only
Candida spp., Aspergillus spp.
(no activity against Cryptococcus
spp. or mucoraceous moulds)
Polyenes
Amphotericin B IV Yeasts and most dimorphic and
lamentous fungi (including
mucoraceous moulds)
Nystatin Topical
Others
5-uorocytosine Oral, IV Yeasts
Griseofulvin Oral Dermatophytes
Terbinane Topical, oral Dermatophytes
(IV = intravenous)
}
}
Triazoles
Fluconazole is effective against yeasts (Candida and Cryptococcus spp.)
and has a long half-life (approximately 30 hours) and an excellent safety
prole. The drug is highly water-soluble and distributes widely to all body
sites and tissues, including CSF. Itraconazole is lipophilic and distributes
extensively, including to toenails and ngernails. Its CSF penetration is
poor. Because oral absorption of itraconazole is erratic, therapeutic drug
monitoring is required. Voriconazole is well absorbed orally but varia-
bility in levels requires therapeutic drug monitoring. It is used mainly in
aspergillosis. Side-effects include photosensitivity, hepatitis and transient
retinal toxicity. Posaconazole and isavuconazole are broad-spectrum
azoles, with activity against Candida spp., Aspergillus spp. and some
mucoraceous moulds. Isavuconazole is non-inferior to voriconazole in
the management of invasive aspergillosis and may be considered as an
alternative when voriconazole cannot be used.
Echinocandins
The echinocandins inhibit β-1,3-glucan synthesis in the fungal cell wall.
They have few signicant adverse effects. Caspofungin, anidulafungin
and micafungin are used to treat systemic candidosis, and caspofungin
is also used in aspergillosis.
150.
Treatment of infectiousdiseases 123
6
6.26 Antiviral agents
Drug
Route(s) of
administration Indications Signicant side-effects
Antiretroviral therapy
(ART, p. 366) Oral HIV infection (including AIDS) CNS symptoms, anaemia, lipodystrophy
Anti-herpesvirus agents
Aciclovir
Topical/oral/IV
Herpes zoster
Chickenpox (esp. in immunosuppressed)
Herpes simplex infections: encephalitis (IV only), genital tract, oral,
ophthalmic
Signicant side-effects rare
Hepatitis, renal impairment and neurotoxicity reported
rarely
Valaciclovir Oral Herpes zoster, herpes simplex As for aciclovir
Famciclovir Oral Herpes zoster, herpes simplex (genital) As for aciclovir
Penciclovir Topical Labial herpes simplex Local irritation
Ganciclovir IV Treatment and prevention of CMV infection in immunosuppressed Gastrointestinal symptoms, liver dysfunction,
neurotoxicity, myelosuppression, renal impairment,
fever, rash, phlebitis at infusion sites
Potential teratogenicity
Valganciclovir Oral Treatment and prevention of CMV infection in immunosuppressed As for ganciclovir but neutropenia is predominant
Cidofovir IV/topical HIV-associated CMV infections and occasionally other viruses (see text) Renal impairment, neutropenia
Foscarnet IV CMV and aciclovir-resistant HSV and VZV infections in
immunosuppressed
Gastrointestinal symptoms, renal impairment,
electrolyte disturbances, genital ulceration, neurotoxicity
Anti-inuenza agents
Zanamivir Inhalation Inuenza A and B Allergic reactions (very rare)
Oseltamivir Oral Inuenza A and B Gastrointestinal side-effects, rash, hepatitis (very rare)
Peramivir IV, IM
Amantadine,
rimantadine
Oral Inuenza A (but see text) CNS symptoms, nausea
Agents used in other virus infections*
Ribavirin Oral/IV/
inhalation
Lassa fever (IV)
RSV infection in infants (inhalation)
Haemolytic anaemia, cough, dyspnoea, bronchospasm
and ocular irritation (when given by inhalation)
Remdesivir IV COVID-19 (experimental, not recommended for clinical use at the
time of writing)
Signicant side-effects rare
*Antiviral agents used in viral hepatitis are discussed on pages 884–890.
(AIDS = acquired immunodeciency syndrome; CMV = cytomegalovirus; CNS = central nervous system; HIV = human immunodeciency virus; HSV = herpes simplex virus; IM = intramuscular;
IV = intravenous; RSV = respiratory syncytial virus; VZV = varicella zoster virus)
}
Ganciclovir
Chemical modication of the aciclovir molecule allows preferential phospho-
rylation by protein kinases of cytomegalovirus (CMV) and other β-herpes-
viruses (e.g. human herpesvirus (HHV) 6/7) and hence greater inhibition of
the DNA polymerase, but at the expense of increased toxicity. Ganciclovir is
administered intravenously or as a prodrug (valganciclovir) orally.
Cidofovir
Cidofovir inhibits viral DNA polymerases with potent activity against CMV,
including most ganciclovir-resistant CMV. It also has activity against aciclo-
vir-resistant herpes simplex virus (HSV) and varicella zoster virus (VZV), HHV6
and occasionally adenovirus, poxvirus, papillomavirus or polyoma virus, and
may be used to treat these infections in immunocompromised hosts.
Foscarnet
This analogue of inorganic pyrophosphate acts as a non-competitive
inhibitor of HSV, VZV, HHV6/7 or CMV DNA polymerase. It does not
require signicant intracellular phosphorylation and so may be effective
when HSV or CMV resistance is due to altered drug phosphorylation. It
has variable CSF penetration.
Letermovir
Letermovir is a CMV DNA terminase inhibitor that plays a role in cleavage
of viral DNA for packaging into mature virions. It is available in oral and
intravenous formulations. It is well tolerated and is approved for proph-
ylaxis of CMV in allogeneic haematopoeitic stem cell transplant recipi-
ents. Gastrointestinal disturbance and drug–drug interactions occur and
resistance may develop.
Anti-inuenza agents
Zanamivir and oseltamivir
These agents inhibit inuenza A and B neuraminidase, which is
required for release of virus from infected cells (see Fig. 6.2). They
are used in the treatment and prophylaxis of inuenza. Administration
within 48 hours of disease onset reduces the duration of symptoms
by approximately 1–1½ days. In the UK, their use is limited mainly
to adults with chronic respiratory or renal disease, signicant car-
diovascular disease, immunosuppression or diabetes mellitus, dur-
ing known outbreaks. Peramivir has been developed as a distinct
chemical structure, which means that it retains activity against some
oseltamivir- and zanamivir-resistant strains. It has poor oral bioavail-
ability and has been developed as an intravenous or intramuscular
formulation for treatment of severe cases of inuenza, e.g. in intensive
care units. It is now approved for use in adults in a number of coun-
tries. An intravenous formulation of zanamivir is also in development
for critically ill patients. Laninamivir is approved as an intranasal for-
mulation in Japan.
151.
124 PRINCIPLESOF INFECTIOUS DISEASE
Other anti-inuenza agents
Amantidine and rimantadine inhibit viral M2 protein ion channel function,
which is required for uncoating (see Fig. 6.2). Resistance is widespread,
and these are only used to treat oseltamivir-resistant inuenza A in
patients unable to take zanamivir (e.g. ventilated patients) and when the
strain is susceptible to these agents. Baloxavir marboxil inhibits a com-
ponent of viral RNA synthesis. It is licensed for uncomplicated inuenza
but is likely to be reserved for neuraminidase-resistant strains. Faviparvir
is an oral or intravenous RNA-dependent RNA polymerase that may
also play a role against resistant strains of inuenza but also other RNA
viruses and has been studied against SARS-CoV-2.
Other agents used to treat viruses
Antiviral agents used to treat hepatitis B and C virus are discussed on pages
886 and 889, and those used against HIV-1 are described on page 366.
Remdesivir
Remdesivir is an intravenous RNA-dependent RNA polymerase inhibitor
that has a broad antiviral spectrum in vitro and has decreased the time to
recovery following SARS-Cov-2 infection. So far it has not demonstrated
efcacy against other viruses clinically. Initial studies have not shown signi-
cant effects on SARS-CoV-2 viral replication, suggesting it may need to be
used early in the course of infection before peak viral replication. The main
side-effects include abnormal liver function tests and infusion reactions.
Ribavirin
Ribavirin is a guanosine analogue that inhibits nucleic acid synthesis in
a variety of viruses. It is used in particular in the treatment of hepatitis C
virus but also against certain viral haemorrhagic fevers, e.g. Lassa fever,
although it has not been useful against Ebola virus.
Antiparasitic agents
Antimalarial agents
Artemisinin (qinghaosu) derivatives
Artemisinin originates from a herb (sweet wormwood, Artemisia annua),
which was used in Chinese medicine to treat fever. Its derivatives,
artemether and artesunate, were developed for use in malaria in the
1970s. Their mechanism of action is unknown. They are used in the
treatment, but not prophylaxis, of malaria, usually in combination with
other antimalarials, and are effective against strains of Plasmodium spp.
that are resistant to other antimalarials. Artemether is lipid-soluble and
may be administered via the intramuscular and oral routes. Artesunate is
water-soluble and is administered intravenously or orally. Serious adverse
effects are uncommon. Current advice for malaria in pregnancy is that
the artemisinin derivatives should be used to treat uncomplicated fal-
ciparum malaria in the second and third trimesters, but should not be
prescribed in the rst trimester until more information becomes available.
Atovaquone
Atovaquone inhibits mitochondrial function. It is an oral agent, used for
treatment and prophylaxis of malaria, in combination with proguanil (see
below), without which it is ineffective. It is also employed in the treatment
of mild cases of Pneumocystis jirovecii pneumonia, or as prophylaxis,
where there is intolerance to co-trimoxazole. Signicant adverse effects
are uncommon.
Folate synthesis inhibitors (proguanil,
pyrimethamine–sulfadoxine)
Proguanil inhibits dihydrofolate reductase and is used for malaria prophy-
laxis. Pyrimethamine–sulfadoxine may be used in the treatment of malaria.
Quinoline-containing compounds
Chloroquine and quinine are believed to act by intraparasitic inhibition
of haem polymerisation, resulting in toxic build-up of intracellular haem.
The mechanisms of action of other agents in this group (quinidine,
amodiaquine, meoquine, primaquine, etc.) may differ. They are employed
in the treatment and prophylaxis of malaria. Primaquine is used for
radical cure of malaria due to Plasmodium vivax and P. ovale (destruction
of liver hypnozoites). Chloroquine may also be given for extraintestinal
amoebiasis.
Chloroquine can cause signicant pruritus. If used in long-term, high-
dose regimens, it causes an irreversible retinopathy. Overdosage leads
to life-threatening cardiotoxicity. The side-effect prole of meoquine
includes neuropsychiatric effects ranging from mood change, nightmares
and agitation to hallucinations and psychosis. Quinine may cause hypo-
glycaemia and cardiotoxicity, especially when administered parenterally.
Primaquine causes haemolysis in people with glucose-6-phosphate
dehydrogenase deciency, which should be excluded before therapy.
Chloroquine is considered safe in pregnancy but meoquine should be
avoided in the rst trimester.
Chloroquine (and its metabolite hydroxychloroquine) exhibits in vitro
activity against the virus SARS-CoV-2 but has not demonstrated clinical
efcacy.
Lumefantrine
Lumefantrine is used in combination with artemether to treat uncom-
plicated falciparum malaria, including chloroquine-resistant strains.
Its mechanism of action is unknown. Signicant adverse effects are
uncommon.
Drugs used in trypanosomiasis
The antiparasitic agents used to treat human African trypanosomiasis
(HAT) and American trypanosomiasis (Chagas’ disease) (benznidazole,
eornithine, fexinidazole, melarsoprol, nifurtimox, pentamidine and
suramin) are discussed in detail on pages 324 and 325.
In addition to its use in HAT, pentamidine is used in leishmaniasis
(p. 327) and in severe Pneumocystis jirovecii pneumonia, if co-trimoxazole
cannot be tolerated or is ineffective. It is administered via intravenous or
intramuscular routes. It is a relatively toxic drug, commonly causing rash,
renal impairment, profound hypotension (especially on rapid infusion),
electrolyte disturbances, blood dyscrasias and hypoglycaemia.
Other antiprotozoal agents
Pentavalent antimonials
Sodium stibogluconate and meglumine antimoniate inhibit protozoal
glycolysis by phosphofructokinase inhibition. They are used parenter-
ally (intravenous or intramuscular) to treat leishmaniasis. Adverse effects
include arthralgia, myalgias, raised hepatic transaminases, pancreatitis
and electrocardiogram changes. Severe cardiotoxicity leading to death
is not uncommon.
Diloxanide furoate
This oral agent is used to eliminate luminal cysts following treatment
of intestinal amoebiasis, or in asymptomatic cyst excreters. The drug
is absorbed slowly (enabling luminal persistence) and has no effect in
hepatic amoebiasis. It is a relatively non-toxic drug, the most signicant
adverse effect being atulence.
Iodoquinol (di-iodohydroxyquinoline)
Iodoquinol is a quinoline derivative with activity against Entamoeba his-
tolytica cysts and trophozoites. It is used orally to treat asymptomatic
cyst excreters or, in association with another amoebicide (e.g. metroni-
dazole), to treat extraintestinal amoebiasis. Long-term use of this drug is
not recommended, as neurological adverse effects include optic neuritis
and peripheral neuropathy.
Nitazoxanide
Nitazoxanide is an inhibitor of pyruvate–ferredoxin oxidoreductase-
dependent anaerobic energy metabolism in protozoa. It is a broad-spec-
trum agent, active against various nematodes, tapeworms, ukes and
152.
Further information 125
6
intestinal protozoa. Nitazoxanide also has activity against some anaero-
bic bacteria and viruses. It is administered orally in giardiasis and crypto-
sporidiosis. Adverse effects are usually mild and involve the gastrointestinal
tract (e.g. nausea, diarrhoea and abdominal pain).
Paromomycin
Paromomycin is an aminoglycoside that is used to treat visceral leish-
maniasis and intestinal amoebiasis. It is not signicantly absorbed when
administered orally, and is therefore given orally for intestinal amoebiasis
and by intramuscular injection for leishmaniasis. It showed early prom-
ise in the treatment of HIV-associated cryptosporidiosis but subsequent
trials have demonstrated that this effect is marginal at best.
Drugs used against helminths
Benzimidazoles (albendazole, mebendazole)
These agents act by inhibiting both helminth glucose uptake, causing
depletion of glycogen stores, and fumarate reductase. Albendazole
is used for hookworm, ascariasis, threadworm, Strongyloides infec-
tion, trichinellosis, Taenia solium (cysticercosis) and hydatid disease.
Mebendazole is used for hookworm, ascariasis, threadworm and whip-
worm. The drugs are administered orally. Absorption is relatively poor but
is increased by a fatty meal. Signicant adverse effects are uncommon.
Bithionol
Bithionol is used to treat uke infections with Fasciola hepatica. It is well
absorbed orally. Adverse effects are mild (e.g. nausea, vomiting, diar-
rhoea, rashes) but relatively common (approximately 30%).
Diethylcarbamazine
Diethylcarbamazine (DEC) is an oral agent used to treat lariasis and loi-
asis. Treatment of lariasis is often followed by fever, headache, nausea,
vomiting, arthralgia and prostration. This is caused by the host response
to dying microlariae, rather than the drug, and may be reduced by
pre-treatment with glucocorticoids.
Ivermectin
Ivermectin binds to helminth nerve and muscle cell ion channels,
causing increased membrane permeability. It is an oral agent, used in
Strongyloides infection, lariasis and onchocerciasis. Signicant side-
effects are uncommon.
Niclosamide
Niclosamide inhibits oxidative phosphorylation, causing paralysis of
helminths. It is an oral agent, used in Taenia saginata and intestinal
T. solium infection. Systemic absorption is minimal and it has few signi-
cant side-effects.
Piperazine
Piperazine inhibits neurotransmitter function, causing helminth mus-
cle paralysis. It is an oral agent, used in ascariasis and threadworm
(Enterobius vermicularis) infection. Signicant adverse effects are uncom-
mon but include neuropsychological reactions such as vertigo, delirium
and convulsions.
Praziquantel
Praziquantel increases membrane permeability to Ca2+
, causing violent
contraction of worm muscle. It is the drug of choice for schistosomi-
asis and is also used in T. saginata, T. solium (cysticercosis) and uke
infections (Clonorchis, Paragonimus) and in echinococcosis. It is admin-
istered orally and is well absorbed. Adverse effects are usually mild and
transient, and include nausea and abdominal pain.
Pyrantel pamoate
This agent causes spastic paralysis of helminth muscle through a
suxamethonium-like action. It is used orally in ascariasis and thread-
worm infection. Systemic absorption is poor and adverse effects are
uncommon.
Thiabendazole
Thiabendazole inhibits fumarate reductase, which is required for energy
production in helminths. It is used orally in Strongyloides infection and
topically to treat cutaneous larva migrans. Signicant adverse effects are
uncommon.
Further information
Websites
cdc.gov. Centers for Disease Control and Prevention, Atlanta, USA. Provides
information on all aspects of communicable disease, including prophylaxis
against malaria.
gov.uk/government/collections/immunisation-against-infectious-disease-the-
green-book. UK Department of Health recommendations for immunisation.
ecdc.europa.eu. European Centre for Disease Prevention and Control. Includes
data on prevalence of antibiotic resistance in Europe.
gov.uk/government/organisations/public-health-england. Public Health England.
Provides information on infectious diseases relating mainly to England,
including community infection control.
idsociety.org. Infectious Diseases Society of America. Publishes up-to-date,
evidence-based guidelines.
who.int. World Health Organization. Provides up-to-date information on global
aspects of infectious disease, including outbreak updates. Also has information
on the ‘World Antibiotic Awareness Week’ campaign.
Multiple Choice Questions
6.1.A 19-year-old student who had not received any childhood
vaccines through parental choice attends a music festival. Nine
days later she develops an illness consisting of conjunctivitis and
malaise with some intraoral lesions, followed by a generalised
maculopapular rash. With respect to the most likely causative
organism, which of the following statements is false?
A. The basic reproduction number (R0
) for this pathogen is one of
the highest encountered for an infectious disease
B. R0
gives an estimate of how many infections are likely to
arise from an infected individual and is dependent on vaccine
uptake and previous infections
C. R0
for this infection is higher than that for SARS-CoV-2 and
Ebola
D. Since R0
for this infection is high the proportion of the
population who need to be immunised to prevent spread of
infection is high
E. Re
gives an estimate of how many infections are likely to arise
from an infected individual taking into account vaccine uptake
in the population
Answer: B.
The case describes a measles infection. R0
is an estimate of the num-
ber of cases that can be expected to be acquired from a single primary
case when all individuals in the population are susceptible to infection,
and is high for measles (usually cited as 12–18). R0
is dependent on the
inherent transmissibility of the organism and assumes susceptibility of
the population. R0
is affected by the social habits and interactions of a
population at the time it is calculated, and therefore the Ro
for a disease
transmitted by close contact will be higher if calculated in a densely pop-
ulated area than a sparsely populated area. Because measles has a high
R0
the number of people in the population who need to be vaccinated
to control measles is high. Re
gives a more accurate assessment of the
number of infections that can be expected to arise within a population
than R0
because it takes into account the number of susceptible indi-
viduals within the population (i.e. vaccinated individuals and those who
have acquired immunity through natural infection) as well as changes
in behaviour (e.g. the introduction of social distancing, use of personal
protective equipment etc.). Re
therefore varies with time and population
behaviour.
6.2. A 23-year-old woman with cystic brosis and a history of multiple
infectious exacerbations is admitted to hospital with increasing
breathlessness. A chest X-ray reveals patchy inltrates throughout
the right lower lung on a background of bronchial dilatation in both
lungs. Sputum culture yields profuse growth of an oxidase-positive
Gram-negative bacillus. The patient has a history of rash with
piperacillin-tazobactam but has tolerated cephalosporins without
previous problems. Which of the following would be a suitable
agent to treat the most likely causative organism?
A. Cefaclor
B. Cefazolin
C. Ceftriaxone
D. Ceftazidime
E. Cephalexin
Answer: D.
The patient has a history of cystic brosis with multiple infectious exac-
erbations. Although an oxidase-positive Gram-negative bacillus could
describe several organisms, the most likely one in a setting of cystic
brosis with multiple infective exacerbations is P. aeruginosa. As the
patient has tolerated cephalosporins previously with no problems, it is
reasonable to treat her with an anti-pseudomonal cephalosporin despite
her history of rash with another beta-lactam antibiotic (piperacillin-
tazobactam). Ceftazidime is the only anti-pseudomonal cephalosporin
on this list. The only other third-generation cephalosporin that has good
Gram-negative coverage on the list is ceftriaxone, but it lacks activity
against P. aeruginosa
6.3. A 58-year-old man with diabetes mellitus is hospitalised with
an acute myocardial infarction. He develops cardiac failure and
acute kidney injury (AKI) and spends a period of time in the
cardiac intensive care unit. He subsequently develops a fever and
his blood cultures are positive for a yeast, which is identied as
Candida krusei. An antifungal agent is selected on the basis of
likely susceptibility and patient comorbidities. The mechanism of
action of this antifungal is:
A. Accumulation of toxic 14α-methylated sterols
B. DNA topoisomerase inhibition
C. Ergosterol synthesis inhibition in the cell membrane
D. Impairment of DNA synthesis due to inhibition of thymidylate
synthase
E. Inhibition of β-1,3-glucan synthesis in the cell wall
Answer: E.
The candidaemia is likely to be related to a central venous catheter
infection and Candida krusei is resistant to antifungal azoles. The best
initial therapy is therefore an echinocandin such as caspofungin, which
acts by inhibiting fungal β-1,3-glucan synthase. It would not be appro-
priate to treat this patient with an amphotericin B preparation because
of his AKI and the potential for nephrotoxicity. Antifungal azoles act by
inhibiting ergosterol synthesis resulting in toxic accumulation of 14α-
methylated sterols. Flucytosine (5-uorocytosine) inhibits DNA synthesis
by its actions on thymidylate synthase. Inhibition of bacterial DNA topo-
isomerases is the mechanism of action of uoroquinolones, but not of
any antifungals.
6.4. A 26-year-old man who grew up on a farm in rural Argentina
is investigated for chronic abdominal pain and weight loss. He
is found on investigation to have infection with Taenia saginata
and treatment is recommended. Guidelines suggest you should
prescribe praziquantel but your pharmacy is not able to obtain any
supplies of this drug. What would be a suitable alternative agent to
use to treat this infection?
A. Bithionol
B. Diethycarbazamine
C. Niclosamide
D. Piperazine
E. Thiabendazole
Answer: C.
Infections with intestinal tapeworms such as T. saginata are often
asymptomatic but can occasionally give rise to symptoms, as in this
case. The preferred treatment for Taenia infections is praziquantel but
niclosamide or nitazoxanide are alternatives. The other agents listed are
used for other helminth infections.
155.
S Clive
M Stares
7
Clinicalexamination of the cancer patient 128
Clinical examination of the patient on cancer treatment 129
The 10 hallmarks of cancer 130
1. Genome instability and mutation 130
2. Resisting cell death 130
3. Sustaining proliferative signalling 131
4. Evading growth suppressors 131
5. Enabling replicative immortality 131
6. Inducing angiogenesis 132
7.Activating invasion and metastasis 132
8. Deregulating cellular energetics 133
9.Tumour-promoting inammation 133
10. Evading immune destruction 133
Environmental and genetic determinants of cancer 133
Investigations 133
Determining the extent of disease (staging) 133
Establishing the type of cancer 135
Assessing tness 137
Multidisciplinary teams 138
Acute oncology 138
Acute presentation of new cancer 138
Oncological emergencies 139
Other acute presentations in oncology 142
Symptoms from locally advanced cancer or metastatic sites 143
Treatment-related toxicities 145
Therapeutics in oncology 145
Surgical treatment 145
Radiotherapy 145
Systemic anti-cancer therapy 146
Evaluation of treatment 148
Late toxicity of therapy 148
Cancer clinical trials 148
Specic cancers 149
Breast cancer 149
Ovarian cancer 150
Endometrial cancer 150
Cervical cancer 150
Head and neck tumours 151
Survivorship 151
Oncology
156.
128 OncOlOgy
Clinicalexamination of the cancer patient
Observation
Breast asymmetry, lump
Lymph nodes (see p. 923)
Cervical
Supraclavicular
Axillary
Inguinal
Face
Conjunctival pallor
Icterus
Horner syndrome
Cushingoid features
Cardiovascular
Superior vena cava obstruction
(SVCO) (see Box 7.15)
Atrial fibrillation
Pericardial effusion (see Ch. 16)
Hypo-/hypertension
Abdomen (see p. 783)
Surgical scars
Umbilical nodule
Mass in epigastrium
Visible peristalsis
Abdominal distension
Ascites
Hepatomegaly
Splenomegaly
Renal mass
Pelvic or adnexal mass
Cachexia
Dehydration
Asymmetry/lumps
Hands
Clubbing
Signs of smoking
Pallor
Periphery
Calf tenderness, venous
thrombosis
Rash, skin changes (see also p. 1065)
Skin tethering above the
nipple
Finger clubbing in
lung cancer
SVCO in a patient with a
mediastinal mass
Skeletal survey
Focal bone tenderness
(pelvis, spine, long bones)
Wrist tenderness
(hypertrophic pulmonary
osteoarthropathy)
Neurological
Ascites (ovarian carcinoma)
6
5
7
8
9
10
3
4
2
1
6 7
Respiratory
Stridor
Consolidation
Pleural effusion (see p. 481)
4
8
9
10
1
5
3
2
Focal neurological signs
Sensory deficit
Spinal cord compression
Memory deficit
Personality change
130 OncOlOgy
Cancerrepresents a signicant global health, social and economic
burden. In 2018 there were 17 million new cases of cancer worldwide
and 9.6 million cancer deaths, making it the second leading cause of
death. By 2030, it is projected that there will be 26 million new cancer
cases and 17 million cancer deaths per year. The developing world is
disproportionately affected by cancer and in 2018 approximately 70%
of cancer deaths occurred in low- and middle-income countries. These
deaths happen in countries with limited or no access to investigations or
treatment and with low per capita expenditure on health care.
The most common solid organ malignancies arise in the lung, breast
and gastrointestinal tract (Fig. 7.1), but the most common form world-
wide is skin cancer. Cigarette smoking accounts for more than 20% of
all global cancer deaths, 80% of lung cancer cases in men and 50%
of lung cancer cases in women worldwide, which could be prevented
by smoking cessation. Diet and alcohol contribute to a further 30% of
cancers, including those of the stomach, colon, oesophagus, breast
and liver. Lifestyle modication could reduce these if steps were taken
to avoid animal fat and red meat, reduce alcohol, increase bre, fresh
fruit and vegetable intake, avoid obesity and increase physical activity.
Infections account for a further 15% of cancers (25% of cancers in low-
and middle-income countries), including those of the cervix, stomach,
liver, nasopharynx, anus and bladder, and some of these could be pre-
vented by infection control and vaccination.
The 10 hallmarks of cancer
The formation and growth of cancer is a multistep process, during which
normal cells are transformed into malignant cells. Ten key characteristics
that underlie these steps, collectively referred to as the ‘Hallmarks of
Cancer’, have been described.
1. Genome instability and mutation
Random genomic aberrations occur continuously throughout all cells of
the body. This may include somatic point mutations, insertions, deletions
and chromosome structural changes (i.e. copy number changes, chro-
mosomal translocations). Epigenomic aberrations, such as DNA meth-
ylation and histone modication, may also occur. Rarely, aberrations will
confer a selective survival advantage on single cells, ‘driving’ overgrowth
and dominance in local tissue environments. Multistep carcinogenesis
results from successive clonal expansions of pre-malignant cells, each
expansion being triggered by acquisition of a random driver aberration.
Under normal circumstances, genome maintenance systems and
DNA repair mechanisms are so effective that almost all spontaneous
genomic aberrations are repaired, or damaged cells are forced into
senescence or apoptosis. In cancer cells, though, the accumulation
of mutations can be accelerated by compromising these maintenance
mechanisms. In turn, this leads to the accumulation of driver genomic
aberrations which lead to cancer growth and progression. Genomic
sequencing technology demonstrates that the pattern of aberrations
vary dramatically between cancer types. However, defects in genome
maintenance mechanisms leading to genome instability are common
ndings across all cancers. This enabling characteristic may lead to the
acquisition of other hallmarks.
2. Resisting cell death
There are three principal mechanisms through which cell death occurs in
healthy tissues: apoptosis, autophagy and necrosis.
Apoptosis
This is programmed cell death. It is frequently found at markedly reduced
rates in cancers, particularly those of high grade or those resistant to
treatment. The cellular apoptotic system has regulatory elements that
sense intrinsic and extrinsic pro-apoptotic signals. This initiates a cas-
cade of proteolysis and cell disassembly with nuclear fragmentation,
chromosomal condensation and shrinking of the cell with loss of inter-
cellular contact, followed by cellular fragmentation and the formation of
apoptotic bodies that are phagocytosed by neighbouring cells. The most
important regulator of apoptosis is the TP53 tumour suppressor gene,
often described as the ‘guardian of the genome’ as it is able to induce
apoptosis in response to sufcient levels of genomic damage. The larg-
est initiator of apoptosis via TP53 is cellular injury, particularly that due
to DNA damage from cytotoxic chemotherapy, oxidative damage and
ultraviolet (UV) radiation. Disruption of p53 protein function as a result of
mutations in the TP53 gene are found in over half of cancers.
Autophagy
This is a catabolic process during which cellular constituents are
degraded by lysosomal machinery within the cell. It is an important phys-
iological mechanism; it usually occurs at low levels in cells but can be
induced in response to environmental stresses, particularly radiotherapy
and cytotoxic chemotherapy, which induce elevated levels of autophagy
that are cytoprotective for malignant cells, thus impeding rather than per-
petuating the killing actions of these stress situations. Severely stressed
cancer cells have been shown to shrink via autophagy to a state of
reversible dormancy.
Necrosis
This is the premature death of cells and is characterised by the release of
cellular contents into the local tissue microenvironment, in marked con-
trast to apoptosis, where cells are disassembled in a step-by-step fash-
ion and the resulting cellular fragments are phagocytosed. Necrotic cell
Fig. 7.1 The most commonly diagnosed cancers in the UK.
(CNS = central nervous system; NHL = non-Hodgkin lymphoma) Statistics
from Cancer Research UK website (http://info.cancerresearchuk.org)
Breast
Prostate
Lung
Bowel
Melanoma
NHL
Kidney
Head
and
neck
CNS
Pancreas
Bladder
Leukaemia
Uterus
Oesophagus
Cancer
of
unknown
primary
Ovary
Stomach
Liver
Myeloma
Thyroid
Other
0
5
10
15
20
25
30
35
40
45
50
55
60
Number
of
new
cases
(thousands)
Male
Female
159.
The 10 hamarksof aer 131
7
death results in the recruitment of inammatory immune cells, promotion
of angiogenesis and release of stimulatory factors that increase cellular
proliferation and tissue invasion, thereby enhancing rather than inhibiting
carcinogenesis.
3. Sustaining proliferative signalling
The ‘cell cycle’ is tightly controlled at different stages. Normal cells grow
and divide in response to external signals, typically growth factors. These
are able to bind to cell surface-bound receptors that activate an intra-
cellular tyrosine kinase-mediated signalling cascade, ultimately leading to
changes in gene expression that promote cellular proliferation and growth.
The cell cycle
The cell cycle is composed of four ordered, strictly regulated phases
referred to as G1
(gap 1), S (DNA synthesis), G2
(gap 2) and M (mitosis)
(Fig. 7.2). Normal cells grown in culture will stop proliferating and enter
a quiescent state called G0
once they become conuent or are deprived
of serum or growth factors. The rst gap phase (G1
) prior to the initiation
of DNA synthesis represents the period of commitment that separates
M and S phases as cells prepare for DNA duplication. Cells in G0
and
G1
are receptive to growth signals, but once they have passed a restric-
tion point, they are committed to enter DNA synthesis (S phase). Cells
demonstrate arrest at different points in G1
in response to different inhib-
itory growth signals. Mitogenic signals promote progression through G1
to S phase, utilising phosphorylation of the retinoblastoma gene product
(pRB, p. 40). Following DNA synthesis, there is a second gap phase (G2
)
prior to mitosis (M), allowing cells to repair errors that have occurred
during DNA replication and thus preventing propagation of these errors
to daughter cells. Although the duration of individual phases may vary,
depending on cell and tissue type, most adult cells are in a G0
state at
any one time.
Stimulation of the cell cycle
Many cancer cells produce growth factors, which drive their own prolifer-
ation by a positive feedback mechanism known as autocrine stimulation.
Examples include transforming growth factor-alpha (TGF-α) and platelet-
derived growth factor (PDGF). Other cancer cells express growth factor
receptors at increased levels due to gene amplication or express abnor-
mal receptors that are permanently activated. This results in abnormal cell
growth in response to physiological growth factor stimulation or even in
the absence of growth factor stimulation (ligand-independent signalling).
The epidermal growth factor receptor (EGFR) is often over-expressed in
lung and gastrointestinal tumours and the human epidermal growth fac-
tor receptor 2 (HER2)/neu receptor is frequently over-expressed in breast
cancer. Both receptors activate the Ras–Raf–mitogen activated protein
(MAP) kinase pathway, causing cell proliferation.
4. Evading growth suppressors
The cell cycle is orchestrated by a number of molecular mechanisms,
most importantly by cyclins and cyclin-dependent kinases (CDKs).
Cyclins bind to CDKs and are regulated by both activating and inacti-
vating phosphorylation, with two main checkpoints at G1
/S and G2
/M
transition. The genes that inhibit progression play an important part in
tumour prevention and are referred to as tumour suppressor genes (e.g.
TP53, TP21, TP16 genes). The products of these genes deactivate the
cyclin–CDK complexes and are thus able to halt the cell cycle. The com-
plexity of cell cycle control is susceptible to dysregulation, and mutations
within inhibitory proteins are common in cancer.
5. Enabling replicative immortality
Normal cells have a limited number of divisions before they are unable
to divide further (senescence) or before they die (crisis). These limits
are controlled by telomeric DNA sequences, which protect and stabilise
chromosomal ends. During replication, telomeres shorten progressively
as small fragments of telomeric DNA are lost with successive cycles of
replication. This shortening process represents a mitotic clock and even-
tually prevents the cell from dividing further. Telomerase, a specialised
polymerase enzyme, adds nucleotides to telomeres, allowing continued
cell division and thus preventing premature arrest of cellular replication.
The telomerase enzyme is almost absent in normal cells but is expressed
at signicant levels in the majority of human cancers.
Fig. 7.2 The cell cycle and sites of action of chemotherapeutic agents. (CDK = cyclin-dependent kinase; RB = retinoblastoma gene)
DNA replication
checkpoint for:
Damaged DNA
RB blocks
TP53 CDKs blocked
Restriction point
(regulated by growth
factors)
G1
Quiescent
G1
Cell
growth
G0
Cyclin D
CDK4, 6
Cell growth
Cyclin E
CDK2
G
S
M
Cyclin A
CDK2
Cyclin B
CDK
2
1
Terminal
differentiation
Apoptosis
Mitosis
Prophase telophase
Nuclear and cellular division
Terminal differentiation
Apoptosis
G2 checkpoint for:
DNA damage
DNA replication
incomplete
Further growth
or DNA repair
160.
132 OncOlOgy
6.Inducing angiogenesis
All cells and body tissues require sustenance in the form of nutrients
and oxygen, as well as an ability to evacuate metabolic waste products
and carbon dioxide. Tumours require a functional vascular network to
ensure continued growth and are unable to grow beyond 1mm3
without
stimulating the development of a vascular supply through angiogenesis
(Figs. 7.3 and 7.4).
Angiogenesis is dependent on the production of angiogenic growth
factors, of which vascular endothelial growth factor (VEGF) and plate-
let-derived growth factor (PDGF) are the best characterised. During
tumour progression, an angiogenic switch is activated and remains on,
causing normally quiescent vasculature to develop new vessels continu-
ally to help sustain expanding tumour growth. Angiogenesis is governed
by a balance of pro-angiogenic stimuli and angiogenesis inhibitors, such
as thrombospondin (TSP)-1, which binds to transmembrane receptors
on endothelial cells and evokes suppressive signals. A number of cells
can contribute to the maintenance of a functional tumour vasculature
and therefore sustain angiogenesis. These include pericytes and a variety
of bone marrow-derived cells such as macrophages, neutrophils, mast
cells and myeloid progenitors.
7. Activating invasion and metastasis
The ability to invade neighbouring tissue determines whether a tumour
is benign or malignant. Clinically, the presence of metastases often
determines whether a cancer can be cured. The invasion-metastatic
cascade is a complex multistep process. The initiation of this process
is enabled by epithelial-mesenchymal transition (EMT). Cancer cells in a
tumour lose normal cell–cell adhesion through the down-regulation or,
occasionally, mutational inactivation of E-cadherin, a calcium-dependent
cell–cell adhesion glycoprotein. After breaking through the basement
membrane, cancer cells enter the blood stream (intravasation). These cir-
culating tumour cells (CTCs) then exit the blood stream into distant tissues
(extravasation) to form small nodules of cancer cells (micrometastases).
Fig. 7.3 Oncogenesis. The multistep origin of cancer, showing events implicated in cancer initiation, progression, invasion and metastasis.
Basal lamina
First mutation
Inherited or acquired
gain of oncogene
Loss of tumour
suppressor gene
Normal
epithelium
Blood vessel
Connective
tissue
Lymphatic
First mutation
Initial proliferation Clonal expansion Further mutation
Further mutation; subset
selected for rapid growth
Localised
Further mutation
Blood spread
Lymphatic
spread
Ectopic
growth factor
production and
autostimulation
Failed apoptosis
(e.g. TP53 mutation)
Local invasion
through basal
lamina
Angiogenesis
to support
tumour growth
(see Fig. 7.4)
Breakdown of connective tissue
via tumour production of
e.g. collagenase
tissue metalloproteinases
Loss of cell adhesion molecules
e.g. E-cadherin
invasion or metastasis
Inhibition
Loss of
inhibition
Viable tumour cell
Apoptotic
tumour cell
Angiogenic
factors
VEGF
VEGF
receptor Tissue
factor
αvβ3 integrin
Plasminogen
Coagulation factor
Coagulation
Fibrinogen
Fibrin
Cell adhesion
Urokinase
Urokinase
receptor Proteolysis
Plasmin
A
B C
Fig. 7.4 Angiogenesis, invasion and metastasis.
beyond 1mm3
the release of angiogenic factors by the tumour cells and loss of inhibition of the
matrix allow cells to extravasate into the blood stream and metastasise to distant
sites. (VEGF = vascular endothelial growth factor)
Following mesenchymal–epithelial transition these micrometastatic lesions
develop into macroscopic tumours (colonisation) (see Fig. 7.3).
Cross-talk between cancer cells and cells of the surrounding stro-
mal tissue is involved in the acquired capability for invasive growth and
metastasis. Mesenchymal stem cells in tumour stroma have been found
161.
Ivestiatios 133
7
tosecrete CCL5, a protein chemokine that helps recruit leucocytes into
inammatory sites. With the help of particular T-cell-derived cytokines
(interleukin (IL)-2 and interferon-gamma (IFN-γ)), CCL5 induces pro-
liferation and activation of natural killer cells and then acts reciprocally
on cancer cells to stimulate invasive behaviour. Macrophages at the
tumour periphery can foster local invasion by supplying matrix-degrading
enzymes such as metalloproteinases and cysteine cathepsin proteases.
8. Deregulating cellular energetics
Under aerobic conditions, oxidative phosphorylation functions as the
main metabolic pathway for energy production; cells process glucose,
rst to pyruvate via glycolysis and thereafter to carbon dioxide in the
mitochondria. While under anaerobic conditions, glycolysis is favoured
to produce adenosine triphosphate (ATP). Cancer cells can reprogram
their glucose metabolism to limit energy production to glycolysis, even
in the presence of oxygen. This has been termed ‘aerobic glycolysis’.
Up-regulation of glucose transporters, such as GLUT1, is the main
mechanism through which aerobic glycolysis is achieved.
This reprogramming of energy metabolism appears paradoxical, as
overall energy production from glycolysis is signicantly lower (18-fold)
than that from oxidative phosphorylation. One explanation may be that
the increased production of glycolytic intermediates can be fed into var-
ious biosynthetic pathways, including those that generate the nucleo-
sides and amino acids, necessary for the production of new cells.
9. Tumour-promoting inammation
Almost all tumours show inltration with immune cells on pathological inves-
tigation and historically this nding was thought to represent an attempt of
the immune system to eradicate the cancer. It is now clear that tumour-
associated inammatory responses contribute to several hallmark capabili-
ties and promote tumour formation and cancer progression. Immune cells
may promote invasive behaviour, and bioactive molecules such as cytokines,
growth factors and pro-angiogenic factors may be released into the tumour
microenvironment. In particular, the release of reactive oxygen species, which
are actively mutagenic, will accelerate the genetic evolution of surrounding
cancer cells, enhancing growth and contributing to cancer progression.
10. Evading immune destruction
Cancer cells continuously shed antigens into the circulatory system,
prompting an immune response that includes cytotoxic T-cell, natural
killer cell and macrophage production. The immune system is thought
to provide continuous surveillance, with resultant elimination of cells that
undergo malignant transformation. However, if not all cancer cells are
eliminated there may be a period of equilibrium where tumour growth is
controlled by the immune system and cancer cells may enter a dormant
state, sometimes for many years. Alternatively, cancer cells may escape
immune control and grow into clinically apparent tumours. Several fac-
tors may contribute to immune escape, including reduced immune rec-
ognition, increased resistance or survival and the development of an
immunosuppressive tumour microenvironment.
Immune checkpoints are increasingly recognised as key factors in the
immune escape of tumours. These pathways are crucial for regulating
the immune response to normal cells by down-regulating the immune
system and promoting self-tolerance by suppressing T-cell immune
activity. However, some cancers co-opt these protective mechanisms
by stimulating immune checkpoint molecular targets such as the pro-
grammed cell death ligand (PD-L1), a protein that helps stop immune
cells from attacking other cells in the body (see Fig. 7.12).
Environmental and genetic determinants
of cancer
The majority of cancers do not have a single cause but rather are the
result of a complex interaction between genetic factors and exposure
to environmental carcinogens. These are often tumour type-specic but
some general principles do apply.
Environmental factors
Environmental triggers for cancer have mainly been identied through
epidemiological studies that examine patterns of distribution of cancers
in patients in whom parameters such as age, sex, presence of other
illnesses, social class, geography and diet differ. Sometimes, these give
strong pointers to the molecular or cellular causes of the disease, such
as the association between aatoxin production within contaminated
food supplies and hepatocellular carcinomas. For most solid cancers,
however, there is evidence of a multifactorial pathogenesis, even when
there is a principal environmental cause (Box 7.1).
Smoking is now established beyond all doubt as a major cause of lung
cancer, but there are additional predisposing factors since not all smok-
ers develop cancer. Similarly, most carcinomas of the cervix are related
to infection with human papillomavirus (HPV subtypes 16 and 18). For
carcinomas of the bowel and breast, there is strong evidence of an envi-
ronmental component. For example, the risk of breast cancer in women
of Far Eastern origin remains relatively low when they rst migrate to a
country with a Western lifestyle, but rises in subsequent generations to
approach that of the resident population of the host country. The precise
environmental factors that cause this change are unclear but may include
diet (higher intake of saturated fat and/or dairy products), reproductive
patterns (later onset of rst pregnancy) and lifestyle (increased use of
articial light and shift in diurnal rhythm).
Genetic factors
A number of inherited cancer syndromes are recognised and account
for 5%–10% of all cancers (Box 7.2). Their molecular basis is discussed
in Chapter 3, but in general they result from inherited mutations in driver
genes that regulate cell growth, cell death and apoptosis. Although carri-
ers of these gene mutations have a greatly elevated risk of cancer, none
has 100% penetrance and additional modulating factors, both genetic
and environmental, are likely to be operative. Exploration of a possible
genetic contribution is a key part of cancer management. It may inform
anti-cancer therapy decisions in people with cancer. Patients and their
family members may also benet from screening investigations to detect
cancer early, or preventative treatments such as prophylactic mastec-
tomy in women who carry a BRCA1 or BRCA2 mutation.
Investigations
When a patient is suspected of having cancer, a full history should be
taken. Attention should be paid to potential risk factors such as smoking
and occupational exposures, any family history of cancer and elucidating
potential complications of the disease. A thorough clinical examination
is essential to identify both the primary cancer site and possible sites of
metastases, and to discover any other conditions that may have a bear-
ing on the management plan.
In order to plan the most appropriate investigations and management,
information is needed on:
the extent of disease
the type of cancer
the patient’s tness.
Determining the extent of disease (staging)
The extent of disease, such as how large a tumour is and if it has spread,
is determined by the process of staging. It entails clinical examination,
imaging (e.g. CT, MRI, PET), specialised investigations (e.g. colonoscopy,
endoscopy, laparoscopy, mediastinoscopy) and, in some cases, surgery.
The outcome is recorded using a standard staging classication that
162.
134 OncOlOgy
7.1Environmental factors that predispose to cancer
Environmental aetiology Processes Diseases
Occupational exposure
(see ‘Radiation’ below)
Dye and rubber manufacturing (aromatic amines) Bladder cancer
Asbestos mining, construction work, shipbuilding (asbestos) Lung cancer and mesothelioma
Vinyl chloride (PVC) manufacturing Liver angiosarcoma
Petroleum industry (benzene) Acute leukaemia
Chemicals Cytotoxic chemotherapy (e.g. melphalan, cyclophosphamide) Acute myeloid leukaemia
Cigarette smoking Exposure to carcinogens from inhaled smoke Lung, throat, oesophagus and bladder cancer
Viral infection Epstein–Barr virus
Human papillomavirus
Hepatitis B and C viruses
Burkitt lymphoma and nasopharyngeal cancer
Cervical cancer, anal cancer, oropharyngeal cancer
Hepatocellular carcinoma
Bacterial infection Helicobacter pylori Gastric MALT lymphomas, gastric cancer
Parasitic infection Liver uke (Opisthorchis sinensis)
Schistosoma haematobium
Cholangiocarcinoma
Squamous cell bladder cancer
Dietary factors Low-roughage/high-fat content diet
High nitrosamine intake
Aatoxin from contamination of Aspergillus avus
Colonic cancer
Gastric cancer
Hepatocellular cancer
Obesity Reduced physical activity, increased insulin
Increased oestrogen
Colon cancer
Breast and endometrial cancer
Radiation UV exposure Basal cell carcinoma
Melanoma
Non-melanocytic skin cancer
Nuclear fallout following explosion (e.g. Hiroshima, Chernobyl) Leukaemia
Solid tumours, e.g. thyroid
Diagnostic exposure (e.g. CT) Cholangiocarcinoma following Thorotrast usage
Occupational exposure (e.g. beryllium and strontium mining) Lung cancer
Therapeutic radiotherapy Medullary thyroid cancer
Sarcoma
Inammatory diseases Ulcerative colitis Colon cancer
Hormonal Use of diethylstilbestrol
Oestrogens
Vaginal cancer
Endometrial cancer
Breast cancer
(CT = computed tomography; MALT = mucosa-associated lymphoid tissue; UV = ultraviolet)
allows comparisons to be made between different groups of patients.
One of the most commonly used systems is the T (tumour), N (regional
lymph nodes), M (metastatic sites) approach of the International Union
against Cancer (UICC, see Box 7.3). For most cancers staging will dene
patients into having:
early, localised disease, which can be cured with surgery
or other localised therapy (e.g. radiotherapy or ablative
therapies)
locally advanced disease, which can often be cured with a com-
bination of surgery and/or systemic anti-cancer therapy and
radiotherapy
metastatic disease, where the cancer has spread to distant sites,
which can only rarely be cured with systemic anti-cancer therapy,
radiotherapy and/or surgery.
Imaging plays a critical role in the diagnosis and staging of cancer. It is
also used to determine the response to treatment. The imaging modality
employed depends primarily on the site of the disease and likely patterns
of spread.
Computed tomography
Computed tomography (CT) is a key investigation in cancer patients and
is particularly useful in imaging the thorax and abdomen (Fig. 7.5). With
modern scanners it is possible to visualise the large bowel if it is prepared
(CT colonography), allowing accurate detection of colorectal cancers
and adenomas ≥10mm.
Ultrasound
Ultrasound is useful in characterising lesions within the liver, kidney, pan-
creas and reproductive organs. It may be used for guiding biopsies of
tumours in the breast and liver. Endoscopic ultrasound is helpful in stag-
ing upper gastrointestinal and pancreatic cancers, involving a special
endoscope with an ultrasound and biopsy probe attached.
Magnetic resonance imaging
Magnetic resonance imaging (MRI) has a high resolution and is the pre-
ferred technique for brain, bone and pelvic imaging. It is widely employed
for the staging of rectal, cervical and prostate cancers.
Positron emission tomography
Positron emission tomography (PET) visualises metabolic activity of
tumour cells and is often used in combination with CT (PET–CT) to eval-
uate the extent of the disease, particularly in the assessment of potential
distant metastases when a radical treatment approach is being consid-
ered. Not all tumour types are ‘PET-avid’ and the metabolic activity of
163.
Ivestiatios 135
7
7.2Inherited cancer predisposition syndromes
Syndrome Malignancies Inheritance Gene
Ataxia telangiectasia Leukaemia, lymphoma, ovarian, gastric, brain, colon AR AT
Bloom syndrome Leukaemia, tongue, oesophageal, colonic, Wilms' tumour AR BLM
Breast/ovarian Breast, ovarian, colonic, prostatic, pancreatic AD BRCA1, BRCA2
Cowden syndrome Breast, thyroid, gastrointestinal tract, pancreatic AD PTEN
Familial adenomatous polyposis Colonic, upper gastrointestinal tract AD APC, MUTYH
Familial atypical multiple mole
melanoma (FAMMM)
Melanoma, pancreas AD CDKN2A (TP16)
Fanconi anaemia Leukaemia, oesophageal, skin, hepatoma AR FACA, FACC, FACD
Gorlin syndrome Basal cell skin, brain AD PTCH
Hereditary diffuse gastric cancer Diffuse gastric cancer AD E-cadherin
Hereditary non-polyposis colon
cancer (HNPCC)
Colonic, endometrial, ovarian, pancreatic, gastric AD MSH2, MLH1, MSH6, PMS1, PMS2
Li–Fraumeni syndrome Sarcoma, breast, osteosarcoma, leukaemia, glioma,
adrenocortical
AD TP53
Multiple endocrine neoplasia
(MEN) 1
Pancreatic islet cell, pituitary adenoma, parathyroid
adenoma and hyperplasia
AD MEN1
MEN 2 Medullary thyroid, phaeochromocytoma, parathyroid
hyperplasia
AD RET
Neurobromatosis 1 Neurobrosarcoma, phaeochromocytoma, optic glioma AD NF1
Neurobromatosis 2 Vestibular schwannoma AD NF2
Papillary renal cell cancer
syndrome
Renal cell cancer AD MET
Peutz–Jegher syndrome Colonic, ileal, breast, ovarian AD STK11
Prostate cancer Prostate AD HPC1
Retinoblastoma Retinoblastoma, osteosarcoma AD RB1
von Hippel–Lindau syndrome Haemangioblastoma of retina and CNS, renal cell,
phaeochromocytoma
AD VHL
Wilms’ tumour Nephroblastoma, neuroblastoma, hepatoblastoma,
rhabdomyosarcoma
AD WT1
Xeroderma pigmentosum Skin, leukaemia, melanoma AR XPA, XPC, XPD (ERCC2), XPF
(AD = autosomal dominant; AR = autosomal recessive; CNS = central nervous system)
small but malignant lymph nodes may not be detected, limiting the sen-
sitivity of PET-CT.
Biochemical markers
Many cancers produce substances called tumour markers, some of
which may assist in diagnosis, response evaluation and detection of
relapse. Unfortunately, most tumour markers are neither sufciently sen-
sitive nor sufciently specic to be used in isolation for diagnosis and
need to be interpreted in the context of other clinical features. Some
can be used for antibody-directed therapy or imaging, where they have
a greater role in diagnosis. Tumour markers in routine use are outlined
in Box 7.4
Establishing the type of cancer
In most cases a biopsy is required in order to establish the tumour type.
Biopsies can be taken at endoscopy, laparoscopy, surgery or by ultra-
sound- or CT- guided biopsy with the help of interventional radiology. At
times a diagnosis can be obtained by examination of cells (cytology) from
uid (e.g. pleural or peritoneal), from a smear (e.g. cervical) or by doing
ne needle aspiration of a palpable or visible lump (Box 7.5).
7.3 TNM classication
Extent of primary tumour*
TX
T0
T1
T2
T3
T4
Not assessed
No tumour
Increase in primary tumour size or
depth of invasion
Increased involvement of nodes*
NX
N0
N1
N2/3
Not assessed
No nodal involvement
Increases in involvement
Presence of metastases
MX Not assessed
M0 Not present
M1 Present
*Exact criteria for size and region of nodal involvement have been dened for
each anatomical site.
164.
136 OncOlOgy
Fig.7.5 Computed tomography (CT) images.
after cytotoxic chemotherapy showing a partial radiological response to treatment. These remain stable 9 years later.
A B
P P
7.4 Commonly used serum tumour markers
Name Natural occurrence Tumours
Alpha-fetoprotein (AFP) Glycoprotein found in yolk sac and fetal liver tissue.
Transient elevation in liver diseases. Has a role in
screening during pregnancy for the detection of neural
tube defects and Down syndrome
Ovarian non-seminomatous germ cell tumours (80%),
testicular teratoma (80%), hepatocellular cancer (50%)
Beta-2-microglobulin A human leucocyte antigen (HLA) common fragment
present on surface of lymphocytes, macrophages and
some epithelial cells. Can be elevated in autoimmune
disease and renal glomerular disease
Non-Hodgkin lymphoma, myeloma
Calcitonin 32-amino-acid peptide from C cells of thyroid. Used to
screen for MEN 2
Medullary cell carcinoma of thyroid
Cancer antigen 125 (CA-125) Differentiation antigen of coelomic epithelium (Müller’s
duct). Raised in any cause of ascites, pleural effusion or
heart failure. Can be raised in inammatory conditions
Ovarian epithelial cancer (75%), gastrointestinal cancer
(10%), lung cancer (5%) and breast cancer (5%)
CA-19.9 A mucin found in epithelium of fetal stomach, intestine
and pancreas. It is eliminated exclusively via bile and so
any degree of cholestasis can cause levels to rise
Pancreatic cancer (80%), mucinous tumour of the
ovary (65%), gastric cancer (30%), colon cancer (30%)
Carcinoembryonic antigen (CEA) Glycoprotein found in intestinal mucosa during embryonic
and fetal life. Elevated in smokers, cirrhosis, chronic
hepatitis, ulcerative colitis, pneumonia
Colorectal cancer, particularly with liver metastasis,
gastric cancer, breast cancer, lung cancer, mucinous
cancer of the ovary
Human chorionic gonadotrophin (hCG) Glycoprotein hormone, 14kD α subunit and 24kD β
subunit from placental syncytiotrophoblasts. Used for
disease monitoring in hydatidiform mole and as the basis
of a pregnancy test
Choriocarcinoma (100%), hydatidiform moles (97%),
ovarian non-seminomatous germ cell tumours
(50%–80%), seminoma (15%)
Placental alkaline phosphatase (PLAP) Isoenzyme of alkaline phosphatase Seminoma (40%), ovarian dysgerminoma (50%)
Prostate-specic antigen (PSA) Glycoprotein member of human kallikrein gene family.
PSA is a serine protease that liquees semen in excretory
ducts of prostate. Can be elevated in benign prostatic
hypertrophy and prostatitis
Prostate cancer (95%)
Thyroglobulin Matrix protein for thyroid hormone synthesis in normal
thyroid follicles
Papillary and follicular thyroid cancer
(MEN = multiple endocrine neoplasia)
Histopathology
Histopathological analysis of tumour tissue is pivotal in identifying the
type of cancer and provides information that may direct subsequent
management. The results of histopathological analysis are most informa-
tive when combined with knowledge of the clinical picture; biopsy results
should therefore be reviewed and discussed within the context of a multi-
disciplinary team meeting.
Light microscopy
Examination of tumour samples by light microscopy remains the core
method of cancer diagnosis and, in cases where the primary site is
unclear, may give clues to the origin of the tumour:
Signet-ring cells favour a gastric primary.
Presence of melanin favours melanoma.
165.
Ivestiatios 137
7
Mucin is common in gut/lung/breast/endometrial and ovarian
cancers.
Psammoma bodies are a feature of ovarian cancer (mucin +) and
thyroid cancer (mucin ).
Immunohistochemistry
Immunohistochemical (IHC) staining for tumour markers can provide
useful diagnostic information and may help with treatment decisions.
Commonly used examples of IHC in clinical practice include:
Oestrogen (ER) and progesterone (PR) receptor positivity indicate
that the cancer may be sensitive to hormonal manipulation.
Alpha-fetoprotein (AFP) and human chorionic gonadotrophin (hCG)
favour germ-cell tumours.
Prostate-specic antigen (PSA) favours prostate cancer.
Cytokeratins and epithelial membrane antigen (EMA) favour epithelial
carcinomas.
HER2 receptor positivity in breast or gastric cancers indicate that
the tumour may respond to HER2 inhibitor targeted therapy.
T-cell receptor and cluster designation (CD) antigen expression aid in
the diagnosis and classication of lymphomas.
Molecular pathology
Molecular proling of individual tumours is used to better understand
their clinical behaviour and to stratify treatment options. Treatment selec-
tion is increasingly tailored to specic ‘druggable’ molecular pathways
which include:
EGFR mutations – predict response to EGFR inhibitor targeted
therapy in lung cancer
RAS mutations – predict resistance to EGFR inhibitor targeted
therapy in colon cancer
ALK fusion oncogene rearrangement – predict response to ALK
inhibitor targeted therapy in lung cancer
microsatellite instability – predict response to checkpoint inhibitor
immunotherapy in several cancer types
Cytogenetic analysis
Some tumours demonstrate typical chromosomal changes that help in
diagnosis. The utilisation of uorescent in situ hybridisation (FISH) tech-
niques can be useful in Ewing's sarcoma and peripheral neuro-ectodermal
tumours where there is a translocation between chromosome 11 and 22:
t(11;22)(q24;q12). In some cases, gene amplication can be detected via
FISH (e.g. determining over-expression of HER2/neu).
Assessing tness
When making decisions about investigations and management in patients
with cancer it is important to consider an individual’s general condition
and comorbidities. The Eastern Cooperative Oncology Group (ECOG)
performance status (PS) scale is used to formally assess a patient’s t-
ness (Box 7.6). Outcomes for patients with PS 3–4 are worse in almost
all malignancies than for those with PS 0–2, and this has a strong inu-
ence on the approach to treatment in the individual patient. Additionally,
outcomes are typically worse for patients with a low albumin and raised
inammatory markers (i.e. white blood cell count, C-reactive protein).
Comorbidities also frequently inuence treatment options. For example,
hepatic or renal dysfunction means some systemic therapies cannot be
used or can only be given in lower doses. In some patients poor tness
or comorbidities may preclude any anti-cancer therapy options, in which
case comprehensive diagnostic and staging investigations add little clini-
cal value and may not be in the best interests of the patient.
Although the incidence of cancer increases with patient age, the
approach to investigation and management is similar at all ages (Box7.7).
However, assessing tness in this group of patients can be difcult, with
7.5 How to perform a ne needle aspiration (FNA) of a tumour/
lesion
Ensure the patient is in a comfortable position with the lesion accessible to the
operator.
Identify the tumour, conrm that it is easily palpable and make sure there are no
nearby critical structures.
If tumour is not palpable or concern about nearby critical structures then
perform FNA under ultrasound guidance.
Clean the skin over the lesion.
Inject local anaesthetic (if required) into the skin over the lesion.
Label glass slide(s) and have ready near to the patient.
Immobilise the lesion between thumb and forenger of one hand.
Introduce needle (usually with syringe attached) into lesion with the other hand.
When the needle tip is at the edge of the lesion, apply negative pressure by
pulling on the syringe and continue to enter the lesion.
Make multiple rapid passes through the lesion, varying the angles if necessary.
Stop sampling if blood is seen in the syringe hub as blood reduces the quality of
the sample.
Release the negative pressure while the needle is still in the lesion.
Withdraw the needle and expel the cellular material from the needle onto the
labelled glass slide(s) for pathological evaluation.
7.6 Eastern Cooperative Oncology Group (ECOG) performance
status scale
0 Fully active, able to carry on all usual activities without restriction and
without the aid of analgesics
1 Restricted in strenuous activity but ambulatory and able to carry out light
work or pursue a sedentary occupation.This group also contains patients
who are fully active, as in grade 0, but only with the aid of analgesics
2 Ambulatory and capable of all self-care but unable to work. Up and
about more than 50% of waking hours
3 Capable of only limited self-care, conned to bed or chair more than
50% of waking hours
4 Completely disabled, unable to carry out any self-care and conned
totally to bed or chair
7.7 Cancer in old age
Incidence: around 50% of cancers occur in the 15% of the population aged
over 65 years.
Screening: women over 65 in the UK are not invited to breast cancer
screening but can request it. Uptake is low despite increasing incidence with
age.
Presentation: may be later for some cancers. When symptoms are non-
specic, patients (and their doctors) may initially attribute them to age alone.
Life expectancy: an 80-year-old woman can expect to live 8 years, so cancer
may still shorten life and an active approach remains appropriate.
Prognosis: histology, stage at presentation and observation for a brief period
are better guides to outcome than age.
Rate of progression: malignancy may have a more indolent course. This is
poorly understood but may be due to reduced effectiveness of angiogenesis
with age, inhibiting the development of metastases.
Response to treatment: equivalent to that in younger people – well
documented for a range of cancers and for surgery, radiotherapy, cytotoxic
chemotherapy and hormonal therapy.
Toxicity of treatment: may be greater due to subclinically reduced baseline
hepatic, renal and bone marrow function and reduced baseline energy.
Treatment selection: chronological age is of minor importance compared to
comorbid illness and patient choice. Although older patients can be treated
effectively and safely, aggressive intervention is not appropriate for all. Symptom
control may be all that is possible or desired by the patient.
166.
138 OncOlOgy
increasingcomorbidity and declining physiological reserve in particular
placing patients at increased risk of treatment toxicity. Geriatric assess-
ment tools to assess patient tness and toxicity risks have been developed.
Multidisciplinary teams
The multidisciplinary team (MDT) is well established in oncology and
meets on a regular basis to discuss patient progress and provide a
forum for patient-centred, interdisciplinary communication to coor-
dinate care and decision-making. It is a platform on which individual
clinicians can discuss complex cases or situations and draw on the
collective experience of the team membership to decide on the best
approach for an individual patient. This can be particularly important
when discussing patients with a rare condition or in an unusual situation.
As a minimum, most MDTs include a radiologist, pathologist, specialist
physician, specialist surgeon, radiation oncologist, medical oncologist,
nurse specialist, auditor and scheduler. Additional staff may be relevant
to different MDTs, e.g. dermatologists in skin cancer MDTs, dieticians
in upper gastrointestinal cancer MDTs, stoma nurses in colorectal can-
cer MDTs, palliative care doctors in pancreatic cancer or cancer of
unknown primary (CUP) MDTs. Specic roles of the MDT are outlined
in Box 7.8
Acute oncology
Cancer centres are usually based in cities and have busy radiother-
apy and systemic anti-cancer therapy units containing specialist
equipment and teams of highly trained physicians, surgeons, nurses,
pharmacists, radiographers, physicists and support workers amongst
others. Most patients with cancer undergo investigations, assess-
ments and treatment as outpatients. However, patients may present
acutely to any specialty in any hospital with a new cancer, cancer-re-
lated symptoms, emergency complications of cancer or with treat-
ment-related toxicities.
Acute presentation of new cancer
Despite cancer screening initiatives, most patients still present with symp-
tomatic disease. They may have symptoms related to the local effect of
a primary tumour or metastatic deposit, such as nding a lump, dyspha-
gia or persistent pain. These are sometimes called ‘red ag’ symptoms
of cancer (Box 7.9). Patients may present with constitutional symptoms
such as unexplained weight loss, fatigue or fevers. Paraneoplastic syn-
dromes due to the production of biologically active hormones by the
tumour, or as the result of an immune response to the tumour, may also
be the presenting symptom of new cancer (Box 7.10). The pattern of
symptoms, detailed clinical history and examination will most often direct
the initial investigations and early referral to the appropriate specialist
multidisciplinary team.
Cancer of unknown primary (CUP)
When a patient is found to have advanced or metastatic cancer but
history, examination and imaging is unable to dene a primary cancer
site, an assessment of patient tness, disease extent, comorbidities and
patient wishes should be made. Selected investigations should be car-
ried out (Box7.11). However, a primary cancer site does not need to
be ‘hunted down’ in all patients and investigations should be limited to
those that will help management. Specialist investigations (e.g. endos-
copies) are only indicated when there is radiological suspicion or if there
are localising symptoms.
In some cases, such as those with clinical suspicion of germ cell
tumours, high-grade lymphoma or small cell cancer, or those present-
ing with oncological emergencies, urgent referral directly to oncology or
haematology is essential to allow rapid initiation of anti-cancer therapy,
sometimes before investigations are completed (Fig. 7.6).
If patients are t to undergo investigations and t enough to be consid-
ered for treatment then a core biopsy of the most accessible metastatic
7.8 Specic roles of the cancer multidisciplinary
team (MDT)
Agreeing on cancer type and nal stage.
Deciding on the appropriate primary treatment modality (most commonly
surgery but the use of neoadjuvant systemic anti-cancer therapy and/or
radiotherapy before surgery is increasing).
Arranging review by the appropriate team to coordinate management.
Planning surveillance strategies.
Diagnosing cancer recurrence or metastases.
Ensuring the appropriate transition from treatment with curative intent to that of
palliation.
Promoting recruitment into clinical trials.
Agreeing on operational policies to deliver high-quality care to patients.
Planning and reviewing audit data to ensure delivery of quality care to patients.
Identifying late effects of cancer treatments.
7.9 Red ag symptoms of malignancy
Symptom Typical site or possible tumour
Lump Breast, lymph node (any site), testicle
Skin abnormality Melanoma, basal cell carcinoma
Bleeding Stomach, colon, lung, endometrium,
bladder, kidney
Dysphagia, odynophagia Oesophagus, bronchus, gastric, head
and neck
Change in bowel habit Colon, rectum, ovary
Cough, hoarseness, stridor Lung, head and neck, thyroid
Bone pain or fracture Bone (primary sarcoma, secondary
metastasis from breast, prostate,
bronchus, thyroid, kidney)
Abdominal swelling (ascites) Ovary, stomach, pancreas
Unexplained weight loss, anorexia Lung, gastrointestinal tract, CUP
Unexplained fatigue Any
(CUP = cancer of unknown primary)
7.10 Paraneoplastic syndromes
Feature Common cancer site associations
Lambert–Eaton myasthenic
syndrome
Small cell lung cancer
Subacute cerebellar
degeneration
Small cell lung cancer, ovarian cancer
Encephalomyelitis Small cell lung cancer
Retinopathy Non-small cell lung cancer
Dermatomyositis/polymyositis Gastric, lung cancer
Acanthosis nigricans Gastric, oesophageal cancer
Vitiligo Melanoma
Pruritus Lymphoma, leukaemia, central nervous
system tumours
Pemphigus Lymphoma, Kaposi’s sarcoma, thymic
Dermatitis herpetiformis Enteropathy-associated T-cell lymphoma
(EATL)
167.
Aute ooo 139
7
Metastatic malignant disease of undened primary
origin (MUO)
Patients who present as emergencies to hospital with symptomatic new
metastatic cancer often have a short prognosis. Features associated
with poor prognosis, irrespective of subsequent diagnosis, include: poor
performance status (PS 3–4), requirement for hospital admission, bulky
or multi-organ metastatic disease, signicant comorbidity, raised LDH,
raised inammatory markers and low albumin. Further investigations,
including biopsy, may not be warranted and honest conversations about
palliative care and end-of-life planning are often preferable. Appropriate
analgesia, palliative radiotherapy (for bleeding or pain) and interventional/
surgical palliation can all help improve symptoms.
Oncological emergencies
Oncological emergencies are a group of potentially life-threatening con-
ditions that occur as a direct result of cancer or its treatment. Positive
outcomes depend upon prompt recognition and implementation of
appropriate management.
Malignant spinal cord compression
Malignant spinal cord compression complicates 5% of cancers and is
most common in myeloma, prostate, breast and lung cancers that most
frequently involve bone. Malignant spinal cord compression often results
from posterior extension of a vertebral body mass but intrathecal spinal
cord metastases can cause similar signs and symptoms. The thoracic
region is most commonly affected (Fig. 7.7).
Clinical features
The earliest sign is back pain, often rapidly worsening or in a ‘band’,
most marked on coughing and lying at. Subsequently, sensory
changes develop in dermatomes below the level of compression, and
motor weakness distal to the block occurs. Finally, sphincter distur-
bance, causing urinary retention and bowel incontinence, is observed.
7.11 Initial diagnostic tests in patients presenting with
carcinoma of unknown primary
Detailed medical history and examination, including breast, nodal areas, skin,
genital, rectal and pelvic regions
Full blood count, urea and electrolytes, renal function, liver function tests,
albumin, calcium, urinalysis, lactate dehydrogenase, CRP
Myeloma screen (if lytic bone lesions)
CT scan of chest, abdomen and pelvis
CT neck if thyroid lump or neck nodes
Histological examination by biopsy, with immunohistochemistry and/or
molecular pathology if required
Upper or lower gastrointestinal endoscopy if gastrointestinal symptoms
Mammography and/or breast MRI for women with axillary nodes or
adenocarcinoma
Breast MRI for women with adenocarcinoma limited to axillary nodes
Testicular ultrasound (if midline disease in young man or clinical features
suggest germ cell tumour)
Naso-endoscopy for squamous neck nodes
PET scan for single-site disease or squamous neck nodes with no primary on
endoscopic evaluation
Tumour markers: prostate-specic antigen (PSA) in men with bone lesions,
cancer antigen 125 (CA-125) in women with peritoneal malignancy or ascites,
α-fetoprotein (AFP) in liver-limited disease, α-fetoprotein (AFP) and human
chorionic gonadotrophin (hCG) in midline disease in young men
(CRP = C-reactive protein; CT = computed tomography; MRI = magnetic resonance imaging;
PET = positron emission tomography)
Fig. 7.6 Young male patient presenting with a bulky midline tumour and
elevated hCG and AFP suggestive of a non-seminomatous testicular cancer.
P
lesion is usually the quickest way of obtaining tissue for pathological eval-
uation of cancer type. Review of tumour morphology, immunohistochem-
istry and molecular pathology will usually be sufcient to plan appropriate
treatment. Patients with clinico-pathological features suggestive of a
particular cancer should be identied early and referred promptly to the
appropriate multidisciplinary/oncology team for consideration of further
investigations and treatment. Where the primary site remains unclear fol-
lowing comprehensive investigations, a patient is said to have a cancer
of unknown primary (CUP).
Whilst it is crucial to identify patients with highly treatable CUPs
(Box 7.12), some of whom can live for years (see Fig. 7.5), or even
be cured by treatment, most CUPs are poorly differentiated cancers
or adenocarcinomas with liver and/or multi-site metastases. These
cancers often behave aggressively so early discussion with oncology,
within the context of a specialist MDT, is essential to streamline investi-
gations and assess tness for anti-cancer treatment. Whilst responses
to generic systemic anti-cancer treatment are modest, gene expression
tests or molecular tumour proling are increasingly providing greater
information about likely tissue of origin or possible treatment targets
which may ultimately lead to improved understanding of CUP and
better outcomes.
7.12 Cancer of unknown primary – favourable
clinicopathological features
Clinicopathological features Management
Women with serous papillary
adenocarcinoma of the peritoneal
cavity
Treat like ovarian cancer
Women with isolated adenocarcinoma
of axillary lymph nodes
Treat like breast cancer
Men with poorly differentiated
carcinoma with midline distribution
Treat like germ cell cancer
Poorly differentiated neuro-endocrine
carcinoma of unknown origin
Treat like small cell cancer
Well-differentiated neuro-endocrine
tumour (NET)
Treat with somatostatin analogues
or as per NETs of known primary
sites
Squamous cell carcinoma of non-
supraclavicular cervical lymph nodes
Treat like head and neck squamous
cell cancer
Adenocarcinoma with a colorectal-IHC
prole (CK20+
, CK7 , CDX2+
)
Treat like colon cancer
Men with blastic bone metastases
or elevated prostate-specic antigen
(adenocarcinoma)
Treat like prostate cancer
Isolated inguinal adenopathy
squamous carcinoma
Treat like anogenital squamous
cancer
Patients with one small, potentially
resectable tumour
Assess for surgical resection
168.
140 OncOlOgy
Involvementof the lumbar spine may cause conus medullaris or cauda
equina compression (Box 7.13). Physical examination ndings consist-
ent with an upper motor neuron lesion, but lower motor neuron nd-
ings, may predominate early on or in cases of nerve root or cauda
equina compression.
Management
Malignant spinal cord compression is a medical emergency and
should be treated with analgesia and high-dose glucocorticoid therapy
(Box 7.14). Neurosurgical intervention produces superior outcomes and
survival compared to radiotherapy alone, and should be considered rst
for all patients. It is the preferred treatment if there is a single site of
disease, low volume metastatic disease, if a biopsy is required to make
a diagnosis or if it is a treatable cancer (e.g. lymphoma, breast cancer).
Radiotherapy is used for the remaining patients and selected tumour
types when the cancer is likely to be radiosensitive. The prognosis varies
considerably, depending on tumour type, but the degree of neurological
dysfunction at presentation is the strongest predictor of outcome, irre-
spective of the underlying diagnosis. Mobility can be preserved in more
Fig. 7.7 MRI images of patient presenting with back pain and lower limb neurological disturbance.
arising from thoracic vertebra compressing spinal cord.
B
A
7.13 Comparison of features of neurological decit
Clinical feature Spinal cord Conus medullaris Cauda equina
Weakness Symmetrical and profound Symmetrical and variable Asymmetrical, may be mild
Reexes Increased (or absent) knee and ankle
reexes with extensor plantar reex
Increased knee reex, decreased ankle
reex, extensor plantar reex
Decreased knee and ankle reexes with
exor plantar reex
Sensory loss Symmetrical, sensory level Symmetrical, saddle distribution Asymmetrical, radicular pattern
Sphincters Late loss Early loss Often spared
Progression Rapid Variable Variable
7.14 Management of suspected spinal cord compression
Conrm diagnosis with urgent MRI scan of whole spine
Administer high-dose glucocorticoids:
Dexamethasone 16mg IV or oral stat
Dexamethasone 8mg twice daily orally
Ensure adequate analgesia
Refer for surgical decompression or urgent radiotherapy
(IV = intravenous; MRI – magnetic resonance imaging)
169.
Aute ooo 141
7
than 80% of patients who are ambulatory at presentation, but neurologi-
cal function is seldom regained in patients with established decits such
as paraplegia.
Superior vena cava obstruction
Superior vena cava obstruction (SVCO) is a common complication of
cancer that can occur through extrinsic compression or intravascu-
lar blockage. The most common causes of extrinsic compression are
lung cancer, lymphoma and metastatic tumours (Fig. 7.8). Patients with
cancer can also develop SVCO due to intravascular blockage in asso-
ciation with a central catheter or thromboembolism secondary to the
tumour.
Clinical features
The classical presentation is with breathlessness, oedema of the arms
and face, distended neck and arm veins and dusky skin coloration over
the chest, arms and face. Collateral vessels may develop over a period of
weeks and the ow of blood in the collaterals helps to conrm the diag-
nosis. Headache secondary to cerebral oedema arising from the back-
ow pressure may also occur and tends to be aggravated by bending
forwards, stooping or lying down. The severity of symptoms is related to
the rate of obstruction and the development of a venous collateral circu-
lation. Accordingly, symptoms may develop rapidly or gradually. Clinical
features are summarised in Box 7.15
Investigations and management
The investigation of choice is a CT scan of the thorax to conrm the diag-
nosis and distinguish between extra- and intravascular causes.
A biopsy should be obtained when the tumour type is unknown
because tumour type has a major inuence on treatment. CT of the
head may be indicated if cerebral oedema is suspected. Tumours that
are exquisitely sensitive to cytotoxic chemotherapy, such as germ cell
tumours and lymphoma, can be treated with cytotoxic chemotherapy
alone. For most other patients with symptomatic SVCO, SVC stenting
provides the best outcomes although mediastinal radiotherapy may also
be considered. Where possible, these measures should be followed
by treatment of the primary tumour, as long-term outcome is strongly
dependent on the prognosis of the underlying cancer.
Neutropenic fever
Neutropenia is a common complication of malignancy. It is usually sec-
ondary to cytotoxic chemotherapy but may occur with radiotherapy, if
large amounts of bone marrow are irradiated, and some targeted therapy
agents; it may also be a component of pancytopenia due to malignant
inltration of the bone marrow. After cytotoxic chemotherapy neutropenic
fever is most commonly dened as a single oral temperature of ≥38.3°C
or a temperature of ≥38°C sustained for over 1hour in a patient with a
neutrophil count of <0.5 × 109
/L (or <1.0 × 109
/L if the nadir is anticipated
to drop to <0.5 × 109
/L in the next 48hours). The risk of sepsis is greater
with profound neutropenia (neutrophil count <0.1 × 109
/L), prolonged neu-
tropenia (<0.5 neutrophils for ≥7 days) or a rapid rate of decline in neu-
trophils as well as the presence of other risk factors, such as intravenous
cannulae or urinary catheters. Neutropenic fever is an emergency in can-
cer patients as, if left untreated, it can result in sepsis with a high mortality
rate.
Clinical features
The typical presentation is with high fever, and affected patients may
feel non-specically unwell. If patients have been taking paracetamol or
steroids then fever or symptoms may be masked. Examination is usually
unhelpful in dening a primary source of the infection. Hypotension is
an adverse prognostic feature and may progress to systemic circulatory
shutdown and organ failure.
Investigations and management
An infection screen should be performed, including blood cultures (both
peripheral and from central lines), urine culture, chest X-ray and swabs
for culture (throat, central line, wound). High-dose intravenous antibiotics
should be commenced (ideally within 1hour of admission) for all febrile
patients on systemic anti-cancer therapy, without awaiting test results.
The standard approach is to commence empirical broad-spectrum anti-
biotics according to local hospital policies agreed with microbiologists
and based on local antibiotic resistance patterns. Depending on the
patient’s MASCC risk index score (Box 7.16) and likely organisms this
Fig. 7.8 Computed tomography (CT) image of extrinsic compression of the
superior vena cava by a small cell lung cancer (arrow) performed as part of
investigations for an acute presentation of cancer.
7.15 Common symptoms and physical ndings in superior
vena cava obstruction*
Symptoms
Dyspnoea (63%)
Facial swelling and head fullness (50%)
Cough (24%)
Arm swelling (18%)
Chest pain (15%)
Dysphagia (9%)
Physical ndings
Venous distension of neck (66%)
Venous distension of chest wall (54%)
Elevated, non-pulsatile jugular venous
pulse (66%)
Facial oedema (46%)
Cyanosis (20%)
Plethora of face (19%)
Oedema of arms (14%)
*Percentage of patients affected.
7.16 MASCC* risk index score for neutropenic fever
Characteristic Score
Burden of current illness
No or mild symptoms (normal function) 5
Moderate symptoms (uncomfortable or inuences daily
activities)
3
Severe symptoms (signicant discomfort or limits daily activities) 0
No hypotension (systolic BP > 90mmHg) 5
No chronic obstructive pulmonary disease 4
Solid tumour or lymphoma with no previous fungal infection 4
No dehydration requiring parenteral uids 3
Outpatient at onset of fever 3
Age <60 years 2
Score ≥21 = low risk febrile neutropenia
Score <21 = high risk febrile neutropenia
*Multinational Association of Supportive Care in Cancer.
170.
142 OncOlOgy
maybe either monotherapy (e.g. piperacillin–tazobactam or meropenem)
or combination therapy (e.g. with the addition of gentamicin if high risk
on MASCC score, metronidazole if anaerobic infection is suspected, or
vancomycin/teicoplanin where Gram-positive infection is suspected).
Antibiotics should be adjusted according to culture results, although
these are often negative. If there is no clinical improvement after
36–48hours antibiotics should be reviewed with microbiological advice,
and antifungal cover should be considered (e.g. uconazole or liposomal
amphotericin B). Granulocyte–colony-stimulating factor (G–CSF) can be
used to hasten neutrophil recovery in some patients with febrile neutro-
penia, in line with local guidelines. Other supportive therapy, including
intravenous uids, oxygen, inotrope therapy, ventilation or haemoltra-
tion, may be required.
Hypercalcaemia of malignancy
Hypercalcaemia is the most common metabolic disorder in patients with
cancer and has a prevalence of up to 20% in cancer patients. The inci-
dence is highest in myeloma and breast cancer (approximately 40%),
intermediate in non-small cell lung cancer, and uncommon in colon,
prostate and small cell lung carcinomas. It is most commonly due to
over-production of PTHrP (80%), which binds to the PTH receptor and
elevates serum calcium by stimulating osteoclastic bone resorption and
increasing renal tubular reabsorption of calcium. Direct invasion of bone
by metastases accounts for around 20% of cases while other mecha-
nisms, such as ectopic PTH secretion, are rare.
Clinical features
The symptoms of hypercalcaemia are often non-specic and may mimic
those of the underlying malignancy. They include drowsiness, delirium,
nausea and vomiting, constipation, polyuria, polydipsia and dehydration.
Investigations and management
The diagnosis is made by measuring serum total calcium and adjust-
ing for albumin. It is especially important to correct for albumin in can-
cer because hypoalbuminaemia is common and total calcium values
under-estimate the level of ionised calcium. The principles of manage-
ment are outlined in Box 7.17
Patients should initially be treated with intravenous 0.9% saline to
improve renal function and increase urinary calcium excretion. This
alone often results in clinical improvement. Concurrently, intravenous
bisphosphonates should be given to inhibit bone resorption. Calcitonin
acts rapidly to increase calcium excretion and to reduce bone resorp-
tion, and can be combined with uid and bisphosphonate therapy for
the rst 24–48hours in patients with life-threatening hypercalcaemia.
Bisphosphonates will usually reduce the serum calcium levels to nor-
mal within 5 days, but, if not, treatment can be repeated. The dura-
tion of action is up to 4 weeks and repeated therapy can be given at
3–4-weekly intervals in the outpatient department. Hypercalcaemia is
frequently a sign of tumour progression and the patient requires fur-
ther investigation to establish disease status and review the anti-cancer
treatment strategy.
Immune-related adverse events
Immunotherapy has revolutionised the treatment of many cancers
(see ‘Immunotherapy’ below). Checkpoint inhibitor immunotherapy, in
particular, is now widely used in the treatment of many different cancers.
Side-effects of checkpoint inhibitor immunotherapy agents occur when
the immune system is stimulated to attack healthy cells and tissues in
the body. These immune-related adverse events (IRAEs) are increasingly
being recognized as oncological emergencies.
Clinical features
Almost any body system may be affected, but inammation of the colon,
endocrine organs (thyroid, adrenal and pituitary glands) lungs, liver,
skin and nervous system are most frequently seen (see p. 129). IRAEs
may occur at any point in a patient’s treatment, or even after they have
stopped receiving the checkpoint inhibitor immunotherapy agent.
Investigations and management
Investigation and management of IRAEs requires a multidiscipli-
nary approach including oncologists and relevant system specialists.
Specialist investigations may be required depending on the body system
affected. The focus of treatment is on dampening the immune response,
often using high-dose steroids. Additional immunosuppressant therapies
are sometimes required. In the case of endocrine organ inammation,
replacement of decient hormones is also important. In patients who
experience severe IRAEs, checkpoint inhibitor immunotherapy treatment
will usually be permanently discontinued. In other cases, and in patients
with endocrine IRAEs who commence replacement hormone therapy,
it may be possible to reintroduce checkpoint inhibitor immunotherapy
treatment.
Tumour lysis syndrome
The acute destruction of a large number of cells can be associated with
metabolic sequelae and is called tumour lysis syndrome. It is usually
related to bulky, chemosensitive disease, including lymphoma, leukae-
mia and germ cell tumours. More rarely, it can occur spontaneously.
Clinical features
Cellular destruction results in the release of potassium, phosphate,
nucleic acids and purines that can cause transient hypocalcaemia,
hyperphosphataemia, hyperuricaemia and hyperkalaemia. This can lead
to acute impairment of renal function and the precipitation of uric acid
crystals in the renal tubular system. These can manifest with symptoms
associated with multiple underlying electrolyte abnormalities, including
fatigue, nausea, vomiting, cardiac arrhythmia, heart failure, syncope,
tetany, seizures and sudden death.
Investigations and management
Serum biochemistry should be monitored regularly for 48–72 hours
after treatment in patients at risk. Elevated serum potassium may
be the earliest biochemical marker but pre-treatment serum lactate
dehydrogenase (LDH) correlates with tumour bulk and may indicate
increased risk. Good hydration and urine output should be maintained
throughout treatment administration. Prophylaxis with allopurinol
should be considered and recombinant urate oxidase (rasburicase)
can be used to reduce uric acid levels when other treatments fail.
Adequate hydration is vital, as it has a dilution effect on the extracel-
lular uid, improving electrolyte imbalance, and increases circulating
volume, improving ltration in the kidneys. In high-risk patients, hydra-
tion and rasburicase should be commenced 24hours before the start
of treatment. If normal treatment methods fail to correct the problems,
haemodialysis should be considered at an early stage to prevent pro-
gression to irreversibility.
Other acute presentations in oncology
Venous thromboembolism
Venous thromboembolism (VTE) is the second leading cause of death
amongst patients with cancer. Malignancy is a strong risk factor for
7.17 Medical management of severe hypercalcaemia
IV 0.9% saline 2–4L/day
Zoledronic acid 4mg IV or pamidronate 60–90mg IV
For patients with severe, symptomatic hypercalcaemia that is refractory to
zoledronic acid, denosumab (initial dose 60mg SC, with repeat dosing based on
response) is an alternative option
(IV = intravenous; SC = subcutaneous)
171.
Aute ooo 143
7
VTE and may be the rst presenting feature of an underlying cancer.
Several mechanisms lead to a prothrombotic state in cancer. This
includes the ability of cancer cells to produce procoagulant/brinolytic
substances and inammatory cytokines and the physical interaction
between tumour cells and blood (monocytes, neutrophils, platelets) or
endothelial cells. Anti-cancer therapy (i.e. surgery, cytotoxic chemo-
therapy, hormone therapy and radiotherapy) and any in-dwelling
access devices (i.e. central venous catheters) further increase the risk.
As in patients without cancer, the management of cancer-associated
VTE primarily involves anticoagulation therapy. However, the choice of
therapy must take into account bleeding risk and possible interactions
with anti-cancer therapies and their side-effects, such as thrombocy-
topenia. In patients with metastatic cancer, anticoagulation therapy will
often be lifelong.
Ectopic hormone production
Some cancers are associated with metabolic abnormalities due to
ectopic production of hormones by tumour cells, including insulin,
ACTH, vasopressin (antidiuretic hormone, ADH), broblast growth fac-
tor (FGF)-23, erythropoietin and parathyroid hormone-related protein
(PTHrP). This can result in a wide variety of presentations, as summa-
rised in Box 7.18
Neurological paraneoplastic syndromes
These form a group of conditions associated with cancer that are
thought to be due to an immunological response to the tumour that
results in damage to the nervous system or muscle (see Box 7.10).
The cancers most commonly implicated are those of the lung, pan-
creas, breast, prostate, ovary and lymphoma. Many are associated
with specic detectable immune biomarkers such as antibodies to
pre-synaptic calcium channels (Lambert–Eaton syndrome), anti-Hu
antibodies (encephalomyelitis) and anti-Yo or anti-Tr antibodies (cere-
bellar degeneration). However, these are not always specic and neg-
ative results do not exclude the diagnosis. The management of these
syndromes is multidisciplinary and includes treatment of the underly-
ing cancer itself and treatment of the syndrome primarily with immu-
nosuppressive agents.
Cutaneous manifestations of cancer
Cancers can present with skin manifestations that are not due to metas-
tases (see Box 7.10). The clinical features and management of these skin
conditions are discussed in Chapter 27
Symptoms from locally advanced cancer or
metastatic sites
Metastatic disease is the major cause of death in cancer patients and the
principal cause of morbidity. For the majority of patients with metastatic
disease the goal of treatment is to control cancer, maintain quality of life,
treat symptoms and prolong life (i.e. ‘palliative treatment’). Patients of PS
3–4 (Box 7.6) or with low albumin plus high inammatory markers often
have a limited prognosis irrespective of anti-cancer treatments. Systemic
anti-cancer therapies have resulted in improved survival for many can-
cers so that some patients live a good quality life for many years with
metastatic cancer. Treatment of a solitary metastasis or highly treatable
cancers (e.g. germ cell) can be curative.
Brain metastases
Brain metastases occur in 10%–30% of adults and 6%–10% of chil-
dren with cancer and are an increasingly important cause of morbidity.
Cancers of lung, breast, melanoma and gastrointestinal tract most com-
monly metastasise to the brain. Most involve the brain parenchyma but
can also affect the meninges, cranial nerves, the blood vessels and other
intracranial structures. In cases of solitary metastasis to the brain, the use
of surgery followed by adjuvant radiotherapy, or alternatively stereotactic
radiotherapy, has been shown to increase survival in patients whose dis-
ease is otherwise controlled. Outcomes for patients with more advanced
brain metastases depends on the primary cancer, extent of extracranial
disease and what systemic treatment options are available. For patients
with advanced untreatable cancer and multiple brain metastases, prog-
nosis is often short. Glucocorticoids can improve symptoms, particularly
where there is evidence of peri-lesional oedema. In treatable cancers
whole brain radiotherapy can allow steroid dose to be reduced whilst
systemic therapy is used to treat the remaining cancer. With improved
systemic therapies, including targeted therapies and immunotherapy,
some patients are now living for several years with brain metastases.
Clinical features
Presentation is with headaches and nausea (40%–50%), focal neuro-
logical dysfunction (20%–40%), cognitive dysfunction (35%), seizures
(10%–20%) and papilloedema (<10%).
Investigations and management
The diagnosis can be conrmed by CT or contrast-enhanced MRI.
Treatment options include high-dose glucocorticoids for tumour-associated
oedema (dexamethasone 4–12mg daily depending on amount of oedema),
anticonvulsants for seizures, whole-brain radiotherapy and systemic
anti-cancer therapy. Surgery may be considered for single sites of disease
and can be curative; stereotactic radiotherapy may also be considered for
solitary site involvement or where surgery is more difcult or not possible.
Lung metastases
Lung metastases are common in breast cancer, colon cancer, renal can-
cer, sarcoma and tumours of the head and neck. The presentation is usu-
ally with a lesion on chest X-ray or CT. Solitary lesions require investigation,
as single metastases can be difcult to distinguish from a primary lung
tumour. Patients with two or more pulmonary nodules can be assumed
to have metastases. The approach to treatment depends on the extent
of disease in the lung and elsewhere. For solitary lesions, surgery should
be considered, with a generous wedge resection, or radiofrequency abla-
tion if available. Radiotherapy and systemic anti-cancer therapies can be
used, dependent on the underlying primary cancer diagnosis (Fig. 7.9).
Liver metastases
Metastatic cancer in the liver can represent the sole or life-limiting com-
ponent of disease for many with colorectal cancer, ocular melanoma,
7.18 Ectopic hormone production by tumours
Hormone Consequence Tumours
ACTH Cushing’s syndrome SCLC
Erythropoietin Polycythaemia Kidney, HCC,
cerebellar
haemangioblastoma,
uterine broids
FGF-23 Hypophosphataemic
osteomalacia
Mesenchymal tumours
PTHrP Hypercalcaemia NSCLC (squamous
cell), breast, kidney
Vasopressin (ADH) Hyponatraemia SCLC
(ACTH = adrenocorticotrophic hormone; ADH = antidiuretic hormone; FGF = broblast growth
factor; HCC = hepatocellular carcinoma; NSCLC = non-small cell lung cancer; PTHrP =
parathyroid hormone-related protein; SCLC = small cell lung cancer)
172.
144 OncOlOgy
neuro-endocrinetumours (NETs) and, less commonly, other tumour
types. The most common clinical presentations are with right upper
quadrant pain due to stretching of the liver capsule, jaundice, deranged
liver function tests or an abnormality detected on imaging. In selected
cases, resection of the metastasis can be contemplated. In colorectal
cancer, successful resection of metastases improves 5-year survival from
3% to 30%–40%. Other techniques, such as chemoembolisation, radio-
frequency ablation or microwave ablation, can also be used, provided
the number and size of metastases remain small. If these are not feasible,
symptoms may respond to systemic anti-cancer therapy (Fig. 7.5).
Bone metastases
Bone is the third most common organ involved by metastasis, after
lung and liver. Bone metastases are a major clinical problem in patients
with myeloma and breast or prostate cancers, but other tumours that
commonly metastasise to bone include those of the kidney and thyroid.
Bone metastases are increasingly seen in other tumour types that do not
classically target bone, due to effective anti-cancer treatments prolong-
ing survival of patients with many cancers. Accordingly, effective man-
agement of bony metastases has become a focus in the treatment of
patients with many incurable cancers.
Clinical features
The main presentations are with pain, pathological fractures, spinal
cord compression (see above) and hypercalcaemia. Pain tends to be
progressive and worst at night, and may be partially relieved by activ-
ity, but subsequently becomes more constant in nature and is exacer-
bated by movement. Most pathological fractures occur in metastatic
breast cancer (53%); other tumour types associated with fracture
include the kidney (11%), lung (8%), thyroid (5%), lymphoma (5%) and
prostate (3%).
Investigations and management
The most sensitive way of detecting bone metastases is by isotope bone
scan. This can have false-positive results in healing bone, particularly as
a are response following treatment and false-negative results occur in
multiple myeloma due to suppression of osteoblast activity. Plain X-ray
lms or MRI scans are therefore preferred for any sites of bone pain,
as lytic lesions may not be detected by a bone scan. In patients with a
single lesion, it is especially important to perform a biopsy to obtain a
tissue diagnosis, since primary bone tumours may look very similar to
metastases on X-ray. The main goals of management are:
pain relief
preservation and restoration of function
skeletal stabilisation
local tumour control (e.g. relief of tumour impingement on normal
structure).
Surgical intervention may be warranted where there is evidence of
skeletal instability (e.g. anterior or posterior spinal column fracture) or
an impending fracture (e.g. a large lytic lesion on a weight-bearing bone
with more than 50% cortical involvement). Intravenous bisphosphonates
(pamidronate, zoledronic acid or denosumab) are widely used for bone
metastases and are effective at improving pain and in reducing further
skeletal related events, such as fractures and hypercalcaemia. In certain
types of cancer, such as breast and prostate, hormonal therapy may be
effective. Radiotherapy, in the form of external beam therapy or systemic
radionuclides (strontium treatment), can also help pain. In some settings
(e.g. breast carcinoma), systemic anti-cancer therapy may be used in the
management of bony metastases.
Malignant pleural effusion
This is a common complication of cancer and 40% of all pleural effusions
are due to malignancy. The most common causes are lung and breast
cancers, and the presence of an effusion indicates incurable disease.
The presentation may be with dyspnoea, cough or chest discomfort,
which can be dull or pleuritic in nature. Diagnosis and management of
pleural effusion is discussed on page 494.
Investigations and management
Pleural aspirate is the key investigation and may show the presence of
malignant cells. Malignant effusions are commonly blood-stained and
are exudates with a raised uid to serum LDH ratio (>0.6) and a raised
uid to serum protein ratio (>0.5). Treatment should focus on palliation of
symptoms and be tailored to the patient's physical condition, treatment
options and prognosis. Aspiration alone may be an appropriate treat-
ment in frail patients with a limited life expectancy. Those who present
with malignant pleural effusion as the initial manifestation of breast can-
cer, small cell lung cancer, germ cell tumours or lymphoma should have
the uid aspirated and should be given systemic anti-cancer therapy to
try to treat disease in the pleural space. Treatment options for patients
with recurrent pleural effusions include pleurodesis, implanted drainage
catheters, pleurectomy and pleuroperitoneal shunt.
Other common symptoms
Other symptoms that commonly arise from metastatic cancer are:
gastrointestinal obstruction, malignant abdominal ascites, hydronephro-
sis and cancer cachexia.
Fig. 7.9
A B
R
173.
Therapeutis i ooo 145
7
Treatment-related toxicities
Whilst most anti-cancer therapies cause some side-effects, most
of these can be managed with supportive medicines at home. Some
patients will develop severe toxicities and an acute assessment will be
needed. Examination of the patient on anti-cancer treatment (p. 128)
should consider the type of anti-cancer treatment, duration since the
last treatment and other concurrent toxicities. Patients on anti-cancer
therapy can deteriorate quickly and prompt assessment and manage-
ment is required. There may be specic management protocols to help
manage treatment-related toxicities and advice should be sought from
the patient’s cancer centre or acute oncology team.
Therapeutics in oncology
Anti-cancer therapy may be used with either curative or palliative intent,
and this distinction inuences the approach to management of individual
patients. The goal of treatment should be recorded in the medical notes.
Curative therapy is given with the aim of achieving complete remis-
sion. Surgery to remove all macroscopic disease is most frequently
the primary curative intervention. However, in some circumstances
radiotherapy, systemic anti-cancer therapy or a combination of
these may be used with curative, or radical, intent.
Adjuvant therapy is additional therapy given after the primary
curative intervention to lower the risk of disease recurrence.
Radiotherapy and/or systemic anti-cancer therapy may be given
after surgery with the intention of eradicating any micrometastatic
disease that remains.
Neoadjuvant therapy is additional therapy given prior to the primary
curative intervention. Systemic anti-cancer therapy may be admin-
istered prior to planned surgery. The principal aim is to lower the
risk of disease recurrence. Any reduction in the volume of disease,
or ‘downstaging’, may also allow less extensive surgery, increase
the likelihood of successful debulking and improve subsequent
surgical morbidity. Direct evaluation of the surgical specimen allows
assessment of the effectiveness of neoadjuvant therapies, guiding
subsequent management and lending this approach to translational
research.
Palliative therapy is primarily used to treat patients with metastatic
disease, or where curative treatment is not possible. The goal is to
control cancer, with the aim of improving or maintaining quality of
life, treating and preventing symptoms and improving survival. The
choice of therapy depends on the clinical situation and a careful
evaluation of the risks and benets of the intervention.
Surgical treatment
Surgery has a pivotal role in the management of cancer. It is the main
curative management of most solid cancers. In early localised cases of
colorectal, breast and lung cancer, cure rates are high with surgery. There
is evidence that outcome is related to surgical expertise, and most multi-
disciplinary teams include surgeons experienced in the management of
a particular cancer. There are some cancers for which surgery is one of
two or more options for primary management, and the role of the MDT is
to recommend appropriate treatment for an individual patient. Examples
include prostate and transitional cell carcinoma of the bladder, in which
radiotherapy and surgery may be equally effective. Specialised surgical,
or interventional radiology techniques may also be employed with cura-
tive intent. Radiofrequency ablation, microwave ablation or cryotherapy
may be used to treat small renal cell carcinomas or hepatocellular carci-
nomas. Surgery has less of a role in lymphoma, high-grade neuro-endo-
crine tumours or small cell cancer, where systemic anti-cancer therapy is
the main treatment used.
Surgical procedures are often the quickest and most effective way
of palliating symptoms in patients with metastatic disease. Examples
include the treatment of faecal incontinence with a defunctioning
colostomy; xation of pathological fractures; decompression of spinal cord
compression; and the treatment of fungating skin lesions by ‘toilet’ sur-
gery. Debulking cytoreductive surgery may improve survival in some can-
cers, including renal cell carcinoma and ovarian cancer. In very selected
cases, such as patients with oligometastatic disease, resection of metas-
tases may improve survival and reduce the need for other therapies.
Radiotherapy
Radiotherapy (radiation therapy) involves treating the cancer with ionis-
ing radiation such as X-rays, gamma rays, electrons or charged atoms.
Ionising radiation kills cancer cells by two mechanisms; directly damag-
ing cancer DNA or indirectly by triggering the formation of very reactive
molecules (free radicals) that also damage cancer DNA.
Two main types of radiation therapy exist: external beam radiotherapy,
and brachytherapy (internal radiotherapy). External beam radiotherapy is
most commonly delivered by a linear accelerator, which produces elec-
tron or photon beams. As normal tissues can also be damaged by radio-
therapy, treatments are planned to ensure maximum exposure of the
tumour and minimal exposure of surrounding normal tissues. Improved
localisation of the target volume, or tumour, can be achieved by the
use of surgical clips at the site of resection and fusion of radiotherapy-
planning CT scans with diagnostic MRI or PET-CT scans. Treatment-
planning software controls the size and shape of the beam. Intensity
modulated radiotherapy (which allows for more homogeneous dose
distribution), and volumetric radiotherapy (in which the shape of the radi-
ation beam is designed to closely t the tumour), have largely replaced
conventional methods that use a limited number of square or rectangular
beams. Stereotactic radiotherapy is a highly targeted method to deliver
focused radiation beams from many different angles which converge
on the tumour to deliver high doses precisely. Proton therapy uses a
cyclotron to produce beams of high-energy protons, which deposit their
radiation dose by a means that allows further sparing of normal tissues.
Brachytherapy, or internal radiation therapy, involves the direct appli-
cation of a radioactive source onto or into a tumour. This allows the deliv-
ery of a high, localised dose of radiation. Brachytherapy is a common
treatment for cancers of the prostate, uterus and cervix. Most commonly,
an applicator device is used to deliver a radioactive source to the tumour
for a set time, typically 10–20minutes. In other cases, such as in prostate
cancer, small radioactive seeds may be permanently placed, releasing
radiation slowly over several months. Radioactive liquid treatments, such
as 223
radium for bone metastases from prostate cancer or 131
iodine for
thyroid cancers are other examples of internal radiation therapy.
Biological differences between normal and tumour tissues are
exploited to obtain therapeutic benet. Fundamental to this is fractiona-
tion, which entails delivering the radiation as a number of small doses on
a daily basis. This allows normal cells to recover from radiation damage
but recovery occurs to a lesser degree in malignant cells. Fractionation
regimens vary depending on the tumour being treated, the total radia-
tion dose to be delivered and the intent of treatment. Curative, or rad-
ical treatments typically deliver a higher overall dose in 20–35 fractions
over 4–7 weeks. For palliative treatments a smaller dose given over 1–10
fractions is usually adequate. Malignant tissues vary widely in their sen-
sitivity to radiotherapy. Germ cell tumours and lymphomas are extremely
radiosensitive, and relatively low doses are adequate for cure. However,
most other cancers require higher doses. Normal tissue also varies in its
radiosensitivity, with the central nervous system, small bowel and lung
being among the most sensitive.
The side-effects of radiotherapy depend on the site being treated, the
tissue’s radiosensitivity and the dose delivered. For example, skin reac-
tions are common with high-dose radical head and neck cancer treat-
ments, or proctitis and cystitis with treatment to the bladder or prostate.
These acute reactions typically settle within a few weeks after treatment.
Late effects of radiotherapy develop more than 6 weeks after treatment
and occur in 5%–10% of patients. Examples include brachial nerve
damage and subcutaneous brosis after breast cancer treatment. There
174.
146 OncOlOgy
Fig.7.10 Commonly used cytotoxic chemotherapy agents and their mechanisms of action.
Precursors
Purine
biosynthesis
DNA
Proteins
Antimetabolites
Alkylating
agents
Alkaloids
Topoisomerase
inhibitors
Antitumour
antibiotics Enzymes Microtubules
RNA
(transfer, messenger,
ribosomal)
Precursors
Ribonucleotides
Deoxyribonucleotides
Pyrimidine
biosynthesis
Antifolates
Fluoropyrimidines
Mitotic inhibitors
Microtubule
aggregators
Nucleoside analogues
Platinum agents
Nitrogen mustards
Topoisomerase I
inhibitors
Topoisomerase II
inhibitors
Anthracyclines
Bleomycin
Actinomycin D
1. Intercalate with DNA
and inhibit the
progression
of topoisomerase I
2. Induce DNA strand
breaks by oxidative
damage
3. Bind to DNA
to prevent
transcription
Antifolates Fluoropyrimidines Nucleoside
analogues
Nitrogen
mustards
Platinum
agents
Topoisomerase
I/II inhibitors
Mitotic
inhibitors
Microtubule
aggregators
Others –
acting on DNA
Mechanism
of action
Examples 1. Anthracyclines:
epirubicin, doxorubicin
2. Bleomycin
3. Actinomycin D
Inhibit
dihydrofolate
reductase and
so purine and
pyrimidine
synthesis
Inhibit thymidylate
synthase and so
pyrimidine synthesis
Incorporated
into DNA
during
synthesis,
leading to
irreparable
errors
Form covalent
cross-links between
DNA molecules by
adding an alkyl
group to DNA
Form inter-/
intra-strand
cross-links
between DNA
molecules,
not by adding
an alkyl group,
so considered
alkylating-like
agents
Prevent re-ligation
of DNA strands by
topoisomerase I/II
Bind tubulin,
blocking
microtubule
formation
Stabilise the
microtubule
polymer
preventing its
disassembly
Methotrexate 5-Fluorouracil,
capecitabine
Gemcitabine,
fludarabine
Cyclophosphamide,
ifosfamide
Cisplatin,
carboplatin,
oxaliplatin
Etoposide (II);
topotecan,
irinotecan (I)
Vinca alkaloids:
vincristine,
vinblastine,
vinorelbine
Taxanes:
paclitaxel,
docetaxel
is also a risk of inducing new cancer after radiotherapy, which varies
depending on the site treated and on whether the patient has had other
treatment such as cytotoxic chemotherapy.
Systemic anti-cancer therapy
Systemic anti-cancer therapy (SACT) is a collective term to describe the
growing number of differing drug therapies used to treat cancer. These
drugs reach throughout the body to treat cancer cells wherever they may
be. Increasingly, treatment is tailored to a patient’s particular cancer and
its molecular prole, allowing more personalised therapy.
Cytotoxic chemotherapy
Cytotoxic chemotherapy drugs work by interfering with the processes
involved in cell division. They are sub-classied by their mode of action
(Fig. 7.10). Cytotoxic chemotherapies have their greatest activity in prolif-
erating cells and this provides the rationale for their use in the treatment
175.
Therapeutis i ooo 147
7
of cancer. However, they are not specic for cancer cells and the side-
effects of treatment are largely a result of their antiproliferative actions in
normal tissues such as the bone marrow, skin and gut (p. 129). Other
organs, such as the heart, kidney and peripheral nervous system, may
also be affected by some cytotoxic drugs.
The choice of cytotoxic chemotherapy agent, or combination of
treatments, is determined by the cancer type. The dosing schedule is
determined by the choice of treatments and recovery of normal tissues,
usually the bone marrow. For most common cytotoxic chemotherapy
regimens the treatment is administered in cycles. A course of treatment
may constitute a pre-dened number of cycles, or may continue inde-
nitely until evidence of disease progression or until limiting side-effects.
Supportive therapy is used to enable patients to tolerate therapy and
achieve benet. Nausea and vomiting are common, but with modern
antiemetics, regimens such as the combination of dexamethasone and
highly selective 5-hydroxytryptamine (5-HT3
, serotonin) receptor antago-
nists such as ondansetron, most patients now receive cytotoxic chemo-
therapy without any signicant problems. Myelosuppression is common
to almost all cytotoxics and this not only limits the dose of drug but also
can cause life-threatening complications. The risk of neutropenia can
be reduced with the use of specic growth factors that accelerate the
repopulation of myeloid precursor cells. The most commonly employed
is G–CSF, which is widely used in conjunction with cytotoxic chemother-
apy regimens that induce a high rate of neutropenia.
Hormone therapy
Hormones are important cell-signalling molecules and, in some cancers,
may be key drivers of tumour growth. Blocking hormonal signalling path-
ways in these cancers may be a very effective treatment strategy.
Approximately 80% of breast tumours are positive for expression of
the oestrogen receptor (ER). Assessment of ER status is now standard
in the diagnostic workup of breast cancer. Drugs that reduce oestrogen
levels or block the effects of oestrogen on the receptor are widely used in
the management of ER-positive breast cancer. Adjuvant hormone ther-
apy may reduce the risk of relapse and death at least as much as cyto-
toxic chemotherapy and in advanced cases can induce stable disease
and remissions that may last months to years, with acceptable toxicity.
Hormonal manipulation may be effective in other cancers. In prostate
cancer, hormonal therapy (e.g. luteinising hormone releasing hormone
(LHRH) analogues such as goserelin and/or anti-androgens such as
bicalutamide) aimed at reducing androgen levels can provide good long-
term control of advanced disease. The side-effects of hormone therapies
are linked to their hormonal targets.
Targeted therapies
Advances in knowledge about the molecular basis of cancer have
resulted in the development of treatments to target specic genes and
proteins that are involved in the growth and survival of cancer cells
(Fig. 7.11). These signalling pathways may have broad importance to a
range of cancer types or be specic to certain cancers. They may not
be important in all tumours of the same cancer type, requiring specic
molecular testing to predict whether the patient may benet.
Targeted therapies are broadly divided into two groups: monoclonal
antibodies (-mab) and small molecule inhibitors (-ib). The -mab family are
typically utilised for targets that are overexpressed on the outside of the
cancer cell. The -ib family typically target processes within the cell, such
as the cytoplasmic tyrosine kinase, and are designed to be small enough
to enter the cell.
A wide range of targeted therapies are now used routinely in onco-
logical practice. Some of these are described below. The side effects
of targeted therapies are determined by the molecular pathway being
targeted (p. 129).
Epidermal growth factor receptor (EGFR) is an important transmem-
brane signalling protein. Mutations in the EGFR gene lead to overexpres-
sion of the EGFR protein or constitutive activation of the cell-signalling
Fig. 7.11 Examples of targeted anti-cancer therapies and their actions in
relation to the Hallmarks of Cancer. (For abbreviations see text.)
Deregulating
cellular
energetics:
aerobic glycolysis
inhibitors
Resisting
cell death:
BCL-2 inhibitors
Genome instability
and mutation:
PARP inhibitors
Tumour-
promoting
inflammation:
selective anti-
inflammatory
agents
Enabling
replicative
immortality:
telomerase
inhibitors
Avoiding
immune
destruction:
checkpoint
inhibitors as
immunotherapy
Evading growth
suppressors:
CDK4/6
inhibitors
Sustaining
proliferative
signalling:
EGFR, HER2
and BRAF
inhibitors
Inducing
angiogenesis:
VEGF inhibitors
Activating
invasion and
metastases:
HGF/c-MET
inhibitors
pathway, leading to uncontrolled cell division, in several cancer types.
Approximately 15% of lung adenocarcinomas have activating mutations
of EGFR, which may be targeted with drugs such as getinib, erlotinib or
osimertinib. The latter of these agents has been designed to overcome
a particular mutation (i.e. T790M) responsible for 50% of resistance to
older EGFR inhibitors. In colorectal cancer drugs such as cetuximab and
panitumumab are active in patients where molecular testing does not
detect resistance inferred by mutations in the RAS/RAF family of genes.
Vascular endothelial growth factor receptor (VEGFR) inhibitors such
as sunitinib, pazopanib and cabozantinib have been a pillar of renal cell
carcinoma treatment for over a decade. Activation of members of the
VEGFR family play an important role in tumour angiogenesis. VEGFR
small molecule inhibitors are commonly used in the management of
hepatocellular carcinoma and thyroid cancer. Bevacizumab, a monoclo-
nal antibody therapy targeted at VEGF-A, is active in a number of can-
cers, including ovarian, colorectal and breast.
HER2 is a member of the epidermal growth factor receptor family.
Amplication or over-expression of HER2 is found in breast, gastric, pan-
creatic, lung and some uterine cancers. Approximately 20% of breast
cancers are HER2-positive, where it is associated with increased risk of
recurrence and poor prognosis. Several agents have been developed to
target HER2, including trastuzumab and pertuzumab. The agent tras-
tuzumab emtansine is an antibody-drug conjugate consisting of trastu-
zumab covalently linked to the cytotoxic chemotherapy agent emtansine
which is delivered specically to the HER2-positive breast cancer cell.
Immunotherapy
The term immunotherapy encompasses a range of anti-cancer thera-
pies that work by harnessing the immune system to attack cancer cells.
Cytokines, such as interferon alpha and interleukin-2, have been used
with some success in melanoma and renal cell carcinoma. However, in
recent years the development of other immunotherapy treatments has
revolutionized the management of several cancer types.
Immune checkpoints are key regulators of the immune system
which work to prevent the immune response from attacking normal
healthy cells. Cancers may co-opt this mechanism to evade immune
destruction. Targeted therapies that inhibit these checkpoint molecules
(CTLA4, PD-1) or their ligands (PD-L1) (Fig. 7.12) are now licensed in
176.
148 OncOlOgy
themanagement of many cancers, including melanoma, renal cell car-
cinoma, lung, bladder, head and neck, and, more broadly, any cancers
demonstrating microsatellite instability. Checkpoint inhibitor immunother-
apies may be used alone, or in combination with other checkpoint inhibi-
tor immunotherapies, targeted therapies or cytotoxic chemotherapy.
Adoptive cell therapy, also known as cellular immunotherapy, is increas-
ingly being used, particularly in haematological malignancies. Immune
cells isolated from either the patient’s tumour or bloodstream may be acti-
vated and expanded, before being infused back into the patient to attack
cancer cells (i.e. tumour inltrating lymphocyte (TIL) therapy). The immune
cells may also be genetically modied to improve the likelihood that they
identify the cancer cells (i.e. engineered T-cell (TCR) therapy, chimeric
antigen receptor T-cell (CAR-T) therapy). Cancer treatment vaccines are
also in development. These aim to boost the immune system’s ability to
recognise and destroy antigens using a number of different techniques.
Immunotherapy treatment is likely to become more prevalent in
the future for many types of cancers and indications. In patients with
advanced or metastatic disease a key observation has been the poten-
tial for durable treatment responses. For example, average survival for
patients with metastatic melanoma has improved from 6–8 months in the
pre-immunotherapy era to over 5 years with the use of combination anti-
CTLA4 and anti-PD-1 checkpoint inhibitors in clinical trials. However,
many patients do not respond and others are affected by potentially
life-threatening immune-related adverse events (see above).
Evaluation of treatment
In advanced or metastatic cancer the evaluation of treatment is an
ongoing process and includes assessments of treatment response, tox-
icity and quality of life. Clinical evaluation alongside radiological imaging
or biochemical monitoring are commonly used to determine whether
a treatment is being effective. Uniform criteria have been established
to measure these, including the response evaluation criteria in solid
tumours (RECIST, Box 7.19) and common toxicity criteria (e.g. common
terminology criteria for adverse events (CTCAE)). This allows clinicians to
inform patients accurately about the prognosis, effectiveness and toxicity
of systemic anti-cancer therapy and empowers patients to take an active
role in treatment decisions.
Late toxicity of therapy
The late toxicities of treatment for cancer are particularly important
for patients where multimodality therapy is given with curative intent,
where the patient is young and as more patients are living longer.
This can cause considerable morbidity: for example, radiotherapy
can retard bone and cartilage growth, impair intellect and cognitive
function, and cause dysfunction of the hypothalamus, pituitary and
thyroid glands. Late consequences of cytotoxic chemotherapy include
heart failure due to cardiotoxicity, pulmonary brosis, nephrotoxicity
and neurotoxicity.
Premature gonadal failure can result from cytotoxic chemotherapy
or radiotherapy and leave a patient subfertile. Patients should be made
aware of this before treatment is initiated, as it may be possible to store
sperm for male patients before treatment starts; this should always be
offered, if practical. Egg storage or embryo banking after in vitro fertil-
isation may be an option for young women. Sterility develops at higher
radiotherapy doses but erectile dysfunction is seen in patients receiving
high radiotherapy doses to the pelvis, as in prostate cancer. Additional
social or psychological support may be required to address these issues.
Infertility and pubertal delay are potential late effects of therapy in chil-
dren, especially boys.
Second malignancies may be induced by cancer treatment and
occur at greatest frequency following chemoradiation. Secondary
acute leukaemia (mostly AML) can occur 1–2 years after treatment
with topoisomerase II inhibitors, or 2–5 years after treatment with
alkylating agents. The most common second malignancy within a
radiation eld is osteosarcoma but others include soft tissue sarcoma
and leukaemia.
Cancer clinical trials
Cancer clinical trials are embedded within routine practice in oncol-
ogy. Close collaboration with laboratory scientists, active recruitment
of cancer patients into clinical trials and robust translational research
have led to many new cancer treatments, personalised therapies and a
transformation of cancer management over the last 20 years. As many
Fig. 7.12
activation and tumour cell death. (PD-1 = programmed cell death protein 1;
PD-L1 = programmed cell death ligand 1)
A
T cell T-cell
inhibited
T-cell
activated
T-cell receptor
Antigen
T-cell receptor
Antigen
Tumour cell
PD-L1
PD-1
B
T cell
Tumour cell
death
PD-L1
PD-1
Anti-PD-1
Anti-PD-L1
T-cell
proliferation
T-cell effector
function
7.19 Response evaluation criteria in solid tumours (RECIST)
Response Criteria
Complete response (CR) Disappearance of all target lesions
Partial response (PR) At least a 30% decrease in the sum of the
longest diameter (LD) of target lesions,
taking as reference the baseline sum LD
Progressive disease (PD) At least a 20% increase in the sum of the
LD of target lesions, taking as reference
the smallest sum LD recorded since
the treatment started and at least 5mm
increase or the appearance of one or more
new lesions
Stable disease (SD) Neither sufcient shrinkage to qualify for
PR nor sufcient increase to qualify for PD,
taking as reference the smallest sum LD
since the treatment started
177.
Speifi aers 149
7
anti-cancer drugs are expected to have toxicities, which may ultimately
limit the deliverable dose, clinical trials in cancer differ from trials of other
medicines.
Phase I cancer clinical trials take treatments of interest from labor-
atory studies and test them in patients with advanced cancer for
whom no other standard anti-cancer treatment exists. Phase I trials
assess the safety of a treatment and identify an optimal dose and
dosing schedule. Initial doses are very low and each sequential
cohort of patients receives a higher dose. Doses are escalated in
controlled cohorts of 1–6 patients, according to toxicities, pharma-
cokinetics and pharmacodynamics, until the maximum tolerated
dose is reached.
Phase II cancer clinical trials treat patients with specic cancers of
interest with the trial drug, using the dose established in phase I
trials. Phase II trials may be randomised or non-randomised but will
recruit enough patients to further assess the safety of the treatment
and whether it results in enough anti-cancer activity in a specic
cancer to develop the drug further by way of phase III trials.
Phase III cancer trials are large, multi-centre randomised controlled
trials to compare the new treatment of interest with the current
established therapy for this indication. Cancer response, toxicity,
quality of life and survival data will usually be assessed. Phase III
trials may also compare current standard treatment with a different
treatment, either looking for improved outcome or improved quality
of life with non-inferior outcome. If a treatment is deemed to be safe
and effective it will be licensed for clinical use.
Phase IV trials involve the continuing safety surveillance of a treat-
ment after it receives a licence for clinical use. This can be particu-
larly useful to detect any rare or long-term adverse effects in a much
larger population and longer time period than was possible during
Phase I–III trials.
Specic cancers
As cancer management becomes more complex and personalised, and
incorporates multi-modality treatment approaches, oncology teams are
increasingly subspecialised and work as part of tumour-specic multidis-
ciplinary teams. The diagnosis and management of specic cancers are
discussed in more detail elsewhere in the book (Box 7.20). Here we dis-
cuss the pathogenesis, clinical features, investigation and management
of some common tumours that are not covered elsewhere.
Breast cancer
Globally, the incidence of breast cancer is second only to that of lung
cancer, and the disease represents the leading cause of cancer-re-
lated deaths among women. Invasive ductal carcinoma with or with-
out ductal carcinoma in situ (DCIS) is the most common histology,
accounting for 70%, whilst invasive lobular carcinoma accounts for
most of the remaining cases. DCIS constitutes 20% of breast cancers
detected by mammography screening. It is multifocal in one-third of
women and has a high risk of becoming invasive (10% at 5years fol-
lowing excision only). Pure DCIS does not cause lymph node metas-
tases, although these are found in 2% of cases where nodes are
examined, owing to undetected invasive cancer. Lobular carcinoma in
situ (LCIS) is a predisposing risk factor for developing cancer in either
breast (7% at 10 years). The survival for breast cancer by stage is
outlined in Box 7.21
Pathogenesis
Both genetic and hormonal factors play a role: about 5%–10% of breast
cancers are hereditary and occur in patients with mutations of BRCA1,
BRCA2, AT or TP53 genes. Prolonged oestrogen exposure associated
with early menarche, late menopause and use of hormone replacement
therapy (HRT) has been associated with an increased risk. Other risk
factors include obesity, alcohol intake, nulliparity and late rst pregnancy.
There is no denite evidence linking use of the contraceptive pill to
breast cancer.
Clinical features
Breast cancer usually presents as a result of mammographic screen-
ing or as a palpable mass with nipple discharge in 10% and pain
in 7% of patients. Less common presentations include inammatory
carcinoma with diffuse induration of the skin of the breast, and this
confers an adverse prognosis. Around 40% of patients will have axil-
lary nodal disease, with likelihood correlating with increasing size of
the primary tumour. Distant metastases are infrequently present at
diagnosis and the most common sites of spread are bone (70%), lung
(60%), liver (55%), pleura (40%), adrenals (35%), skin (30%) and brain
(10%–20%).
Investigations
Following clinical examination, patients should undergo imaging with mam-
mography or ultrasound evaluation, and a biopsy using ne needle aspi-
ration for cytology or core biopsy for histology. Histological assessment
should be carried out to assess tumour type and to determine oestrogen
and progesterone receptor (ER/PR) status and HER2 status. If distant
spread is suspected, CT of the thorax and abdomen and an isotope bone
scan are required. Molecular subtyping is being used to classify tumours
into four major subtypes: luminal A, luminal B, HER2 type and basal-like
7.20 Specic cancers covered in other chapters
Bladder cancer p. 607
Colorectal cancer p. 826
Familial cancer syndromes p. 55
Gastric cancer p. 817
Hepatocellular carcinoma p. 901
Leukaemia p. 963
Lung cancer p. 528
Lymphoma p. 971
Mesothelioma p. 546
Myeloma p. 976
Oesophageal cancer p. 809
Pancreatic cancer p. 855
Prostate cancer p. 610
Renal cancer p. 606
Seminoma p. 611
Skin cancer p. 1082
Teratoma p. 611
Thyroid cancer p. 665
7.21 Five-year survival rates for breast cancer by stage
Tumour stage Stage denition 5-year survival (%)
I Tumour <2cm, no lymph
nodes
98
II Tumour 2–5cm and/or
mobile axillary lymph nodes
90
III Chest wall or skin xation
and/or xed axillary lymph
nodes
72
IV Metastasis 26
178.
150 OncOlOgy
(oftencalled ‘triple negative’, as these tumours are ER-, PR- and HER2-
negative). This may allow more targeted selection of therapies in future.
Management
Surgery is the mainstay of curative treatment. This can range from a
lumpectomy, where only the tumour is removed, to mastectomy, where
the whole breast is removed. Breast-conserving surgery is as effective as
mastectomy if complete excision with negative margins can be achieved.
Lymph node sampling is performed at the time of surgery.
There is signicant evidence to support the use of additional therapies
to reduce the risk of breast cancer recurrence. Adjuvant radiotherapy is
given to reduce the risk of local recurrence. In those patients considered
at high risk of recurrence (i.e. tumour of >1cm, ER-negative disease or the
presence of involved axillary lymph nodes) cytotoxic chemotherapy may
be offered. In patients with HER2-positive breast cancer adjuvant trastu-
zumab, a humanised monoclonal antibody to HER2, may be used along-
side standard cytotoxic chemotherapy. These treatments are increasingly
being used in the neoadjuvant setting, with the aim of achieving a complete
pathological response when the cancer is resected, often with a more
organ-preserving surgical procedure. In patients with ER-positive tumours
adjuvant hormonal therapy may gain additional disease-free and overall
survival benets. Patients at low risk of recurrence (i.e. small, ER-positive
disease) may require only adjuvant hormonal therapy. In post-menopausal
women adjuvant bisphosphonate therapy may also be used.
The treatment of metastatic breast cancer is complex. Radiotherapy
may be used to palliate painful bone metastases. SACT decisions are
made with consideration of ER status, HER2 status, the distribution
of metastatic disease and previous neo/adjuvant treatment, alongside
assessments of performance status (PS) and comorbidities. For exam-
ple, in a post-menopausal patient with ER-positive, HER2-negative
bone-only metastatic disease who is PS 0, hormonal therapy (i.e. an
aromatase inhibitor) in combination with a CDK4/6 targeted therapy
(i.e. palbociclib, abemaciclib or ribociclib) may be used as rst-line
treatment. In a similar patient with additional symptomatic liver metas-
tases, cytotoxic chemotherapy may be more appropriate.
Ovarian cancer
Ovarian cancer is the most common gynaecological tumour in Western
countries. Most ovarian cancers are epithelial in origin (90%), and up
to 7% of women with ovarian cancer have a positive family history.
Patients often present late in ovarian cancer with vague abdominal dis-
comfort, low back pain, bloating, altered bowel habit and weight loss.
Occasionally, peritoneal deposits are palpable as an omental ‘cake’ and
nodules in the umbilicus (Sister Mary Joseph nodules).
Pathogenesis
Genetic and environmental factors play a role. The risk of ovarian cancer
is increased in patients with BRCA1 or BRCA2 mutations, and Lynch
type II families (a subtype of hereditary non-polyposis colon cancer,
HNPCC) can have ovarian, endometrial, colorectal and gastric tumours
due to mutations of mismatch repair enzymes. Advanced age, nulliparity,
ovarian stimulation and European descent all increase the risk of ovarian
cancer, while suppressed ovulation appears to protect, so pregnancy,
prolonged breastfeeding and the contraceptive pill have all been shown
to reduce the risk of ovarian cancer.
Investigations
Initial workup for patients with suspected ovarian cancer includes imaging in
the form of ultrasound and CT. Serum levels of the tumour marker CA-125
are often measured. Surgery plays a key role in the diagnosis, staging and
treatment of ovarian cancer, and in early cases, palpation of viscera, perito-
neal washings and biopsies are generally performed to dene disease extent.
Management
In early disease, surgery followed by adjuvant cytotoxic chemotherapy
with carboplatin, or carboplatin plus paclitaxel, is the treatment of choice.
Surgery should include removal of the tumour along with total abdom-
inal hysterectomy, bilateral salpingo-oophorectomy, and omentectomy.
Even in advanced disease, surgery is undertaken to maximally debulk the
tumour and is followed by cytotoxic chemotherapy, typically using carbo-
platin and paclitaxel. Bevacizumab, a targeted therapy against VEGFR,
is indicated for patients with high-grade tumours that are suboptimally
debulked or those with a more aggressive biological pattern. Subsequent
treatment decisions are made with consideration of response to rst-line
cytotoxic chemotherapy and germline BRCA mutation status. Options
include further platinum/paclitaxel combination, liposomal doxorubicin or
targeted therapy against poly-ADP ribose polymerase (PARP, e.g. olapa-
rib, niraparib or rucaparib).
The serum tumour marker CA-125 and clinical examination may be
used to monitor treatment response in ovarian cancer, with CT imaging
for those with suspected progressive disease.
Endometrial cancer
Endometrial cancer accounts for 4% of all female malignancies, produc-
ing a 1 in 73 lifetime risk. The majority of patients are post-menopausal,
with a peak incidence at 50–60 years of age. Mortality from endome-
trial cancer is currently falling. The most common presentation is with
post-menopausal bleeding, which often results in detection of the dis-
ease before distant spread has occurred.
Pathogenesis
Oestrogen plays an important role in the pathogenesis of endometrial
cancer, and factors that increase the duration of oestrogen exposure,
such as nulliparity, early menarche, late menopause and unopposed HRT,
increase the risk. Endometrial cancer is 10 times more common in obese
women and this is thought to be due to elevated levels of oestrogens.
Investigations
The diagnosis is conrmed by endometrial biopsy.
Management
Surgery is the treatment of choice and is used for staging. A hysterec-
tomy and bilateral salpingo-oophorectomy are performed with peritoneal
cytology and, in some cases, lymph node dissection. Where the tumour
extends beyond the inner 50% of the myometrium or involves the cer-
vix and local lymph nodes, or there is lymphovascular space invasion,
adjuvant pelvic radiotherapy is recommended. Cytotoxic chemotherapy
is used as adjuvant therapy, and hormonal therapy and cytotoxic chemo-
therapy are used to palliate symptoms in recurrent disease.
Cervical cancer
Cervical cancer is the fourth most common cancer in women and the
leading cause of death from gynaecological cancer worldwide. The
incidence is decreasing in high-income industrialised countries but con-
tinues to rise in low- and middle-income nations. The most common
presentation in the UK is with an abnormal smear test, but with locally
advanced disease the presentation is with vaginal bleeding, discomfort,
discharge or symptoms attributable to involvement of adjacent struc-
tures, such as bladder, or rectal or pelvic wall. Occasionally, patients
present with distant metastases to bone and lung.
Pathogenesis
Almost all cases of cervical cancer are linked to high-risk human papil-
lomaviruses (HPV), transmitted through sexual contact. This has under-
pinned the introduction of programmes to immunise adolescents against
HPV in an effort to prevent up to 90% of cervical cancer.
Investigations
Diagnosis is made by smear or cone biopsy. Further examination may
require cystoscopy and exible sigmoidoscopy if there are symptoms ref-
erable to the bladder, colon or rectum. In contrast to other gynaecological
179.
Further iformatio 151
7
Management
In general, the majority of patients with early or locally advanced disease
are treated with curative intent. In localised disease where there is no
involvement of the lymph nodes, long-term remission can be achieved in
up to 90% of patients with surgery or radiotherapy. The choice of surgery
versus radiotherapy often depends on patient preference, as surgical
treatment can be mutilating with an adverse cosmetic outcome. Patients
with lymph node involvement are treated with a combination of surgery
and radiotherapy (often with a radiosensitising agent such as cisplatin
or cetuximab), and this produces long-term remission in approximately
60%–70% of patients. Recurrent or metastatic tumours may be palli-
ated with further surgery or radiotherapy to aid local control, or systemic
cytotoxic chemotherapy or immunotherapy may be used. Second malig-
nancies are common (3% per year) following successful treatment for pri-
mary disease, and all patients should be encouraged to give up smoking
and drinking alcohol to lower their risk.
Survivorship
Advances in cancer prevention, diagnosis and treatment mean that
more people are surviving cancer. Cancer survival has doubled in the last
40years in the UK. There are an estimated 2 million people living with, or
beyond, cancer in the UK today and 50% of those diagnosed with can-
cer will survive their disease for 10 years or more. Cancer survivorship
has at least two common meanings:
completing treatment for cancer and having no signs of cancer after
nishing treatment
living with, through and beyond cancer, thus including people who
receive curative treatment and people who receive intermittent
anti-cancer treatment to control their cancer over a longer time.
Many people feel that life is never the same after a cancer diagno-
sis. There are often long-lasting physical, social and emotional conse-
quences of both cancer and its treatment. These start at diagnosis and
last through rst treatment (acute survivorship), continue through and
beyond cancer treatments (extended survivorship) and can be long-
lasting, even when risk of cancer recurrence is low (permanent survivor-
ship). An increasing awareness of survivorship, the impact of a cancer
diagnosis and its wide-ranging effects on patients has highlighted the
need for holistic and patient-centred care, support and services through-
out and after cancer treatment.
Further information
Books and journal articles
Cassidy J, Bissett D, Spence RAJ, etal. Oxford handbook of oncology, 4th edn.
Oxford: Oxford University Press; 2015.
Hanahan D, Weinberg RA. The hallmarks of cancer: perspectives for cancer
medicine. In: Kerr DJ, Haller DG, van de Velde CJH, Baumann M, eds. Oxford
textbook of oncology, 3rd edn. Oxford: Oxford University Press; 2016. Oxford
Medicine Online DOI: 10.1093/med/9780199656103.003.0001.
Tobias J, Hochhauser D. Cancer and its management, 7th edn. Chichester:
Wiley–Blackwell; 2014.
Websites
cancer.org American Cancer Society: clinical practice guidelines
ctep.cancer.gov/reporting/ctc.html Common toxicity criteria
info.cancerresearchuk.org/cancerstats/ Cancer statistics that can be sorted by
type or geographical location
7.22 Common presenting features by location in head and
neck cancer
Hypopharynx
Dysphagia
Odynophagia
Referred otalgia
Enlarged lymph nodes
Mouth and tongue
Non-healing ulcers Ipsilateral otalgia
Nasal cavity and sinuses
Discharge (bloody) or obstruction
Nasopharynx
Nasal discharge or obstruction
Conduction deafness
Atypical facial pain
Diplopia
Hoarse voice
Horner syndrome
Oropharynx
Dysphagia
Pain
Otalgia
Salivary gland
Painless swelling Facial nerve palsy
malignancies, cervical cancer is a clinically staged disease, although MRI
is often used to characterise the primary tumour. CT of the chest, abdo-
men and pelvis is performed to look for metastases in the lungs, liver and
lymph nodes, and to exclude hydronephrosis and hydroureter.
Management
This depends on the stage of disease. Pre-malignant disease can be
treated with laser ablation or diathermy, whereas in microinvasive dis-
ease a large loop excision of the transformation zone (LLETZ) or a simple
hysterectomy is employed. Invasive but localised disease requires rad-
ical surgery, while cytotoxic chemotherapy and radiotherapy, including
brachytherapy, may be given as primary treatment, especially in patients
with adverse prognostic features such as bulky or locally advanced dis-
ease, or lymph node or parametrium invasion. In metastatic disease,
platinum-based cytotoxic chemotherapy may be benecial in improving
symptoms but does not increase survival signicantly.
Head and neck tumours
Head and neck cancers are typically squamous tumours that arise in
the nasopharynx, hypopharynx and larynx. They are most common in
older adult males but oropharyngeal cancers now occur with increasing
frequency in a younger cohort of patients, including in women. The rising
incidence of oropharyngeal cancers, especially in high-income countries,
is thought to be secondary to HPV infection. Presentation depends on
the location of the primary tumour and the extent of disease. For exam-
ple, early laryngeal cancers may present with hoarseness, while more
extensive local disease may present with pain due to invasion of local
structures or with a lump in the neck. Patients who present late often
have pulmonary symptoms, as this is the most common site of distant
metastases (Box 7.22).
Pathogenesis
The tumours are strongly associated with a history of smoking and
excess alcohol intake, but other recognised risk factors include Epstein–
Barr virus for nasopharyngeal cancer and HPV infection for oropharyn-
geal tumours.
Investigations
Careful inspection of the primary site is required as part of the staging
process, and most patients will require endoscopic evaluation and exami-
nation under anaesthesia. Tissue biopsies should be taken from the most
accessible site. CT of the primary site and the thorax is the investigation of
choice for visualising the tumour, while MRI may be useful in certain cases.
Multiple Choice Questions
7.1.A 54-year-old woman presents to the emergency department
with fevers and a sore throat. She has recently been diagnosed
with left-sided breast cancer and associated axillary lymph
node disease. Twelve days ago she received her rst cycle of
neoadjuvant cytotoxic chemotherapy (5-uorouracil, epirubicin
and cyclophosphamide). On clinical examination her temperature
is 38.3°C, she appears dehydrated and there is evidence of
oral candidiasis. An intravenous catheter is placed and bloods,
including blood cultures, taken. What is the most appropriate next
step in this patient’s care?
A. Start oral uconazole
B. Await blood test results to inform further management
C. Start intravenous uids
D. Perform a chest X-ray, collect a urine sample and throat
swab to complete the infection screen
E. Start high-dose broad-spectrum intravenous antibiotic
therapy immediately
Answer: E.
The clinical features here are of fever in a patient at high risk of neu-
tropenia. This is an oncological emergency. Patients are at risk of neu-
tropenia at any point during their systemic anti-cancer therapy treatment
cycle, with the highest risk typically 10–14 days after a treatment. In
patients with potential neutropenic sepsis high-dose broad spectrum
intravenous antibiotics should be commenced, ideally within 1hour of
admission, without awaiting test results. In this patient uconazole (A)
for oral candidiasis, intravenous uids (C) and further tests (D) are also
appropriate, but should not delay the rst dose of antibiotic therapy.
Test results may later inform changes to antimicrobial therapy and its
duration.
7.2. A 61-year-old man recently diagnosed with colon cancer and
associated liver and lung metastases is seen in the oncology
clinic. He is keen to talk about treatment for his cancer. His ECOG
performance status is 1 and he has no signicant comorbidities.
He has some mild symptoms of abdominal discomfort and has
lost 5kg in weight. His routine bloods demonstrate anaemia
(Hb105g/L) and mild elevations in his liver enzymes. A biopsy of
his tumour has shown a moderately differentiated adenocarcinoma
which is KRAS/NRAS wildtype and BRAF mutant. What is the
most appropriate treatment option for this patient?
A. Cytotoxic chemotherapy (5-uorouracil, folinic acid and
oxaliplatin) alone
B. Surgery to all cancer sites
C. Cytotoxic chemotherapy (5-uorouracil, folinic acid and
oxaliplatin) with an EGFR inhibitor (cetuximab)
D. Cytotoxic chemotherapy (5-uorouracil, folinic acid and
oxaliplatin) with a BRAF inhibitor (dabrafenib)
E. Refer to palliative care
Answer: A.
This man wishes to pursue anti-cancer therapy and, from the informa-
tion provided, appears t to do so. In patients with metastatic disease
systemic anti-cancer therapies are most often used, unless there is a
need to palliate a specic symptom. Cytotoxic chemotherapy would be
an appropriate option here. As the tumour is BRAF mutant he is unlikely to
respond to an EGFR inhibitor such as cetuximab. BRAF inhibitors such
as dabrafenib are not used in colon cancer regardless of the mutational
status.
7.3. A 59-year-old woman is seen in the emergency department with
a 2-day history of severe diarrhoea. She has had 10 loose stools
today, the most recent of which have been bloody and associated
with crampy abdominal pain. Her past medical history includes
metastatic melanoma and she had her fourth cycle of ipilimumab
and nivolumab therapy 20 days ago. At the time of her treatment she
was constipated and the outpatient systemic anti-cancer therapy unit
doctor prescribed her a macrogol laxative. She notes that the evening
prior to the diarrhoea starting she had reheated some leftover rice for
her supper. What is the most important likely diagnosis?
A. Bacillus cereus-associated food poisoning
B. Overow diarrhoea
C. Laxative overuse
D. Immunotherapy-related colitis
E. Clostridioides difcile infection
Answer: D.
Immune-related adverse events should be considered in all patients
who present acutely unwell following immunotherapy treatments for
cancer. Although the other options are on the differential diagnosis list,
severe IRAEs are oncological emergencies and prompt recognition and
management are vital. This is a grade 3 colitis and should be treated
with IV methylprednisolone in the rst instance. Investigations such as
bloods, stool sample, radiological imaging and exible sigmoidoscopy or
colonoscopy are also appropriate.
7.4. A 58-year-old man is seen in clinic with a diagnosis of metastatic
lung cancer. He is coping well at home, where he lives alone.
However, he has taken early retirement as a brick-layer as he feels
unable to work due to increasing fatigue. He awakes early each
morning and goes to the shops to collect his newspaper, but has
taken to napping for an hour in the afternoon. What is this man’s
ECOG performance status?
A. 0
B. 1
C. 2
D. 3
E. 4
Answer: B.
This man is ambulatory and capable of self-care, but unable to under-
take strenuous activity. He is up for more than 50% of waking hours.
Performance status is a key assessment tool in patients with cancer.
7.5. You have been asked to see a 70-year-old man on a medical
ward. He was admitted 2 weeks ago following a fall. On admission
he had abnormal liver function tests and an elevated CRP. He
has been treated for a urinary tract infection. A CT scan has
demonstrated multiple liver, lung and bone metastases, but no
obvious primary cancer. The ward nurses tell you he remains
mildly confused at times, sleeps for most of the day and does not
wish to engage with physiotherapy. What is the most appropriate
management for this man?
A. Refer to the Hepatobiliary Pancreatic Cancer
Multidisciplinary Team
B. Ultrasound-guided liver biopsy
C. Colonoscopy
D. Palliative care
E. Cytotoxic chemotherapy
Answer: D.
182.
This man hasa new diagnosis of metastatic cancer of undened pri-
mary origin (MUO). His ECOG performance status (3), requirement for
hospital admission, multi-organ metastatic disease, end-organ dys-
function and raised inammatory markers are poor prognostic features.
Further investigations may not be warranted and honest conversations
about the likely diagnosis, palliative care and end-of-life care, taking into
account the patient’s wishes are often preferable.
7.6. A 56-year-old woman is seen in oncology clinic following surgery
to remove a localised clear cell renal cell carcinoma. She is eligible
for a phase III clinical trial comparing adjuvant immunotherapy
to the current clinical standard of observation. Patients will be
randomised to receive either the immunotherapy treatment or
a placebo every 4 weeks for 1 year. Neither the patient nor the
investigator will know what treatment she is receiving. Which of the
following statements regarding randomised controlled trials (RCTs)
is true?
A. Patients may choose which treatment they receive
B. RCTs are always ‘double blinded’
C. RCTs are considered to be the ‘gold standard’ for determin-
ing efcacy and safety in clinical research
D. RCTs always have a control arm that uses placebo
E. RCTs always equally divide patients between each
treatment arm
Answer: C.
Phase III RCTs are often used to determine whether a treatment
should be licensed for clinical use. Patients are randomised to each arm,
sometimes with stratication to ensure equal allocation of patient sub-
groups. RCTs may be double-blinded, as is the case here, single-blinded
or ‘open-label’. The control arm should be the current clinical standard
treatment. In this example a placebo is used in the place of observation
only to ensure blinding and avoid potential bias of results. Patients may
be divided in alternative ratios. This is sometimes the case when earlier
studies suggest large differences between the investigational and current
treatment.
183.
Pain and palliativecare
8
LA Colvin
M Fallon
Clinical examination in pain and palliative care 154
Clinical evaluation and management in a patient with chronic pain or in the
palliative care setting 155
Pain 156
Functional anatomy and physiology 156
Investigations 158
Principles of management 160
Interventions 162
Chronic pain syndromes 165
Palliative care 167
Presenting problems in palliative care 167
Death and dying 172
184.
154 PAINAND PALLIATIVE CARE
Venous system
SVC obstruction due to lung
carcinoma
Mouth
Increased secretions due to
dysphagia
Extremities
Neuropathic pain due to CRPS type I
1
Abdomen
Abdominal swelling secondary to
malignant ascites
Lower limb
Phantom limb pain (CRPS type II)
following amputation
Lungs
Dyspnoea due to pleural effusion
3
4
5
2
Hand
Muscle wasting and pain due
to nerve compression
1
2
Spine
Back pain due to bone metastases
3
4
5 6 6
7
9 9
8
7
8
Nervous system
Drowsiness due to hypercalcaemia
or brain metastases
Insets (SVC obstruction, wasting in the hand, pleural effusion, ascites, amputation) From Forbes CD, Jackson WF. Color atlas and text of clinical
medicine, 3rd edn. Edinburgh: Mosby; 2002. (CRPS = chronic regional pain syndrome; SVC = superior vena cava)
Clinical examination in pain and palliative care
185.
Clinical evaluation andmanagement in a patient with chronic pain or in the palliative care setting 155
8
VAS
score
Record severity of pain
Take careful history, recording
character and radiation of pain
Assess pinprick, fine touch
and heat/cold sensation
Formulate management plan
with patient and set goals
Consider TENS and
acupuncture
Consider nerve block
or ablation
Consider psychological
therapies and mindfulness
Increase physical activity
Conduct biopsychosocial
assessment
Conduct general examination
Educate patient on nature of pain
and promote self-management
Consider yoga, pilates or tai chi
Assess mood and screen
for depression
Check gait and whether
using a walking aid
Conduct neurological
examination
Optimise medication
(TENS = transcutaneous electrical nerve stimulation)
Clinical evaluation and management in a patient with chronic pain or in the palliative care setting
186.
156 PAINAND PALLIATIVE CARE
Pain
Pain is dened as ‘an unpleasant sensory and emotional experience
associated with actual or potential tissue damage or described in terms
of such damage’. It is one of the most common symptoms for which
people seek health-care advice. Our understanding of the mechanisms
of pain has evolved considerably from Hippocrates’ suggestion in 450
BC that pain arose as a result of an imbalance in vital uids. We now
know that pain is a complex symptom that is inuenced and modied
by many social, cultural and emotional factors, as illustrated in Figure 8.1.
The sensation of acute pain that occurs in response to inammation or
tissue damage plays an important role in protection from further injury.
Chronic pain serves no useful function but results in signicant distress
and suffering for the patient affected, as well as having a wider societal
impact.
Functional anatomy and physiology
The functional anatomy of the somatosensory system is shown in Figures
28.3 and 28.6. Here, discussion will focus on the mechanisms and medi-
ators that are involved in pain processing.
Peripheral nerves
Peripheral nerves contain several types of neuron. These can be classi-
ed into two groups, depending on whether or not they are surrounded
by a myelin sheath. Myelinated neurons have a fast conduction velocity
and are responsible for transmission of various sensory signals, such as
proprioception, light touch, heat and cold, and the detection of localised
pains, such as pin-prick. Unmyelinated bres have a much slower con-
duction velocity and are responsible for transmitting diffuse and poorly
localised pain, as well as other sensations (Box 8.1).
Sensory neurons (also known as primary afferent neurons) connect
the spinal cord to the periphery and supply a dened territory or a der-
matome, which can be used to identify the position of a nerve lesion
(see Fig. 28.10). In healthy individuals, dermatomes have distinct bor-
ders, but in pathological pain syndromes these may become blurred as
the result of neuronal plasticity, which means that pain may be felt in an
area adjacent to that supplied by a specic nerve root. Autonomic neu-
rons also contain pain bres and are responsible for transmitting visceral
sensations, such as colic. In general, visceral pain is diffuse and less well
localised than pain transmitted by sensory neurons.
Anatomical features of the afferent pain pathway are illustrated in
Figure 8.2. Pain signals are transmitted from the periphery to the spinal
cord by sensory neurons. These have the following components:
A cell body, containing the nucleus, which is situated in the dorsal
root ganglion close to the spinal cord. The cell body is essential for
survival of the neuron, production of neurotransmitters and neuronal
function.
The nerve bre (axon) and peripheral nerve endings, which are
located in the periphery and contain a range of receptors in the
neuronal membrane.
Specialised receptors in the periphery, consisting of bare nerve end-
ings known as nociceptors or pain receptors, which are activated by
various mediators. They are situated mainly in the epidermis.
The central termination, which travels to the dorsal horn of the spinal
cord to form the rst central synapse with neurons that transmit pain
sensation to the brain.
When a noxious stimulus is encountered, activation of nociceptors
leads to generation of an action potential, which travels upwards to the
dorsal root ganglion and also stimulates the release of neurotransmitters
that have secondary effects on surrounding neurons.
Spinal cord
Sensory neurons, through their central termination, synapse with sec-
ond-order neurons in the dorsal horn of the spinal cord. There is consid-
erable modulation of pain messages at this site, both from local neurons
within the spinal cord and from neurons that descend from the brain,
as depicted in Figure 28.11. Several neurotransmitters are involved in
pain processing at this level and these are summarised in Box 8.2. They
include amino acids, such as glycine and γ-aminobutyric acid (GABA),
which are inhibitory, and glutamate, which is excitatory; neuropeptides,
such as substance P and calcitonin gene-related peptide (CGRP); and
endorphins. Whether or not they increase or decrease pain perception
depends on the connectivity of the neurons on which they act.
Illness and
pain behaviour
(underlying disease, impact
on quality of life, fear avoidance)
Social
and cultural
(socioeconomics,
religion, family)
Affect and
emotional state
(mood, self-efficacy)
Cognition
(catastrophising,
acceptance)
Sensory
(genetic, anatomical,
biomedical)
Fig. 8.1 The biopsychosocial model of pain. The perception of pain as a
symptom is dependent not only on sensory inputs but also on the individual’s
cognitive reaction to the pain, their emotional state, their underlying disease and their
social and cultural background.
8.1 Types of nerve bre
Fibre
type
Diameter
(µm)
Conduction
velocity (ms 1
)
Function
Large myelinated
Aα 12–20 70–120 Proprioception
Motor to muscle bres
Aβ 5–12 30–70 Light touch, pressure
A 3–6 15–30 Motor to muscle spindles
Small myelinated
A 2–5 12–30 Well-localised pain
Thermal sensation
B <3 3–15 Pre-ganglionic autonomic
Unmyelinated
C 0.4–1.3 0.5–3 Diffuse pain
Poorly localised thermal
sensation
Post-ganglionic autonomic
187.
Pain 157
8
Centralprocessing of pain
The signals transmitted by second-order neurons in the spinal cord are
relayed to the sensory cortex by third-order neurons, which synapse with
second-order neurons in the thalamus. At this site, perception of pain
is inuenced by interactions between a range of structures in the brain,
where sensory, cognitive and emotional aspects are integrated. This is
termed the pain neuromatrix (see Fig. 8.2). Signals within the neuromatrix
are multidirectional in nature, involving modulation of incoming messages
by the cerebral cortex (top-down regulation), as well as a complex net-
work of connections between other subcortical structures. Under normal
conditions, there is a degree of descending inhibition from the brainstem
that reduces input from peripheral stimuli.
It is thought that chronic widespread pain (CWP) and opioid-induced
hyperalgesia may result, at least in part, from abnormalities in central pro-
cessing of pain signals. It has also been suggested that variations in the
levels of descending inhibition between individuals may make some peo-
ple more vulnerable than others to developing chronic pain. Over recent
years, there has been increasing interest in the role that glial cells (see Fig.
28.1) play in pain processing. Both astrocytes and microglial cells can
become activated in chronic pain states and release pro-inammatory
cytokines, as well as altering re-uptake of excitatory neurotransmitters
such as glutamate, which can inuence pain perception considerably.
As our understanding of these processes improves, there is increasing
potential to develop novel therapies targeted at these mediators, with
some early clinical studies in neuropathic pain (pain related to nerve injury
or disease, with characteristic neurobiological changes).
Sensitisation
Sensitisation is one of the key features of pain processing. It refers to the
fact that both peripheral and central nervous systems adapt rapidly to the
presence of pain, especially in response to tissue damage. This adaptive
process is called neuronal plasticity. In some situations, neuronal plas-
ticity can lead to prolonged changes in the pathways that are involved
in detecting and processing nociceptive stimuli, resulting in chronic pain
syndromes. The specic changes in key neurotransmitters and recep-
tors differ between chronic pain states, with implications for the efcacy
of treatments. For example, mu opioid receptors are down-regulated in
neuropathic pain, potentially leading to limited opioid responsiveness.
Peripheral sensitisation
Peripheral sensitisation can occur in association with a variety of clin-
ical conditions, including sepsis, cancer, inammatory disease, injury,
Primary afferent
neuron
Second-order
neuron
First central
synapse
Spinothalamic
tract
Dorsal root
ganglion
Peripheral
input
Spinal cord
Medulla
Mid-brain
Cell
body
C fibre
Nociceptors: chemicals,
changes in pH, cytokines
Thalamus
PAG
RVM
Synapse
Hypothalamus
Amygdala
Emotional
Cognitive Sensory Sensory
Emotional
Cerebral cortex
Fig. 8.2 Ascending and descending pain pathways. Ascending pathways are shown in blue and descending in red. Pain signals are detected in the periphery by
nociceptors, which are activated by chemicals, changes in pH and cytokines. The signal is transmitted by the primary afferent neuron to the spinal cord, where there is a
synapse with a second-order neuron, which transmits the signal onwards to the thalamus. Thereafter, the pain signal is transmitted to the cerebral cortex. The intensity of pain
signals is subject to extensive modulation at several levels within the nervous system. Cognitive inuences derived from the frontal lobe, coupled with sensory inuences from
cortex and emotional inuences from the amygdyla, affect pain perception in the mid-brain around the periaqueductal grey matter (PAG) and the rostroventrolateral medulla
(RVM) in the medulla. These structures form part of the descending modulatory systems, which, under normal circumstances, inhibit pain perception. In some chronic pain
states, however, dysfunction of the descending pathways can occur, increasing pain.
188.
158 PAINAND PALLIATIVE CARE
surgery and obesity. The nal common pathway by which sensitisation
takes place in all of these conditions is inammation. Inammation is
accompanied by increased capillary permeability and tissue oedema
with the release of a diverse range of mediators, including bradykinin,
hydrogen ions, prostaglandins and adenosine, which bind to receptors
and ion channels on nociceptors of primary afferent neurons (Fig. 8.3).
The signalling pathways activated by these mediators generate action
potentials, which are transmitted by sensory neurons to the spinal cord.
If these pain-provoking stimuli persist, the activation threshold of sensory
neurons is reduced, resulting in an increased transmission of pain signals
to the spinal cord.
Central sensitisation
Sensitisation may also take place at the level of the spinal cord in
response to a sustained painful stimulus. It can occur acutely and rapidly,
such as immediately after surgery, or may progress to chronic changes,
such as chronic infection, cancer, repeated surgery or multiple trau-
matic episodes. Glutamate, acting via the N-methyl-D-aspartate (NMDA)
receptor complex, plays a key role in central sensitisation (Fig. 8.4). In
response to a sustained peripheral painful stimulus, increased amounts
of glutamate are released in the spinal cord, overcoming the inhibitory
action of magnesium ions and resulting in activation of the NMDA recep-
tor. This initiates a cascade of intracellular signalling events that lead to
prolonged modications of somatosensory processing, with amplica-
tion of pain responses within the spinal cord and continued neuronal
ring, even after the noxious stimulus has stopped. This phenomenon is
termed ‘after-discharge’. In neuropathic pain, prolonged activation of the
NMDA pathway results in a decrease in the number of inhibitory interneu-
rons, which further potentiates pain.
Genetic determinants of pain perception
There are marked ethnic and individual variations in how people respond
to painful stimuli and studies in twins have estimated that the heritability
of CWP ranges between 30% and 50%. In the general population, the
individual variants in response to pain and perception of pain are most
likely due to a complex interaction between genetic and environmental
inuences. Few variants have been identied with robust evidence of
association with CWP. Several rare syndromes have been described,
however, in which insensitivity to pain or heightened pain responses
occur as the result of a single gene disorder, as summarised in Box 8.3.
Most are due to mutations affecting ion channels that play a key role in
neurotransmission (see Fig. 8.3), but other causes include mutations in
the NTKR1 gene, which encodes the receptor for nerve growth factor,
and mutations in the PDRM12 transcription factor, which is involved in
neuron development.
Investigations
Pain can be a presenting feature of a wide range of disorders and the
rst step in evaluation of a patient with pain should be to perform what-
ever investigations are required to dene the underlying cause of the
pain, unless this is already known. However, with most chronic pain
syndromes, such as bromyalgia, complex regional pain syndrome and
CWP, investigations are negative and the diagnosis is made on the basis
of clinical history and exclusion of other causes. Specic investigations
that are useful in the assessment of selected patients with chronic pain
are discussed below.
8.2 Neurotransmitters and receptors involved in pain processing in the spinal cord
Neurotransmitter Receptor(s) Receptor type Comments*
Amino acids
Glutamate AMPA Ion channel Excitatory; permeable to cations: can be Ca2+
, Na+
or K+
, depending on
subunit structure
NMDA Ion channel Excitatory; blocked by Mg2+
in the resting state; block can be altered if
membrane potential changes; permeable to Ca2+
, Na+
and K+
Kainate Ion channel Post synaptic – excitatory
Gp I GPCR Pre-synaptic – inhibitory through GABA release; permeable to Na+
and K+
Gp II GPCR Activates a range of signalling pathways; long-term effects on synaptic
excitability
Gp III GPCR Probably inhibitory; can decrease cAMP production; pre-synaptic; decreases
glutamate release
Glycine GlyR Ion channel Mainly inhibitory; permeable to Cl ; blocked by caffeine
GABA GABAA
Ion channel Mainly inhibitory in spinal cord; permeable to Cl ; indirectly modulated by
benzodiazepines (increased ion channel opening); not specically involved in
nociception, generally depressant effect on spinal cord activity
GABAB
GPCR Predominantly inhibitory; activated by baclofen
Neuropeptides
Substance P Neurokinin receptors GPCR Mainly excitatory; increased in inammation, decreased in neuropathic pain
Cholecystokinin CCKRs1–8 GPCR Excitatory; clinical trials of antagonists in progress
Calcitonin gene-related
peptide
CALCRL GPCR Excitatory; slows degradation of substance P; implicated in migraine
Opioids
Dynorphin DOP GPCR Excitatory?; may be pro-nociceptive
β-endorphin MOP GPCR Inhibitory
Nociceptin NOP GPCR Inhibitory; also expressed by immune cells
(AMPA = α-amino 3-hydroxy, 5-methyl, 4-isoxazole propionic acid; CALCRL = calcitonin receptor-like receptor; cAMP = cyclic adenosine monophosphate; CCKR = cholecystokinin receptor;
DOP = delta opioid receptor; GABA = γ-aminobutyric acid; Gp = group; GPCR=G-protein-coupled receptor; MOP = mu opioid receptor; NMDA = N-methyl-D-aspartate;
NOP = nociceptin/orphan receptor)
*Excitatory = increased pain; inhibitory = reduced pain.
189.
Pain 159
8
Magneticresonance imaging
Magnetic resonance imaging (MRI) can be helpful in the assessment of
an underlying cause in patients with focal pain that follows a nerve root
or peripheral nerve distribution. Imaging is seldom helpful in individuals
with CWP.
Blood tests
Blood tests are not generally helpful in the diagnosis of chronic pain,
except in patients with peripheral neuropathy; in this case, a number of
blood tests may be required to investigate the underlying causes of the
neuropathy. Full details are provided in Box 28.85. Genetic testing may
Stimulus
ATP
Temperature or low pH
Osmosis
Cold or limitants
Cool
Low pH
Mechanical
Bradykinin
Prostanoids
ATP
NGF
Transducer
P2X
TRPV1/2
TRPV4
TRPA1
TRPM8
ASIC
Unknown
BK1/2
EP
P2Y
NTRK1
Kv
HCN2
Nav1.7
Sensitisation
Stimulus
Agonists
[H+]
Receptors
Response
Nav1.8 Nav1.9
B
A
C
Fig. 8.3
channels that act as mediators of pain. They include sodium channels implicated in congenital pain syndromes; the purinergic 2X (P2X) and purinergic 2Y (P2Y) receptor for
adenosine triphosphate (ATP); members of the transient receptor potential (TRP) superfamily of ion channel receptors, which detect changes in osmolality and temperature;
acid-sensing ion channel (ASIC) receptors, which detect hydrogen ions; G-protein-coupled receptors, which detect bradykinin (BK), prostaglandins and ATP; sodium-potassium
hyperpolarization-activated cyclic nucleotide-gated channels (HCN2), and voltage gated potassium channels (Kv) and the neurotrophic tyrosine kinase 1 (NTRK1) receptor, which
+
] and high temperature (>42°C) amplies action potentials, which increase pain
signals and cause peripheral sensitisation. (EP = E-prostanoid receptor) Adapted from Bennett DL, Woods CG. Painful and painless channelopathies. The Lancet Neurol 2014;
13:587–599; reproduced with permission from Elsevier.
Glutamate
Amino acids
(and other
neurotransmitters)
Pain
signal
Amplified
signal
Kinase
Dorsal root
ganglia
Neurotransmitter
changes
Glycine
NR1 NR2
Mg2+
Regulation of
pain response
NMDA receptor
Fig. 8.4 Mechanisms of central sensitisation. Post-synaptic activation of the N-methyl-D-aspartate (NMDA) receptor requires the amino acids glycine and glutamate, which
bind to the NR1 and NR2 subunits, respectively; these amplify pain signals at the level of the spinal cord. In contrast, magnesium ions block receptor activation.
190.
160 PAINAND PALLIATIVE CARE
be of value in patients with clinical features that point to an inherited
disorder of pain processing (see Box 8.3).
Quantitative sensory testing
Quantitative sensory testing can be helpful in the detailed assessment
of patients with chronic pain. A simple set of tools can be used in the
clinical setting (Fig. 8.5). Lightly touching the skin with a brush, swab or
cotton-wool ball can be used to test for abnormalities of ne touch. This
may include allodynia, where a normally non-painful stimulus is perceived
as painful. Assessing the patient’s response to a pin-prick can be used
to test for abnormalities in mechanical hyperalgesia. Finally, touching the
patient’s skin with warm and cool thermal rollers can be used to test for
abnormalities of thermal sensation. An unaffected area of skin should
be tested rst, to establish normal sensation, before testing the affected
area.
Nerve conduction studies
Nerve conduction studies can be helpful in demonstrating and quanti-
fying a denitive nerve lesion, either peripherally or centrally. They can
be used to help differentiate between central and peripheral neuropathic
pain. They do not, however, effectively examine small nerve bre function.
Nerve blocks
Performing a nerve block with inltration of a local anaesthetic such as
1% lidocaine can be used diagnostically, in assessing whether a pain
syndrome is due to involvement of a specic nerve or nerve root. Where
inammation and or swelling may be contributing to the underlying pain –
for example, if there is compression of a nerve root – then a mixture of
local anaesthetic and depot glucocorticoid may be helpful in alleviating
pain. Nerve blockade can also be used to determine whether more radi-
cal therapies, such as nerve ablation, might be helpful in controlling pain,
particularly that related to cancer.
Pain scoring systems
Various questionnaires and other instruments have been devised to
localise pain, rate its severity and assess its impact on quality of life.
Some of the most widely used are listed in Box 8.4. The distribution of
pain can be documented on a diagram of the body, on which the patient
can mark the sites that are painful. Similarly, other methods have been
developed with which to assess the severity of pain using verbal, numer-
ical and behavioural rating scales. Visual scoring systems employing dif-
ferent facial expressions may be of value in paediatric patients and those
with cognitive impairment. Documenting changes in pain scores using
questionnaires can be helpful in indicating to what extent drug treat-
ments have been successful and can reduce the time taken to achieve
pain control.
Principles of management
Effective management of chronic pain depends in part on the underlying
cause but some general principles can be applied. In general terms, the
treatment goals are to:
educate the patient
promote self-management
optimise function
enhance quality of life
control pain.
8.3 Genetic regulators of pain perception
Gene (protein) Mutation (inheritance) Protein function Phenotypes
SCN9A (Na,1.7) LoF (AR) Ion channel Absent pain, hypohydrosis, anosmia
SCN9A (Na,1.7) GoF (AD) Ion channel Erythromelalgia, paroxysmal pain, burning pain,
autonomic dysfunction
SCN11A (Na,1.9) GoF (AD) Ion channel Absent pain, hyperhydrosis, muscular weakness,
gut dysmotility
SCN10A (Na,1.8) GoF (AD) Ion channel Burning pain, autonomic dysfunction
TRPA1 (TRPA1) GoF (AD) Ion channel Absent pain
PDRM12 (PDRM12) LoF (AR) Transcription factor; neuron development Absent pain
NTRK1 (high-afnity
NGF receptor)
LoF (AR) Tyrosine kinase; promotes neuron
development
Absent pain; anhydrosis, mental retardation,
increased cancer risk
(AD = autosomal dominant; AR = autosomal recessive; GoF = gain of function; LoF = loss of function; NGF = nerve growth factor)
Cotton wool Neurology pin
Allodynia
Warm and cool thermal rollers
Increased or decreased thermal sensation
Hyperalgesia
Fig. 8.5 Equipment for bedside sensory testing.
191.
Pain 161
8
Clinicalhistory
Biopsychosocial assessment
A full biopsychosocial assessment should be performed in all patients
with chronic pain. Although this is time-consuming, the time invested is
likely to pay dividends in improving the long-term outcome for patients.
A biopsychosocial assessment takes account of the underlying neurobi-
ology of the condition in the context of wider inuences, including cogni-
tion and beliefs, emotions, and social and cultural factors. For example,
an individual with abdominal pain might respond differently if a close
relative had recently died of gastric cancer than if a colleague had been
off work with gastric upset.
An accurate clinical history is important, taking note of the dura-
tion of pain, any precipitating and relieving factors, its location and,
if the pain is located at more than one site, which site is the one
that impacts most on the patient’s quality of life. The characteris-
tics of the pain should be documented, by assessing whether it is
described as dull, sharp, aching or burning. Associated features,
such as hypersensitivity to ne touch or temperature, numbness,
paraesthesia, tingling and formication (the feeling of insects crawl-
ing over the skin), should be noted. It is important to determine to
what extent the pain is interfering with normal daily activities, such as
work, leisure pursuits and sleep. The patient’s social circumstances
and cultural background should be documented, including any car-
ing responsibilities, employment status and social and family sup-
port. The intensity of pain should also be recorded, preferably using
a validated questionnaire (see Box 8.4). The patient’s mood should
be assessed and, if evidence of low mood is detected, a suicide risk
assessment should be considered (see Box 31.14). The past medi-
cal and medication history should be recorded and specic enquiry
made about substance misuse and any previous history of physical or
mental abuse. This should be approached sympathetically, with infor-
mation about how to access appropriate support if required. It is also
useful to enquire specically about the patient’s beliefs as to what
is causing their pain, as well as what their expectation of treatment
is; unless these are addressed, management may be less effective.
There are some patient populations in whom particular challenges
arise, often related to differences in communication ability. Strategies
that can be used to overcome these difculties are summarised
in Box 8.5
Examination
The patient’s general appearance should be noted, including ability to
walk and use of a walking aid. In those with focal pain, neurological
examination should be performed, focusing particularly on any areas of
abnormal sensation, reexes and evidence of muscle wasting. A general
examination should be carried out to determine whether there is any
evidence of an underlying physical disorder that can account for the pain.
In addition to the use of investigations to nd the underlying cause of
pain, patients with persistent or chronic pain may benet from sensory
testing or diagnostic nerve blocks to explore the underlying mechanisms
and direct treatment. For example, a combined femoral and sciatic nerve
block may be used in a patient with lower limb amputation to assess
whether the pain is predominantly peripherally or centrally generated.
If the pain is not improved by an effective nerve block, then peripherally
directed therapies are unlikely to be effective.
8.4 Instruments used in the assessment of pain and its impact
Instrument Comments
Brief Pain Inventory Developed for use in cancer pain,
validated and widely employed for
chronic pain; based on 0–10 ratings
of pain intensity and the impact
of pain on a range of domains,
including sleep, work and enjoyment
of life
Pain Detect, s-LANSS, DN-4 A number of screening
questionnaires to aid diagnosis of
neuropathic pain
Pain Catastrophising Scale Developed to assess individual levels
of catastrophising, encompassing
three different domains:
helplessness, rumination and
magnication
Tampa Scale of Kinesiophobia Measures how much an individual is
fearful of movement
Pain Self-efcacy Questionnaire Assesses individual beliefs about
self-efcacy in the context of chronic
pain, and how this impacts on
function
Visual analogue scale (VAS) Patient marks pain intensity on a
horizontal line
Localisation of pain Body chart, allowing the patient to
indicate where pain is situated
Beck Depression Inventory Assesses emotional function
SF-36/EQ-5D Assesses health-related quality of life
(DN-4 = Douleur Neuropathique questionnaire; EQ-5D = EuroQol 5-Domain questionnaire;
SF-36 = Short Form 36; s-LANSS = self-completed Leeds Assessment of Neuropathic Signs
and Symptoms)
8.5 Challenges in pain assessment in particular patient populations
Patient population Challenges Solutions
Paediatric Assessment needs to be appropriate to developmental stage Consider visual tools to aid pain assessment
Older adults May have impaired cognitive function
Cultural factors may reduce self-reporting of pain
Risk of adverse effects of medication increased
Consider formal assessment of cognitive function
Consider non-verbal assessment
Consider visual tools to assess pain
Employ a number of tools assessing pain
behaviours
Cognitive impairment Reporting and expression of pain may change
Increased sensitivity to central nervous system effects of
analgesics
Perform formal assessment of cognitive function
Use non-verbal assessment: facial expressions,
vocalisations, body movements, changes in social
interactions
Substance misuse Response to analgesics altered
Increased tolerance
Increased risk of addiction
Substance misuse may affect reporting of pain
Seek specialist support early
Ensure prescribing is safe
192.
162 PAINAND PALLIATIVE CARE
Interventions
Probably the most effective mode of treatment for pain is to identify the
underlying cause. Examples include the use of immunosuppressive med-
ication in inammatory disease, chemotherapy, radiotherapy or hormone
therapy in cancer, and antimicrobial therapy in patients with infection.
There are many circumstances, however, in which the underlying cause
of pain cannot be treated or the treatments available are incompletely
effective. Under these circumstances, several management options are
available. In all cases, a multidisciplinary approach is necessary that
combines pharmacological management with supported self-manage-
ment, and other specic interventions when appropriate.
Supported self-management
Self-management strategies are useful in the treatment of chronic pain.
Self-management works best if the patient has some understanding of
their chronic pain, and acceptance that it is unlikely to resolve completely.
The aim is for patients to maximise their quality of life and function despite
ongoing pain. Support for self-management can be delivered by health-
care professionals, patients who suffer from the same condition or lay
people, either on an individual basis, in a group setting or, increasingly,
through web-based resources. There is a strong educational component
to supported self-management, which seeks to generate an interaction
between patient and tutor. The key aspects include:
increasing activity levels, while understanding and practising pacing
techniques (not overdoing things and cycling between over- and
under-activity)
using relaxation and mindfulness techniques as part of daily
management
using medication when appropriate
having a plan to manage pain ares.
All these are covered in formal pain management programmes (see
‘Psychological therapies’), where a structured approach is used, in a
group-based setting, to address all these aspects with expert multidis-
ciplinary input, that may include psychologists, occupational therapists,
nurses, pharmacists and pain medicine specialists.
For less complex cases, or to provide ongoing support, there are a
number of useful online self-help resources (see ‘Further information’).
Physical therapies
There is strong evidence that exercise can help in the management of
chronic pain. Several types of exercise have been successfully delivered
in various ways, through physiotherapists, exercise classes or individual
tuition. In choosing a form of exercise therapy, it is important to tailor
the approach most likely to be acceptable to the individual patient. A
successful exercise programme can help overcome ‘fear avoidance’, a
well-recognised problem in chronic pain, where patients associate activ-
ity with an increase in pain and therefore do progressively less activity,
with resultant deconditioning. Because of this it is important to pace
physical activity to ensure that patients do not cycle from over-activity,
with a are in pain, to fatigue and deconditioning. This can be done by
working with patients to establish their baseline level of activity and using
an individually tailored, graded exercise programme (Box 8.6). This may
include normal household activities, as well as targeted exercises and
stretches. Manual therapy covers a variety of hands-on treatments,
including manipulation, mobilisation and massage. Manual therapy can
be provided by a range of therapists, including physiotherapists, osteo-
paths and chiropractors. There is some evidence of short-term benet
for manual therapy but limited evidence of long-term efcacy.
Pharmacological therapies
A range of analgesics can be used in the management of chronic pain
but, for most of these, the evidence of long-term benet is limited, and
there may be considerable risks associated with long-term use for some
agents. If using analgesics as part of a holistic treatment plan (e.g. to
manage short-term are-ups, or target neuropathic pain), in general, it
is advisable to use a multimodal approach, choosing different drugs to
target pain processing at multiple points (Box 8.7). By employing different
classes of analgesic, it is possible to use lower doses of each, thereby
improving the side-effect prole and reducing risk. There is considera-
ble inter-individual variability in response to analgesics, even within the
same class. There are many reasons for this, including genetic variations
in the enzymes that metabolise drugs. For example, the CYP2D6 gene
encodes for a liver enzyme, cytochrome P450 2D6, which metabolises
a number of commonly used analgesics. Genetic variation in CYP2D6
can inuence circulating levels of many drugs, depending on whether
someone is a rapid or poor metaboliser. This is particularly important if
metabolites are active, as is the case with codeine and dihydrocodeine,
which are metabolised to morphine. Genetic variations have also been
described in the opioid receptors and downstream pathways that they
affect, with good pre-clinical evidence that variations in mu opioid recep-
tors alter analgesic response to different opioids. Because of this there is
a good rationale to try different drugs, even ones from the same class, if
there is an inadequate response or there are unacceptable side-effects
with one agent.
Whatever drug or combination of drugs is chosen, the key to suc-
cessful pharmacological management is careful assessment and review,
aiming for an acceptable balance between the benets of treatment in
providing pain relief, maximising function, and improving quality of life
and adverse effects. Specic drug treatments are described below.
Non-opioid analgesics
Paracetamol
Paracetamol is widely used in the treatment of mild to moderate pain.
Its mechanism of action is incompletely understood but it is known to
be a weak inhibitor of the cyclo-oxygenase type 1 (COX-1) and cyclo-
oxygenase type 2 (COX-2) enzymes, providing weak anti-inammatory
properties. There is also some evidence that it activates inhibitory
descending spinal pathways, via a serotonergic mechanism. Other
postulated mechanisms include endocannabinoid re-uptake inhibi-
tion, and inhibition of nitric oxide and tumour necrosis factor alpha. For
migraine and tension-type headache it has moderate efcacy at a dose
of 1000mg. It is used widely for musculoskeletal disorders and osteo-
arthritis, with very little high-quality evidence that it is much better than
placebo, even at doses of up to 4000mg per day. Acute liver failure is a
well-recognised complication of paracetamol overdose but this risk may
also be increased with long-term use, even within the recommended
dose range. In view of this, it should be employed with caution in older
patients and those weighing less than 50kg.
Non-steroidal anti-inammatory drugs
Non-steroidal anti-inammatory drugs (NSAIDs) are widely used in the
treatment of inammatory pain and osteoarthritis. These drugs can be
given systemically or locally and are discussed in more detail in Chapter 26.
They are also useful in the management of pain in cancer patients, as
discussed later in this chapter. Although widely prescribed, there is
8.6 Physical therapies for chronic pain
Land-based
Walking
Gym work
Exercise classes
Yoga
Pilates
Tai-chi
Water-based
Hydrotherapy
Swimming
Exercise classes
193.
Pain 163
8
limitedhigh-quality evidence of long-term efcacy in chronic pain, and
concerns about risks of long-term use particularly around cardiovascular
and renal effects. There is a clear need for further studies in this area.
Topical analgesics
Topical capsaicin cream (0.025 or 0.075%) has some efcacy for osteo-
arthritis and may be used for neuropathic pain, although evidence of
benet is limited. A single application (done by a trained health-care
professional) of a high-dose 8% capsaicin patch can give around
12 weeks of pain relief for neuropathic pain and can be repeated there-
after. Capsaicin is an agonist at the transient receptor potential vanilloid
1 (TRPV1) ion channel, found on some C bres. Capsaicin activates the
channel, causing an initial sensation of heat, but an analgesic effect sub-
sequently results due to desensitisation of the channel, with a reduction
in intra-epidermal nerve bre density.
Lidocaine 5% patches can also be helpful in focal neuropathic pain
and should be applied for 12 hours out of 24 hours, with up to 4–6
weeks before maximum benet is seen. The mode of action is blockade
of sodium channels in primary afferent neurons and nociceptors, which
reduces peripheral input to the spinal cord.
Anti-neuropathic agents
Anti-neuropathic agents are also termed ‘adjuvant analgesics’. This term
is used to cover a range of medicines that are employed in the treatment
of neuropathic pain, and in certain patients with CWP. It should be noted
that these medicines should only be used as part of a holistic management
plan, including physical and psychological therapies, sometimes in com-
bination with classical analgesics. Typically, these agents do not produce
an immediate reduction in pain, but rather exert an analgesic effect over a
longer timeframe through their effects on central processing of pain. They
are of particular value when used in combination in the management of
pain with a neuropathic component but require careful dose titration over a
number of weeks, to reach a dose that balances efcacy with side-effects.
While the response to individual agents is variable, it is often possible to
nd an agent or combination of agents that works for most patients.
In the majority of current recommendations, including those from
the International Association for the Study of Pain, rst-line treatments
include antidepressants such as serotonin noradrenaline reuptake
inhibitors (mainly duloxetine) or tri-cyclic anti-depressants such as ami-
triptyline and nortriptyline. Anti-epileptic agents such as gabapentin or
pregabalin are also recommended for use as second-line treatments,
although there is increasing concern around their abuse potential and
association with an increase in drug deaths.
Opioid analgesics
Opioids have been used for centuries to reduce pain, and were origi-
nally obtained from the resin of the opium poppy (Papaver somniferum).
Over recent years, new synthetic opioids such as fentanyl, oxycodon
and tramadol have been introduced. The pharmacological effects of opi-
oids are mediated by binding to a number of opioid receptors which are
G-protein-coupled receptors, as summarised in Box 8.8. When opioids
bind to their receptors, several intracellular signalling pathways are acti-
vated, increasing cyclic adenosine monophosphate (cAMP) levels and
altering calcium and potassium permeability of neurons. Opioids are tra-
ditionally divided into subclasses of weak opioids, such as codeine and
dihydrocodeine, and strong opioids, such as morphine and oxycodone.
While tramadol is a weak agonist at the mu opioid receptor, it is classi-
ed as a strong opioid in some countries. Codeine and dihydrocodeine
are metabolised in the liver to morphine by CYP2D6 at a rate that is
genetically determined. Similarly, tramadol is metabolised by CYP2D6 to
yield o-desmethyltramadol, which has greater afnity than tramadol for
opioid receptors. Up to 10% of people are rapid metabolisers and this
may be associated with differences in efcacy and adverse effects of
these drugs between individuals (see Box 2.5). The dosages and char-
acteristics of commonly prescribed opioids are shown in Box 8.9. There
has been a large increase in the use of strong opioids for chronic pain
over the last few decades, to the extent that it has been referred to as
an ‘opioid epidemic’. A number of factors contribute to this, including a
rising incidence of chronic pain with an ageing population, reluctance
to use NSAIDs because of cardiovascular and gastrointestinal adverse
effects, changes in patient expectation, societal attitudes and availa-
bility of new formulations of opioids. There is very limited evidence of
short- to medium-term benet for strong opioids in low back pain and
osteoarthritis with more recent longer-term evidence nding no improve-
ment in function, worse pain and increased side-effects in people on
opioids compared to non-opioid analgesia. More good-quality studies of
8.7 Pharmacological management of chronic pain
Drug or class of drug Mechanism of action
Paracetamol Central inhibition of COX-1 and COX-2 enzymes
Mechanisms of action incompletely understood
Non-steroidal anti-inammatory drugs Inhibition of prostaglandin production
Opioids Agonists at opioid receptors at multiple levels in the central nervous system
Blockade of ascending pain pathways
Ketamine Antagonist of NMDA receptors
Reduction of central sensitisation
Gabapentin and pregabalin Inhibition of glutamate release by primary afferent neurons at rst central synapse
Decrease of excitatory neuronal activity
Tricyclic antidepressants Inhibition of serotonin and noradrenaline (norepinephrine) re-uptake at synapses in
the spinal cord, and also potential effects in the limbic system
Inhibition of Na+
channels in neurons
Serotonin (5-hydroxytryptamine, 5-HT) and
noradrenaline (norepinephrine) re-uptake inhibitors
Inhibition of serotonin and noradrenaline re-uptake at synapses in the spinal cord,
and also potential effects in the limbic system
Lidocaine patches Inhibition of Na+
in sensory neurons
Capsaicin patch Activation of TRPV1 channels on subset of C bres, causing selective
pharmacological denervation, with a decrease in intra-epidermal nerve bre
density
Nerve blocks with lidocaine and glucocorticoids Temporary denervation due to blockade of Na+
channels in sensory neurons
Local anti-inammatory effect
(COX = cyclo-oxygenase; NMDA = N-methyl-D-aspartate; OP = opioid; TRPV1 = transient receptor potential vanilloid 1)
194.
164 PAINAND PALLIATIVE CARE
long-term use are needed. Additionally, there is increasing concern about
potential harm from long-term use. This includes addiction, dependence,
opioid-induced hyperalgesia, endocrine dysfunction, fracture risk (espe-
cially in older people), overdose and cardiovascular events, with many
of these adverse effects being dose-related. Doses of more than 50mg
morphine equivalents per day may be detrimental, with an increased in
harm at doses of >90mg morphine equivalents per day. National and
international guidelines have changed to reect this, with most only rec-
ommending short- to medium-term use of opioids in carefully selected
patients, as part of a holistic management plan. Regular review is essen-
tial to assess ongoing benet, and any opioid trial should have clear
goals, and a plan for cessation if these are not reached. A suggested
strategy for using strong opioids in chronic pain is shown in Box 8.10
Psychological therapies
The aims of psychological therapy are to increase coping skills and
improve quality of life when facing the challenges of living with chronic
pain. There are a range of ways in which psychological therapies can
be delivered, including individual one-to-one sessions, group sessions,
multidisciplinary pain management programmes, or web-based or tele-
phone-based programmes.
There is good evidence for the use of a cognitive behavioural therapy
(CBT)-based approach for chronic pain, delivered either individually or in
a group. The overall aim is to reduce negative thoughts and beliefs, and
develop positive coping strategies. The interaction between thoughts,
behaviours and emotions is explored, and a problem-focused approach
is used in therapy delivery.
Relaxation techniques, such as biofeedback and mindfulness medi-
tation, require a degree of stillness and withdrawal, with regular practice
required for sustained benet (see ‘Further information’). Acceptance
and commitment therapy (ACT) is based on CBT principles but also uses
components of mindfulness to improve psychological exibility in the
context of living with chronic pain.
Stimulation therapies
These range from minimally invasive procedures like acupuncture and
transcutaneous electrical nerve stimulation (TENS) to more invasive
techniques such as spinal cord stimulation.
8.8 Endogenous opioids and opioid receptors
Endogenous ligand Receptor
(IUPHAR)
Alternative
classication
Potential sites Pharmacological effects
Endomorphin 1 and 2
Met-enkephalin
Dynorphin A
Dynorphin B
MOP Mu Brain, spinal cord, peripheral nerves,
immune cells
Analgesia, reduced gastrointestinal
motility, respiratory depression, pruritus
Leu-enkephalin
Met-enkephalin
β-endorphin
DOP Delta Brain, spinal cord, peripheral nerves Analgesia, cardioprotection,
thermoregulation
Dynorphin A
Dynorphin B
β-endorphin
KOP Kappa Brain (nucleus accumbens, neocortex,
brainstem, cerebellum)
Analgesia, neuroendocrine
(hypothalamic–pituitary axis), diuresis,
dysphoria
Orphanin FQ (nociceptin) NOP Orphan Nucleus raphe magnus, spinal cord,
afferent neurons
Opioid tolerance, anxiety, depression,
increased appetite
It is thought that opioids used clinically act through the MOP.
(IUPHAR = International Union of Basic and Clinical Pharmacology)
8.9 Commonly used opioids
Opioid Typical starting dose Route Oral morphine equivalent Comments
Morphine 10mg Oral 10mg Most widely used
Codeine 30–60mg; max 240mg/24hr Oral 3–6mg Metabolised to morphine byCYP2D6 enzyme
Dihydrocodeine 60mg; max 240mg/24hr Oral 6mg Metabolised to morphine byCYP2D6 enzyme
Tramadol 100mg; max 400mg/24hr Oral 10mg Metabolised to o-desmethyl tramadol by
CYP2D6 enzyme
Oxycodone 6.6mg Oral 10mg More predictable bioavailability than morphine
Buprenorphine 5µg/hr Transdermal 12mg/day Patch change usually every 7 days
(frequency of change dependent on
manufacturer and dose); advantages
in impaired renal function
Fentanyl 12µg/hr Transdermal 30mg/day Use with care in opioid-naïve patients;
patch change usually every 72 hrs
Tapentadol 50mg; max 600mg/24hr Oral 20mg Use with care in opioid-naïve patients
Hydromorphone 2mg Oral 10mg Semi-synthetic; hepatic metabolism
Morphine 3mg Subcutaneous,
intramuscular, intravenous
10mg Mainly used for acute pain or palliative care
195.
Pain 165
8
Acupuncture(Fig. 8.6) has been used successfully in Eastern med-
icine for centuries. The mechanisms are incompletely understood,
although endorphin release may explain, in part, the analgesic effect.
Acupuncture is particularly effective in pain related to muscle spasm,
with some evidence of short-term benet for patients with low back pain.
Similar mechanisms probably apply to TENS, which is worth consider-
ing in many types of chronic pain. Neuromodulation, using implanted
electrodes in the epidural space (or, more recently, adjacent to periph-
eral nerves), has been shown to be an effective option for neuropathic
pain, including failed back surgery syndrome and chronic regional pain
syndrome (see below). Specialist assessment and ongoing support is
necessary, as there are many potential complications, including infection,
malfunction and battery failure. The likelihood of success is increased
when this technique is used within the context of multidisciplinary
assessment and management.
Complementary and alternative therapies
Complementary techniques, such as herbal medicines, vitamins,
homeopathy and reexology, have been used for the treatment of
chronic pain but with little evidence of efcacy. It should be noted that
herbal medications may interact with conventional drugs, causing
adverse effects as the result of drug–drug interactions. St John’s wort
(Hypericum perforatum) interacts with many drugs, including many
antidepressants used in chronic pain, with increased serotonergic
effects. Grapefruit may also increase the risk of serotonergic effects
with some antidepressants. Ginkgo biloba may interact with paraceta-
mol to increase bleeding time.
Nerve blocks and nerve ablation
The use of specialist nerve blocks and nerve ablation therapy can be
considered for pain that is unresponsive to less invasive approaches.
If these are being considered, they should form part of a multidis-
ciplinary management plan, with the aim of restoring function and
reducing pain. Local anaesthetic with or without depot glucocorticoid
(non-particulate for neuraxial administration) can be effective in some
circumstances. Examples include occipital nerve blocks for migraine
or cervicogenic headache and trigger point injections for myofascial
pain. If there is limited compression of a spinal nerve root, the nerve
root injections into the epidural space may help settle symptoms and
avoid the need for surgical intervention. Neurodestructive procedures
can also be employed for intractable pain but are rarely used outside
the palliative care setting.
Chronic pain syndromes
Chronic pain is a feature of several recognised syndromes, which are
discussed in more detail below.
Neuropathic pain
Neuropathic pain is dened as ‘pain associated with a lesion or disease
of the somatosensory nervous system’. Neuropathic pain may be acute,
such as in sciatica, which occurs as the result of a prolapsed disc, but
is most problematic when it becomes chronic. Neuropathic pain causes
major morbidity; in a recent study, 17% of those affected rated their
8.10 Use of opioids in chronic pain: Always use as part of a holistic management plan
Step Factors to take into account Comment
1. Assess suitability for opioids Type of pain Neuropathic pain and chronic widespread pain less likely to respond
Likelihood of dependence Increased risk in those with history of alcohol and substance misuse
Co-morbidity Avoid use in conditions where adverse effects more likely:
Chronic obstructive pulmonary disease
Chronic liver disease
Chronic kidney disease
2. Discuss with patient Discuss potential benets Improvement in pain
Improvement in function
Discuss adverse effects Nausea
Constipation
Drowsiness
Establish treatment goal Improvement in function
3. Plan treatment trial Set timescale Dene duration of treatment
Agree frequency of review
Agree on dose Aim for lowest effective dose
Set upper dose limit
Agree on stopping rules Consider stopping if:
Treatment goal is not met
There is no dose response
Tolerance develops rapidly
Fig. 8.6 Acupuncture.
196.
166 PAINAND PALLIATIVE CARE
quality of life as ‘worse than death’. The clinical features of neuropathic
pain are summarised in Box 8.11. The diagnosis is easily missed and
so careful assessment is vital, in order to make the diagnosis in the rst
place and then to direct management appropriately. An algorithm for the
management of neuropathic pain is provided in Figure 8.7. It is important
to recognise the negative impact of neuropathic pain on quality of life,
which has been shown to be greater than with other types of chronic
pain. As a result, appropriate support and multidisciplinary management
should always be considered in addition to pharmacological therapies.
Complex regional pain syndrome
Complex regional pain syndrome (CRPS) is a type of neuropathic pain that
affects one or more limbs. It was previously termed reex sympathetic dystro-
phy (RSD), reecting the fact the disease is thought to be caused in part by an
abnormality in the autonomic nervous system. It is a rare syndrome, occur-
ring in about 20 per 100000 individuals, and is more common in females,
typically presenting between the ages of 35 and 50. It is classied into type
1 CRPS, which may be precipitated by a traumatic event such as a fracture
but is not associated with peripheral nerve damage, and type 2 CRPS, which
is associated with a peripheral nerve lesion. The diagnosis is primarily clinical,
with the current standard being based on the Budapest criteria, as outlined in
Box 8.12. Other diagnostic tools, none of which provide a denitive diagno-
sis, include thermography (temperature difference of >1°C), and electromy-
ography (if myoclonus is a feature). Increased tracer uptake on radionuclide
bone scan and bone marrow oedema on MRI scan may be observed in
CRPS but the diagnosis is primarily clinical, as outlined in Box 8.12
Prompt diagnosis and early treatment with physiotherapy, and addi-
tional approaches such as desensitisation, and graded motor imagery,
may prevent progression of symptoms. Pharmacological management
is similar to that for neuropathic pain. Specic approaches (with varia-
ble quality evidence) that may be considered include bisphosphonates
or calcitonin. Intravenous regional block with guanethidine is not rec-
ommended, as there is limited evidence of benet, and an increased
risk of adverse effects. If medical management is incompletely effective,
consideration should be given to the appropriateness of a spinal cord
stimulator, with reasonable evidence of efcacy.
Phantom limb pain
Phantom limb is a common complication of amputation, occurring
in up to 70% of patients. It is a form of neuropathic pain but can
8.11 Clinical features of neuropathic pain
Characteristic Symptom or
clinical feature*
Descriptive term
Spontaneous pain No stimulus required
to evoke pain
Positive sensory
disturbance
Light touch painful
Pressure painful
Increased pain on
pin-prick
Cool and warm
temperatures
painful
Dynamic allodynia
Punctate allodynia
Hyperalgesia
Thermal allodynia
Negative sensory
disturbance
Numbness
Tingling
Loss of temperature
sensitivity
Loss of sensation
Paraesthesia
Other features Feeling of insects
crawling over skin
Affected area feels
abnormal
Formication
Dysaesthesia
*Symptoms may cluster, with a predominance of either positive or negative symptoms, or a
mixture of both, reecting differences in underlying mechanisms.
Are there clinical features of
neuropathic pain? (Box 8.11)
Assess likelihood of
neuropathic pain
Tricyclic antidepressant
Gabapentin or pregabalin
SNRI
First line
(moderate to high
evidence; strong
recommendation)
Probable
Possible Definite
Response? Continue
Yes
Capsaicin patch
Lidocaine patch
Tramadol
Second line
(moderate evidence;
weak recommendation)
No
Yes
Response? Continue
Botulinum toxin
Strong opioids
Third line
(moderate evidence;
weak recommendation)
No
Yes
Fig. 8.7 Algorithm for pharmacological management of neuropathic pain.
(SNRI = serotonin noradrenaline (norepinephrine) re-uptake inhibitor) Adapted from
SIGN 136 and NeuPSIG recommendations; reproduced from Finnerup NB, Attal N,
Haroutounian S, et al. Pharmacotherapy for neuropathic pain in adults: a systematic
review and meta-analysis. The Lancet Neurol 2015; 14:162–173, with permission
from Elsevier.
be particularly distressing, as the pain is felt in the area where the
absent limb was previously. Although usually presenting after limb
amputation, reports of phantom pain in other body parts have been
reported, such as phantom breast pain following mastectomy. It is
very often associated with phantom sensations, which are described
as non-painful sensations in the absent body part and pain in the
stump.
Diagnostic nerve blocks may be helpful in directing therapy, with
use of anti-neuropathic medications as outlined in Box 8.7. If there
is a denite neuroma at the stump site that is interfering with pros-
thesis use, surgical review may be necessary. Management should
use rehabilitation approaches with physical therapy. Additional
approaches such as mirror visual feedback and desensitisation may
also be considered.
Chronic widespread pain
Chronic widespread pain (CWP) is often associated with other features,
such as fatigue and irritable bowel syndrome. Fibromyalgia is a subtype
of CWP in which there are myofascial trigger points, and is often associ-
ated with sleep disturbance. Clinical features and management of bro-
myalgia are discussed in more detail in Chapter 26.
Joint hypermobility syndrome
Hypermobility can be associated with chronic musculoskeletal pain
that often targets the joints and periarticular tissues. It is thought to be
caused by abnormal stresses being placed on the joints and surrounding
197.
Palliative care 167
8
soft tissues due to ligament laxity, although the mechanisms are poorly
understood since many people with hypermobile joints do not suffer
pain. It is described in more detail in Chapter 26
Palliative care
Palliative care is the term used to describe the active total care of
patients with incurable disease. It can be distinguished from end-of-life
care, which refers to the care of patients with far advanced, rapidly pro-
gressive disease that will soon prove fatal. The focus of palliative care
is on symptom control alongside supportive care. While palliative care
can and should be delivered at any stage of an incurable illness along-
side optimal disease control, the focus of end-of-life care is on quality of
life rather than prolongation of life or cure. Palliative care encompasses
a distinct body of knowledge and skills that all good physicians must
possess to allow them to care effectively for patients. Palliative care is
seen traditionally as a means of managing distress and symptoms in
patients with cancer, when metastatic disease has been diagnosed and
death is seen as inevitable. However, prognosis in metastatic cancers
has evolved signicantly with the emergence of improved anti-cancer
treatments, hence the principle of delivering palliative and supportive
care whenever a patient needs it, as opposed to a specic stage of dis-
ease, is now even more important. There is also a growing recognition
that the principles of palliative care and some of the interventions it uses
are equally applicable in other conditions. Palliative care may be applied
to any chronic disease state. Which elements are used will depend on
the individual patient’s need.
For conditions other than cancer, the challenge is recognising when
patients have entered the terminal phase of their illness, as there are
fewer clear markers and the course of the illness is much more variable.
Different chronic disease states progress at different rates, allowing some
general trajectories of illness or dying to be dened (Fig. 8.8). These tra-
jectories are useful in decision-making for individual patients but also in
the planning of services.
The ‘rapid decline’ trajectory following a gradual decline, as occurs
in cancer, is the best-recognised pattern of the need for palliative care,
although a similar trajectory may be observed in other conditions,
such as motor neuron disease. Many traditional hospice services are
designed to meet the needs of people on this trajectory. Over recent
years, improvements in management of malignant disease mean that
some types of cancer may follow an erratic or intermittent decline
trajectory.
Many chronic diseases, such as advanced chronic obstructive pul-
monary disease (COPD) and intractable congestive heart failure, carry
as high a burden of symptoms as cancer, as well as psychological and
family distress. The ‘palliative phase’ of these illnesses may be more dif-
cult to identify because of periods of relative stability interspersed with
acute episodes of severe illness. However, it is still possible to recognise
those patients who may benet from a palliative approach to their care.
The challenge is that symptom management needs to be delivered at
the same time as treatment for acute exacerbations. This leads to dif-
cult decisions as to the balance between symptom relief and aggressive
management of the underlying disease. The starting point of need for
palliative care in these conditions is the point at which consideration of
comfort and individual values becomes important in decision-making,
often alongside management of the underlying disease.
The third major trajectory is categorised by years of poor function and
frailty before a relatively short terminal period; it is exemplied by demen-
tia but is also increasingly true for patients with many different chronic
illnesses. As medical advances extend survival, this mode of dying is
being experienced by increasing numbers of people. The main challenge
lies in providing nursing care and ensuring that plans are agreed for the
time when medical intervention is no longer benecial.
In a situation where death is inevitable and foreseeable, palliative care
aims to strike a balance between addressing the wishes, expectations
and values of the patient with a realistic assessment of the benets of
medical interventions. This often results in a greater focus on comfort,
symptom control and support for patient and family, and may enable
withdrawal of both futile and burdensome interventions. In cases of
prognostic uncertainty, open, honest and gentle communication with
the patient and family is important. The most common symptoms in
palliative care are discussed in the next section. The central networks
we have heard about above in relation to chronic pain also exist for all
other symptoms. In addition there is an extra complexity in cancer as a
result of the additional layer of biological complexity resulting from the
factors released by the tumour such as pro-inammatory cytokines and
the body’s response to these factors.
Presenting problems in palliative care
Pain
Pain is a common problem in palliative care. It has been estimated that
about two-thirds of patients with cancer experience moderate or severe
pain, and a quarter have three or more different sites of pain. Many of
8.12 The Budapest criteria for diagnosis of complex regional
pain syndrome (CRPS)
Category Symptom or sign
Sensory Allodynia to:
Temperature
Light touch
Deep somatic pressure
Movement
Hyperalgesia to pin-prick
Vasomotor Temperature asymmetry
Skin colour change and/or
asymmetry
Sudomotor Oedema
Sweating change and/or
asymmetry
Motor/trophic Reduced range of motion
Motor dysfunction:
Weakness
Tremor
Dystonia
Trophic changes:
Hair
Nails
Skin
Type 1 CRPS: without evidence of major nerve damage; type 2 CRPS: evidence of major nerve
damage. For a positive diagnosis, the patient should report at least one symptom in at least
three out of the four categories, and at least one sign should be detected in two out of the four
categories. Other causes that might explain the signs and symptoms should be excluded.
High
Death
Cancer
Organ failure
Physical and cognitive frailty
Function
Low
Time
Fig. 8.8 Archetypal trajectories of dying. From Murray SA, Kendall M, Boyd K,
et al. Illness trajectories and palliative care. BMJ 2005; 330:7498; reproduced with
permission from the BMJ Publishing Group.
198.
168 PAINAND PALLIATIVE CARE
these are of a mixed aetiology and about half of patients with cancer-
associated pain have a neuropathic element.
Clinical assessment
The commonest barrier to assessment of pain is the development of idio-
syncratic shortcuts in the belief that these are also easier for the patient.
A common example is, ‘How is your pain?’ The patient will often give a
reex response such as, ‘OK’, which might be documented by the clini-
cian as 0, 1 or 2 out of 10. If the same patient were asked, ‘Have you had
any pain in the last 24 hours, on a scale of 0–10, where 0 is none at all
and 10 is the worst imaginable?’ the same patient might record a score
as high as 8 or 9 out of 10. This emphasises the importance of using a
validated question that screens for pain effectively.
Improved control of cancer-associated pain has been demonstrated
with use of the Edinburgh Pain Assessment and management Tool (EPAT).
This involves administration of a simple screening question about worst
pain on a scale of 0–10 in the last 24 hours, followed by questions to
identify specic pain/s such as neuropathic or bone, in addition to distress.
The identied pains are linked to management algorithms. Review of pain
and opioid side-effects completes the loop (see ‘Further information’).
Clinical features and suggested management strategies for common
types of pain in cancer are shown in Box 8.13. The majority of patients
with cancer-associated pain can be managed effectively using a step-
wise approach, as outlined below.
Management: pharmacological treatments
Pharmacological treatments are the mainstay of management in can-
cer-associated pain, however, they have to be underpinned by an
appropriate assessment of pain and associated symptoms. In addition,
non-pharmacological treatments and disease-modifying treatments run
through each step of pain control. A stepwise approach is adopted, fol-
lowing the principles of the World Health Organization (WHO) analgesic
ladder (Fig. 8.9), in which analgesia that is appropriate for the degree of
pain is prescribed rst. Patients with mild pain should be started on a
non-opioid analgesic drug, such as paracetamol (maximum: 1g 4 times
daily) or an NSAID (step 1). If the patient fails to respond adequately
or has moderate pain, a weak opioid, such as codeine (60mg 4 times
daily), should be added (step 2). This can be prescribed separately or
in the form of the compound analgesic co-codamol. If pain relief is still
not achieved or if the patient has severe pain, a strong opioid should be
substituted for the weak opioid (step 3). If the pain is severe at the outset,
strong opioids should be prescribed and increased or titrated according
to the patient’s response. It is important not to move ‘sideways’ (change
from one drug to another of equal potency), which is a common problem
during step 2 of the analgesic ladder.
In recent years there has been a tendency to use a low dose of a step
3 opioid rather than a traditional step 2 opioid such as codeine. It has
now been demonstrated that it is safe, effective and has cost savings
to miss out step 2 of the analgesic ladder. This is of great importance in
low- and middle-income countries where step 2 drugs are usually much
more expensive than morphine.
Opioids
Opioid analgesia plays a key role in patients with moderate to severe
pain. Its successful use depends on appropriate assessment and a
detailed explanation to the patient and carer about the benets and
potential side-effects of therapy. Morphine is the most commonly pre-
scribed strong opioid, although there are several alternatives, as outlined
in Box 8.9
Oral morphine takes about 20 minutes to exert an effect and usu-
ally provides pain relief for 4 hours. Most patients with continuous pain
should be prescribed oral morphine every 4 hours initially, as this will
provide continuous pain relief over the whole 24-hour period. Controlled-
release morphine lasts for 12 or 24 hours, depending on the formulation,
and if clinical circumstances dictate, a controlled-release formulation can
be used to initiate and titrate morphine. The median effective morphine
equivalent dose for cancer pain is about 200mg per 24 hours.
In addition to the regular dose of morphine, an extra dose of imme-
diate-release (IR) morphine should be prescribed ‘as required’ for the
treatment of breakthrough pain that has not been controlled by the reg-
ular prescription. As a rule of thumb, this additional dose should be one-
sixth of the total 24-hour dose of opioid. The frequency of breakthrough
8.13 Common types of pain in cancer
Type of pain Features Management options
Bone pain Tender area over bone
Possible pain on movement
NSAIDs
Bisphosphonates
Radiotherapy
Increased
intracranial
pressure
Headache, worse in the morning,
associated with vomiting and
occasionally delirium
Glucocorticoids
Radiotherapy
Opioid
Abdominal
colic
Intermittent, severe,
spasmodic, associated with
nausea or vomiting
Antispasmodics
Hyoscine
butylbromide
Antiemetic and opioid
may be required for
intestinal obstruction
Liver capsule
pain
Right upper quadrant
abdominal pain, often associated
with tender enlarged liver
Responds poorly to opioids
Glucocorticoids
Neuropathic
pain
Spontaneous pain
Light touch, pressure and
temperature changes are painful;
increased pain on pin-prick
Numbness, tingling or loss of
temperature sensation
Skin feels abnormal
Anticonvulsants:
Gabapentin
Pregabalin
Antidepressants:
Amitriptyline
Duloxetine
Ketamine
Ischaemic
pain
Diffuse, severe, aching pain
associated with evidence of poor
perfusion
Responds poorly to opioids
NSAIDs
Ketamine
Incident pain Episodic pain usually
related to movement
Intermittent
short-acting opioids
Nerve block
(NSAIDs = non-steroidal anti-inammatory drugs)
Opioid for moderate
to severe pain
± non-opioid
± adjuvant
Opioid for mild to
moderate pain
± non-opioid
± adjuvant
Non-opioid
± adjuvant
Pain persisting
or increasing
Pain persisting
or increasing
Freedom from
cancer pain
Pain
Fig. 8.9 The WHO analgesic ladder. From WHO. Cancer pain relief, 2nd edn.
Geneva: WHO; 1996.
199.
Palliative care 169
8
doses should be dictated by their efcacy and any side-effects, rather
than by a xed time interval. A patient may require breakthrough analge-
sia as frequently as hourly if pain is severe, but this should lead to early
review of the regular prescription. The patient or carer should note the
timing of any breakthrough doses and the reason for them. These should
be reviewed daily and the regular 4-hourly dose increased for the next
24 hours on the basis of:
the frequency of and reasons for breakthrough analgesia
the degree and acceptability of side-effects.
The regular dose should be increased by adding the total of the break-
through doses over the previous 24 hours, unless there are signicant
problems with unacceptable side-effects. When the correct dose has
been established, a continuous release (CR) preparation can be pre-
scribed, usually twice daily. Breakthrough analgesia used for move-
ment-related pain is generally not included in background opioid dose
titration. Attempts to control movement-related pain with background
opioid dose will usually lead to over-dosing and opioid-related side-ef-
fects. This can be a risk in metastatic bone pain.
Some patients may have concerns about using opioids and it is vital
for these to be explored. Patients should be reassured that psycholog-
ical dependence is rare when opioids are used for cancer pain, unless
a pre-existing dependence problem exists. Recent survey data from the
United States have suggested that opioid abuse may occur in up to 20%
of cancer patients, but to-date, these data have not been replicated in
other countries. In fact, the majority of the world’s population do not have
any access to opioids for cancer pain relief. Good prescribing practice,
clear communication and reassessment of benets and side-effects are
all critical in achieving good cancer pain relief. Pharmacological tolerance
is not usually a clinically relevant problem; however, physical depend-
ence, which is physiological, as manifest by a physical withdrawal syn-
drome, can occur if opioids are suddenly discontinued.
Nearly all types of cancer pain respond to morphine to some degree
but there is a spectrum of response, such that in some patients the
dose of opioid required to control neuropathic pain and all elements of
metastatic bone pain may be high and associated with unacceptable
side-effects. In these situations, other methods of analgesia, both phar-
macological and non-pharmacological, should be explored and consid-
ered at an early stage.
The most effective and appropriate route of morphine administration
is oral but transdermal preparations of strong opioids (usually fentanyl)
are useful in certain situations, such as in patients with dysphagia or
those who are reluctant to take tablets on a regular basis. Diamorphine
is a highly soluble strong opioid used for subcutaneous infusions, par-
ticularly in the last few days of life, but is only available in certain coun-
tries and morphine is now the most commonly prescribed parenteral
opioid.
Opioid-related adverse effects
Adverse effects are a common problem with opioids, especially on initiat-
ing treatment and on increasing the dose. The most common side-effects
are nausea, drowsiness, constipation and dry mouth, as summarised
in Box 8.14. Nausea and vomiting can occur initially but usually settle
after a few days. Drowsiness is usually transient at opioid initiation and
dose increase. If it is persistent, an alternative opioid and/or a non-opioid
should be considered. In acute dosing, respiratory depression can occur
but this is rare in patients on regular opioids or in those starting on small,
regular doses with appropriate titration.
Tolerance usually develops to nausea, vomiting and drowsiness but
not to constipation or dry mouth. All patients should therefore be pre-
scribed a laxative, unless suffering from diarrhoea, and have access to
an antiemetic and good mouth care, along with rationalisation of any
concomitant medication that might exacerbate drowsiness. Newer
developments include the use of preparations in which opioids are com-
bined with opioid antagonists, such as naloxone. The naloxone is poorly
absorbed and does not antagonise the systemic analgesic effect but
rather acts locally to block opioid receptors in the gut, thereby reduc-
ing opioid-related constipation. Vivid dreams, visual hallucinations (often
consisting of a sense of movement at the periphery of vision), delirium
and myoclonus are typical of opioid-related toxicity and, if present,
require urgent reassessment of the opioid dose. Biochemistry should
also be checked to exclude renal impairment, dehydration, electrolyte
disturbance or hypercalcaemia.
Since opioid toxicity can occur at any dose, side-effects should be
assessed regularly, but particularly after a dose increase. Pain should
be reassessed to ensure that appropriate adjuvants are being used.
Parenteral rehydration is often helpful to speed up excretion of active
metabolites of morphine. The dose of opioid may need to be reduced or
the opioid changed to a strong alternative.
Different opioids have different side-effect proles in different peo-
ple. If a patient develops side-effects, switching to an alternative strong
opioid may be helpful. Options include oxycodone, transdermal fenta-
nyl, hydromorphone and occasionally methadone, any of which may
produce a better balance of benet against side-effects. Fentanyl has
no renally excreted active metabolites and may be particularly useful in
patients with renal failure. Buprenorphine is also particularly useful in
signicant renal impairment. It is possible to switch between opioids
but great care must be taken when doing so to make sure the dose is
correct and to avoid prescribing too much or too little opioid.
Adjuvant analgesics
An adjuvant analgesic is a drug that has a primary indication other than
pain but which provides analgesia in some painful conditions and may
enhance the effect of the primary analgesic. Commonly used adjuvant
analgesics in the palliative care setting are shown in Box 8.15. Some
adjuvant analgesics may enhance the side-effect prole of the primary
analgesic, and dose reductions of opioids may be required when an
adjuvant analgesic is added. At each step of the WHO analgesic ladder,
an adjuvant analgesic should be considered, the choice depending on
the type of pain.
Management: non-pharmacological treatments
Neurodestructive interventions
Neurodestructive techniques have an important role in the management
of cancer pain, where life expectancy is limited. They should be used as
part of an overall management plan and considered when the response
to drug treatment has been inadequate. Intrathecal analgesia, delivered
via either an external pump or a fully implanted device, is a good option,
8.14 Opioid side-effects
Side-effect Management
Constipation Regular laxative
Opioid/naloxone oral combination, in resistant
constipation
Dry mouth Frequent sips of iced water, soft white parafn
to lips, chlorhexidine mouthwashes twice daily,
sugar-free gum, water or saliva sprays
Nausea/vomiting Oral haloperidol 0.5–1mg at night, oral
metoclopramide 10mg 3 times daily or oral
domperidone 10mg 3 times daily
If constant, haloperidol or levomepromazine may
be given parenterally to break the nausea cycle
Sedation Explanation is very important
Symptoms usually settle in a few days
Avoid other sedating medication where possible
Ensure appropriate use of adjuvant analgesics
that can have an opioid-sparing effect
May require an alternative opioid
200.
170 PAINAND PALLIATIVE CARE
particularly where life expectancy is more than 3 months. Coeliac plexus
blocks can be helpful for visceral pain, such as in pancreatic cancer.
Lateral cordotomy to disrupt the spinothalamic tracts (either open or per-
cutaneous) may be considered for unilateral chest wall pain, such as may
occur in mesothelioma, where life expectancy is limited.
Radiotherapy
Radiotherapy is the treatment of choice for pain from bone metastases
(see Box 8.13) and can also be considered for metastatic involvement at
other sites. All patients with pain secondary to bone metastases should
be considered for palliative radiotherapy, which can usually be given in a
single dose. Some patients experience a transient are of pain after radi-
otherapy and this can be managed by 24–48 hours of dexamethasone
(4–8mg once in the morning).
Physiotherapy
Physiotherapy has a key role in the multidisciplinary approach to a wide
spectrum of cancer-related symptoms, including the prevention and
management of pain, muscle spasm, reduced mobility, muscle wast-
ing and lymphoedema. Rehabilitation in palliative care has expanded
and now includes pre-habilitation, which involves the use of proactive
focused exercise to maintain muscle mass during cancer chemotherapy
and in other chronic conditions such as COPD.
Psychological techniques
As with chronic pain, there is increasing use of psychological techniques
in cancer pain management, which train the patient to use coping strat-
egies and behavioural techniques. Other issues related to the specic
experience of a cancer diagnosis and cancer treatment may be complex,
and individual therapy in addition to group-based approaches can be
helpful.
Stimulation therapies
Acupuncture and TENS are low-risk stimulation therapies that may be
useful in palliative care for management of pain and nausea. Both are
particularly useful for secondary muscle spasm and TENS is increasingly
used for bone pain.
Complementary and alternative therapies
Palliative care patients often seek symptom relief from both comple-
mentary and alternative therapies. While the evidence base is poorly
8.15 Adjuvant analgesics in cancer pain
Drug Example Indications Side-effects*
NSAIDs Diclofenac Bone metastases, soft tissue inltration,
liver pain, inammatory pain
Gastric irritation and bleeding, uid retention,
headache
Caution in renal impairment
Glucocorticoids Dexamethasone 8–16mg per day,
titrated to lowest dose that controls
pain
Raised intracranial pressure, nerve
compression, soft tissue inltration,
liver pain
Gastric irritation if used together with
NSAID, uid retention, proximal muscle
myopathy, delirium, Cushingoid appearance,
candidiasis, hyperglycaemia
Anticonvulsants Evidence strongest for:
Duloxetine
Gabapentin
Pregabalin
Neuropathic pain of any aetiology Mild sedation, tremor, delirium
Exacerbation of opioid-related side-effects
Tricyclic antidepressants Amitriptyline
Nortriptyline (less sedative)
Neuropathic pain of any aetiology Sedation, dizziness, delirium, dry mouth,
constipation, urinary retention
Avoid in cardiac disease
Exacerbation of opioid-related side-effects
NMDA receptor blockers Ketamine Severe neuropathic pain (only under
specialist supervision)
Delirium, anxiety, agitation, hypertension
*In old age, all drugs can cause delirium.
(NMDA = N-methyl-D-aspartate; NSAIDs = non-steroidal anti-inammatory drugs)
developed, individual patients can gain signicant benets from the
complementary therapies as outlined earlier in this chapter. It is critically
important that patients are encouraged to discuss any alternative med-
icines they are considering, given the potential interactions with other
therapies.
Breathlessness
Breathlessness is one of the most common symptoms in palliative care
and is distressing for both patients and carers. Patients with breathless-
ness should be fully assessed to determine whether there is a reversible
cause, such as a pleural effusion, heart failure or bronchospasm; if so,
this should be managed in the normal way. If symptoms persist, addi-
tional measures may be necessary. There are many potential causes of
dyspnoea in cancer patients and in other chronic diseases; apart from
direct involvement of the lungs, muscle loss secondary to cachexia, anx-
iety and fear can all contribute. A cycle of panic and breathlessness,
often associated with fear of dying, can be dominant. Exploration of
precipitating factors is important and patient education about breath-
lessness and effective breathing has been shown to be effective. Non-
pharmacological approaches that include using a hand-held fan, pacing
and following a tailored exercise programme can help. There is no evi-
dence to suggest that oxygen therapy reduces the sensation of breath-
lessness in advanced cancer any better than cool airow, and oxygen is
indicated only if there is signicant hypoxia. Opioids, through both their
central and their peripheral action, can palliate breathlessness. Both oral
and parenteral opioids are effective and are now licensed for this indica-
tion in Australia. A low dose should be used initially and titrated against
symptoms, unless opioids are already being prescribed for pain, in which
case the existing dose can be increased further. If anxiety is considered
to be playing a signicant role, a quick-acting benzodiazepine, such as
lorazepam (used sublingually for rapid absorption), may also be useful.
Cough
Persistent unproductive cough can be helped by opioids, which have
an antitussive effect. Troublesome respiratory secretions can be treated
with hyoscine hydrobromide (400–600µg every 4–8 hours), although
dry mouth is a common adverse effect. As an alternative, glycopyrro-
nium can be useful and is given by subcutaneous infusion (0.6–1.2mg in
24 hours).
FCPS Single Best
201.
Palliative care 171
8
Nausea and vomiting
The presentation of nausea and vomiting differs depending on the under-
lying cause, of which there are many. Large-volume vomiting with little
nausea is common in intestinal obstruction, whereas constant nau-
sea with little or no vomiting is often due to metabolic abnormalities or
adverse effects of drugs. Vomiting related to raised intracranial pressure
is worse in the morning. Different receptors are activated, depending on
the cause or causes of the nausea (Fig. 8.10). For example, dopamine
receptors in the chemotactic trigger zone in the fourth ventricle are stim-
ulated by metabolic and drug causes of nausea, whereas gastric irritation
stimulates histamine receptors in the vomiting centre via the vagus nerve.
Reversible causes, such as hypercalcaemia and constipation, should be
treated appropriately. Drug-induced causes should be considered and
the offending drugs stopped if possible. As different classes of antiemetic
drug act at different receptors, antiemetic therapy should be based on
a careful assessment of the probable causes and a rational decision to
use a particular class of drug (Box 8.16). The subcutaneous route is often
required initially to overcome gastric stasis and poor absorption of oral
medicines.
Gastrointestinal obstruction
Gastrointestinal obstruction is a frequent complication of intra-abdomi-
nal cancer. Patients may have multiple levels of obstruction and symp-
toms may vary greatly in nature and severity. Surgical mortality is high
in patients with advanced disease and obstruction should normally be
managed without surgery. The key to effective management is to address
the presenting symptoms – colic, abdominal pain, nausea, vomiting,
intestinal secretions – individually or in combination, using parenteral
drugs that do not cause or worsen other symptoms. This can be prob-
lematic when a specic treatment worsens another symptom. Cyclizine
improves nausea and colic responds well to anticholinergic agents, such
as hyoscine butylbromide, but both slow gut motility. Nausea will improve
with metoclopramide, although this is usually contraindicated in the pres-
ence of colic because of its prokinetic effect. There is some low-quality
evidence that glucocorticoids (dexamethasone 8mg) can shorten the
length of obstructive episodes. Somatostatin analogues, such as octre-
otide, will reduce intestinal secretions and therefore large-volume vomits.
Occasionally, a nasogastric tube is required to reduce gaseous or uid
distension.
Weight loss
Patients with cancer lose weight for a variety of reasons, including
reduced appetite or the effects of drug treatment, or as a consequence of
low mood and anxiety. There is, however, a particularly challenging syn-
drome associated with weight loss, which is known as cancer cachexia.
This results from an alteration of metabolism caused by a complex inter-
action of tumour-related factors and the body’s response to these fac-
tors, resulting in muscle loss, along with anorexia. Treatment involves
prescribing exercise to maintain muscle mass and strengthen muscles,
ensuring that there is an adequate calorie intake and providing nutri-
tional supplements. Anti-inammatory medication to attenuate systemic
inammation is the subject of research and many patients self-medicate
with sh oil. Glucocorticoids can temporarily boost appetite and general
well-being but may cause false weight gain by promoting uid retention.
Their benets need to be weighed against the risk of side-effects, and
glucocorticoids should generally be used on a short-term basis only.
Anxiety and depression
Anxiety and depression are common in palliative care but the diagnosis
may be difcult, since the physical symptoms of depression are similar
to those of advanced cancer. It is therefore important to acknowledge
that these symptoms are not inevitable in advanced cancer. Patients
should still expect to look forward to things and to enjoy them, within the
context of the situation. Simply asking the question ‘Do you think you
are depressed?’ can be very useful in deciding with the patient whether
antidepressants or psychological interventions may be of benet. In this
regard, psycho-oncology has been evolving rapidly and there is now
good evidence for the role of ‘talk therapy’ in palliative care, along with
other appropriate management of anxiety and depression. If antidepres-
sants are required, citalopram and mirtazapine are good choices since
they are generally well tolerated in patients with advanced disease.
Delirium and agitation
Many patients become confused or agitated in the last days of life. It
is important to identify and treat potentially reversible causes unless
the patient is too close to death for this to be feasible. Early diagno-
sis and effective management of delirium are extremely important. As in
Higher centres
Vestibular input
Vomiting centre
H1, ACh, 5-HT
Chemotactic trigger
zone
D2, 5-HT
Vagal
afferents
Metabolic
toxins
Chemoreceptors
Mechanoreceptors
Retroperistalsis
Gastric pyloric
contraction
Abdominal and
thoracic wall
contraction
Peripheral
circulation
Fig. 8.10 Mechanisms of nausea. (ACh = acetylcholine; D2
= dopamine;
5-HT = 5-hydroxytryptamine, serotonin; H1
= histamine)
8.16 Receptor site activity of antiemetic drugs
Area Receptors Drugs
Chemotactic trigger zone Dopamine2
5-HT
Haloperidol
Metoclopramide
Vomiting centre Histamine1
Acetylcholine
Cyclizine
Levomepromazine
Hyoscine
Gut (gastric stasis) Metoclopramide
Gut distension (vagal
stimulation)
Histamine1
Cyclizine
Gut (chemoreceptors) 5-HT Levomepromazine
(5-HT = 5-hydroxytryptamine, serotonin)
202.
172 PAINAND PALLIATIVE CARE
other palliative situations, it may not be possible to identify and treat the
underlying cause, and the focus of management should be to ensure
that the patient is comfortable. It is important to distinguish between
behavioural change due to pain and that due to delirium, as opioids
will improve one and worsen the other. The management of delirium is
detailed in Chapter 34. It is important, even in the care of the actively
dying patient, to treat delirium with antipsychotic medicines, such as
haloperidol, or olanzapine if under 70 years, rather than to regard it as
distress or anxiety and use benzodiazepines only.
Dehydration
Deciding whether to give intravenous uids can be difcult when a
patient is very unwell and the prognosis is uncertain. A patient with a
major stroke, who is unable to swallow but is expected to survive the
event, will develop renal impairment and thirst if not given uids and
should be hydrated. On the other hand, when a patient has been deteri-
orating and is clearly dying, parenteral hydration needs very careful con-
sideration and it is very important to manage this on an individual basis.
Patient comfort and avoidance of distress in the family are the primary
aims. Where a patient and family are happy with meticulous oral hygiene
and care to reduce the sensation of dryness in the mouth, this is usually
more appropriate and effective at the end of life than parenteral hydra-
tion, which by itself will not necessarily improve the sensation of dryness.
In some patients, parenteral hydration will simply exacerbate pooling
of secretions, causing noisy and distressing breathing. Each decision
should be individual and discussed with the patient’s family.
Death and dying
Planning for dying
There have been dramatic improvements in the medical treatment and
care of patients with cancer and other illnesses over recent years but the
inescapable fact remains that everyone will die at some time. Planning
for death should be actively considered in patients with chronic dis-
eases when the death is considered to be foreseeable or inevitable.
Doctors rarely know exactly when a patient will die but are usually
aware that an individual is about to die and that medical interventions
are unlikely to extend life or improve its quality signicantly. Most people
wish their doctors to be honest about this situation to allow them time
to think ahead, make plans and address practical issues. A few do
not wish to discuss future deterioration or death; if this is felt to be the
case, avoidance of discussion should be respected. For doctors, it is
helpful to understand an individual’s wishes and values about medical
interventions at this time, as this can help guide decisions about inter-
ventions. It is important to distinguish between interventions that will
not provide clinical benet (a medical decision) and those that do not
confer sufcient benet to be worthwhile (a decision that can only be
reached with a patient’s involvement and consent). A common exam-
ple of this would be decisions about not attempting cardiopulmonary
resuscitation.
In general, people wish for a dignied and peaceful death and most,
but not all, prefer to die at home. Families also are grateful for the chance
to prepare themselves for the death of a relative, by timely and gentle
discussion with the doctor or other health professionals. Early discussion
and effective planning improve the chances that an individual’s wishes
will be achieved. There are two important caveats: rstly, wishes can
and do change as the terminal situation evolves, and secondly, planning
in general can only be done over time as patients form a relationship
with professionals and evolve an understanding of the situation in which
they nd themselves. Attempts to carry out and nalise advanced care
planning at a single consultation, especially if a rst meeting, are usually
unsatisfactory.
Structures for assessment and planning around end-of-life care are
for guidance only and the focus should evolve with the individual patient.
Diagnosing dying
When patients with cancer or other conditions become bed-bound,
semi-comatose, unable to take tablets and only able to take sips of
water, with no reversible cause, they are likely to be dying and many will
have died within 2 days. Doctors are sometimes poor at recognising this
and should be alert to the views of other members of the multidisciplinary
team. A clear decision that the patient is dying should be agreed and
recorded.
Management of dying
Once the conclusion has been reached that a patient is going to die in
days to a few weeks, there is a signicant shift in management (Box 8.17).
Symptom control, relief of distress and care for the family become the
most important elements of care. Medication and investigation are justi-
able only if they contribute to these ends. When patients can no longer
drink because they are dying, intravenous uids are usually not neces-
sary and may cause worsening of bronchial secretions; however, this is
a decision that can be made only on an individual basis. Management
should not be changed without discussion with the patient and/or family.
Medicines should always be prescribed for the relief of symptoms. For
example, morphine or diamorphine may be used to control pain, levo-
mepromazine to control nausea, haloperidol to treat delirium, diazepam
or midazolam to treat distress, and hyoscine hydrobromide to reduce
respiratory secretions. Side-effects, such as drowsiness, may be accept-
able if the principal aim of relieving distress is achieved. It is important to
discuss and agree the aims of care with the patient’s family. Poor com-
munication with families at this time is one of the most common reasons
for family distress afterwards and for formal complaints.
Ethical considerations
The overwhelming force in caring for any patient must be to listen to
that patient and family and take their wishes on board. Patients know
8.17 How to manage a patient who is dying
Patient and family awareness
Assess patient’s and family’s awareness of the situation
Ensure patient, if able, and family understand plan of care
Medical interventions
Stop non-essential medications that do not contribute to symptom control
Stop inappropriate investigations and interventions, including routine
observations
Resuscitation
Complete Do Not Attempt Cardiopulmonary Resuscitation (DNACPR) form
Deactivate implantable debrillator
Symptom control
Ensure availability of parenteral medication for symptom relief
Support for family
Make sure you have contact details for family, that you know when they want to
be contacted and that they are aware of facilities available to them
Religious and spiritual needs
Make sure any particular wishes are identied and followed
Ongoing assessment
Family’s awareness of condition
Management of symptoms
Need for parenteral hydration
Care after death
Make sure family know what they have to do
Notify other appropriate health professionals
Multiple Choice Questions
8.1.A 45-year-old woman consults her GP with gradually worsening
low back pain that started over 2 years ago. There is no radiation,
nor any red ags. She can’t take non-steroidal anti-inammatory
drugs (NSAIDs) due to gastrointestinal upset. She takes
co-codamol 30/500, at a dose of 8 tablets per day, with limited
benet. She works as a cleaner in the local school and is a single
parent, with three children ranging in age from 7 to 18. She has
been off work for the last 6 months after a are-up that has not
settled, and is worried about her job and her nances. Her mother
lives nearby and is very supportive, helping out with child care and
shopping. She saw a physiotherapist for one session, but didn’t
go back as she felt the exercises were too hard and made the
pain worse. She worries that if the pain gets worse she must be
damaging her back, and she spends much of her time in bed or
lying on the couch. What management approach should be used
to increase her physical activity?
A. Refer back to the physiotherapist for more effective
exercises
B. Prescribe stronger analgesics, such as morphine, to control
the pain better, and thus allow her to do more.
C. Assess what barriers there are to increasing her physical
activity before agreeing on management
D. Refer for further investigations, such as X-ray or magnetic
resonance imaging (MRI), of her lumbar spine to exclude
any sinister causes, such as malignancy, or to identify any
surgical target
E. Add in diazepam to help with muscle spasm and reduce her
anxiety about the impact of the pain
Answer: C.
There is good evidence that increasing physical activity is an effective
way to manage chronic pain, but it is acknowledged that there may be
a number of barriers preventing people who live with chronic pain from
doing this successfully. These can include fear avoidance, not pacing
activities well, nancial problems or other commitments that prevent their
ability to attend appointments or go to a leisure centre. Advice alone is
not enough and personal preference/previous experience will impact on
patient engagement. There is no one type of exercise that will work for
everyone (see Box 8.6). Strong opioids are not recommended for long-
term use in chronic pain, and combining these with other sedative drugs
may be harmful. There is no good evidence that benzodiazepines are
helpful in chronic low back pain. Continued investigation in the absence
of specic concerns is not recommended.
8.2. A 27-year-old man was involved in a severe road trafc accident,
and suffered multiple lower limb fractures requiring orthopaedic
intervention and several surgeries. His pain control was challenging
whilst he was an in-patient, and he was discharged on oramorph
10–20 mg as required for pain, prescribed up to 6 times daily, plus
MST 40 mg 2 times daily and paracetamol 1 g 4 times daily. The
plan was that he would be reviewed in the orthopaedic clinic, but
he defaulted from attending. He lived alone, with no family nearby
or supportive friends. He attended his GP complaining of severe,
poorly controlled pain, nightmares, ashbacks to the accident
and with very poor sleep and low mood, to the extent that he felt
that life was not worth living. He was asking for an increase in his
oramorph to help him cope, as he had tried taking a higher dose
(40 mg) and felt it was effective, especially in improving sleep. What
would be the rst step in your management plan?
A. Increase his oramorph as a short-term solution in order to
relieve distress and improve sleep
B. Assess suicide risk and institute appropriate urgent support
if required
C. Refer back to the orthopaedic team for consideration of
further investigations or surgery
D. Add in anti-neuropathic agents, such as gabapentin, in case
there is a neuropathic component to his pain
E. Refer for assessment and management of possible
post-traumatic stress disorder (PTSD) syndrome
Answer: B.
This is the most urgent action as he has a number of risk factors,
including co-morbid mental health issues, high levels of distress, iso-
lation, limited social support, being prescribed strong opioids and not
using them according to prescribing advice. Suicide risk assessment
should include risk factors, evidence of planning and means to carry it
out, and protective factors. Further management depends on the degree
and urgency of risk, ranging from urgent involvement of the duty mental
health team to liaison with GP/other relevant health-care professionals
and offering information on support available (local, telephone, online).
Increasing oramorph with the aim of improving sleep and/or mood
would not be appropriate or safe (see Box 8.10). Similarly, adding in gab-
apentin may increase the risk of death, and further assessment of neuro-
pathic features would be needed if being considered at a later date (see
Fig. 8.7). Additionally, consideration should be given to using controlled
dispensing arrangements to help improve safety (e.g. daily pick-up from
the pharmacy). Further orthopaedic involvement and treatment of PTSD
symptoms would be appropriate, but do not address the immediate
problem.
8.3. John is a 73-year-old, retired joiner with a 3-month history of
right-sided chest pain and cough. He has been receiving palliative
radiotherapy for right-sided Pancoast’s tumour. He lives with his
wife, and his family are not nearby.
Timeline
1. John was reviewed during radiotherapy because of pain in his right
shoulder and arm. He experiences constant severe background
pain, rated as 9/10. He also experiences intermittent, excruciating
shooting pain associated with pins and needles and a tight sensa-
tion, rated as 10/10.
2. John is unable to sleep. He says he feels miserable and hopeless,
and that he ‘doesn’t look forward to anything, each day is a chal-
lenge’. He has also lost weight and has a poor appetite.
3. His medication is:
Co-codamol (30/500 mg) 2 tablets 4 times a day
Laxidol (laxative) 1 sachet daily
4. He was able to give a clear history that a codeine and paracetamol
combination worked within 40 minutes, but lasted for just an hour,
reducing background pain from 10/10 to 8/10. John has normal
biochemistry.
Which of the following did John’s doctor suggest?
A. Stop co-codamol and start titration with a strong opioid to
5 mg of normal release morphine orally 6 times daily, with
the same dose provided for breakthrough analgesia (1/6 of
the total 24-hour dose). Advice with regard to continuing
with the laxative and a metoclopramide made available in
case of nausea. Arranged a review in 48 hours
B. As in answer A, but also continue with co-codamol
C. His doctor was concerned about commencing a strong
opioid too soon, as John was not yet actively dying. So, he
decided to try tramadol instead of co-codamol.
206.
D. In viewof John’s mood and inability to sleep, analgesia
should be left unchanged; however, night sedation and an
antidepressant can be introduced
E. The doctor knew it was important to control John’s pain as
quickly as possible, therefore slow-release morphine 60 mg
2 times daily was commenced
Answer: A.
Pain affects, and is affected by, many other symptoms and aspects of
daily living. Time spent on the initial detailed assessment of all aspects of
the patient is key to successful management. For John, rapid proposed
solutions to his myriad of problems would only lead to therapeutic chaos.
The key decision is what is the greatest driver of the symptoms and how
does this potentially interact with other symptoms and distress.
Our patient had very severe pain with a strong neuropathic compo-
nent, as would be expected from pressure on and/or invasion of the
brachial plexus by the tumour. It is not surprising that he was not sleep-
ing, because of this extremely severe pain, and felt thoroughly miserable
and hopeless. His poor appetite and weight loss could be a result of
the uncontrolled pain; however, it could also be the result of the cancer
process via a cachexia mechanism. It is important to recognise that neu-
ropathic pain is particularly associated with mood disturbances.
When assessing complexities, it is important to ground decisions on
information obtained directly from the patient. John was able to give a
clear history that a codeine and paracetamol combination worked within
40 minutes but lasted for just an hour, and the clue here is that his pain
was at least partially opioid responsive. It is critical not to move sideways
on an analgesic ladder and if maximum dose of codeine is not effective,
the patient should move to a strong opioid, such as morphine. The usual
starting dose in this situation of normal renal function and such severe
pain should be 5 mg of normal release morphine orally 6 times daily. If
starting modied-release morphine, the dose is 10–15 mg 2 times daily.
For breakthrough analgesia the dose is usually 1/6 of the total 24-hour
dose, so 5 mg in this case. The patient should be advised that if break-
through pain relief is required it will take 30 minutes to start having an
effect. The patient should be instructed to take a careful note of duration
onset and duration of effect of the regular dose, and frequency of onset
and duration and effect of the breakthrough dose.
8.4. On review after 48 hours, John reported that his background pain
reduced from 9/10 to 4/10 within 40 minutes of taking 5 mg of
normal-release morphine. The improvement lasted for 2 hours,
after which background pain returned to 9/10. A breakthrough
dose of 5 mg immediate-release morphine then reduced
background pain to 4/10 again and this lasted for another 2 hours.
In total 5 regular doses and 6 breakthrough doses of 5 mg normal-
release morphine were used in 24 hours.
Crucially, there was no sedation, sleepiness or vivid dreams. Bowels
were moving normally with the laxative prescribed and no nausea was
present.
The severe, intermittent, shooting pain was less frequent and reduced
from 10/10 to 5/10.
Sleep, appetite, mood and interactions with others were much
improved.
What did John’s doctor decide to do next?
A. Switch to slow-release morphine, 15 mg 2 times daily and
continue to encourage use of breakthrough analgesia with
5 mg normal release morphine as required
B. As A, but also add in amitriptyline 10 mg at bedtime
C. Continue on normal-release morphine 5 mg 6 times daily
and increase breakthrough to 10 mg normal release
morphine as required and review in 48 hours
D. Switch to slow-release morphine, 30 mg 2 times daily and
prescribe normal-release morphine 10 mg for breakthrough
pain
E. No changes to opioid regimen, but add dexamethasone for
brachial plexus pressure, appetite and general wellbeing
Answer: D.
The strong history provided by John is of an opioid-responsive pain.
It is clear that a dose of 10 mg 6 times daily would bring background
pain from 9/10 to 4/10. At present there are no opioid-related side
effects, therefore a switch to a sustained-release morphine of 30 mg
2 times daily is reasonable, instead of 10 mg normal-release 6 times
daily. Breakthrough normal-release morphine should be increased to 10
mg and used as required. Ongoing review of common opioid side effects
is crucial.
On further review of pain relief, breakthrough opioid required, intermit-
tent component of pain and other symptoms, such as mood, appetite
and weight, may lead to further manipulation of John’s management at
follow-up.
207.
Acute medicine and
criticalillness
VR Tallentire
MJ MacMahon
Clinical examination in critical care 176
Monitoring 177
Acute medicine 178
The decision to admit to hospital 178
Ambulatory care 178
Presenting problems in acute medicine 178
Chest pain 179
Acute breathlessness 181
Anaphylaxis 183
Syncope/presyncope 184
Headache 186
Unilateral leg swelling 188
Acute abdomen 189
Identication and assessment of deterioration 191
Early warning scores and the role of the medical emergency team 191
Immediate assessment of the deteriorating patient 191
Selecting the appropriate location for ongoing management and anticipatory
care planning 191
Common presentations of deterioration 191
Tachypnoea 193
Hypoxaemia 194
Tachycardia 195
Hypotension 195
Hypertension 197
Decreased conscious level 197
Decreased urine output/deteriorating renal function 198
Disorders causing critical illness 198
Sepsis and the systemic inammatory response 198
Acute respiratory distress syndrome 201
Acute circulatory failure (cardiogenic shock) 202
Cardiac arrest 202
Post cardiac arrest 205
Other causes of multi-organ failure 206
9
Critical care medicine 206
Decisions around intensive care admission 206
Stabilisation and institution of organ support 207
Respiratory support 207
Cardiovascular support 210
Renal support 212
Neurological support 212
Daily clinical management in intensive care 212
Clinical review 212
Infection surveillance 212
Sedation and analgesia 212
Delirium in intensive care 213
Weaning from respiratory support 213
Extubation 214
Tracheostomy 214
Nutrition 214
Other essential components of intensive care 214
Complications and outcomes of critical illness 214
Adverse neurological outcomes 215
Airway complications 215
Micro- and macrovascular complications 215
Other complications 216
The older patient 216
Withdrawal of active treatment and death in intensive care 216
Discharge from intensive care 217
Critical care scoring systems 218
208.
176 ACUTEMEDICINE AND CRITICAL ILLNESS
Clinical examination in critical care
Airway
Is the airway patent?
Is the end-tidal CO2 trace
normal?
Are there any signs of airway
obstruction?
A B
Circulation
Is the physiology normal
(heart rate, blood pressure,
peripheral temperature,
lactate, urine output)?
How much support is required
(inotrope,vasopressor)?
C
D
E
Glucose
What is the glucose level?
Is insulin being administered?
G
G
D
Haematology
What are the haemoglobin/
platelet levels?
Are there any signs of
bleeding?
H
Infection
What is the temperature?
Review recent infective
markers and trend
What antibiotics are being
given and what is the
duration of treatment?
I
Enteral/exposure
Feeding regimen
Stool frequency
Abdominal tenderness/bowel
sounds present?
Disability
Level of responsiveness
Delirium screen
Pupillary responses
Doses of sedative drugs
F Fluids, electrolytes and
renal system
What is the fluid balance?
Urine volume and colour?
Is there any oedema?
Review the renal biochemistry
and electrolyte levels
Breathing
Is the physiology normal
(SpO2, respiratory rate, tidal
volume)?
What is the level of support?
Are there any abnormal signs
on chest examination?
Review the ventilator settings,
arterial blood gases and
recent chest X-ray
F
E
C
B
A
209.
Monitoring 177
9
Basicprinciples
Uses the different red and infrared
absorption proles of oxyhaemoglobin
and deoxyhaemoglobin to estimate
arterial oxyhaemoglobin saturation
(SaO2
)
Only pulsatile absorption is
measured
A poor trace correlates with poor
perfusion
Sources of error
Carboxyhaemoglobin – absorption prole is the
same as oxyhaemoglobin: falsely elevated SpO2
Methaemoglobinaemia – SpO2
will tend towards
85%
Ambient light/poor application of probe/severe
tricuspid regurgitation (pulsatile venous ow):
falsely depressed SpO2
Reduced accuracy below 80% saturation
Hyperbilirubinaemia does not affect SpO2
Monitoring
Electrocardiography
Heart rate, rhythm and QRS morphology
Arterial line trace
Size of the area under the curve is proportional to stroke
volume
Narrow peaks suggest low stroke volume as shown here
Oxygen saturation
Saturation of haemoglobin measured by plethysmography
(SpO2). Gives an indication of adequacy of oxygenation,
and the quality of tissue perfusion can also be inferred –
a flat trace suggests poor peripheral perfusion
Central venous pressure trace
A non-specific guide to volume status and right ventricular
function. Increased values in fluid overload and right
ventricular failure
Capnography
Numerical value of end-tidal CO2 (ETCO2) is less than
arterial PCO2 (PaCO2) by a variable amount. Shape of
trace can signify airway displacement/obstruction,
bronchospasm or a low cardiac output (as shown below)
kPa mmHg
Time (secs)
0
5
40
Normal
Steep ‘upstroke’
in early expiration
Bronchospasm
Shallow ‘upstroke’
in early expiration
Partial obstruction/displacement
of airway device
Decreasing cardiac output
Decreasing size of
ETCO2 waveform
No ventilation
(from any cause)
Bedside physiological data commonly monitored in an intensive care unit setting.
Pulse oximetry (SpO2
)
210.
178 ACUTEMEDICINE AND CRITICAL ILLNESS
Hospital medicine is becoming ever-more specialised and people are
living longer while accruing increasing numbers of chronic disease diag-
noses. Rather than diminishing the role of the generalist, these factors
paradoxically create a need for experts in the undifferentiated presenta-
tion. In the UK such physicians are known as ‘general physicians’, while
in the United States they are referred to as ‘hospitalists’.
Acute illness can present in a large variety of ways, depending on the
nature of the illness, the underlying health of the individual, and their cul-
tural and religious background. The skills of prompt diagnosis formation
and provision of appropriate treatment rely on the integration of infor-
mation from all the available sources, along with careful consideration of
underlying chronic health problems.
Patients who deteriorate while in hospital make up a small but impor-
tant cohort. If they are well managed, in-hospital cardiac arrest rates will
be low. This can be achieved through the combined effects of prompt
resuscitation and appropriate end-of-life decision-making. Early recog-
nition of deterioration by ward teams and initial management by health-
care professionals operating within a functioning rapid response system
are the central tenets of any system designed to improve the outcomes
of deteriorating ward patients.
Intensive care medicine has developed into a prominent specialty,
central to the safe functioning of a modern acute hospital. Scientic
endeavour has resulted in a much better understanding of the molecu-
lar pathophysiology of processes such as sepsis and acute respiratory
distress syndrome, which account for much premature death worldwide.
Acute medicine
Acute medicine is the part of general medicine that is concerned with the
immediate and early management of medical patients who require urgent
care. As a specialty, it is closely aligned with emergency medicine and
intensive care medicine, but is rmly rooted within general medicine. Acute
physicians manage the adult medical take and lead the development of
acute care pathways that aim to reduce variability, improve care and cut
down hospital admissions. In order to achieve these aims, acute physicians
must use their knowledge, combined with high-level clinical reasoning and
decision-making skills, to minimise both diagnostic error and the risks of
over-investigation. These concepts are explained more fully in Chapter 1
The decision to admit to hospital
Every patient presenting to hospital should be assessed by a clinician
who is able to determine whether or not admission is required. The
requirement for admission is determined by many factors, including the
severity of illness, the patient's physiological reserve, the need for urgent
investigations, the nature of proposed treatments and the patient's social
circumstances. In many cases, it is clear early in the assessment process
that a patient requires admission. In such cases, a move into a med-
ical receiving unit – often termed a medical admissions unit (MAU) or
acute medical unit (AMU) – should be facilitated as soon as the initial
assessment has been completed and urgent investigations and/or treat-
ments have been instigated. In hospitals where such units do not exist,
patients will need to be moved to a downstream ward once treatment
has been commenced and they have been deemed sufciently stable. In
suspected cases of airborne-transmissible infectious diseases, patients
should be isolated initially and may require cohorting in specic areas
of the hospital once diagnoses have been conrmed. Following the ini-
tial assessment, it may be possible to discharge stable patients home
with a plan for early follow-up (such as a rapid-access specialist clinic
appointment).
Ambulatory care
In some hospitals, it is increasingly possible for patient care to be coordi-
nated in an ambulatory setting, negating the need for a patient to remain
in hospital overnight. In the context of acute medicine, ambulatory care
can be employed for conditions that are perceived by either the patient
or the referring practitioner as requiring prompt clinical assessment by
a competent decision-maker with access to appropriate diagnostic
resources. The patient may return on several occasions for investigation,
observation, consultation or treatment. Some presentations, such as a
unilateral swollen leg (p. 188), lend themselves to this type of manage-
ment (Box 9.1). If indicated, a Doppler ultrasound can be arranged, and
patients with conrmed deep vein thrombosis can be anticoagulated on
an outpatient basis. Successful ambulatory care requires careful patient
selection; while many patients may cherish the opportunity to sleep at
home, others may nd frequent trips to hospital or clinic too difcult due
to frailty, poor mobility or transport difculties.
Presenting problems in acute medicine
Thissectiondetailssomeofthemostcommonpresentationstoacutemed-
icine. However, many people present to hospital with physical complaints
that do not appear to be the symptoms of a medical condition, referred
to as ‘medically unexplained’ or ‘functional’ symptoms (see Chs 23, 28
and 31). It is thought that such symptoms may account for up to half
of all new visits to hospital in the UK. Rather than providing reassur-
ance, a lack of understanding of the cause of the symptoms can result in
more distress for patients. When the unexplained symptoms relate to the
nervous system (such as limb weakness, numbness, shaking or black-
outs), the term ‘functional neurological disorder’ is used (see p. 1152).
Medically unexplained symptoms are more common in women, younger
people, those who have previously suffered from depression or anxiety,
recently bereaved people and those recovering from a recent physical
9.1 Groups of patients who are potentially suitable for ambulatory care
Group Example(s) Quality and safety issues
Diagnostic exclusion group Chest pain – possible myocardial infarction;
breathlessness – possible pulmonary embolism
Even when a specic condition has been excluded, there is still
a need to explain the patient's symptoms through the diagnostic
process
Low-risk stratication group Non-variceal upper gastrointestinal bleed with
low Blatchford score (Box 23.16); community-
acquired pneumonia with low CURB-65 score
(see Fig. 17.32)
Appropriate treatment plans should be in place
Specic procedure group Replacement of percutaneous endoscopic
gastrostomy (PEG) tube; drainage of pleural
effusion/ascites
The key to implementation is how ambulatory care for this group
of patients can be delivered when they present out of hours
Outpatient group with
supporting infrastructure
Deep vein thrombosis (DVT); cellulitis These are distinct from the conditions listed above because the
infrastructure required to manage them is quite different
211.
Presenting problems inacute medicine 179
9
illness. For every acute medical presentation, there will be a cohort of
patients for whom the diagnosis remains elusive. In such circumstances,
consideration should be given to the potential harm of over-investiga-
tion and the alternative approaches required to manage medically unex-
plained symptoms. Further detail can be found on p. 1256.
Chest pain
Chest pain is a common symptom in patients presenting to hospital. The
differential diagnosis is wide (Box 9.2), and a detailed history and thor-
ough clinical examination are paramount to ensure that the subsequent
investigative pathway is appropriate.
Presentation
Chest ‘pain’ is clearly a subjective phenomenon and may be described
by patients in a variety of different ways. Whether the patient describes
‘pain’, ‘discomfort’ or ‘pressure’ in the chest, there are some key fea-
tures that must be elicited from the history.
Site and radiation
Pain secondary to myocardial ischaemia is typically located in the centre
of the chest. It may radiate to the neck, jaw and upper or even lower
arms. Occasionally, it may be experienced only at the sites of radiation or
in the back. The pain of myocarditis or pericarditis is characteristically felt
retrosternally, to the left of the sternum, or in the left or right shoulder. The
severe pain of aortic dissection is typically central with radiation through
to the back. Central chest pain may also occur with tumours affecting the
mediastinum, oesophageal disease (p. 806) or disease of the thoracic
aorta (p. 342). Pain situated over the left anterior chest and radiating
laterally is unlikely to be due to cardiac ischaemia and may have many
causes, including pleural or lung disorders, musculoskeletal problems or
anxiety. Rarely, sharp, left-sided chest pain that is suggestive of a mus-
culoskeletal problem may be a feature of mitral valve prolapse (p. 455).
Characteristics
Pleurisy, a sharp or ‘catching’ chest pain aggravated by deep breathing
or coughing, is indicative of respiratory pathology, particularly pulmonary
infection or infarction. However, the pain associated with myocarditis
or pericarditis is often also described as ‘sharp’ and may ‘catch’ dur-
ing inspiration, coughing or lying at. It typically varies in intensity with
movement and the phase of respiration. A malignant tumour invading
the chest wall or ribs can cause gnawing, continuous local pain. The
pain of myocardial ischaemia is typically dull, constricting, choking or
‘heavy’, and is usually described as squeezing, crushing, burning or ach-
ing. Patients often emphasise that it is a discomfort rather than a pain.
Angina occurs during (not after) exertion and is promptly relieved (in less
than 5minutes) by rest. It may also be precipitated or exacerbated by
emotion but tends to occur more readily during exertion, after a large
meal or in a cold wind. In crescendo or unstable angina, similar pain may
be precipitated by minimal exertion or at rest. The increase in venous
return or preload induced by lying down may also be sufcient to provoke
pain in vulnerable patients (decubitus angina). Patients with reversible
airways obstruction, such as asthma, may also describe exertional chest
tightness that is relieved by rest. This may be difcult to distinguish from
myocardial ischaemia. Bronchospasm may be associated with wheeze,
atopy and cough (p. 489). Musculoskeletal chest pain is variable in site
and intensity but does not usually fall into any of the patterns described
above. The pain may vary with posture or movement of the upper body,
or be associated with a specic movement (bending, stretching, turning).
Many minor soft tissue injuries are related to everyday activities, such as
driving, manual work and sport.
Onset
The pain associated with myocardial infarction (MI) typically takes sev-
eral minutes or even longer to develop to its maximal intensity; similarly,
angina builds up gradually in proportion to the intensity of exertion. Pain
that occurs after, rather than during, exertion is usually musculoskele-
tal or psychological in origin. The pain of aortic dissection (severe and
‘tearing’), massive pulmonary embolism (PE) or pneumothorax is usually
very sudden in onset. Other causes of chest pain tend to develop more
gradually, over hours or even days.
Associated features
The pain of MI, massive PE or aortic dissection is often accompanied by
autonomic disturbance, including sweating, nausea and vomiting. Some
patients describe a feeling of impending death, referred to as ‘angor
animi’. Breathlessness, due to pulmonary congestion arising from transient
ischaemic left ventricular dysfunction, is often a prominent feature of myo-
cardial ischaemia. Breathlessness may also accompany any of the res-
piratory causes of chest pain and can be associated with cough, wheeze
or other respiratory symptoms. Patients with myocarditis or pericarditis
may describe a prodromal viral illness. Gastrointestinal disorders, such as
gastro-oesophageal reux or peptic ulceration, may present with chest
pain that is hard to distinguish from myocardial ischaemia; it may even be
precipitated by exercise and be relieved by nitrates. However, it is usually
possible to elicit a history relating chest pain to supine posture or eating,
drinking or oesophageal reux. The pain of gastro-oesophageal reux often
radiates to the interscapular region and dysphagia may be present. Severe
chest pain arising after retching or vomiting, or following oesophageal
instrumentation, should raise the possibility of oesophageal perforation.
Anxiety-induced chest pain may be associated with breathlessness
(without hypoxaemia), throat tightness, perioral tingling and other evi-
dence of emotional distress. It is important to remember, however, that
chest pain itself can be an extremely frightening experience, and so psy-
chological and organic features often coexist. Anxiety may amplify the
effects of organic disease and a confusing clinical picture may result.
9.2 Differential diagnosis of chest pain
Central
Cardiac
Myocardial ischaemia (angina)
Myocardial infarction
Myocarditis
Pericarditis
Mitral valve prolapse syndrome
Aortic
Aortic dissection Aortic aneurysm
Oesophageal
Oesophagitis
Oesophageal spasm
Mallory–Weiss syndrome
Oesophageal perforation (Boerhaave
syndrome)
Pulmonary embolus
Mediastinal
Malignancy
Anxiety/emotion1
Peripheral
Lungs/pleura
Pulmonary infarct
Pneumonia
Pneumothorax
Malignancy
Tuberculosis
Connective tissue disorders
Musculoskeletal2
Osteoarthritis
Rib fracture/injury
Acute vertebral fracture
Costochondritis (Tietze syndrome)
Intercostal muscle injury
Epidemic myalgia (Bornholm disease)
Neurological
Prolapsed intervertebral disc
Herpes zoster
1
May also cause peripheral chest pain. 2
Can sometimes cause central chest pain.
212.
180 ACUTEMEDICINE AND CRITICAL ILLNESS
A detailed and clear history is key to narrowing the differential diagno-
sis of chest pain. Figure 9.1 shows how certain features of the history,
particularly when combined, can tip the balance of evidence towards or
away from ischaemic cardiac chest pain.
Clinical assessment
Cardiorespiratory examination may detect clinical signs that help guide
ongoing investigation. Patients with a history compatible with myocardial
ischaemia should have a 12-lead electrocardiogram (ECG) performed
while clinical examination proceeds. Ongoing chest pain with clinical fea-
tures of shock or pulmonary oedema, or ECG evidence of ventricular
arrhythmia or complete heart block, should prompt urgent cardiology
review and referral to a higher level of care.
Chest pain that is accompanied by clinical evidence of increased intra-
cardiac pressure (especially a raised jugular venous pressure) increases
the likelihood of myocardial ischaemia or massive PE. The legs should be
examined for clinical evidence of deep vein thrombosis.
A large pneumothorax should be evident on clinical examination, with
absent breath sounds and a hyper-resonant percussion note on the
affected side. Other unilateral chest signs, such as bronchial breathing
or crackles, are most likely to indicate a respiratory tract infection, and a
chest X-ray should be expedited.
Pericarditis may be accompanied by a pericardial friction rub. In aortic
dissection, syncope or neurological decit may occur. Examination may
reveal asymmetrical pulses, features of undiagnosed Marfan syndrome
(p. 445) or a new early diastolic murmur representing aortic regurgitation.
Any disease process involving the pleura may restrict rib movement
and a pleural rub may be audible on the affected side. Local tenderness
of the chest wall is likely to indicate musculoskeletal pain but can also be
found in pulmonary infarction.
Subdiaphragmatic inammatory pathology, such as a liver abscess,
cholecystitis or ascending cholangitis, can mimic pneumonia by causing
fever, pleuritic chest pain and a small sympathetic pleural effusion, usu-
ally on the right. Likewise, acute pancreatitis can present with thoracic
symptoms, and an amylase or lipase level should be requested where
appropriate. It is imperative that the abdomen is examined routinely in all
patients presenting with pleuritic chest pain.
Initial investigations
Chest X-ray, ECG and biomarkers (e.g. troponin, D-dimer) play a piv-
otal role in the evaluation of chest pain. However, indiscriminate order-
ing of such investigations may result in diagnostic confusion and
over-investigation. The choice of investigation(s) is intimately linked to the
history and examination ndings. A chest X-ray and 12-lead ECG should
be performed in the vast majority of patients presenting to hospital with
chest pain. Pregnancy is not a contraindication to chest X-ray, but par-
ticular consideration should be given to whether the additional diagnostic
information justies breast irradiation.
The chest X-ray may conrm the suspected diagnosis, particularly in
the case of pneumonia. Small pneumothoraces are easily missed, as are
rib fractures or small metastatic deposits, and all should be considered
individually during chest X-ray review. A widened mediastinum suggests
acute aortic dissection but a normal chest X-ray does not exclude the
diagnosis. Provided it has been more than 1hour since the onset of pain,
chest X-ray in oesophageal rupture may reveal subcutaneous emphy-
sema, pneumomediastinum or a pleural effusion.
Patients with a history compatible with myocardial ischaemia require
an urgent 12-lead ECG. Acute chest pain with ECG changes indicating
a ST segment elevation myocardial infarction (STEMI) suggests that the
patient is likely to benet from immediate reperfusion therapy. Specic
information relating to cocaine or amphetamine use should be sought,
particularly in younger patients. In the context of a compatible history,
an ECG showing ischaemic changes that do not meet STEMI criteria
should prompt regular repeat ECGs and treatment for non-ST segment
elevation myocardial infarction (NSTEMI)/unstable angina. Measurement
of serum troponin concentration on admission is often helpful in cases
where there is diagnostic doubt, but a negative result should always
prompt a repeat sample 6–12hours after maximal pain. Acute coronary
syndrome may be diagnosed with condence in patients with a convinc-
ing history of ischaemic pain (see Fig. 9.1) and either ECG evidence of
ischaemia or an elevated serum troponin. If an elevated serum troponin is
found in a patient who has an atypical history or is at low risk of ischae-
mic heart disease, then alternative causes of raised troponin should be
considered (Box 9.3). Further management of acute coronary syndromes
is discussed on page 432.
In the absence of convincing ECG evidence of myocardial ischaemia,
other life-threatening causes of chest pain, such as aortic dissection,
massive PE and oesophageal rupture, should be considered. Suspicion
of aortic dissection (background of hypertension, trauma, pregnancy
or previous aortic surgery) should prompt urgent thoracic computed
tomography (CT) or transoesophageal echocardiography. An ECG in the
context of massive PE most commonly reveals only a sinus tachycardia,
but may show new right axis deviation, right bundle branch block or a
dominant R wave in V1
. The classical nding of S1
Q3
T3
(a deep S wave in
Ischaemic
cardiac chest pain Non-cardiac chest pain
Relieving factors
Respiratory, gastrointestinal,
locomotor or psychological
Rest
Quick response to nitrates
Precipitation
Spontaneous, not related to exertion,
provoked by posture, respiration or palpation
Precipitated by exertion
and/or emotion
Character Tight, squeezing, choking
Radiation Other or no radiation
Jaw/neck/shoulder/arm
(occasionally back)
Location Peripheral, localised
Central, diffuse
Associated
features
Not relieved by rest
Variable or no response to nitrates
Breathlessness
Fig. 9.1 Identifying ischaemic cardiac pain: the ‘balance’ of evidence.
213.
Presenting problems inacute medicine 181
9
lead I, with a Q wave and T wave inversion in lead III) is rare. If massive PE
is suspected and the patient is haemodynamically unstable, a transthor-
acic echocardiogram, to seek evidence of right heart strain and exclude
alternative diagnoses such as tamponade, is extremely useful.
If the patient is deemed to be at low risk of PE, a D-dimer test can
be informative, as a negative result effectively excludes the diagnosis.
The D-dimer test should be performed only if there is clinical suspicion
of PE, as false-positive results can lead to unnecessary investigations. If
the D-dimer is positive, there is high clinical suspicion, or there is other
convincing evidence of PE (such as features of right heart strain on the
ECG), prompt imaging should be arranged (p. 546 and Fig. 17.67).
Acute breathlessness
In acute breathlessness, the history, along with a rapid but careful exam-
ination, will usually suggest a diagnosis that can be conrmed by routine
investigations including chest X-ray, 12-lead ECG and arterial blood gas
(ABG) sampling.
Presentation
A key feature of the history is the speed of onset of breathlessness. Acute
severe breathlessness (over minutes or hours) has a distinct differential
diagnosis list to chronic exertional breathlessness. The presence of asso-
ciated cardiovascular (chest pain, palpitations, sweating and nausea) or
respiratory (cough, wheeze, haemoptysis, stridor) symptoms can narrow
the differential diagnosis yet further. A previous history of left ventricular
dysfunction, asthma or exacerbations of chronic obstructive pulmonary
disease (COPD) is important. A high temperature, cough (productive or
non-productive) and/or viral prodrome may indicate respiratory infection
and, if so, relevant infection control precautions should be taken from ini-
tial assessment. In the severely ill patient, it may be necessary to obtain
the history from accompanying witnesses. In children, the possibility of
inhalation of a foreign body (Fig. 9.2) or acute epiglottitis should always be
considered. There is often more than one underlying diagnosis; a thorough
assessment should continue, even after a possible diagnosis has been
reached, particularly if the severity of symptoms does not seem to be ade-
quately explained. The causes of acute severe breathlessness are covered
here; chronic exertional dyspnoea is discussed further on page 489).
Clinical assessment
Airway obstruction, anaphylaxis and tension pneumothorax require
immediate identication and treatment. If any of these is suspected,
treatment should not be delayed while additional investigations are per-
formed, and anaesthetic support is likely to be required. In the absence of
an immediately life-threatening cause, the following should be assessed
and documented:
level of consciousness
degree of central cyanosis
work of breathing (rate, depth, pattern, use of accessory muscles)
adequacy of oxygenation (SpO2
)
ability to speak (in single words or sentences)
cardiovascular status (heart rate and rhythm, blood pressure (BP)
and peripheral perfusion).
Pulmonary oedema is suggested by a raised jugular venous pressure
and bi-basal crackles or diffuse wheeze, while asthma or COPD is charac-
terised by wheeze and prolonged expiration. A hyper-resonant hemithorax
with absent breath sounds raises the possibility of pneumothorax, while
severe breathlessness with normal breath sounds may indicate PE. Leg
swelling may suggest cardiac failure or, if asymmetrical, venous thrombosis.
The presence of wheeze is not always indicative of bronchospasm. In
acute left heart failure, an increase in the left ventricular diastolic pressure
causes the pressure in the left atrium, pulmonary veins and pulmonary
capillaries to rise. When the hydrostatic pressure of the pulmonary capil-
laries exceeds the oncotic pressure of plasma (about 25–30mmHg), uid
moves from the capillaries into the interstitium. This stimulates respiration
through a series of autonomic reexes, producing rapid, shallow respira-
tion, and congestion of the bronchial mucosa may cause wheeze (some-
times known as cardiac asthma). Sitting upright or standing may provide
some relief by helping to reduce congestion at the apices of the lungs.
The patient may be unable to speak and is typically distressed, agitated,
sweaty and pale. Respiration is rapid, with recruitment of accessory
muscles, coughing and wheezing. Sputum may be profuse, frothy and
blood-streaked or pink. Extensive crepitations and rhonchi are usually
audible in the chest and there may also be signs of right heart failure.
Any arrhythmia may cause breathlessness, but usually does so only
if the heart is structurally abnormal, such as with the onset of atrial bril-
lation in a patient with mitral stenosis. In such cases, the classic mid-di-
astolic rumbling murmur may be heard. Patients sometimes describe
chest tightness as ‘breathlessness’. However, myocardial ischaemia may
also induce true breathlessness by provoking transient left ventricular
dysfunction. When breathlessness is the dominant or sole feature of
9.3 Causes of elevated serum troponin other than acute
coronary syndrome
Cardiorespiratory causes
Pulmonary embolism
Acute pulmonary oedema
Tachyarrhythmias
Myocarditis/myopericarditis
Aortic dissection
Cardiac trauma
Cardiac surgery/ablation
Non-cardiorespiratory causes
Prolonged hypotension
Severe sepsis
Severe burns
Stroke
Subarachnoid haemorrhage
End-stage renal failure
A
B
Fig. 9.2 Inhaled foreign body.
covering mucous lm.
214.
182 ACUTEMEDICINE AND CRITICAL ILLNESS
myocardial ischaemia, it is known as ‘angina equivalent’. A history of
chest tightness or close correlation with exercise should be sought.
Initial investigations
As shown in Box 9.4, amalgamation of a clear history and thorough
clinical examination with chest X-ray, ECG and ABG ndings will usu-
ally indicate the primary cause of breathlessness. In cases of suspected
infection, a viral throat swab should be obtained early in the course of
the assessment; increasingly point-of-care testing of viral throat swabs is
available, providing important and rapid diagnostic information. If avail-
able, sputum should be sent for culture. If bronchospasm is suspected,
measurement of peak expiratory ow will assist in the assessment of
severity and should be performed whenever possible.
An ABG will often provide additional information to SpO2
measure-
ment alone, particularly if there is clinical evidence (drowsiness, delirium,
asterixis) or a strong likelihood of hypercapnia. An acute rise in PaCO2
will increase the HCO3
by only a small amount, resulting in inadequate
buffering and acidaemia. Renal compensation and a large rise in HCO3
will take at least 12hours. In acute type II respiratory failure (p. 496), the
rate of rise of PaCO2
is a better indicator of severity than the absolute
value. An ABG can also give a carboxyhaemoglobin level after smoke
inhalation (although this can also be measured on a venous sample), and
is central to the identication of metabolic acidosis or the diagnosis of
psychogenic hyperventilation (see Box 9.4).
If pulmonary embolism is suspected, calculating pre-test probability
(Ch. 1) is key. In a patient with a pre-test probability of less than 15%,
the pulmonary embolism rule-out criteria (PERC) can rule out pulmonary
embolism clinically, negating the need for further imaging, if none of the
criteria listed in Box 9.5 is met. Further detail on the investigation and
management of pulmonary embolus is given on page 546.
Procalcitonin (PCT) and N-terminal pro-hormone brain natriuretic pro-
tein (NT-proBNP) can be measured in venous blood. While these bio-
markers can give an indication of aetiology in shortness of breath, they
are probably of more value in tracking clinical progression and response
to treatment. Elevated PCT is a biomarker for bacterial infection and may
be useful, in addition to clinical assessment, in helping decide the need
for and duration of antibiotic therapy in patients with conrmed viral res-
piratory disease (such as COVID-19) who may have additional bacterial
super-infection. Elevated NT-proBNP is suggestive of underlying left ven-
tricular failure (p. 393), although it can be elevated in other conditions
such as renal failure, COPD, pulmonary hypertension and pulmonary
embolism. Measurement of NT-proBNP may be considered if there is no
clear-cut evidence of pulmonary oedema on a chest X-ray and can be
particularly useful as a ‘rule-out’ test, as a normal NT-proBNP has high
negative predictive value for heart failure. Individuals with suspected heart
failure should undergo early echocardiography (p. 402).
CT imaging (with or without pulmonary angiography) is a useful
investigation in many respiratory conditions as interstitial changes,
tumours or consolidation may not be evident on chest X-ray. If breath-
lessness is suspected to be an ‘angina equivalent’, objective evidence
9.5 PERC rule for pulmonary embolism
Rules out PE if none of the eight criteria is present and pre-test probability
is less than or equal to 15% (1.8% was chosen as the point of equipoise
between the benets and risks of further investigations for PE, and the benets
and risks of not investigating further).
Age ≥50
Heart rate ≥100
Oxygen saturation on room air <95%
Unilateral leg swelling
Haemoptysis
Recent surgery or trauma (≤4 weeks ago requiring hospitalisation)
Prior venous thromboembolism (VTE)
Hormone use (oral contraceptives), hormone replacement or oestrogenic
hormone use in male or female patients
9.4 Clinical features in acute breathlessness
Condition History Signs Chest X-ray ABG ECG
Pulmonary
oedema
Chest pain, palpitations,
orthopnoea, cardiac
history*
Central cyanosis, ↑JVP,
sweating, cool extremities,
basal crackles*
Cardiomegaly,
oedema/pleural
effusions*
↓PaO2
↓PaCO2
Sinus tachycardia,
ischaemia*, arrhythmia
Massive
pulmonary
embolus
Risk factors, chest pain,
pleurisy, syncope*,
dizziness*
Central cyanosis, ↑JVP*,
absence of signs in the
lung*, shock (tachycardia,
hypotension)
Often normal
Prominent hilar
vessels, oligaemic
lung elds*
↓PaO2
↓PaCO2
Sinus tachycardia,
RBBB, S1
Q3
T3
pattern↑T(V1
–V4
)
Acute severe
asthma
History of asthma,
asthma medications,
wheeze*
Tachycardia, pulsus
paradoxus, cyanosis (late),
→JVP*, ↓peak ow,
wheeze*
Hyperination only
(unless complicated
by pneumothorax)*
↓PaO2
Normal PaCO2
(↑PaCO2
in
extremis)
Sinus tachycardia
(bradycardia in
extremis)
Acute
exacerbation
of COPD
Previous episodes*,
smoker. If in type II
respiratory failure, may
be drowsy
Cyanosis, hyperination*,
signs of CO2
retention (apping
tremor, bounding pulses)*
Hyperination*,
bullae, complicating
pneumothorax
↓ or ↓↓PaO2
↑PaCO2
in type II failure
± ↑H+
, ↑HCO3
in
chronic type II failure
Normal, or signs of right
ventricular strain
Pneumonia/
lower
respiratory tract
infection
Prodromal illness*,
fever*, rigors*, pleurisy*
Fever, delirium, pleural
rub*, consolidation*,
cyanosis (if severe)
Pneumonic
consolidation*
↓PaO2
↓PaCO2
(↑ in extremis)
Tachycardia
Metabolic
acidosis
Evidence of diabetes
mellitus or renal
disease, aspirin or
ethylene glycol overdose
Fetor (ketones), hyperventilation
without heart or lung signs*,
dehydration*, air hunger
Normal PaO2
normal
↓↓PaCO2
, ↑H+
↓HCO3
Psychogenic Previous episodes,
digital or perioral
dysaesthesia
No cyanosis, no heart or
lung signs, carpopedal spasm
Normal PaO2
normal*
↓↓PaCO2
, ↓H+
*
*Valuable discriminatory feature.
(ABG = arterial blood gas; COPD = chronic obstructive pulmonary disease; JVP = jugular venous pressure; RBBB = right bundle branch block)
215.
Presenting problems inacute medicine 183
9
of myocardial ischaemia from stress testing may help to establish the
diagnosis, although coronary artery angiography (either by CT or car-
diac catheterisation) is often performed early in the investigation path-
way (p. 432).
Anaphylaxis
Anaphylaxis is a potentially life-threatening, systemic allergic reaction
characterised by a variable combination of circulatory collapse, bron-
chospasm, laryngeal stridor, angioedema and urticaria. The risk of death
is increased in patients with pre-existing asthma, particularly if this is
poorly controlled, and in situations where treatment with adrenaline (epi-
nephrine) is delayed.
Presentation
Anaphylaxis occurs when an allergen binds to and cross-links mem-
brane-bound IgE on mast cells in a susceptible individual, causing
release of histamine, tryptase and other vasoactive mediators from mast
cells. These mediators have a variety of effects, including vasodilatation,
increased capillary permeability leading to hypotension, and broncho-
constriction. It can be difcult to distinguish IgE-mediated anaphylaxis
clinically from non-specic degranulation of mast cells on exposure to
drugs, chemicals or other triggers where IgE is not involved, previously
known as anaphylactoid reactions.
Clinical assessment
The clinical features of anaphylaxis are summarised in Figure 9.3. Several
other conditions can mimic anaphylaxis and these are listed in Box 9.6
It is important to assess the severity of the reaction, and the time
between allergen exposure and onset of symptoms provides a guide.
Enquiry should be made about potential triggers. If a trigger is immedi-
ately obvious, a detailed history of the previous 24hours may be helpful.
The most common triggers of anaphylaxis are foods, latex, insect venom
and drugs. A history of previous local allergic responses to the offending
agent is common. The route of allergen exposure may inuence the prin-
cipal clinical features of a reaction; for example, if an allergen is inhaled,
the major symptom is frequently wheezing. Features of anaphylaxis may
overlap with the direct toxic effects of drugs and venoms (Chs 7 and 8).
Potentiating factors, such as exercise alcohol or fatigue, can lower the
threshold for an anaphylactic event. It is important to identify precipitating
factors so that appropriate avoidance measures may be taken in the
longer term.
Investigations and management
Measurement of serum mast cell tryptase concentrations is useful to
conrm the diagnosis. It is important to measure tryptase in serial tests
to demonstrate the temporal relationship to the triggering event. Specic
IgE tests may be useful in conrming hypersensitivity and may be prefer-
able to skin-prick tests when investigating patients with a history of ana-
phylaxis. The cornerstone of management of a severe case is the early
administration of adrenaline (epinephrine), which supports the cardiovas-
cular system, reduces bronchospasm and has some disease-modifying
effects. Secondary management depends upon which organs are most
affected. The emergency management of anaphylaxis is summarised in
Box 9.7. If present, angioedema of the orofacial area requires careful
observation and timely intervention, possibly with endotracheal intuba-
tion. The investigation and management of angioedema is discussed
further on page 82.
Individuals who have recovered from an anaphylactic event should
be referred for specialist assessment. The aims are to identify the trig-
ger, educate the patient regarding avoidance and management of sub-
sequent episodes, and to establish whether specic treatment, such
as immunotherapy, is indicated. If the trigger factor cannot be identi-
ed or avoided, recurrence is common. Patients who have previously
Itching of
palms, soles of feet
and genitalia
Feeling of impending doom,
loss of consciousness
Conjunctival injection
Flushing
Sweating
Hypotension
Urticaria
Wheeze,
bronchoconstriction
Angioedema
of lips and mucous
membrane
Abdominal
pain
Diarrhoea
Cardiac
arrhythmias
Laryngeal obstruction
Stridor
Wasp sting
Fig. 9.3 Clinical manifestations of anaphylaxis. In this example, the response is to an insect sting containing venom to which the patient is allergic. This causes release of
histamine and other vasoactive mediators, which lead to the characteristic features of anaphylaxis that are illustrated.
9.6 Differential diagnosis of anaphylaxis
Causes of hypotension
Vasovagal syncope
Cardiac arrhythmia
Cardiogenic shock
Causes of respiratory distress
Status asthmaticus Pulmonary embolus
Causes of laryngeal obstruction
C1 inhibitor deciency Idiopathic angioedema
Causes of generalised ushing
Systemic mastocytosis
Carcinoid syndrome
216.
184 ACUTEMEDICINE AND CRITICAL ILLNESS
experienced an anaphylactic event should be prescribed self-injectable
adrenaline (epinephrine) and they and their families or carers should be
instructed in its use (Box 9.8). The use of a MedicAlert (or similar) brace-
let will increase the likelihood of the injector being administered in an
emergency. There is an important role for a specialist dietitian when a
food trigger is the suspected allergen. Allergy in adolescence requires
additional consideration and management, as set out in Box 9.9
Syncope/presyncope
The term ‘syncope’ refers to sudden loss of consciousness due to
reduced cerebral perfusion. ‘Presyncope’ refers to lightheadedness, in
which the individual thinks he or she may ‘black out’. Dizziness and pre-
syncope are particularly common in old age (Box 9.10). Symptoms are
disabling, undermine condence and independence, and can affect a
person's ability to work or to drive.
There are three principal mechanisms that underlie recurrent presyn-
cope or syncope:
cardiac syncope due to mechanical cardiac dysfunction or arrhythmia
neurocardiogenic syncope (also known as vasovagal or reex syn-
cope), in which an abnormal autonomic reex causes bradycardia
and/or hypotension
postural hypotension, in which physiological peripheral vasocon-
striction on standing is impaired, leading to hypotension.
There are, however, other causes of loss of consciousness, and dif-
ferentiating syncope from seizure is a particular challenge. Psychogenic
blackouts (also known as non-epileptic seizures or pseudoseizures) also
need to be considered in the differential diagnosis.
Presentation
The history from the patient and a witness is the key to establishing a
diagnosis. The terms used for describing the symptoms associated with
syncope vary so much among patients that they should not be taken
for granted. Some patients use the term ‘blackout’ to describe a purely
visual symptom, rather than loss of consciousness. Some may under-
stand ‘dizziness’ to mean an abnormal perception of movement (vertigo),
9.8 How to prescribe self-injectable adrenaline (epinephrine)
Prescription (normally initiated by an immunologist or allergist)
Specify the brand of autoinjector, as they have different triggering mechanisms
Prescribe two devices
Indications
Anaphylaxis to allergens that are difcult to avoid:
Insect venom
Foods
Idiopathic anaphylactic reactions
History of severe localised reactions with high risk of future anaphylaxis:
Reaction to trace allergen
Likely repeated exposure to allergen
History of severe localised reactions with high risk of adverse outcome:
Poorly controlled asthma
Lack of access to emergency care
Patient and family education
Know when and how to use the device
Carry the device at all times
Seek medical assistance immediately after use
Wear an alert bracelet or necklace
Include the school in education for young patients
Other considerations
Caution with β-adrenoceptor antagonists (β-blockers) in anaphylactic patients
as they may increase the severity of an anaphylactic reaction and reduce the
response to adrenaline (epinephrine)
9.9 Allergy in adolescence
Resolution of childhood allergy: most children affected by allergy to milk,
egg, soybean or wheat will grow out of their food allergies by adolescence, but
allergies to peanuts, tree nuts, sh and shellsh are frequently life-long.
Risk-taking behaviour and fatal anaphylaxis: serious allergy is increasingly
common in adolescents and this is the highest risk group for fatal, food-induced
anaphylaxis. This is associated with increased risk-taking behaviour, and
food-allergic adolescents are more likely than adults to eat unsafe foods, deny
reaction symptoms and delay emergency treatment.
Emotional impact of food allergies: some adolescents may neglect to carry
a prescribed adrenaline (epinephrine) autoinjector because of the associated
nuisance and/or stigma. Surveys of food-allergic teenagers reveal that many
take risks because they feel socially isolated by their allergy.
9.10 Dizziness in old age
Prevalence: common, affecting up to 30% of people aged >65 years.
Symptoms: most frequently described as a combination of unsteadiness and
lightheadedness.
Most common causes: postural hypotension and cardiovascular disease. Many
patients have more than one underlying cause.
Arrhythmia: can present with lightheadedness either at rest or on activity.
Anxiety: frequently associated with dizziness but rarely the only cause.
Falls: multidisciplinary workup is required if dizziness is associated with falls.
9.7 Emergency management of anaphylaxis
Treatment Comment
Prevent further contact with allergen Prevents ongoing mast cell activation
Ensure airway patency Prevents hypoxia
Administer adrenaline (epinephrine)
promptly:
0.3–0.5mL 1:1000 solution IM
in adults (0.3–0.5mg)
Repeat at 5–10min intervals if
initial response is inadequate
Intramuscular route important
because of peripheral
vasoconstriction
Acts within minutes
Increases blood pressure
Reverses bronchospasm
Administer antihistamines:
Chlorphenamine 10mg IM or slow
IV injection
Blocks effect of histamine on target
cells
Administer glucocorticoids:
Hydrocortisone 200mg IV
Reduces cytokine release
Prevents rebound symptoms in
severe cases
Provide supportive treatment:
Nebulised adrenaline
(epinephrine) (e.g. 3mg)
Nebulised β2
-agonists (e.g.
salbutamol 5mg)
IV uids
Oxygen
Reduces airway oedema
Reverses bronchospasm
Restores plasma volume
Reverses hypoxia
(IM = intramuscular; IV = intravenous)
some will consider this a feeling of faintness, and others will regard it
as unsteadiness. The clinician thus needs to elucidate the exact nature
of the symptoms that the patient experiences. The potential differential
diagnosis of syncope and presyncope, on the basis of the symptoms
described, is shown in Figure 9.4.
The history should always be supplemented by a direct eye-witness
account if available. Careful history with corroboration will usually establish
whether there has been full consciousness, altered consciousness, vertigo,
transient amnesia or something else. Attention should be paid to potential
triggers (e.g. medication, micturition, exertion, prolonged standing), the
217.
Presenting problems inacute medicine 185
9
patient's appearance (e.g. colour, seizure activity), the duration of the epi-
sode, and the speed of recovery (Box 9.11). Cardiac syncope is usually
sudden, but can be associated with premonitory lightheadedness, palpita-
tion or chest discomfort. The blackout is usually brief and recovery rapid.
Exercise-induced syncope can be the presenting feature of a number of
serious pathologies (such as hypertrophic obstructive cardiomyopathy or
exercise-induced arrhythmia) and always requires further investigation.
Neurocardiogenic syncope will often be associated with a situational trig-
ger (such as pain or emotion), and the patient may experience ushing,
nausea, malaise and clamminess for several minutes afterwards. Recovery
is usually quick and without subsequent delirium, provided the patient
has assumed a supine position. There is often some brief stiffening and
limb-twitching, which requires differentiation from seizure-like movements.
It is rare for syncope to cause injury or to cause amnesia after regaining
awareness. Patients with seizures do not exhibit pallor, may have abnormal
movements, usually take more than 5 minutes to recover and are often
confused. Aspects of the history that can help to differentiate seizure from
syncope are shown in Box 9.12
A diagnosis of psychogenic blackout (also known as non-epileptic
seizure, pseudoseizure or psychogenic seizure) may be suggested by
specic emotional triggers, dramatic movements or vocalisation, or by
very prolonged duration (hours). A history of rotational vertigo is sug-
gestive of a labyrinthine or vestibular disorder (p. 1143). Postural hypo-
tension is normally obvious from the history, with presyncope or, less
commonly, syncope occurring within a few seconds of standing. The
history should include enquiry about predisposing medications (diuretics,
vasodilators, antidepressants) and conditions (such as diabetes mellitus
and Parkinson's disease).
Clinical assessment
Examination of the patient may be entirely normal, but may reveal clinical
signs that favour one form of syncope. The systolic murmurs of aortic
Labyrinthine dysfunction
Infection
Benign positional vertigo
Ménière’s disease
Ischaemia/infarction
Trauma
Other (e.g. drugs, otosclerosis)
Central vestibular dysfunction
‘Physiological’ (visual–
vestibular mismatch)
Demyelination
Migraine
Posterior fossa mass lesion
Vertebro-basilar ischaemia
Other (e.g. disorders of
cranio-vertebral junction)
Ataxia
Weakness
Loss of joint position sense
Gait dyspraxia
Joint disease
Visual disturbance
Fear of falling (Chs 26 and 28)
Impaired cerebral perfusion
Cardiac disease
Arrhythmia
Left ventricular dysfunction
Aortic stenosis
Hypertrophic obstructive
cardiomyopathy
Other causes
Vasovagal syncope
Postural hypotension
Micturition syncope
Cough syncope
Carotid sinus sensitivity
Hypoglycaemia (Ch. 21)
Anxiety*
Hyperventilation
Post-concussive syndrome
Panic attack
Non-epileptic attack
Epileptic seizure
Presyncope
(reduced cerebral
perfusion)
Syncope
(loss of cerebral
perfusion)
Sensation of
movement?
(vertigo)
Loss of
balance?
Lightheaded?
Other
description
‘Funny turn’
or blackout
* Anxiety is the most
common cause of
dizziness in those
under 65 years
Loss of
consciousness
(‘blackout’)
Fig. 9.4 The differential diagnosis of syncope and presyncope.
218.
186 ACUTEMEDICINE AND CRITICAL ILLNESS
to bradycardia, but the key to establishing a diagnosis is to obtain an
ECG recording while symptoms are present. Since minor rhythm distur-
bances are common, especially in older adults, symptoms must occur
at the same time as a recorded arrhythmia before a diagnosis can be
made. Ambulatory ECG recordings are helpful only if symptoms occur
several times per week. Patient-activated ECG recorders are useful for
examining the rhythm in patients with recurrent dizziness, but are not
helpful in assessing sudden blackouts. When these investigations fail to
establish a cause in patients with presyncope or syncope, an implantable
ECG recorder can be sited subcutaneously over the upper left chest.
This device continuously records the cardiac rhythm and will activate
automatically if extreme bradycardia or tachycardia occurs. The ECG
memory can also be tagged by the patient, using a hand-held activator
as a form of ‘symptom diary’. Stored ECGs can be accessed by the
implanting centre, using a telemetry device in a clinic, or using a home
monitoring system via an online link.
Head-up tilt-table testing is a provocation test used to establish the
diagnosis of vasovagal syncope. It involves positioning the patient supine
on a padded table that is then tilted to an angle of 60–70° for up to
45minutes, while the ECG and BP responses are monitored. A pos-
itive test is characterised by bradycardia (cardio-inhibitory response)
and/or hypotension (vasodepressor response), associated with typical
symptoms.
Headache
Headache is common and causes considerable worry amongst both
patients and clinicians, but rarely represents sinister disease. The causes
may be divided into primary or secondary, with primary headache syn-
dromes being vastly more common (Box 9.13).
Presentation
The primary purpose of the history and clinical examination in patients
presenting with headache is to identify the small minority of patients with
serious underlying pathology. Key features of the history include the tem-
poral evolution of a headache; a headache that reached maximal inten-
sity immediately or within 5 minutes of onset requires rapid assessment
for possible subarachnoid haemorrhage. Other ‘red ag’ symptoms are
shown in Box 9.14
It is important to establish whether the headache comes and goes,
with periods of no headache in between (usually migraine), or whether
it is present all or almost all of the time. Associated features, such as
preceding visual symptoms, nausea/vomiting or photophobia/phono-
phobia, may support a diagnosis of migraine but others, such as pro-
gressive focal symptoms or constitutional upset like weight loss, may
suggest a more sinister cause. The headache of cerebral venous throm-
bosis may be ‘throbbing’ or ‘band-like’ and associated with nausea,
vomiting or hemiparesis. Raised intracranial pressure (ICP) headache
9.11 Typical features of cardiac syncope,neurocardiogenic
syncope and seizures
Cardiac syncope Neurocardiogenic
syncope
Seizures
Premonitory
symptoms
Often none
Lightheadedness
Palpitation
Chest pain
Breathlessness
Nausea
Lightheadedness
Sweating
Delirium
Hyperexcitability
Olfactory
hallucinations
‘Aura’
Unconscious
period
Extreme ‘death-
like’ pallor
Pallor Prolonged
(>1min)
unconsciousness
Motor seizure
activity*
Tongue-biting
Urinary
incontinence
Recovery Rapid (< 1min)
Flushing
Rapid
Nausea
Lightheadedness
Prolonged
delirium
(>5mins)
Headache
Focal
neurological
signs
*N.B. Cardiac syncope can also cause convulsions by inducing cerebral anoxia.
9.12 How to differentiate seizures from syncope
Seizure Syncope
Aura (e.g. olfactory) + −
Cyanosis + −
Lateral tongue-biting + −/+
Post-ictal delirium + −
Post-ictal amnesia + −
Post-ictal headache + −
Rapid recovery − +
9.13 Primary and secondary headache syndromes
Primary headache syndromes
Migraine (with or without aura)
Tension-type headache
Trigeminal autonomic cephalalgia (including cluster headache)
Primary stabbing/coughing/exertional/sex-related headache
Thunderclap headache
New daily persistent headache syndrome
Secondary causes of headache
Medication overuse headache (chronic daily headache)
Intracranial bleeding (subdural haematoma, subarachnoid or intracerebral
haemorrhage)
Raised intracranial pressure (brain tumour, idiopathic intracranial hypertension)
Infection (meningitis, encephalitis, brain abscess)
Inammatory disease (temporal arteritis, other vasculitis, arthritis)
Referred pain from other structures (orbit, temporomandibular joint, neck)
stenosis or hypertrophic obstructive cardiomyopathy are important nd-
ings, particularly if paired with a history of lightheadedness or syncope
on exertion. BP taken when supine and then after 1 and 3minutes of
standing may, when combined with symptoms, provide robust evidence
of symptomatic postural hypotension.
Clinical suspicion of hypersensitive carotid sinus syndrome (sensitiv-
ity of carotid baroreceptors to external pressure such as a tight collar)
should prompt monitoring of the ECG and BP during carotid sinus pres-
sure, provided there is no carotid bruit or history of cerebrovascular dis-
ease. A positive cardio-inhibitory response is dened as a sinus pause
of 3seconds or more; a positive vasodepressor response is dened as
a fall in systolic BP of more than 50mmHg. Carotid sinus pressure will
produce positive ndings in about 10% of older adults, but fewer than
25% of these experience spontaneous syncope. Symptoms should not,
therefore, be attributed to hypersensitive carotid sinus syndrome unless
they are reproduced by carotid sinus pressure.
Initial investigations
A 12-lead ECG is essential in all patients presenting with syncope or
presyncope. Lightheadedness may occur with many arrhythmias, but
blackouts (Stokes–Adams attacks) are usually due to profound brady-
cardia or malignant ventricular tachyarrhythmias. The ECG may show
evidence of conducting system disease (e.g. sinus bradycardia, atrioven-
tricular block, bundle branch block), which would predispose a patient
219.
Presenting problems inacute medicine 187
9
retro-orbital pain, clouding of the cornea, decreased visual acuity and,
often, systemic upset. Temporal headaches in patients over 60 should
prompt examination for enlarged or tender temporal arteries and palpa-
tion of temporal pulses (often absent in temporal arteritis). Visual acuity
should be assessed promptly, as visual loss is an important complication
of temporal arteritis.
Initial investigations
If there is any alteration of conscious level, focal neurological signs,
new-onset seizures or a history of head injury, then CT scanning of the
head is indicated. The urgency of scanning will depend on the clinical
picture and trajectory but in many circumstances will be immediately
9.14 ‘Red ag’ symptoms in headache
Symptom Possible explanation
Sudden onset (maximal immediately
or within 5min)
Subarachnoid haemorrhage
Cerebral venous sinus thrombosis
Pituitary apoplexy
Meningitis
Focal neurological symptoms (other
than for typically migrainous)
Intracranial mass lesion:
Vascular
Neoplastic
Infection
Constitutional symptoms:
Weight loss
General malaise
Pyrexia
Meningism
Rash
Meningitis
Encephalitis
Neoplasm (lymphoma or metastases)
Inammation (vasculitis)
Raised intracranial pressure (worse
on waking/lying down, associated
vomiting)
Intracranial mass lesion
New-onset aged >60 years Temporal arteritis
9.15 Identification of bacterial meningitis
In patients presenting with headache, identication of those with bacterial
meningitis is a top priority to facilitate rapid antibiotic treatment. In almost all cases
there will be one of the following features:
meningism (neck stiffness, photophobia, positive Kernig's sign)
fever >38°C
signs of shock (tachycardia, hypotension, elevated serum lactate)
rash (not always petechial).
Hello, Mr XXX.
Can you open your
eyes, please?
Score best eyes/
motor/verbal response
Trapezius pinch Score response looking
at arms for localisation/
flexion/abnormal flexion
Pressure on
supra-orbital ridge
Score response
Firm nail-bed
pressure
Score if withdrawal present
or any eye/verbal response
No response
No response
No response
Fig. 9.5 Assessment of the Glasgow Coma Scale (GCS) score in an obtunded
patient. Avoid using a sternal rub, as it causes bruising.
tends to be worse in the morning and when lying at or coughing, and
associated with nausea and/or vomiting.
A description of neck stiffness along with headache and photophobia
should raise the suspicion of meningitis (Box 9.15), although this may
present in atypical ways in immunosuppressed, alcoholic or pregnant
patients. The behaviour of the patient during headache is often instruc-
tive; migraine patients typically retire to bed to sleep in a dark room,
whereas cluster headache often induces agitated and restless behaviour.
The pain of a subarachnoid haemorrhage frequently causes signicant
distress.
Headache duration is also important to elicit; headaches that have
been present for months or years are almost never sinister, whereas
new-onset headache, especially in older adults, is more of a concern. In
a patient over 60 years with head pain localised to one or both temples,
scalp tenderness or jaw claudication, temporal arteritis (Ch. 26) should
be considered.
Clinical assessment
An assessment of conscious level (using the Glasgow Coma Scale
(GCS); Fig. 9.5) should be performed early and constantly reassessed.
A decreased conscious level suggests raised ICP and urgent CT scan-
ning (with airway protection if necessary) is indicated. A full neurological
examination may provide clues as to the pathology involved; for example,
brainstem signs in the context of acute-onset occipital headache may
indicate vertebrobasilar dissection. Neurological signs may, however,
be ‘falsely localising’, as in large subarachnoid haemorrhage or bacte-
rial meningitis. Care should be taken to examine for other evidence of
meningitis such as a rash (not always petechial), fever or signs of shock.
Unilateral headache with agitation, ipsilateral lacrimation, facial sweat-
ing and conjunctival injection is typical of cluster headache. Conjunctival
injection may also be seen in acute glaucoma, accompanied by peri- or
220.
188 ACUTEMEDICINE AND CRITICAL ILLNESS
required. Intracranial haemorrhage or a space-occupying lesion with
mass effect should prompt urgent neurosurgical referral. If bacterial men-
ingitis is suspected (see Box 9.15), cerebrospinal uid (CSF) analysis is
required to make a denite diagnosis. Antibiotics should not be delayed
for lumbar puncture (LP), which needs to be preceded by CT scanning
only if raised ICP is suspected. In cases of thunderclap headache (peak
intensity within 5 minutes and lasting over an hour), a normal CT scan
should be followed by an LP performed more than 12 hours after head-
ache onset, to look for evidence of xanthochromia. A negative CT scan
within 6 hours of headache onset has a high degree of sensitivity for
detecting subarachnoid blood. In such cases, a discussion of risks and
benets with the patient may conclude that an LP is not necessary to
exclude subarachnoid haemorrhage, although a CT angiogram may be
considered to exclude other pathology, such as arterial dissection. Many
headaches require prompt involvement of specialists. Features of acute
glaucoma, for example, require immediate ophthalmological review for
measurement of intraocular pressures. Suspected temporal arteritis with
an erythrocyte sedimentation rate (ESR) of >50mm/hr should prompt
immediate glucocorticoid therapy and rheumatological referral (see
Ch. 26 for management). Features of raised ICP in the absence of a
mass lesion on neuroimaging may indicate idiopathic intracranial hyper-
tension; CSF opening pressure is likely to be informative.
Unilateral leg swelling
Most leg swelling is caused by oedema, the accumulation of uid within
the interstitial space. There are three explanatory mechanisms for devel-
opment of oedema, which are described in Box 9.16. Unilateral swelling
usually indicates a localised pathology in either the venous or the lym-
phatic system, while bilateral oedema often represents generalised uid
overload combined with the effects of gravity. However, all causes of
unilateral leg swelling may present bilaterally, and generalised uid over-
load may present with asymmetrical (and therefore apparently unilateral)
oedema. Fluid overload may be the result of cardiac failure, pulmonary
hypertension (even in the absence of right ventricular failure), renal failure,
hypoalbuminaemia or drugs (calcium channel blockers, glucocorticoids,
mineralocorticoids, non-steroidal anti-inammatory drugs (NSAIDs) and
others); see Box 16.14 for other causes. The remainder of this section
focuses on the causes of ‘unilateral’ oedema.
Presentation
Any patient who presents with unilateral leg swelling should be assessed
with the possibility of deep vein thrombosis (DVT) in mind. The pain and
swelling of a DVT is often fairly gradual in onset, over hours or even days.
Sudden-onset pain in the posterior aspect of the leg is more consistent
with gastrocnemius muscle tear (which may be traumatic or spontane-
ous) or a ruptured Baker's cyst. Leg swelling and pain associated with
paraesthesia or paresis, or in the context of lower limb injury or reduced
conscious level, should always prompt concern regarding the possibility
of compartment syndrome (Box 9.17).
Clinical assessment
Lower limb DVT characteristically starts in the distal veins, causing an
increase in temperature of the limb and dilatation of the supercial veins.
Often, however, symptoms and signs are minimal.
Cellulitis is usually characterised by erythema and skin warmth local-
ised to a well-demarcated area of the leg and may be associated with
an obvious source of entry of infection (e.g. leg ulcer or insect bite). The
patient may be febrile and systemically unwell. Supercial thrombophle-
bitis is more localised; erythema and tenderness occur along the course
of a rm, palpable vein.
Examination of any patient presenting with leg swelling should include
assessment for malignancy (evidence of weight loss, a palpable mass
or lymphadenopathy). Malignancy is a risk factor for DVT, but pelvic or
lower abdominal masses can also produce leg swelling by compress-
ing the pelvic veins or lymphatics. Early lymphoedema is indistinguish-
able from other causes of oedema. More chronic lymphoedema is rm
and non-pitting, often with thickening of the overlying skin, which may
develop a ‘cobblestone’ appearance.
Chronic venous insufciency is a cause of long-standing oedema
that, particularly when combined with another cause of leg swelling,
may acutely worsen. Characteristic skin changes (haemosiderin depo-
sition, hair loss, varicose eczema, ulceration) and prominent varicosities
are common, and sometimes cause diagnostic confusion with cellulitis.
See Box 9.17 for the examination ndings associated with compartment
syndrome.
Initial investigations
Factors predisposing to venous thromboembolism are covered in detail
in Chapter 25. Clinical criteria can be used to rank patients according to
their likelihood of DVT, by using scoring systems that determine pre-test
probability (see Ch. 1), such as the Wells score (Box 9.18). Figure 9.6
gives an algorithm for investigation of suspected DVT based on initial
Wells score. In patients with a low (‘unlikely’) pre-test probability of DVT,
D-dimer levels can be measured; if these are normal, further investiga-
tion for DVT is unnecessary. Further information on the interpretation of
D-dimer is given in Box 9.19. In those with a moderate or high (‘likely’)
probability of DVT or with elevated D-dimer levels, objective diagnosis of
DVT should be obtained using appropriate imaging, usually a Doppler
ultrasound scan. Therefore, in the same way as for pulmonary embo-
lus, the investigative pathway for DVT differs according to the pre-test
probability of DVT. For low-probability DVT, the negative predictive value
of the D-dimer test (the most important parameter in this context) is
over 99%; if the test is negative, the clinician can discharge the patient
with condence. In patients with a high probability of DVT, the negative
predictive value of a D-dimer test falls to somewhere in the region of
97%–98%. While this may initially appear to be a high gure, to discharge
2 or 3 patients in every 100 incorrectly would generally be considered
an unacceptable error rate. Hence, with the exception of pregnancy
(Box 9.20), a combination of clinical probability and blood test results
should be used in the diagnosis of DVT.
If cellulitis is suspected, serum inammatory markers, skin swabs
and blood cultures should be sent, ideally before antibiotics are given.
Ruptured Baker's cyst and calf muscle tear can both be readily diag-
nosed on ultrasound. If pelvic or lower abdominal malignancy is sus-
pected, a prostate-specic antigen (PSA) level should be measured in
males and appropriate imaging with ultrasound (transabdominal or trans-
vaginal) or CT should be undertaken.
9.16 Mechanisms of oedema
There are three explanatory mechanisms for the development of oedema that may
occur in isolation or combination:
increased hydrostatic pressure in the venous system due to increased
intravascular volume or venous obstruction
decreased oncotic pressure secondary to a decrease in the plasma proteins
that retain uid within the circulation
obstruction to lymphatic drainage (‘lymphoedema’).
9.17 Identification of compartment syndrome
Compartment syndrome classically occurs following extrinsic compression of
a limb due to trauma or reduced conscious level (especially when caused by
drugs or alcohol).
It usually presents with a tense, rm and exquisitely painful limb.
The pain is characteristically exacerbated by passive muscle stretching or
squeezing the compartment.
Altered sensation may be evident distally.
Absent peripheral pulses are a late sign and their presence does not exclude
the diagnosis.
Clinical suspicion of compartment syndrome should prompt measurement of
creatine kinase and urgent surgical review.
190 ACUTEMEDICINE AND CRITICAL ILLNESS
Symptoms and signs
of peritonitis
Resuscitation
Pain
No clear evidence
of peritonitis
Blood tests ↑ Amylase/lipase
Acute
pancreatitis
Free air Perforation
Dilated loops
of bowel
Intestinal
obstruction/ileus
Gallstones
and thickened
gallbladder wall
Cholecystitis
Pseudo-obstruction
CT scan
Pancreatitis
Abscess
Aortic aneurysm
Malignancy
Erect
chest X-ray
Abdominal
X-ray
Ultrasound
Contrast
radiology
Abnormality
detected
Abnormality
detected
Perforation
Inconclusive
investigations
No diagnosis
No free air
No abnormality
No abnormality
No abnormality
No abnormality
Symptoms persist
Symptoms
settle
Laparotomy
Laparoscopy
Observe
Fig. 9.7 Management of acute abdominal pain: an algorithm.
pancreatitis. Urinalysis is useful in suspected renal colic and pyelone-
phritis. An erect chest X-ray may show air under the diaphragm, sug-
gestive of perforation, and a plain abdominal lm may show evidence
of obstruction or ileus (see Fig. 23.11). An abdominal ultrasound may
help in identifying gallstones, renal stones, free uid or an intra-abdominal
abscess. Abdominal CT scanning is the most useful investigation, and
is essential in differentiating pseudo-obstruction and mechanical large
bowel obstruction. Concerns regarding contrast-induced nephropathy
should be secondary to the imperative to reach an early diagnosis. CT
may also be useful to seek evidence of pancreatitis, retroperitoneal col-
lections or masses, including aortic aneurysm.
Diagnostic laparotomy should be considered when the diagnosis has
not been revealed by other investigations. All patients must be carefully
and regularly reassessed (every 2–4 hours) so that any change in condi-
tion that might alter both the suspected diagnosis and clinical decision
can be observed and acted on early.
Management
The general approach is to close perforations, treat inammatory condi-
tions with antibiotics or resection, and relieve obstructions. Aspiration of
gastric contents is a major risk in all acute abdominal conditions, espe-
cially intestinal obstruction. This occurs once the laryngeal reexes are
overwhelmed by the volume of regurgitated material, and is particularly
common in frailer individuals or where there is a coexisting reduction in
conscious level. The placement of a large-bore nasogastric tube should
therefore be considered whenever bowel obstruction is suspected. The
speed of intervention and the necessity for surgery depend upon vari-
ous factors, of which the presence of peritonitis is the most pertinent.
A treatment summary of some of the more common surgical conditions
follows:
Acute appendicitis: This should be treated by early surgery, since
there is a risk of perforation and recurrent attacks with non-operative
treatment. The appendix can be removed through a conventional
right iliac fossa skin crease incision or by laparoscopic techniques.
Acute cholecystitis: This can be successfully treated non-operatively
but the high risk of recurrent attacks and the low morbidity of sur-
gery have made early laparoscopic cholecystectomy the treatment
of choice unless there are other considerations (such as pregnancy).
Acute diverticulitis: Conservative therapy is standard, but if perfora-
tion has occurred, resection is advisable. Depending on peritoneal
contamination and the degree of shock (risk of anastomotic leak
correlates with severity of shock), primary anastomosis is pref-
erable to a Hartmann's procedure (oversew of rectal stump and
end-colostomy).
Small bowel obstruction: If the cause is obvious and surgery inevita-
ble (such as with a strangulated hernia), an early operation is appro-
priate. If the suspected cause is adhesions from previous surgery,
only those patients who do not resolve within the rst 48 hours or
who develop signs of strangulation (colicky pain becoming constant,
peritonitis, tachycardia, fever, leucocytosis) should have surgery.
Large bowel obstruction: Pseudo-obstruction should be treated
non-operatively. Some patients benet from colonoscopic decom-
pression, but mechanical obstruction merits resection, usually with a
primary anastomosis. Differentiation between the two can be made
by water-soluble contrast enema.
223.
Common presentations ofdeterioration 191
9
Mesenteric ischaemia: This can be due to an arterial or venous throm-
bosis, or a structural abnormality of the gut such as a volvulus. Early
surgery can be successful in revascularisation of the gut or removing
ischaemic tissue, although acute mesenteric arterial thrombosis has a
poor prognosis, especially when the diagnosis is delayed.
Acute pancreatitis: CT scanning can be useful to rule out other
pathologies that may be amenable to operative management (e.g. ret-
roperitoneal perforation), but is not essential in the early management
of the disease. Management is usually conservative, and the majority
of patients will improve. A minority will develop pancreatic necro-
sis that will require prolonged supportive care and organ support.
Antibiotics are not usually indicated in the early phase of the disease.
Perforated peptic ulcer: Surgical closure of the perforation is stand-
ard practice, but some patients without generalised peritonitis can
be treated non-operatively once a water-soluble contrast meal has
conrmed spontaneous sealing of the perforation. Adequate and
aggressive resuscitation with intravenous uids, antibiotics and anal-
gesia is mandatory before surgery.
For a more detailed discussion of acute abdominal pain, the reader
is referred to the companion volume of this text, Principles and Practice
of Surgery
Identification and assessment of
deterioration
Early warning scores and the role of the medical
emergency team
There are many systems that have been developed with the aim of rapidly
identifying and managing physiological deterioration. These are referred to
as ‘rapid response systems’. One popular example of a rapid response
system is a medical emergency team (MET). A MET system operates on
the basis that when a patient meets certain physiological criteria, the team
is alerted. The team is expected to make a rapid assessment and institute
immediate management. This may include escalation to critical care or, fol-
lowing liaison with the parent clinical team, ongoing ward-based care.
The trigger for a ‘MET’ call may be a single parameter – such as a low
BP or tachycardia – or may consist of a composite early warning score.
Early warning score systems function by the observer allocating a value
between 0 and 3 for abnormalities in respiratory rate, SpO2
, tempera-
ture, BP, heart rate and neurological response (Fig. 9.8). The values are
summed and the composite score gives an indication of the severity of
physiological derangement. Early warning systems can be automated
into an electronic format that calculates the score and even alerts the
responsible clinician(s) by email or text message.
There are advantages and disadvantages to having a separate MET
system, compared with allowing the responsible clinical team to manage
deterioration, and to having a composite score or a single parameter
detection system. These are outlined in Box 9.22
Immediate assessment of the deteriorating patient
An approach to assessment of the deteriorating patient can be summa-
rised by the mnemonic ‘C-A-B-C-D-E’.
C – Control of obvious problem
For example, if the patient has ventricular tachycardia on the monitor or
signicant blood loss is apparent, immediate action is required.
A and B – Airway and breathing
If the patient is talking in full sentences, then the airway is clear and
breathing is adequate. A rapid history should be obtained while the
initial assessment is undertaken. Breathing should be assessed with a
focused respiratory examination. Oxygen saturations and ABGs should
be checked early (p. 194).
C – Circulation
A focused cardiovascular examination should include heart rate and
rhythm, blood pressure, jugular venous pressure, evidence of bleeding,
signs of shock and abnormal heart sounds. The carotid pulse should be
palpated in the collapsed or unconscious patient, but peripheral pulses
also should be checked in conscious patients. The radial, brachial, foot
and femoral pulses may disappear as shock progresses, and this indi-
cates the severity of circulatory compromise.
D – Disability
Conscious level should be assessed using the GCS (see Fig. 9.5 and
Box 9.29). A brief neurological examination looking for focal signs should
be performed. Capillary blood glucose should always be measured to
exclude hypoglycaemia or severe hyperglycaemia.
E – Exposure and evidence
‘Exposure’ indicates the need for targeted clinical examination of the
remaining body systems, particularly the abdomen and lower limbs.
‘Evidence’ may be gathered via a collateral history from other health-care
professionals or family members, recent investigations, prescriptions or
monitoring charts.
Selecting the appropriate location for ongoing
management and anticipatory care planning
Two groups of patients frequently gain benet from admission to criti-
cal care: those with organ dysfunction severe enough to require organ
support and those in whom the disease process is clearly setting them
on a downward trajectory and in whom early, aggressive manage-
ment may alter the outcome. Whether an individual patient should be
admitted to the intensive care or high-dependency unit (ICU/HDU) will
depend on local arrangements. A useful tool to assist with the decision
regarding location is the ‘level of care’ required (Box 9.23). Many inten-
sive care units are a mix of level 2 and level 3 beds, which streamlines
the admission process. When a patient requires admission with a sus-
pected transmissible disease that poses a risk to other patients and
staff, the patient should be admitted to an isolated room located within
an appropriate area of the hospital that can provide an appropriate
level of care.
Anticipatory care planning (ACP) refers to the process of decision-mak-
ing relating to treatments or support that would be appropriate if a patient’s
condition was to deteriorate. Admission can be a valuable opportunity for
discussion with the patient and family members/carers about the appro-
priate ‘ceiling of care’; for example, would resuscitation in the event of a
cardiac arrest and/or admission to a critical care area be appropriate. Such
discussions may result in the placement of a ‘do not resuscitate order’
and a decision that ‘ward-level’ care may be the ‘ceiling’ for that individual.
It is important that discussions and decisions are clearly documented in
the patient record. Further details of ACP decision-making are found on
page 206.
Common presentations of deterioration
As patients become critically unwell, they usually manifest physiological
derangement. The principle underpinning critical care is the simultane-
ous assessment of illness severity and the stabilisation of life-threatening
physiological abnormalities. The goal is to prevent deterioration and
224.
192 ACUTEMEDICINE AND CRITICAL ILLNESS
Full Name
NEWS key
A+B
0 1 2 3 Date of Birth
A+B
C
C
D
E
SpO2 Scale 2†
Oxygen saturation (%)
Date
Time
≥25
21–24
18–20
15–17
12–14
9–11
≤8
Date of Admission
3
2
1
1
1
2
2
3
1
2
2
3
1
2
3
1
2
1
1
3
3
3
2
1
3
3
3
3
≥25
21–24
18–20
15–17
12–14
9–11
≤8
≥97 on O2
95–96 on O2
93–94 on O2
≥93 on air
88–92
86–87
84–85
≤83%
≥220
201–219
181–200
161–180
141–160
121–140
111–120
101–110
91–100
81–90
71–80
61–70
51–60
≤50
≥131
121–130
111–120
101–110
91–100
81–90
71–80
61–70
51–60
41–50
31–40
≤30
≥131
121–130
111–120
101–110
91–100
81–90
71–80
61–70
51–60
41–50
31–40
≤30
≥220
201–219
181–200
161–180
141–160
121–140
111–120
1